sat full reading explanations (qas 9-19) · sat full reading explanations (qas 9-19) sat #9 reading...

297
SAT Full Reading Explanations (QAS 9-19) SAT #9 Reading 1 1.R.9 Answer Choice C is the correct answer because Lines 19-22 say, “Moths were shuddering round and round the lantern, though the old man was oblivious to them. He had come out because he had heard the throbbing of the engine in the distance.” These Lines tell us most clearly that the old man was standing on the verandah because he had the car, matching answer choice C which says that he came out because “he had heard what he believed to be the visitors’ car.” Answer Choice A is incorrect because the Passage doesn’t tell us he was looking for cars traveling down the road. Answer Choice B is incorrect because the Passage doesn’t show an exchange between the boys and the old man, and them telling him that the visitors would arrive soon. Answer Choice D is incorrect because the Passage doesn’t say that the old man enjoyed listening to the evening. How to solve this? The Question asks, “According to the passage, the old man was standing on the verandah because [...]?” To solve this [...]. 2.R.9 Answer Choice A is the correct answer because Lines 37-39 say, “‘This is too much,’ she insisted, with the air of one who has just received the Kohinoor diamond as a birthday present.” These Lines tell us most clearly that the yoghurt and sweetmeats are compared to the “Kohinoor diamond,” or to a “jewel,” matching Answer Choice A. Answer Choices B, C and D are incorrect because the Passage doesn’t compare the yoghurt and sweetmeats to cuisine, a wedding gift or a generous donation. How to solve this? The Question asks, “In the passage, the yoghurt and sweetmeats are compared to a [...]?” To solve this [...].

Upload: others

Post on 11-Oct-2020

151 views

Category:

Documents


1 download

TRANSCRIPT

Page 1: SAT Full Reading Explanations (QAS 9-19) · SAT Full Reading Explanations (QAS 9-19) SAT #9 Reading 1 1.R.9 Answer Choice C is the correct answer because Lines 19-22 say, “Moths

SAT Full Reading Explanations (QAS 9-19)

SAT #9

Reading 1

1.R.9

Answer Choice C is the correct answer because Lines 19-22 say, “Moths were shuddering round and round the lantern, though the old man was oblivious to them. He had come out because he had heard the throbbing of the engine in the distance.” These Lines tell us most clearly that the old man was standing on the verandah because he had the car, matching answer choice C which says that he came out because “he had heard what he believed to be the visitors’ car.” Answer Choice A is incorrect because the Passage doesn’t tell us he was looking for cars traveling down the road. Answer Choice B is incorrect because the Passage doesn’t show an exchange between the boys and the old man, and them telling him that the visitors would arrive soon. Answer Choice D is incorrect because the Passage doesn’t say that the old man enjoyed listening to the evening.

How to solve this? The Question asks, “According to the passage, the old man was standing on the verandah because [...]?” To solve this [...].

2.R.9

Answer Choice A is the correct answer because Lines 37-39 say, “‘This is too much,’ she insisted, with the air of one who has just received the Kohinoor diamond as a birthday present.” These Lines tell us most clearly that the yoghurt and sweetmeats are compared to the “Kohinoor diamond,” or to a “jewel,” matching Answer Choice A. Answer Choices B, C and D are incorrect because the Passage doesn’t compare the yoghurt and sweetmeats to cuisine, a wedding gift or a generous donation.

How to solve this? The Question asks, “In the passage, the yoghurt and sweetmeats are compared to a [...]?” To solve this [...].

Page 2: SAT Full Reading Explanations (QAS 9-19) · SAT Full Reading Explanations (QAS 9-19) SAT #9 Reading 1 1.R.9 Answer Choice C is the correct answer because Lines 19-22 say, “Moths

3.R.9

Answer Choice C is the correct answer because the word “air” in Lines 37 and 40 is used to express the attitude or “demeanor” the characters giving and receiving their gifts. Answer Choice A is incorrect because “atmosphere” doesn’t provide a good substitute for expressing attitude. Answer Choice B is incorrect because it doesn’t make sense to say that the characters acted with the “absence” of something. Answer Choice D is incorrect because the characters didn’t act with a “melody,” or actual song.

● “This is too much,” she insisted, with the air of one who has just received the Kohinoor diamond as a birthday present.

● “This is too much,” she insisted, with the demeanor of one who has just received the Kohinoor diamond as a birthday present.

How to solve this? The Question asks, “As used in lines 37 and 40, ‘air’ most nearly means [...]?”

4.R.9 & 5.R.9

Answer Choices B and B are the correct answers because Lines 44-46 say, “they fussed and fussed and created the illusion that it was something, something unique and untasted and unencountered.” These Lines most clearly tell us how the characters acted during the gift giving, and that they made the exchange more important than it was in reality. This most closely matches Answer Choice B, which says that the characters’ behavior serves to “inflate the significance of the gesture.” For Question 5, Answer Choice A is incorrect because these Lines only tell us the main character’s opinion of the gifts and not directly how the other characters behaved. Answer Choice C is incorrect because these Lines come after the exchange of gifts and concern the greeting between Chhotomama and the couple. Answer Choice D is incorrect because these Lines also come after the gift exchange and primarily concern the greeting. For Question 4, Answer Choice A is incorrect because none of the Lines mention how lavish or expensive the gifts are, and, in fact, the Passage tells us that the gifts “were nothing.” Answer Choice C is incorrect because neither the Texts nor the Passage show the characters being indifferent towards the gifts, but instead show them having strong emotional reactions. Answer Choice D is incorrect because neither the Texts nor the Passage talk about polite behavior or its necessity.

How to solve this? Question 4 asks, “The characters’ behavior during the gift giving mainly serves to [...]?” To solve this [...].

Page 3: SAT Full Reading Explanations (QAS 9-19) · SAT Full Reading Explanations (QAS 9-19) SAT #9 Reading 1 1.R.9 Answer Choice C is the correct answer because Lines 19-22 say, “Moths

6.R.9

Answer Choice A is the correct answer because Lines 54-57 say, “This was half a token gesture towards modesty, and half towards the new, ‘modern’ India — Nehru’s secular India, free of ritual and religion,” while Lines 60-63 say, “This was half a token gesture towards modesty, and half towards the old, ‘traditional’ India — Gandhi’s India of ceremony and custom.” These Lines come in the interaction between Chhotomama and the son, where the son responds by kissing Chhotomama’s feet and Chhotomama reacts by telling him to stop, and tell us that the men differed in respect to the old and new India. This most closely matches Answer Choice A, which says that the description of the interaction serves to “show how the characters diverge in their approaches to cultural practices.” Answer Choice B is incorrect because, while their different cultural beliefs might show some complexity between the two, the Lines don’t quite establish them as having a relationship or an especially complex one [with many, etc.], but instead emphasize their different cultural beliefs. Answer Choice C is incorrect because, while the Passage does reference an old, “traditional” India, it doesn’t tell us that either man misunderstands Indian history. Answer Choice D is incorrect because the Passage doesn’t indicate that these gestures or this exchange became part of the mens’ routine. How to solve this? The Question asks, “The description of Chhotomama and the son’s interaction mainly serves to [...]?”

7.R.9

Answer Choice D is the correct answer because Lines 64-69 say, “Sandeep, meanwhile, had come to the conclusion that the grown-ups were mad, each after his or her own fashion. Simple situations were turned into complex, dramatic ones; not until then did everyone feel important and happy. Will they never grow up? thought Sandeep irately.” These Lines most clearly tell us a realization that Sandeep comes to have over the course of the Passage, and tell us that he wonders if the adults will ever grow up. This most closely matches Answer Choice D, which says the Sandeep comes to view the adults as “immature.” Answer Choice A is incorrect because the Passage doesn’t tell us that Sandeep views the adults as strict. Answer Choice B is incorrect because nowhere does the Passage suggest that he views the adults as reserved. Answer Choice C is incorrect, because the Passage tells us that Sandeep views the adults immature and “mad,” not sophisticated.

How to solve this? The Question asks, “Over the course of the passage, Sandeep comes to view the adults as [...]?” To solve this [...].

Page 4: SAT Full Reading Explanations (QAS 9-19) · SAT Full Reading Explanations (QAS 9-19) SAT #9 Reading 1 1.R.9 Answer Choice C is the correct answer because Lines 19-22 say, “Moths

8.R.9

Answer Choice C is the correct answer because Lines 66-68 say, “Simple situations were turned into complex dramatic ones; not until then did everyone feel important and happy.” These Lines most clearly tell us that Sandeep would be most critical of actions where people make a bigger deal out of something than it is, like the gift-giving scene where everyone “fussed and fussed and created the illusion that it was something, something unique and untasted and unencountered.” This matches Answer Choice C, which says that Sandeep would be critical of “the old lady’s reaction to the gift.” Answer Choice A is incorrect because the Passage doesn’t say much about the boys playing carrom, and doesn’t tell us that they ever turned simple situations into complex ones. Answer Choice B is incorrect because the Passage doesn’t indicate that Mamima’s question about the gender of the child made a situation more complex or dramatic than it had to be. Answer Choice D is incorrect because, while the greeting scene between the son and daughter-in-law and Chhotomama may qualify as being overly dramatic and complex, the two of them simply waiting in the anteroom wouldn’t be.

How to solve this? The Question asks, “Sandeep would be most critical of which action from the passage?” To solve this [...].

9.R.9

Answer Choice C is the correct answer because Lines 54-57 say “This was half a token gesture towards modesty, and half towards the new, “modern” India — Nehru’s secular India, free of ritual and religion.” These Lines most clearly tell us that there’s a new India, free of some of the older traditions, and that India has experienced social change. Answer Choice A is incorrect because these Lines only tell us how the old lady reacted to the gifts, not about broad social change in India. Answer Choice B is incorrect because these Lines only show the son and daughter-in-law stooping to touch Chhotomama’s feet, but don’t tell us anything directly about social change in India. Answer Choice D is incorrect because these Lines tell us about Sandeep’s view on the difference between poverty and austerity, and not about any broad social change in India.

How to solve this? The Question asks, “Which lines from the passage most strongly suggest that India has experienced social change?” To solve this [...].

Page 5: SAT Full Reading Explanations (QAS 9-19) · SAT Full Reading Explanations (QAS 9-19) SAT #9 Reading 1 1.R.9 Answer Choice C is the correct answer because Lines 19-22 say, “Moths

10.R.9

Answer Choice A is the correct answer because the “word” impression in Line 72 is used to describe how the room looked, and the way it “appeared’ to Sandeep. Answer Choice B is incorrect because it’s unclear how the room could give a “belief.” Answer Choice C is incorrect because the word “impression” isn’t emphasizing any kind of imitation or resemblance. Answer Choice D is incorrect because the Lines don’t tell us that Sandeep had a recollection or remembrance.

● Despite its bareness, the impression it gave was of austerity rather than poverty. ● Despite its bareness, the appearance it gave was of austerity rather than poverty.

How to solve this? The Question asks, “As used in line 72, ‘impression’ most nearly means [...]?”

Reading 2

Passage Outline

Mindwise: How We Understand What Others Think, Believe, Feel, and Want

● It can be hard to know your own reputation. Several studies tested whether people can predict how others in a group feel about them. (Paragraph 1)

● People are pretty good at guessing the group as a whole feels about them. (Paragraph 2)

● But people are pretty bad at predicting how any individual within that group feels about them. (Paragraphs 3 & 4)

● People aren’t even good at knowing whether individuals like them or not. (Paragraph 5)

11.R.9

Answer Choice D is the correct answer because the first sentence of the Passage says, “Knowing your own reputation can be surprisingly difficult,” meaning that we have to find a Text that support this claim. Lines 54-58 say, “Although you might have some sense of how smart your coworkers think you are, you appear to have no clue about which coworkers in particular find you smart and which do not,” which tell us clearly that knowing how others feel about us can be difficult. Answer Choice A is incorrect because these Lines only introduce a set of experiments meant to test the first claim of the Passage, but don’t directly support the sentence itself. Answer Choice B is incorrect because these Lines only give general statistical information about how the results will be measured, without providing support for the fact that knowing your reputation can be difficult. Answer Choice C is incorrect because these Lines also only continue to give general statistical information.

How to solve this? The Question asks, “Which choice best supports the claim in the first sentence of the passage?” To solve this [...].

Page 6: SAT Full Reading Explanations (QAS 9-19) · SAT Full Reading Explanations (QAS 9-19) SAT #9 Reading 1 1.R.9 Answer Choice C is the correct answer because Lines 19-22 say, “Moths

12.R.9

Answer Choice D is the correct answer because Lines 23-27 say, “Statistically speaking, you measure relationships like these with a correlation, where perfect correspondence yields a correlation of 1 and no correspondence yields a correlation of 0. The closer the correlation is to 1, the stronger the relationship.” These Lines come in the context of describing a series of experiments, and primarily give the reader background information for how to understand the basic statistics behind the study, and most clearly match Answer Choice D, which says that the information is presented in order to “provide context for a way in which the results of the experiments will be discussed.” Answer Choice A is incorrect because, while these Lines do explain how some researchers quantify their experiments, the information isn’t presented as a way to correct common misunderstandings. Answer Choice B is incorrect because these Lines aren’t presented in a way that suggests that they’re meant to give a counterargument to potential objections. Answer Choice C is incorrect because these Lines only discuss how the data in the studies was measured, but doesn’t point to a finding or pattern in the conclusion of the experiment.

How to solve this? The Question asks, “The information about statistical measurement in Lines 23-27 (“Statistically … relationship”) is presented in order to [...]?” To solve this [...].

13.R.9 & 14.R.9

Answer Choices B and B are the correct answers because Lines 28-33 say, “These experiments suggested that people are pretty good, overall, at guessing how a group of others would evaluate them, on average.” These Lines most clearly tell us in what situation someone could accurately predict how others feel about them, and say that we’re good at understanding how large groups feel about us. This most closely matches Answer Choice B from Question 13, which says that the situation when someone could best understand how others feel about them is when “a manager predicts the collective opinion of employees about her ability.” For Question 14, Answer Choice A is incorrect because these Lines only explain the traits that were measured in the series of experiments looked at. Answer Choice C is incorrect because these Lines begin the discussion of how people are bad at predicting how others feel about them, and don’t provide us with the information we need. Answer Choice D is incorrect because these Lines give a series of rhetorical statements which the author doesn’t believe are true and goes on to show as false, and so can’t tell us about what situations people are good at understanding. For Question 13, Answer Choice A is incorrect because the Passage tells us that people are good predicting how groups feel about them but bad at predicting how individuals feel about them, like in this situation. Answer Choice C is incorrect because the Passage makes the case that we’re good on average at predicting how a group feels about us, but not individuals, which better matches B than C, which only concerns assessing two students. Answer Choice D is incorrect because this shows a person trying to predict how another individual feels about them, which the Passage suggests we’re pretty bad at.

How to solve this? The Question asks, “Based on the passage, in which situation would an individual stand the greatest chance of accurately predicting how he or she is perceived?”

Page 7: SAT Full Reading Explanations (QAS 9-19) · SAT Full Reading Explanations (QAS 9-19) SAT #9 Reading 1 1.R.9 Answer Choice C is the correct answer because Lines 19-22 say, “Moths

15.R.9

Answer Choice A is the correct answer because the word “magnitude” in Line 35 is used to describe a statistical correlation, and its common to describe that correlation using “strength.” Answer Choice B is incorrect because it’s unclear how “influence” would work in this sentence and its unidiomatic. Answer Choice C is incorrect because [...]. As well as the others

● To put that in perspective, this is roughly the same magnitude as the correlation between the heights of fathers and the heights of sons (around .5).

● To put that in perspective, this is roughly the same strength as the correlation between the heights of fathers and the heights of sons (around .5).

How to solve this? The Question asks, “As used in line 35, ‘magnitude’ most nearly means?”

16.R.9

Answer Choice A is the correct answer because the words “clueless” and “mind-reading” are more casual, informal words and are used in slightly humorous ways in the Text, like in Lines 61-62, which say, “But perhaps this is holding your mind-reading abilities to too high a standard?” This most closely matches Answer Choice A, which says that the words affect the tone of the Passage by contributing “to a casual and gently humorous tone that renders a potentially specialized discussion more approachable.” Answer Choice B is incorrect because these its not clear that the words contribute to a “mocking and disapproving tone,” and because the author isn’t critical of the researchers’ conclusions. Answer Choice C is incorrect because it’s too strong to say that these words contribute to a pessimistic tone and because the author doesn’t say that we can ever know how others perceive us, and in fact says that we do have a “tiny glimmer of insight.” Answer Choice D is incorrect because neither these words nor the Passage try to cast doubt on “the real-world usefulness of experimental data.”

How to solve this? The Question asks, “What main effect do the words ‘clueless’ (line 38) and ‘mind-reading” (line 61) have on the tone of the passage?” To solve this [...].

17.R.9

Answer Choice C is the correct answer because Lines 58-60 say, “‘People seem to have just a tiny glimmer of insight into how they are uniquely viewed by particular other people.’” These Lines tell us that this author believes that people have some insight into how they’re perceived by others, but not much. This most closely matches Answer Choice C, which says that the author believes the study reveals “that one individual hardly knows what another individual thinks of him or her.” Answer Choice A is incorrect because of the word “promising,” which adds an extra element to the claim that isn’t directly supported by the Passage. Answer Choice B is incorrect because the quote comes in the context of predicting how we’re perceived by other individuals, and the Passage suggests that we’re generally pretty good at predicting how we’re regarded by groups of people. Answer Choice D is incorrect because the quote tells us that we at least have a “tiny glimmer of insight” into how we’re viewed by others, which is better than random guesses.

Page 8: SAT Full Reading Explanations (QAS 9-19) · SAT Full Reading Explanations (QAS 9-19) SAT #9 Reading 1 1.R.9 Answer Choice C is the correct answer because Lines 19-22 say, “Moths

How to solve this? The Question asks, “The author quoted in lines 58-60 expresses which view of the study’s results?” To solve this [...].

18.R.9

Answer Choice D is the correct answer because Lines 78-85 say, “The same barely-better-than-guessing accuracy is also found in experiments investigating how well speed daters can assess who wants to date them and who does not, how well job candidates can judge which interviewers were impressed by them and which were not, and even how well teachers can predict their course evaluations.” These Lines apply to same findings from the study to these different scenarios and show the broader application of the findings. This most closely matches Answer Choice D which says that information is presented to “offer examples of situations in which a certain finding holds true.” Answer Choice A is incorrect because there’s nothing in these Lines or their surrounding context that tell us that we should have caution in making assumptions about others’ motives. Answer Choice B is incorrect because the Passage is making a comparison between the three scenarios, not trying to draw a distinction between them. Answer Choice C is incorrect because the Passage doesn’t say that we more research needs to be conducted in these fields.

How to solve this? The Question asks, “The main reason that the author includes the information about speed daters, job candidates, and teachers in lines 78-85 is to [...]?” To solve this [...].

19.R.9

Answer Choice C is the correct answer because the Figure measures six scenarios, represented along the x-axis, and measures the correlation along the y-axis. The bar that most nearly approaches “complete agreement” is the dark gray bar for correlation A, which rises to a correlation of about .9. This correlation measures how a person views themselves versus how they think someone they know really well will view them. This most closely matches Answer Choice C which says that the strongest correlation holds between individuals’ self-perception and “the individuals’ predictions of how they are perceived by people with whom they are well acquainted.” Answer Choice A is incorrect because this claim refers to the light gray bar for correlation B, which rises to just under .3. Answer Choice B is incorrect because this refers to the dark gray bar for correlation B, which rises to about .25. Answer Choice D is incorrect because this choice refers to both the dark and gray bar for correlation A, where the light gray bar rises to about .55.

How to solve this? The Question asks, “According to the figure, the mean correlation that most nearly approaches complete agreement exists between individuals’ self-perception and [...]?” To solve this.

Page 9: SAT Full Reading Explanations (QAS 9-19) · SAT Full Reading Explanations (QAS 9-19) SAT #9 Reading 1 1.R.9 Answer Choice C is the correct answer because Lines 19-22 say, “Moths

20.R.9

Answer Choice B is the correct answer because the description for correlation C says it describes the “correlation between individuals’ predictions of how others perceive them and actual perception of those individuals by others.” This means the comparison between how you think others will view you versus how others actually view you. This most closely matches Answer Choice B, which says that correlation C is exemplified by a situation where “Sally expects that others will say she is outgoing, but many describe her as reserved.” Answer Choice A is incorrect because it describes the distinction between how you view yourself versus how you think others will view you, which matches correlation A. Answer Choice C is incorrect because this doesn’t describe a distinction but only says that people who know you well describe you in one way. Answer Choice D is incorrect because this describes a difference between how you act in one setting versus another, which isn’t measured by any of the correlations in the figure.

How to solve this? The Question asks, “Which statement best exemplifies the distinction made by correlation C in the figure?”

21.R.9

Answer Choice C is the correct answer because the Figure primarily differences between people’s understanding of themselves, predictions about how others will perceive them and how those others actually perceive them. This could best answer the question from Answer Choice C, which says “To what degree are people able to predict how individual acquaintances perceive them?” because this is what correlation C measures. Answer Choice A is incorrect because the figure doesn’t give any information about why the researchers chose the specific traits that they did. Answer Choice B is incorrect because the Figure only concerns individuals and not groups, and doesn’t say anything about why this distinction might hold. Answer Choice D is incorrect because the Figure doesn’t say anything about why people may have different opinions of trustworthiness.

How to solve this? The Question asks, “Information in the figure is most useful for addressing which question provoked by the passage?” To solve this [...].

Page 10: SAT Full Reading Explanations (QAS 9-19) · SAT Full Reading Explanations (QAS 9-19) SAT #9 Reading 1 1.R.9 Answer Choice C is the correct answer because Lines 19-22 say, “Moths

Reading 3

Passage Outline

Has Pluto Sent Us a Message in Ceres?

● the dwarf planet Ceres may have traveled from where Pluto is to the asteroid belt between Mars and Jupiter. (paragraph 1)

● it is the biggest planet in the asteroid belt. it also has a high proportion of water ice, unlike other objects in the belt. it may also have ammonium-rich clay on its surface, unlike other objects. (paragraphs 2 and 3)

● how did it move to the asteroid belt? the Nice model says that Uranus and Neptuen move through the outer solar system and dragged objects along with it, like Ceres. (paragraph 4)

● two scientists led studies which support this idea. some of the asteroids in the belt have a dark, reddish colour which suggests they were once icy objects that had their outer layer burned off. (paragraph 5)

● another scientist believes that some objects may have traveled to the asteroid belt, but doesn’t believe Ceres is one of them. (paragraph 6)

22.R.9 & 23.R.9

Answer Choices B and D are the correct answers because Lines 18-20 say, “This material has never been found in the fragments of asteroids that have fallen to Earth, but fits the expected ammonia-rich composition of a TNO” These Lines tell us that one of the things that makes Ceres different from other objects in the asteroid belt is that its surface is made up of a different material than most other asteroids. This most closely matches Answer Choice B from Question 22 which says that “the surface composition of Ceres is dissimilar to the composition of the other objects.” For Question 23, Answer Choice A is incorrect because, while these Lines do explain a possible difference between Ceres and other objects in the asteroid belt, they don’t come from McKinnon’s point of view. Answer Choice B is incorrect because, while these Lines do tell us about a difference between Ceres and other objects in the belt and do come from McKinnon’s point of view, they don’t match any Answer Choice from Question 22. Answer Choice C is incorrect because, while these Lines do introduce the fact that ammonia-rich clay is on the surface of Ceres, they less clearly tell us that this material is unique for asteroids than Answer Choice D does. For Question 22, Answer Choice A is incorrect because neither the Lines nor the Passage tell us about the surface temperature of Ceres and how it’s lower than other objects. Answer Choice B is incorrect because these Lines don’t emphasize that Ceres has the largest dimensions of all objects in the asteroid belt, but that these dimensions have varied the most, a point that neither the Passage nor the Text makes. Answer Choice D is incorrect because the Passage only says that light reflected from Ceres shows “a dip” in the light spectrum, not that the reflected light is more intense than for other objects.

How to solve this? Question 22 asks, “In McKinnon’s view, Ceres differs from other objects in the asteroid belt in which significant way?” To solve this [...].

Page 11: SAT Full Reading Explanations (QAS 9-19) · SAT Full Reading Explanations (QAS 9-19) SAT #9 Reading 1 1.R.9 Answer Choice C is the correct answer because Lines 19-22 say, “Moths

24.R.9

Answer Choice A is the correct answer because Lines 27-30 say, “According to this idea — known as the Nice model — Uranus and Neptune went rampaging through the outer solar system around 3.9 billion years ago.” These Lines tell us most clearly what the Nice Model is, and that it involves Uranus and Neptune moving through the outer solar system at one point in time. This most closely matches Answer Choice A which says that the Nice model is based on the idea that “Uranus and Neptune were not always locked into their current orbital paths.” Answer Choice B is incorrect because the Passage doesn’t tell us that Ceres traveled farther than any other object in the solar system and suggests that Ceres and other objects in the asteroid belt migrated about the same distance. Answer Choice C is incorrect because, while this might be true of some objects in the inner solar system, the Passage doesn’t mention this. Answer Choice D is incorrect because the Passage goes on to tell us that, according to the Nice model, Ceres would have been formed in the outer solar system and then dragged inward.

How to solve this? The Question asks, “According to the passage, the Nice model is based on the idea that [...]?”

25.R.9

Answer Choice D is the correct answer because the word “simply” in Line 34 has the same meaning as “just” or “only,” which most closely matches “merely.” Answer Choice A is incorrect because the Passage isn’t emphasizing the entirety of Ceres being the largest of these immigrants. Answer Choice B is incorrect because the Lines aren’t emphasizing the “sincerity” of this viewpoint, which wouldn’t make much sense. Answer Choice C is incorrect because the words aren’t emphasizing the obviousness or “plainness” of the claim.

● Ceres would simply be the largest of these immigrants. ● Ceres would merely be the largest of these immigrants.

How to solve this? The Question asks, “As used in line 34, ‘simply’ most nearly means [...]?”

Page 12: SAT Full Reading Explanations (QAS 9-19) · SAT Full Reading Explanations (QAS 9-19) SAT #9 Reading 1 1.R.9 Answer Choice C is the correct answer because Lines 19-22 say, “Moths

26.R.9 & 27.R.9

Answer Choices C and D are the correct answers because Lines 49-53 say, “Bottke and Levison’s computer simulations show that the observed number of objects is about right if they are immigrants, though they have assumed many of the objects broke up after transport.” These Lines tell us most clearly that the two scientists base their conclusions on the assumption that a lot of the objects that made it to the asteroid belt broke up after migrating. If this assumption weren’t true, it would weaken their argument. This matches Answer Choice C from Question 26 which says that the scientists’ conclusion would be weakened by a study that “demonstrated that very few objects broke up after migrating to the asteroid belt.” For Question 27, Answer Choice A is incorrect because these Lines only explain what these two scientists studied and don’t offer any suggestion of what would weaken their study. Answer Choice B is incorrect because these Lines also only explain what the scientists focused on in their study, without saying any assumption they based their findings on or any of their reasoning, making it hard to find something that would weaken their study. Answer Choice C is incorrect because these Lines also mostly give facts about what the scientists focused on and because it doesn’t match any of the answer choices for Question 26. For Question 26, Answer Choice A is incorrect because Bottke and Levison’s conclusion depends on the fact that the Sun burned away the outermost layer of some objects, so this would support their conclusions. Answer Choice B is incorrect because Bottke and Levison’s conclusions don’t depend on the stability of objects in the inner solar system. Answer Choice D is incorrect because their conclusions doesn’t depend on immigrants from the outer solar system surviving in the asteroid belt, and actually suggest the opposite, that they “broke up” after transport.

How to solve this? Question 26 asks, “Based on the passage, Bottke and Levison’s conclusions would be most weakened by a study that [...]?” To solve this [...].

28.R.9

Answer Choice A is the correct answer because the word “raw” in Line 60 is used to indicate the materials that were already in the asteroid belt, or “original” there. Answer Choice B is incorrect because the Lines aren’t emphasizing that the materials are young, which wouldn’t make much sense. Answer Choice C is incorrect because the Lines aren’t emphasizing that the materials are “exposed” or laid bare. Answer Choice D is incorrect because the Lines aren’t emphasizing that the materials are “inexperienced,” which wouldn’t make sense when applied to materials.

● Its ice-to-rock ratio matches the expected composition of the raw materials that would have been available at its current position early on, he says.

● Its ice-to-rock ratio matches the expected composition of the original materials that would have been available at its current position early on, he says.

Page 13: SAT Full Reading Explanations (QAS 9-19) · SAT Full Reading Explanations (QAS 9-19) SAT #9 Reading 1 1.R.9 Answer Choice C is the correct answer because Lines 19-22 say, “Moths

29.R.9

Answer Choice C is the correct answer because Lines 64-66 say, “New measurements of Ceres’s composition by NASA’s Dawn mission, for which McCord is a team member, could help pin down its birthplace.” These Lines tell us clearly that a new mission regarding Ceres could help scientists finally understand its origin. This most clearly matches Answer Choice C, which says that the last sentence of the passage serves to “allude to a potential resolution to a puzzle.” Answer Choice A is incorrect because these Lines don’t mention any other claim or weakness in it. Answer Choice B is incorrect because these Lines don’t talk about a crucial decision, but a new study. Answer Choice D is incorrect because these Lines aren’t talking about reconciling the two opposing positions in the Passage, but about determining which is correct, and only suggest the future outcomes of research without making any specific claims.

How to solve this? The Question asks, “The last sentence of the passage serves mainly to [...]?” To solve this [...].

30.R.9

Answer Choice C is the correct answer because the last column in the table measures density, and Saturn has a density of 0.7, the lowest of all the other objects.

31.R.9

Answer Choice B is the correct answer because the first column shows the average distance for each of these objects from the Sun, with Juno showing a distance of 2.7 and Ceres a distance of 2.8. This most closely matches Answer Choice B which says that “Juno’s average distance from the Sun is less than that of Ceres.” Answer Choice A is incorrect because, while the table does show Earth having a greater overall density than Ceres, it doesn’t show anything about the change in density. Answer Choice C is incorrect because the only other object in the asteroid belt depicted in the table, Juno, has a smaller mass than Ceres does. Answer Choice D is incorrect because Pluto, another dwarf planet, has a radius that is twice as large as Ceres’s.

How to solve this? The Question asks, “Which statements is supported by data represented in the table?” To solve this [...].

Page 14: SAT Full Reading Explanations (QAS 9-19) · SAT Full Reading Explanations (QAS 9-19) SAT #9 Reading 1 1.R.9 Answer Choice C is the correct answer because Lines 19-22 say, “Moths

32.R.9

Answer Choice A is the correct answer because Lines 13-17 say, “McKinnon points out that Ceres has a low density, which suggests it is 25 to 30 percent water ice. That’s a high proportion for an asteroid, but closely matches Pluto and other icy objects native to the outer solar system, [...].” The fact that Pluto and Ceres have relatively similar densities would support his argument. This most closely matches Answer Choice A, which says that he would find it useful that, “The average density of Pluto is similar to that of Ceres.” Answer Choice B is incorrect because nowhere does the Passage suggest that McKinnon’s view depends on the relative distance of Neptune and Ceres. Answer Choice C is incorrect because the table shows the mass of Ceres as being much larger than Juno, and because, while McKinnon does suggest that Ceres is the largest of all the objects in the asteroid belt, he doesn’t make a point about it having the largest mass. Answer Choice D is incorrect because nowhere in the Passage does McKinnon make his argument depend on the density of Ceres.

How to solve this? The Question asks, “Which data presented in the table would McKinnon find most useful to his argument?” To solve this [...].

Reading 4

Passage Outline

Passage 1

● If a mistake is made in this new government, the US may be lost forever. (Paragraph 1) ● The members of the convention believed they had to make a unified government, but its

a danger. (Paragraph 2) ● Why did they justify themselves by saying, “We the people?” It’s the states, not people,

that are the agents of the country. (Paragraph 3) ● They didn’t have the power to make this decision. The Convention should have just fixed

the old system, not make a new one. (Paragraph 4)

Passage 2

● Patrick Henry objects to the phrase, “We, the people.” What’s wrong with it? The Union should be one of the people and not the states. (Paragraphs 1 & 2)

● The power of the Convention is doubted. But the Convention had the power to fix every problem, and didn’t have to follow the old system. The choice is between the old Confederation and new Constitution, and the Constitution is better. It’s not true that the Confederation brought us through the war, but the spirit of America. (Paragraph 3)

Page 15: SAT Full Reading Explanations (QAS 9-19) · SAT Full Reading Explanations (QAS 9-19) SAT #9 Reading 1 1.R.9 Answer Choice C is the correct answer because Lines 19-22 say, “Moths

33.R.9

Answer Choice A is the correct answer because the subtitle of the Passage tells us that Henry delivers his speech in Virginia, and because Lines 30-33 say, “Disorders have arisen in other parts of America; but here, sir, no dangers, no insurrection or tumult have happened; every thing has been calm and tranquil.” Together, these Lines tell us that Henry believes, in contrast to other parts of the country, Virginia is a place of peace and safety. This most closely matches Answer Choice A which says that Henry believes that in Virginia “there is no evidence of civil unrest.” Answer Choice B is incorrect because the Passage doesn’t say [...].

How to solve this? The Question asks, “In Passage 1, Henry states that Virginia differs from other areas of the country in that [...]?” To solve this [...].

34.R.9

Answer Choice B is the correct answer because Lines 33-35 say, “But, notwithstanding this, we are wandering on the great ocean of human affairs. I see no landmark to guide us.” In context, these Lines communicate a sense of the unknown or a general “uncertainty,” matching Answer Choice B. Answer Choice A is incorrect because [...].

How to solve this? The Question asks, “The figurative language in Lines 33-35 (“But, notwithstanding … guide us”) serves mainly to suggest [...]?”

35.R.9

Answer Choice B is the correct answer because, in Lines 52-57, Pendleton responds to the criticism of the phrase “We, the people,” by asking, “Who but the people have a right to form government?” and saying, “The expression is a common one, and a favorite one with me.” This tells us that Pendleton views the phrase favorably and as appropriate, matching Answer Choice B which says that he views the phrase as “appropriate for the proposed type of government.” Answer Choice A is incorrect because [...].

36.R.9

Answer Choice D is the correct answer because the word “issue” in Line 79 is used to talk about the result or “outcome” of the war. Answer Choice A is incorrect because the word doesn’t mean reluctance or “misgiving,” emphasizing a sense of regret. Answer Choice B is incorrect because the word isn’t used to mean a publication. Answer Choice C is incorrect because the word “issue” is used to talk about a finished result, not an ongoing “proceeding.”

● The latter is no government at all. It has been said that it has carried us, through a dangerous war, to a happy issue.

● The latter is no government at all. It has been said that it has carried us, through a dangerous war, to a happy outcome.

How to solve this? The Question asks, “As used in Line 79, “issue” most nearly means [...]? To solve this […].

Page 16: SAT Full Reading Explanations (QAS 9-19) · SAT Full Reading Explanations (QAS 9-19) SAT #9 Reading 1 1.R.9 Answer Choice C is the correct answer because Lines 19-22 say, “Moths

37.R.9 & 38.R.9

Answer Choices D and C are the correct answers because Lines 79-83 say, “Not that Confederation, but common danger, and the spirit of America, were bonds of our union: union and unanimity, and not that insignificant paper, carried us through that dangerous war.” These Lines most clearly tell us about Pendleton’s view of the Articles of Confederation, and tell us that it wasn’t the Confederation that brought us through the war, but the people and spirit of America. This most closely matches Answer Choice D which says that his view of the Articles is that, “they had little to do with America’s having prevailed in its most recent conflict.” For Question 38, Answer Choice A is incorrect because these Lines mostly talk about Henry’s perspective, without telling us clearly what Pendleton thinks of the Confederation. Answer Choice B is incorrect because these Lines primarily concern Pendleton’s thoughts on the phrase “We the people,” and not his opinion of the Confederation. Answer Choice D is incorrect because these Lines less directly talk about the Confederation, and moreso concern Pendelton’s view of the importance of the spirit of America and how it allowed us to win the war. For Question 37, Answer Choice A is incorrect because none of the Lines tell us that Pendleton viewed the Articles as a source of great concern for Americans. Answer Choice B is incorrect because none of the Texts talk about how the articles gave too much power to elected representatives. Answer Choice C is incorrect because Pendleton argues that it wasn’t the Articles of Confederation that ultimately united the country, but rather the overall Spirit of America.

How to solve this? Question 37 asks, “Based on Passage 2, which statement best reflects Pendelton’s view of the Articles of Confederation?” To solve this [...].

39.R.9

Answer Choice D is the correct answer because Henry, in Passage 1, takes a critical stance towards the Convention, arguing that the members should only have fixed the old system, and not entirely replaced it, and shouldn’t have justified themselves by referencing, “We the people.” Meanwhile, Pendelton, in Passage 2, argues in favor of the Convention and defends the members’ justifications and decisions. This most clearly matches Answer Choice D, which says that “Pendleton disagreed with most of the points made by Henry.” Answer Choice A is incorrect because only Henry disagreed with the conclusions of the federal Convention, while Pendleton supported them. Answer Choice B is incorrect because the two men expressed different views of the New Constitution, with Pendleton generally supporting it and Henry generally criticizing it. Answer Choice C is incorrect because nowhere in the Passage does Pendleton admit to not being able to answer Henry’s questions, and in fact addresses them all.

How to solve this? The Question asks, “Which statement best describes the relationship between the views expressed in the two passages?” To solve this [...].

Page 17: SAT Full Reading Explanations (QAS 9-19) · SAT Full Reading Explanations (QAS 9-19) SAT #9 Reading 1 1.R.9 Answer Choice C is the correct answer because Lines 19-22 say, “Moths

40.R.9

Answer Choice A is the correct answer because Lines 39-41 say, “The federal Convention ought to have amended the old system; for this purpose they were solely delegated [...],” while Lines 79-83 say, “Not that Confederation, but common danger, and the spirit of America, were bonds of our union: union and unanimity, and not that insignificant paper, carried us through that dangerous war.” These Lines tell us that Henry viewed the Confederation as needing to be changed, and Pendleton viewed the Confederation as insignificant, especially in regards to the war. This most closely matches Answer Choice A, which says of the Confederation that, “Henry felt it was flawed but correctable, while Pendleton felt that it had served no useful purpose.” Answer Choice B is incorrect because, while Henry does take a positive stance towards the Confederation, he doesn’t make the strong claim that it’s perfect, and Pendleton doesn’t specifically call it authoritarian. Answer Choice C is incorrect because the Passage tells us that Henry believed the Confederation needed to be changed and nowhere does Pendleton make the case that it was only a transitional system. Answer Choice D is incorrect because Pendleton doesn’t make the strong claim that the Confederation harmed our nation’s future.

How to solve this? The Question asks, “Which statement best expresses Henry’s and Pendleton’s respective views of the Confederation?”

41.R.9 & 42.R.9

Answer Choices B and C are the correct answers because Pendleton, in Lines 65-68 say, “But the power of the Convention is doubted. What is the power? To propose, not to determine. This power of proposing was very broad; it extended to remove all defects in government [...].” These Lines tell us that Pendleton claims that the members of the Convention had full power to create a new proposal for government, a point that Henry disagrees with in Lines 39-42, which say, “The federal Convention ought to have amended the old system; for this purpose they were solely delegated; the object of their mission extended to no other consideration.” These Lines tell us that Henry believed that the Convention only had the power to make changes to the Confederation and not entirely replace them, and match Answer Choice B which says that the members “should not have proposed an entirely new form of government.” For Question 42, Answer Choice A is incorrect because these Lines only tell us that Henry believes that the members of the Convention found it necessary to form a consolidated government, but don’t directly address the extent of the power of the Convention. Answer Choice B is incorrect because these Lines only emphasize Henry’s belief that the new plan is a consolidated government and is dangerous, but don’t directly address the power of the Convention. Answer Choice D is incorrect because these Lines only make a rhetorical apology for the questions that Henry is asking, without addressing the power of the Convention. For Question 41, Answer Choice A is incorrect because neither the Texts nor the Passage tell us that Henry thinks the Convention didn’t sufficiently address the defects of the Confederation, and, in fact, he thinks they overstepped their boundaries. Answer Choice C is incorrect because its unlikely that Henry would respond by saying the Convention was only trying to follow the wishes of the American people, since he thinks that authority should lie instead with individual states, and because this doesn’t match any of the Texts. Answer Choice D is incorrect because neither the Texts nor the Passage tell us that Henry thinks that the Convention didn’t understand the

Page 18: SAT Full Reading Explanations (QAS 9-19) · SAT Full Reading Explanations (QAS 9-19) SAT #9 Reading 1 1.R.9 Answer Choice C is the correct answer because Lines 19-22 say, “Moths

importance of not taking action, and because, if anything, Henry thinks they took too much action.

How to solve this? Question 41 asks, “Henry would most likely have responded to Pendleton’s claim about the members of the Convention by asserting that they [...].” To solve this [...].

Reading 5

Passage Outline

A Tangled Tale of Plant Evolution

● Lignin is used in plants to seal in water and hold them up. Recently, it’s been found in some alga. This suggests that lignin evolved a long time ago. (Paragraphs 1-3)

● Martone, the scientist who discovered this, wasn’t trying to do so. He was looking at how a certain species of alga survived its harsh environment. (Paragraphs 4 & 5)

● When they examined the seaweed, they found structures that suggested lignin. Experts in lignin confirmed this. (Paragraphs 6-8)

● There’s less lignin in seaweed than land plants, but more in parts that are stressed. Martone believes that lignin developed a long time ago, in the ancestor of both red and green algae. Another scientist thinks the evidence for lignin is strong, but that lignin developed separately in red and green algae. (Paragraphs 9-11)

43.R.9

Answer Choice A is the correct answer because Paragraphs 4-8 primarily describe how scientists Martone and Estevez discovered the presence of lignin in a red alga, and Lines 13-16 say, “The finding suggests that a biological building block fundamental to the success of land plants has roots that stretch back far deeper — and maybe wider — through evolutionary time than was known.” Together, these tell us that the Passage primarily concerns the discovery of lignin and some possible consequences of that discovery. This most closely matches Answer Choice A, which says that the Passage is primarily concerned with “narrating how a finding was arrived at and indicating possible implications.” Answer Choice B is incorrect because the Passage doesn’t look at different scientific specialists and compare them, like, for instance, noting the difference between a [...] and a biomechanist. Answer Choice C is incorrect because the phenomenon of the presence of lignin isn’t characterized as troubling or “vexing,” and the Passage keeps things open-ended without “endorsing a single explanation.” Answer Choice D is incorrect because, while the Passage does describe some properties of an organism, it doesn’t try to show how they can be exploited.

How to solve this? The Question asks, “The passage is primarily concerned with [...]?” To solve this [...].

Page 19: SAT Full Reading Explanations (QAS 9-19) · SAT Full Reading Explanations (QAS 9-19) SAT #9 Reading 1 1.R.9 Answer Choice C is the correct answer because Lines 19-22 say, “Moths

44.R.9

Answer Choice B is the correct answer because Lines 9-12 say, “Now a University of British Columbia botanist and some highly specialized chemists have strong evidence for lignin in a red alga called Calliarthron cheilosporioides.” These Lines tell us that the conclusion about the presence of lignin in this alga is based on strong evidence, matching Answer Choice B, which says that it’s “founded on empirical evidence and thus persuasive.” Answer Choice A is incorrect because the Passage presents the discovery of lignin as novel one, and not something thought about before by the scientific community. Answer Choice C is incorrect because, while the Passage does say the discovery suggests that previous knowledge about the evolution of algae might be changed, it doesn’t make the strong claim that the discovery is “certain to disprove” earlier theories. Answer Choice D is incorrect because the presence of lignin isn’t based on speculation or “conjectural reports,” but instead on strong empirical evidence.

How to solve this? The Question asks, “As presented in the passage, the conclusion that lignin is present in C. cheilosporioides can best be described as [...]?”

45.R.9 & 46.R.9

Answer Choices D and C are the correct answers because Lines 23-26 say, “Martone didn’t set out to locate lignin in algae. The biomechanist simply wanted to better understand the toughness of C. cheilosporioides, which dwells in the harsh habitat of intertidal zones along rocky shores.” These Lines tell us most clearly that Martone originally wasn’t interested in finding lignin, but in learning more about the toughness of the alga. This most closely matches Answer Choice D, which says that his research interest was “initially secondary, because it was undertaken to support particular questions about C. cheilosporioides.” For Question 46, Answer Choice A is incorrect because it only explains the adaptive advantage of lignin without telling us anything about Martone or his research interest in lignin. Answer Choice B is incorrect because these Lines only explain the possible implications of the discovery of lignin in the alga, without telling us anything about Martone and his research. Answer Choice D is incorrect because these Lines tell us an aspect of lignin that Martone doesn’t yet understand, but doesn’t tell us about his research interest. For Question 45, Answer Choice A is incorrect because none of the Texts suggest that Martone’s research has changed the way lignin is understood, and the Passage only makes the case that his research may change our view of the evolutionary history of lignin. Answer Choice B is incorrect because neither the Passage nor the Texts suggest that Martone based his conclusions on an atypical sample. Answer Choice C is incorrect because neither the Passage nor the Texts make the case that Martone was cautious, and checked his findings multiple times.

How to solve this? Question 45 asks, “It can reasonably be inferred from the passage that Martone’s research interest in lignin should be considered [...]?” To solve this [...].

Page 20: SAT Full Reading Explanations (QAS 9-19) · SAT Full Reading Explanations (QAS 9-19) SAT #9 Reading 1 1.R.9 Answer Choice C is the correct answer because Lines 19-22 say, “Moths

47.R.9

Answer Choice D is the correct answer because Lines 30-32 say, “The calcified, or rigid-bodied seaweed has multiple noncalcified joints that make it flexible yet strong enough to handle that setting.” These Lines tell us most clearly about the structure of C. cheilosporioides, and that it has a hard, rigid body, yet soft joints that allow it to bend and be flexible. This most closely matches Answer Choice D, which says that the structure of the alga has components that “operate together to allow for suppleness.” Answer Choice A is incorrect because the Passage doesn’t indicate that the seaweed can change its density. Answer Choice B is incorrect because, although the Passage does present the structure of the alga as crucial to how it withstands its harsh environment, it doesn’t specifically make the case that these structural components protect its cell interior. Answer Choice C is incorrect because the Passage doesn’t say that the seaweed has components that can regenerate as it matures.

How to solve this? The Question asks, “The passage indicates that the structure of C. cheilosporioides consists of components that [...]?” To solve this [...].

48.R.9

Answer Choice B is the correct answer because the word “handle” in Line 32 is used to describe how the seaweed it able to live, or “survive in” in its harsh environment. Answer Choice A is incorrect because it doesn’t make sense to say that the seaweed “trained for” its environment, like an athlete. Answer Choice C is incorrect because the Lines are emphasizing the seaweed’s mere ability to live in that environment, and not its ability to “engage” or interact with it. Answer Choice D is incorrect because the Lines aren’t describing the seaweed as acting on its environment.

● The calcified, or rigid-bodied seaweed has multiple noncalcified joints that make it flexible yet strong enough to handle that setting.

● The calcified, or rigid-bodied seaweed has multiple noncalcified joints that make it flexible yet strong enough to survive in that setting.

How to solve this? Question 48 asks, “As used in line 32, “handle” most nearly means [...]?”

Page 21: SAT Full Reading Explanations (QAS 9-19) · SAT Full Reading Explanations (QAS 9-19) SAT #9 Reading 1 1.R.9 Answer Choice C is the correct answer because Lines 19-22 say, “Moths

49.R.9 & 50.R.9

Answer Choices A and D are the correct answers because Lines 42-45 say, “John Ralph and colleagues at the University of Wisconsin-Madison’s Great Lakes Bioenergy Research Center detected lignin in C. cheilosporioides.” The sentence after elaborates that these scientists “found the same telltale components [...] used to describe lignins in terrestrial plants.” Because John Ralph is introduced as an expert in lignin (Lines 37-38), these Lines tell us that experts were useful overall to Martone and Estevez because they found parts in the alga that were similar to land plants containing lignin. This most closely Answer Choice B, which says that scientists were useful to Martone and Estevez because “C. cheilosporioides was found to contain features typically found in plants known to contain lignin.” For Question 50, Answer Choice A is incorrect because these Lines only tell us about Martone’s original research interest and not why experts in lignin were useful to him. Answer Choice B is incorrect because these Lines only explain the structure of the alga, and don’t suggest why experts in lignin were useful. Answer Choice C is incorrect because these Lines describe Estevez and Martone themselves, and not other experts who were useful to them. For Question 49, Answer Choice A is incorrect because neither the Passage nor the Texts describe the toughness of the alga making it difficult to examine. Answer Choice C is incorrect because neither the Texts nor the Passage talk about the seaweed containing molecules of a strong chemical compound. Answer Choice D is incorrect because doesn’t match our Text for 50, and because the Passage doesn’t directly describe an unexpected measurement.

How to solve this? Question 49 asks, “Scientists who specialized in lignin were useful to Martone and Estevez because [...]?” To solve this [...].

51.R.9

Answer Choice A is the correct answer because the word “strong” in Line 76 is used to describe Martone’s evidence for lignin, indicating that its persuasive or “compelling.” Answer Choice B is incorrect because the sentence isn’t highlighting the uniqueness or distinctiveness of Martone’s finding. Answer Choice C is incorrect because its too strong to describe his evidence as impervious. Answer Choice D is incorrect because it doesn’t make sense to describe his evidence as “vigorous.”

● Karl J. Niklas, a Cornell University botanist, considers Martone’s evidence for lignin in C. cheilosporum exceptionally strong.

● Karl J. Niklas, a Cornell University botanist, considers Martone’s evidence for lignin in C. cheilosporum exceptionally compelling.

How to solve this? The Question asks, “As used in line 76, ‘strong’ most nearly means [...]?”

Page 22: SAT Full Reading Explanations (QAS 9-19) · SAT Full Reading Explanations (QAS 9-19) SAT #9 Reading 1 1.R.9 Answer Choice C is the correct answer because Lines 19-22 say, “Moths

52.R.9

Answer Choice C is the correct answer because Lines 76-78 say that Niklas believes “that red and green algae evolved from separate endosymbiotic events.” This stands in contrast to Martone’s view that “both red and green algae descend from one endosymbiotic event, when a eukaryote cell engulfed a photosynthesizing cyanobacterium and gained the ability to make its own food.” This shows us that the two scientists differ in their view of the evolutionary history of red and green algae because Martone thinks they evolved from a single endosymbiotic event, while Niklas believes they evolved from separate ones. This most closely matches Answer Choice C, which says that Niklas counters Martone by suggesting that the “evolutionary lines leading to the two kinds of algae might have diverged before they acquired the ability to photosynthesize.” Answer Choice A is incorrect because, while Martone may believe this view, Niklas believes that the separate progenitors of either algae contained genes conducive to these pathways. Answer Choice B is incorrect because, while this statement is true, Niklas doesn’t make any point about it and Martone himself admits that he doesn’t have evidence for this. Answer Choice D is incorrect because both Passages suggest that similar molecular pathways for the production of lignin exist between C. cheilosporioides and its ancestral species.

How to solve this? The Question asks, “Niklas counters Martone’s proposal about the evolutionary history of red and green algae by suggesting that the [...]?” To solve this [...].

Page 23: SAT Full Reading Explanations (QAS 9-19) · SAT Full Reading Explanations (QAS 9-19) SAT #9 Reading 1 1.R.9 Answer Choice C is the correct answer because Lines 19-22 say, “Moths

SAT #10

1.R.10 (medium)

Answer Choice D is correct because the Passage is best described as an “accidental meeting that reveals the narrator’s relationship with a character.” Answer Choice A is incorrect, because we have no evidence that the narrator was forced to “reevaluate his perspective on his childhood.” Answer Choice B is incorrect because no connection is established in the Passage between the past experiences and futures of the narrator and the Swede. Answer Choice C is incorrect because there’s no reflection on people who “give up on their childhood dreams.”

How to solve this? The Questions asks, “The main purpose of the passage is to [...]?” Because this is a [Big Picture Question], we might be better off solving it after having gone through the other Questions, although we can still solve it upfront. Since this is a literary passage, however, we’re unlikely to find any theses or [high-density information sentences] that give us the main purpose directly. Instead, we should make sure we have a decent grasp of the events in the passage, and then go directly to the answer choices and try to disprove three answer choices, while selecting one other.

Answer Choice A says that the purpose of the passage is to, “show how an event forced the narrator to reevaluate his perspective on his childhood.’ We should focus on the latter part of this sentence and see if the Passage tells us whether or not the narrator had to reevaluate his perspective. Going back through, we should conclude that he doesn’t and eliminate this option. Answer Choice B says that the passage analyzes “how past experiences shaped the narrator’s and another character’s future.” It’s clear that the Passage does concern past experiences, but it’s not clear that the connection is made between these experiences and the narrator’s and character’s future. If we go back through the Passage, we should find that this claim isn’t supported and eliminate this option. Answer Choice C says that the passage reflects “upon the changes that people go through as they give up on their childhood dreams.” We should conclude that this answer choice is also incorrect, because there’s no discussion of or reflection on individuals giving up on their childhood dreams. Answer Choice D says that the main purpose of the passage is to “describe an accidental meeting that reveals the narrator’s relationship with a character.” This answer choice should strike us as pretty general, since it only says that the passage describes a meeting and reveals a relationship, leaving very little to go wrong with it. Because it contains no errors and matches our text, we should Answer Choice D as our correct answer.

Page 24: SAT Full Reading Explanations (QAS 9-19) · SAT Full Reading Explanations (QAS 9-19) SAT #9 Reading 1 1.R.9 Answer Choice C is the correct answer because Lines 19-22 say, “Moths

2.R.10 (medium)

Answer Choice B is the correct answer because in Lines 57-62 we’re told that the narrator felt “almost as wonderfully singled out as I had the one time before, at the age of ten, when the Swede had got so personal as to recognize me [...]” and in Lines 65-66 a woman tells the narrator, “You should have seen your face [...]. I saw just what you looked like as a boy.” These Texts support Answer Choice B which says that a main theme of the passage is that “encountering a memorable person from the past can make an adult feel like a child again.” Answer Choice A is incorrect because the the two friends who meet in the Passage don’t find that everything has changed. Rather, in the description of the Swede, it’s suggested that things haven’t changed much at all. Answer Choice C is incorrect because we have no evidence that the narrator and the Swede played sports together, and that that’s the reason why they’re still connected even to this day. Answer Choice D is incorrect because although the two characters remember the past fondly, there’s no evidence to suggest that they imagine it better than it actually was.

How to solve this? The Question asks, “A main theme of the passage is that [...]?” To solve this, we should take a more general approach and begin working through the answer choices directly, looking for possible errors in any of them to help narrow down on our correct answer choice. Going through our options, we should note that [...].

3.R.10 (medium)

Answer Choice B is the correct answer because the word “exquisitely” in Line 11 can be best replaced by the word “perfectly,” which emphasizes that it fit him perfectly as opposed to imperfectly. Answer Choice A is incorrect because it doesn’t make sense to describe the suit as fitting the Swede skillfully, because this makes the suit an agent. Also, the word isn’t trying to emphasize the skillful fit of the suit as opposed to the unskillful fit. Answer Choice C is incorrect because “primly” means “done in an excessively proper manner,” which doesn’t match the way the Swede is described. Answer Choice D is incorrect because the Passage isn’t trying to emphasize the formal fit as opposed to the informal fit.

How to solve this? [...]

Page 25: SAT Full Reading Explanations (QAS 9-19) · SAT Full Reading Explanations (QAS 9-19) SAT #9 Reading 1 1.R.9 Answer Choice C is the correct answer because Lines 19-22 say, “Moths

4.R.10 (easy)

Answer Choice A is the correct answer because Lines 18-21 describe Swede’s son Chris as a “kid under a Mets cap pounding away a first baseman's mitt that dangled, as had the Swede’s, from his left hand,” and best support the conclusion that Chris reminds the narrator of the Swede. Answer Choices B, C and D are incorrect because these Lines give us no indication that the boy reminds the narrator of the Swede but insteaded describe the Swede as another baseball player, describe a feature of that baseball player and show a small exchange between the narrator and the boy, respectively.

How to solve this? The Question asks “Which choice best supports the conclusion that Chris, the Swede’s son, reminds the narrator of the Swedge?” To solve this, we should go through each of the answer choices, one by one, until we find the option that best describes the narrator finding that Chris reminds him of the Swede.

Answer Choice A says, “A skinny, fair-haired boy of about seven or eight was walking alongside the Swede, a kid under a Mets cap pounding away at a first basemen’s mitt that dangled, as had the Swede’s, from his left hand.” The phrase “as had the Swede’s” should stand out to us as good evidence that the narrator is reminded of the Swede by Chris, so we should keep this option. Answer Choice B says, “Do you know that?’ I said to his son. ‘Your dad was our Hernandez.’’ This choice does show an interaction between Chris and the narrator but concerns the Swede directly, and not how Chris reminds the narrator of him, so we should eliminate this option. Answer Choice C says, “‘Hernandez is left-handed,’ he replied,” which also doesn’t give us evidence for the narrator being reminded of the Swede. Answer Choice D says, “‘Well, that’s the only difference,’ I said to the literal literalist, and put out my hand again to his father.” This answer choice also doesn’t describe Chris as reminding the narrator of the Swede, so we should eliminate this option as well. That leaves us with Answer Choice A as our correct answer.

5.R.10 & 6.R.10 (medium)

Answer Choices B and B are the correct answers because Lines 34-38 describes the Swede acknowledging the narrator by saying to his son, “‘You know who this is?’ the Swede said the boy. ‘The guy who wrote those books. Nathan Zuckerman.’” This shows a meaningful irony (something that happens contrary to expectations) because, while the narrator had admired the Swede as young boy, now we see the Swede showing admiration for Zuckerman. Answer Choice B best captures this by saying that the Swede, “appreciated the narrator’s accomplishments as an adult.” Answer Choice A is incorrect because neither our Passage nor our Text describe the Swede training his son to follow in the narrator’s footstep. Answer Choice C is incorrect because neither the Passage nor our Text describe the Swede’s failure to achieve his earlier promise. Answer Choice D is incorrect because while it recognizes the irony that the Swede now shows appreciation for the narrator in their adult lives, it goes too far by describing this appreciation as envy.

How to solve this? Question 5 asks, “A meaningful irony in the passage is that, while the narrator had admired the young Swede’s accomplishments, the Swede [...]?” Because this is a Paired Question we should first to the answer choices for Question 6 and look for an end to the Question which would complete the irony. Because an irony is something that violates our

Page 26: SAT Full Reading Explanations (QAS 9-19) · SAT Full Reading Explanations (QAS 9-19) SAT #9 Reading 1 1.R.9 Answer Choice C is the correct answer because Lines 19-22 say, “Moths

reasonable expectations, we should expect that an irony related to the narrator admiring the Swede in their youth would involve the Swede now admiring the narrator in their adulthood, and we should look for an option like this.

For Question 6, Answer Choice A describes the narrator seeing the Swede and his son from afar and approaching them. This answer choice doesn’t provide us with any possible irony, so we should eliminate this option. Answer Choice B describes the Swede recognizing and acknowledging the narrator, finally saying to his son, “‘You know who this is?’ the Swede said the boy. ‘The guy who wrote those books. Nathan Zuckerman.’” This shows a degree of admiration that the Swede now has for the narrator, so we should keep this option. Answer Choice C shows a simple farewell that the Swede gives to the narrator, but doesn’t show any meaningful irony, so we should eliminate this option. Answer Choice D shows another character laughing and stating that “the greatest athlete in the history of Weequahic High called you ‘Skip.’” This answer choice might seem plausible, because it completes the irony by showing how the Swede used an affectionate and personal name with the narrator, showing a reversal in admiration or respect. We can keep this option, although we should note that the Swede calling the narrator ‘Skip’ shows a smaller level of appreciation than recognizing him as the author of certain books. Moreover, as we’ll see later, this comment is made in a tongue-in-cheek fashion, rendering it less useful to us here.

For Question 5, Answer Choice A says that the Swede “had trained his son to follow in [the narrator’s] footsteps.” This would show a meaningful irony, but we should note that the phrase “follow in his footsteps” is a relatively “loud” answer choice, and carries a large burden of proof which the Text just doesn’t provide, so we can eliminate this option. Answer Choice B says that the Swede now appreciates “the narrator’s accomplishments as an adult.” This is fairly neutral answer choice, and matches our two possible Texts, so we can keep this option. Answer Choice C says that the Swede “had failed to achieve his promise as the years went by.” This wouldn’t perfectly complete the irony set up in the question, since the irony involves the narrator appreciating the Swede early on and the Swede appreciating the narrator later in life, not that the Swede failed in later life. More importantly, neither our Texts nor our Passage supports this claim, so we can eliminate this option. Answer Choice D says that the Swede “envied the achievements of his more scholarly classmates.” This answer choice might seem to vaguely match our two texts and complete the irony, but it has a few things wrong with it. First, it has too much [pejorative flair], making it unlikely to be our answer choice unless it has very strong evidence to support it. Second, it uses the plural classmates, whereas the only possible envy he might feel that we would know about would have to address Zuckerman directly. And third, there’s just no evidence in our Passage or our Text that the Swedge feels envy, only a kind of appreciation or admiration. This leaves us with Answer Choice B for Question 5.

Now we have to decide between Answer Choice B and D for Question 6. Earlier we mentioned a few problems for Question 6, that the Swede calling the narrator ‘Skip’ shows a smaller level of appreciation than recognizing him as the author of certain books. Now that we chose Answer Choice B for Question 5, we should also note that only the Text for Answer Choice B shows the Swede appreciating the narrator for his accomplishments, thus we should choose Answer Choice B for Question 5.

Page 27: SAT Full Reading Explanations (QAS 9-19) · SAT Full Reading Explanations (QAS 9-19) SAT #9 Reading 1 1.R.9 Answer Choice C is the correct answer because Lines 19-22 say, “Moths

7.R.10 (easy)

Answer Choice C is the correct answer because in Lines 47-50, the narrator compares the Swede to Hernandez and Chris, the son, responds by saying “Hernandez is left-handed,” thereby “pointing out a problem with the comparison.” Answer Choice A is incorrect because there’s no point at which Chris compares his father to a different player. Answer Choice B is incorrect because Chris doesn’t show any admiration for his father in this instance. Answer Choice D is incorrect because Chris also doesn’t show any gratitude to the narrator. How to solve this? The Question asks, “Chris, the Swede’s son, responds to the narrator’s comparison of the father to another baseball player by [...]?” To solve this, we have to go back to the Passage to find the point at which the narrator compares the Swede. We can find this Text by memory, by searching and by using [the Question Order rule], which tells us the Text should be between about Lines 35 and Lines 55. Going back to the Passage, we should find Lines 44-51, where the narrator says of the Swede that he, “Played first base like Hernandez [...] Do you know that?’ I said to his son. “Your dad was our Hernandez.” The son responds by saying “Hernandez is left-handed,” indicating a difference between the two men for which the narrator calls him a “little literalist.” Thus, we should expect our answer to have something to do with the son dismissing the comparison on the grounds that the two men favor different hands.

Answer Choice A says the son responds by, “comparing his father to a different player,” for which we have no evidence in either the Passage or the Text. Answer Choice B says he responds by, “revealing his admiration for his father,” for which we also don’t have any evidence in either the Passage or the Text. Answer Choice C says he responds by, “pointing out a problem with the comparison,” which does our match our Text, since the son points out that Hernandez is left-handed while the Swede isn’t. Answer Choice D says the son responds by, “showing his gratitude to the narrator,” for which we also don’t have any evidence in either our Passage or our Text, so we should eliminate this option. That leaves Answer Choice C, which we should choose for our correct answer.

8.R.10 (medium)

Answer Choice C is the correct answer because when someone repeats the narrator’s phrase “the greatest athlete in the history of Weequahic High,” we have to recognize its use in a tongue-in-cheek manner. This description of the Swede is used to tease the narrator and his excitement over meeting the Swede, which most closely matches Answer Choice C. Answer Choice A is incorrect because the phrase is used in a joking manner, and not as any kind of admittance about the Swede’s greatness as an athlete. The same can be said about Answer Choice B, which doesn’t make sense given the joking manner in which the comment is made. Answer Choice D is also incorrect because the phrase doesn’t recall any particular incident, and so wouldn’t be useful in helping the narrator remember something.

How to solve this? The Question asks, “When someone repeats the narrator’s phrase “the greatest athlete in the history of Weequahic High” (lines 55-56), the main effect is to [...]?” To solve this Question, we first have to go back to the Lines referenced. The Lines tell us, “He laughed, we parted, and someone was saying to me, ‘Well, well, the greatest athlete in the history of Weequahic High called you ‘Skip.’’’ At this point, to solve the question correctly, we have to recognize the tongue-in-cheek use of the phrase, “greatest athlete in the history of

Page 28: SAT Full Reading Explanations (QAS 9-19) · SAT Full Reading Explanations (QAS 9-19) SAT #9 Reading 1 1.R.9 Answer Choice C is the correct answer because Lines 19-22 say, “Moths

Weequahic High.” It can sometimes be hard to recognize this kind of use, but the Passage gives us three indications its being used in this way: (1) the contrast between the phrase “greatest athlete in the history” and “Weequahic High,” where the first is illustrious and grand while the second is very local, (2) the context of what we already know about Swede’s and the narrator’s relationship, where the narrator admires him so much that another character can say of him, “you might as well have told us he was Zeus. I saw just what you looked like as a boy,” and (3) the use of “well, well” in the introduction of the phrase which gives it a sarcastic tone in the opening. Noticing these three indications, we can predict our answer choice will have something to do with the tongue-in-cheek use of the phrase.

Answer Choice A says the effect of the phrase is to, “admit that the narrator was right about the Swede.” Because of the joking use of the phrase, it doesn’t make sense to call this an admittance about the narrator being correct about the Swede, so we can eliminate this option. Answer Choice B says the effect is to, “show appreciation for the Swede’s accomplishments,” which also doesn’t make sense given the joking manner in which the phrase is used, so we should eliminate this option. Answer Choice C says the effect is to, “tease the narrator for his enthusiasm in meeting the Swede.” This does match our Text, since it shows the character teasing the narrator with the use of the phrase, so we should keep this option. Answer Choice D says the effect is to, “help the narrator remember an incident involving the Swede,” which doesn’t make sense because the phrase doesn’t seem to recall any particular incident and so would be unlikely to help the narrator remember one, so we should eliminate this option. This leaves Answer Choice C, which we should choose as our correct answer.

9.R.10

Answer Choice B is the correct answer because in Lines 57-62 the narrator indicates his amazement at being called “Skip” when he says, “‘I know. I can’t believe it.’ And I did feel almost as wonderfully singled out as I had the one time before, at the age of ten, when the Swede had got so personal as to recognize me by the playground nickname I’d acquired because of two grades I skipped in grade school.” This explanation for his amazement, that he felt “wonderfully singled out,” most clearly matches Answer Choice B, that he “still felt lucky to receive personal attention from the Swede.” Answer Choice A is incorrect, because nowhere in the Passage does the narrator say he thinks that “adults should refer to each other by their formal names,” and because he’s so appreciative at being called “Skip.” Answer Choice C is incorrect because, although it may be true that the narrator hasn’t “been called ‘Skip’ since he was ten,” the Passage gives us no evidence for this. Answer Choice D is incorrect because the narrator is “aware that the Swede had [known] his nickname,” because the Swede used it when the narrator was ten.

How to solve this? The Question asks, “Based on the passage, the reason the narrator was amazed that the Swede had called him ‘Skip’ was most probably that the narrator [...]?” To solve this we should go back to the Passage and look for the lines where Swede calls the narrator “Skip” to see how he responds and why he’s amazed. Based on memory, searching or using the [Question Order rule], we should find our Text around Lines 57-62, where the narrator responds to being called “Skip” by the Swede by saying, “‘I know. I can’t believe it.’ And I did feel almost as wonderfully singled out as I had the one time before, at the age of ten, when the Swede had got so personal as to recognize me by the playground nickname I’d acquired because of two

Page 29: SAT Full Reading Explanations (QAS 9-19) · SAT Full Reading Explanations (QAS 9-19) SAT #9 Reading 1 1.R.9 Answer Choice C is the correct answer because Lines 19-22 say, “Moths

grades I skipped in grade school.” This shows us that he finds himself amazed because, just like at 10, he feels singled out and proud that the Swede recognizes him individually. So, we should expect our answer to have something to do with this.

Answer Choice A says the narrator was amazed because he, “thought adults should refer to each other by their formal names.” Because we don’t have evidence for this either in the Passage or our Text and because the narrator reacts positively to being called “Skip,” we should eliminate this option. Answer Choice B says the narrator, “still felt lucky to receive personal attention from the Swede.” This does match our answer choice, especially the two phrases “wonderfully singled out” and “particular attention,” so we should keep this option. Answer Choice D says the narrator “was not aware that the Swede had ever known his nickname,” which is incorrect because we already know that the Swede used his nickname when the narrator was ten, and because neither the Passage nor our Text give us evidence for this. This leaves Answer Choice B, which we should choose as our answer choice.

10.R.10

Answer Choice A is the correct answer because Lines 64-67 say, “‘You should have seen your face — you might as well have told us he was Zeus. I saw just what you looked like as a boy.’” The use of the name “Zeus” in these lines is used as a reference for a notable, admirable character to which the narrator’s conception of the Swede is compared, explaining his looking “like a boy.” Answer Choice B is incorrect because the context surrounding the use of the word “Zeus” doesn’t indicate that its being used as a reference to an intimidating character, nor is the Swede described as intimidating in the rest of the Passage. Answer Choice C is incorrect because “Zeus” isn’t used in context as a reference to show the narrator’s surprise at being recognized by the Swede. Answer Choice D is incorrect because “Zeus” is also not used in context as a reference for the narrator’s shock.”

How to solve this? The Question asks, “The reference to ‘Zeus’ in line 66 mainly serves to [...]?” To solve this, we should go back to the reference to try and figure out how it’s being used. Lines 64-67 say, “‘You should have seen your face — you might as well have told us he was Zeus. I saw just what you looked like as a boy.’” Here, we see “Zeus” being used as a reference for an entity so amazing and awe-inspiring that it would make the narrator respond similarly to how he responds to the Swede — by acting as a child or a boy. So, we should expect our answer choice to involve this explanation of the use of “Zeus” as referencing an amazing, or awe-inspiring character. Answer Choice A says that the reference to “Zeus” mainly serves to “emphasize that the narrator held the Swede in high regard.” This does match our Text, which shows “Zeus” being used as a reference to someone awe-inspiring, so we should keep this option. Answer Choice B says the reference serves to, “show that the Swede intimidated those around him.” But the context surrounding the use of the word “Zeus” doesn’t indicate that its being used as a reference to an intimidating character, nor is the Swede described as intimidating in the rest of the Passage, so we should eliminate this option. Answer Choice C says that the reference serves to, “suggest that the narrator was surprised that the Swede had recognized him.” Although this may be true, the context of the reference to “Zeus” doesn’t indicate that it’s being used to show surprise at being recognized, so we should eliminate this option. Answer Choice D says the reference is used to “indicate the narrator’s shock at seeing a man from his past,” which also doesn’t make sense because it doesn’t match the context of the

Page 30: SAT Full Reading Explanations (QAS 9-19) · SAT Full Reading Explanations (QAS 9-19) SAT #9 Reading 1 1.R.9 Answer Choice C is the correct answer because Lines 19-22 say, “Moths

reference nor would Zeus serve as a great reference for someone shocking from the narrator’s past, so we should eliminate this option as well. This leaves Answer Choice A, which we should choose as our correct answer.

11.R.10

Answer Choice C is the correct answer because, as the title indicates, the Passage details the “Mental Subordination” of women, and all the ways in which this manifests, which best matches “alert readers to an urgent societal problem.” Answer Choice A is incorrect because, while the Passage does discuss what it sees as a general historical trend, it doesn’t highlight or analyze specific events. Answer Choice B is incorrect because the Passage doesn’t support a particular practice, and especially not an unusual one. Answer Choice D is incorrect because the Passage doesn't mention any political change, so also doesn’t describe the underlying causes of one.

How to solve this? Because this is a [Big Picture Question], we’re best off trying to solve it after answering the other questions related to this Passage. Whenever we’re ready to solve it though, we should try to: (1) get a general sense of the overall meaning of the Passage and (2) identify specific [high density information] texts that will give us the thesis and overall meaning. In general, the Passage: (1) points out that women are criticized no matter how they act, either generously or prudently, (2) asks why women are given books to read and think about without the freedom to act on them or think how they want, (3) that knowledge without freedom only points out women’s plight, (4) that men only care for women insofar as it benefits them and (5) that if women were to act the same way, society would deteriorate. Specifically, we can look at the title of the Passage which is called “A Letter to the Women of England on the Injustice of Mental Subordination.” Taken together, we can say the Passage primarily involves detailing the injustices which women face, especially in the realm of the mind, so we should expect our answer to relate to this.

Answer Choice A says that the main purpose of the passage is to, “analyze a series of historical events.” The Passage does imply the long history of the subordination of women but looks at it generally, so it’s incorrect to say that it analyzes specific historical events, so we should eliminate this option. Answer Choice B says the purpose of the passage is to “persuade readers to support an unusual practice.” Because the passage doesn’t advocate any practice directly, especially not any unusual practice, we should eliminate this option. Answer Choice C says that the purpose of the passage is to “alert readers to an urgent societal problem.” This answer choice seems plausible, since the author clearly presents the subordination of women as an “urgent societal problem,” so we should keep this option. Answer Choice D says the purpose of the passage is to “describe the underlying causes of a political change.” This answer choice might seem plausible at first, because the Passage might seem to imply the need for a political change in the status of women and thus detail the underlying tension that would cause it. But, going through the Passage, there’s no mention of any specific political change, only the outlining of a problem, so we should eliminate this option as well. That leaves us with Answer Choice C, which we should choose as our correct answer.

Page 31: SAT Full Reading Explanations (QAS 9-19) · SAT Full Reading Explanations (QAS 9-19) SAT #9 Reading 1 1.R.9 Answer Choice C is the correct answer because Lines 19-22 say, “Moths

12.R.10 (hard)

Answer Choice B is the correct answer because at multiple points in the Passage the author indicates that women, although they have some educational access, remain unable develop and use their intellectual capabilities completely. In Lines 44-49, she writes, “She, like Tantalus, is placed in a situation where the intellectual blessing she sighs is within her view; but she is not permitted to attain it: she is conscious of possessing equally strong mental powers; but she is obliged to yield, as the weaker creature.” Earlier, she says, “Let man remember that, ‘A little learning is a dangerous thing.’” These quotes most clearly support Answer Choice B, that “women are hindered from fully developing and using their intellectual capabilities.” Answer Choice A is incorrect, because while it’s probably true that the author believes that women “have as much right to rigorous education as men have,” she places less emphasis on the right and more emphasis on the injustice women face in receiving only a partial education with partial freedoms. Answer Choice C is incorrect because, while the author does reference how an incomplete education with incomplete freedoms has taught women “to discriminate just sufficiently to know her own unhappiness,” she doesn’t go so far as to argue that it has “prevented women from realizing their goals.” Answer Choice D is incorrect because, while it’s probably true that the author believes that “methods of education need to be developed that appeal equally to men and to women,” she doesn’t focus especially on the need the development of these methods.

How to solve this? The Question asks, “The author’s central claim in the passage is that [...]?” To solve this, like the question before it, we have to first establish a general picture of the meaning of the Passage (which we can often do better after answering the remaining questions first) as well as locate a few specific Texts that give us the central claim. Following the previous question, we can say that, in general, the Passage: (1) points out that women are criticized no matter how they act, either generously or prudently, (2) asks why women are given books to read and think about without the freedom to act on them or think how they want, (3) that knowledge without freedom only points out women’s plight, (4) that men only care for women insofar as it benefits them and (5) that if women were to act the same way, society would deteriorate. Specifically, we can look at the title of the Passage which is called “A Letter to the Women of England on the Injustice of Mental Subordination.” Taken together, we can say the Passage primarily involves detailing the injustices which women face, especially in the realm of the mind.

This Question proves difficult because all of the answer choices are related to each other and because all of them seem plausible based on parts of the text or the author’s expected view. Once we do a first pass over these answer choices, we should recognize the need to be more careful and to focus particularly on finding errors within three of them. Answer Choice A says, “women have as much right to a rigorous education as men have.” This answer choice should strike us as immediately plausible, given what we know about the author’s point of view. However, because this is a difficult question, we need to approach it more discriminately. Does the author focus on detailing the right that women have? Or, does she assume that women do have this right and instead outline the injustice of it not being served to them? We should eventually arrive at the conclusion that this answer choice is incorrect, because while it seems a plausible position for the author to take, we have insufficient information to say that she focuses on this right. Answer Choice B says that, “women are hindered from fully developing and using

Page 32: SAT Full Reading Explanations (QAS 9-19) · SAT Full Reading Explanations (QAS 9-19) SAT #9 Reading 1 1.R.9 Answer Choice C is the correct answer because Lines 19-22 say, “Moths

their intellectual capabilities.” This answer choice also seems plausible given the point of view of the author. Moreover, it has two things working in its favor: (1) it’s a very general answer choice, (2) we have evidence that the author believes that women are hindered from “developing and using their intellectual capabilities,” implying that they do have some access but that its incomplete. So we should keep this answer choice. Answer Choice C says that “education has prevented women from realizing their goals rather than helping meet them.” Although this answer choice seems to take a negative point of view towards education, in contrast to the author’s favorable view, it might also seem plausible given what she says about the downsides of receiving a partial education (Lines 44-49). However, her particular point is that receiving “a little education” with only partial freedoms is an injustice that makes their limited freedom more acute, but not necessarily one which prevents them from realizing their goals, so we can eliminate this answer choice. Answer Choice D says that “methods of education need to be developed that appeal equally to men and to women.” This answer choice also seems plausible, given what we know about the author’s point of view. However, the Passage doesn’t focus especially on the need for these methods to be developed, and we can only say that the author would support this implicitly, so we can eliminate this option. That leaves us only with Answer Choice B, which we should choose as our correct answer.

13.R.10

Answer Choice D is the correct answer because Lines 63-65 say that “there is no law of Nature which forbids them,” where “them,” in context, refers to women behaving like men and acting “upon the same principle of egotism, consulting their own inclinations, interest, and amusement only.” Answer Choice A is incorrect because it only asks why women should receive just enough education to understand the “tyranny of custom.” Answer Choice B is incorrect because it only gives us a metaphor advising men to not expect much in the way of a “mental harvest” given the quality of women’s education and freedoms afforded to them. Answer Choice C is incorrect because it only details elements of women’s plight.

How to solve this? The Question asks, “Which choice best supports the idea that women, if they choose, are entitled to act as men do?” To solve this, we should go through each of our answer choices, looking for any texts that tell us that women are entitled to act as men do. Answer Choices A-C don’t meet this requirement because of the reasons given previously. Answer Choice D, however, says that, “there is no law of Nature which forbids them; none of any species but that which is framed by man,” which, if we remember to follow [the Pronoun Rule], we find comes immediately after her question about what would happen if women were to act like men. This answer choice, then, clearly tells us that “there is no law of Nature” prohibiting women from acting like men, or that, if they choose, women are entitled to act as men do. So, we should choose Answer Choice D as our correct answer.

Page 33: SAT Full Reading Explanations (QAS 9-19) · SAT Full Reading Explanations (QAS 9-19) SAT #9 Reading 1 1.R.9 Answer Choice C is the correct answer because Lines 19-22 say, “Moths

14.R.10

Answer Choice B is the correct answer because the word “faculties” in Line 20 can best be replaced by “intellectual abilities.”

● If WOMAN is not permitted to assert a majesty of mind, why fatigue her natural instincts with the labours of any species of education?

● If WOMAN is not permitted to assert a majesty of mind, why fatigue her intellectual abilities with the labours of any species of education?

● If WOMAN is not permitted to assert a majesty of mind, why fatigue her practical capabilities with the labours of any species of education?

● If WOMAN is not permitted to assert a majesty of mind, why fatigue her granted privileges with the labours of any species of education?

We can also use the [Antonym Technique].

15.R.10 (medium)

Answer Choice D is the correct answer, because the adage in Line 37, “A little learning is a dangerous thing,” is used, in context, to point out to men that giving women only a partial education and partial freedom (in expanding “the female heart, merely to render it more conscious that it is, by the tyranny of custom, rendered vulnerable”) can be “a dangerous thing.” Answer Choice A is incorrect because there’s no suggestion of a new innovation or insight in the Line. Answer Choice B is incorrect because the author isn’t deriding the adage, but instead quoting to men as something she obviously finds true. Answer Choice C is incorrect because, although we might be able to implicitly claim that the belief is overlooked (so that she has to remind men of it), there’s no evidence for the moderately [loud] claim that this is an old-fashioned belief.

How to solve this? The Question asks, “the purpose of the adage in Line 37 is to [...]?” To solve this, we should go back to Line quoted and try to determine its meaning given its content and context. Just prior to the Line, the author asks a series of questions which make a similar point — why give women a partial education and partial freedoms when it only makes them more aware of their plight? She then says, “Let man remember, that ‘A little learning is a dangerous thing,’” and continues to elaborate on the idea that nothing good will come from women’s partial education. So, we should expect our answer choice to say something about the lines demonstrating the danger of partial education or a “little learning.” Answer Choice A says the purpose of the Line is to, “suggest an innovative way of analyzing a pervasive challenge.” Because the Line doesn’t give any proposal of a new of analyzing a challenge, we should eliminate this option as well. Answer Choice B says the purpose of the Line is to “deride a viewpoint that has been gaining popularity,” but, since the author herself offers the adage as something true, it doesn't make sense to say that she’s deriding it, so we should eliminate this option. Answer Choice C says the Line’s purpose is to “summarize an old-fashioned belief that is often overlooked.” Because we have no evidence for the claim that this belief is an “old-fashioned” one, we should eliminate this option. Thus, only Answer Choice D remains, so we should this option as our correct answer.

Page 34: SAT Full Reading Explanations (QAS 9-19) · SAT Full Reading Explanations (QAS 9-19) SAT #9 Reading 1 1.R.9 Answer Choice C is the correct answer because Lines 19-22 say, “Moths

16.R.10

Answer Choice B is the correct answer because the words “sighs for” in Line 45 is best replaced by “craves.”

● we might at first replace this with want. ● verb

● She, like Tantalus, is placed in a situation where the intellectual blessing she dismisses is within her view; but she is not permitted to attain it: she is conscious of possessing equally strong mental powers; but she is obliged to yield, as the weaker creature.

● She, like Tantalus, is placed in a situation where the intellectual blessing she craves for is within her view; but she is not permitted to attain it: she is conscious of possessing equally strong mental powers; but she is obliged to yield, as the weaker creature.

● She, like Tantalus, is placed in a situation where the intellectual blessing she exhales is within her view; but she is not permitted to attain it: she is conscious of possessing equally strong mental powers; but she is obliged to yield, as the weaker creature.

● She, like Tantalus, is placed in a situation where the intellectual blessing she suffers is within her view; but she is not permitted to attain it: she is conscious of possessing equally strong mental powers; but she is obliged to yield, as the weaker creature.

17.R.10 & 18.R.10 (hard)

Answer Choices B and C are the correct answers, because Lines 49-52 say, “Man says, ‘you shall be initiated in all the arts of pleasing; but you shall, in vain, hope that we will contribute to your happiness one iota beyond the principle which constitutes our own,’” which best matches Answer Choice B, “A pragmatic impulse to maximize contentment.” Answer Choice A is incorrect because the claim that men’s behavior towards women is primarily motivated by “a selfish desire to deprive women of even the smallest joy,” is too strong and not supported by the Passage. Answer Choice C is incorrect because there’s no evidence in the Passage for men being primarily motivated by “a cruel tendency to afford and then withhold affections.” Answer Choice D is incorrect because there’s no evidence in the Passage for men having a “well-meaning but ultimately ineffectual intent to act fairly.”

How to solve this? Question 17 asks, “What does the author suggest primarily motivates men’s behavior toward women?” Because this is a Paired Question, we should begin by going through the answer choices for Question 18, looking for any texts that would tell us a motivation for men’s behavior toward women. We can also the [Question Order Rule] and predict that our Text is likely to come around Lines 45-55.

For Question 18, Answer Choice A says, “If WOMAN is not permitted to assert a majesty of mind, why fatigue her faculties with the labours of any species of education?” Here the author asks why women, if they’re not allowed to have complete freedom of mind, should go through the trouble of learning at all. Because this answer choice doesn’t tell us anything about men and their motivation in treating women, we should eliminate it. Answer Choice B says, “She, like Tantalus, is placed in a situation where the intellectual blessing she sighs for is within her view.”

Page 35: SAT Full Reading Explanations (QAS 9-19) · SAT Full Reading Explanations (QAS 9-19) SAT #9 Reading 1 1.R.9 Answer Choice C is the correct answer because Lines 19-22 say, “Moths

Here she notes that women can see the “intellectual blessing” of knowledge within view, but are still unable to attain it. We’re also not given any evidence here for what motivates men in their treatment of women, so we should eliminate this option as well. Answer Choice C says, “Man says, ‘you shall be initiated in all the arts of pleasing; but you shall, in vain, hope that we will contribute to your happiness one iota beyond the principle which constitutes our own.’” This tells us that the author believes that men want women to be taught all the “arts of pleasure,” but only to the degree that it benefits them. This does tell us about men and their possible motivation, so we should keep this option. Answer Choice D says, “Woman is absolutely necessary to your felicity; nay, even to your existence: yet she must not arrogate to herself the power to interest your actions.” Because this is addressed to men, and concerns their interests, it could also work as an answer choice, so we should keep this option. Both Answer Choices C and D, then, remain, and both suggest that the primary motivation for men’s treatment of women is for their happiness (“arts of pleasing” or “necessary to your felicity”), so we should expect our answer choice for Question 17 to follow this.

For Question 17, Answer Choice A says that men are motivated by, “a selfish desire to deprive women of even the smallest joy.” This certainly matches the author’s negative opinion of men and their treatment of women, but we should note that this is both a particularly [“loud”] claim and also that it has a high degree of [pejorative flair], both of which carry a high burden of proof. Because there’s no evidence in the Passage or our Texts for this motivation, we can eliminate this option. Answer Choice B says that men are motivated by, “a pragmatic impulse to maximize contentment.” This answer choice does match our two possible Texts, since they both generally refer to a desire to “maximize contentment,” so we should keep this option. Answer Choice C says that men are motivated by, “a cruel tendency to afford and then withhold affections.” This answer choice also has a high degree of [pejorative flair] which we should be skeptical of. Moreover, neither our Passage nor our Texts support this dynamic of giving and then withholding affections, so we can eliminate this option. Answer Choice D says that men are motivated by “a well-meaning but ultimately ineffectual intent to act fairly.” This answer choice is fairly neutral, although it offers a near positive evaluation of men, in contrast to the author’s negative one. Moreover, neither our Texts nor our Passage supports this idea that men are well meaning in their desire to act fairly, but only fail at the actual execution, so we should eliminate this option. Thus, we should choose Answer Choice B for Question 17.

Now, we’re left having to choose between Answer Choice C or D for Question 18. To solve this, we should keep the answer choice for Question 17 clearly in mind, that men are motivated by “a pragmatic impulse to maximize contentment.” We should notice that Lines 53-56 describe the fact that women are necessary for Man’s “felicity,” but only Lines 49-52 give us the language of what men want and are motivated by because of the use of the word “shall,” implying their desire and motivation. Thus, we should Answer Choice B for Question 17 and Answer Choice C for Question 18.

Page 36: SAT Full Reading Explanations (QAS 9-19) · SAT Full Reading Explanations (QAS 9-19) SAT #9 Reading 1 1.R.9 Answer Choice C is the correct answer because Lines 19-22 say, “Moths

19.R.10 (hard)*

Answer Choice A is the correct answer because the exclamation in Line 53, “Sensual Egotists!” is a strong negative declaration against men wanting women to please them without allowing or offering them happiness in return. Answer Choice B is incorrect because, “although the sentiment is true of the Passage as a whole and of the author, in its immediate context the exclamation is not used to show anger specially about men’s failure “to provide women with useful educations.” Answer Choice C is incorrect because although the Text does describe men selfishly putting their needs ahead of women, it doesn’t necessarily specify that they “always put their own needs ahead of those of their loved ones.” Answer Choice D is incorrect because nowhere in the Passage or surrounding context of the Text does the author “indicate frustration about the unwillingness of men to demonstrate openly their sensitivity.”

How to solve this? The Question asks, “In line 53, the author includes the exclamation most likely to [...]?” To solve this, we should go back the Lines mentioned, read the exclamation and try to understand its purpose in context. Lines 49-53 say, “Man says, ‘you shall be initiated in all the arts of pleasing; but you shall, in vain, hope that we will contribute to your happiness one iota beyond the principle which constitutes our own.’ Sensual Egotists!” Thus we see the author’s exclamation comes immediately after she details how men want women to be “initiated in all the arts of pleasing” but only insofar as it benefits men, and with no regard to happiness of women. Going back through the answer choices, we should notice that almost all of the answer choices vaguely match this angry exclamation at men for their selfishness and treatment of women. This means that we’ll have to go through each one of the answer choices more discriminately, trying to identify errors in any of them more particularly.

Answer Choice A says the exclamation is used to, “express contempt about the excessive regard of men for their presumed privileges.” Although this answer choice is both [“loud”] and [pejorative], the rare exclamation and tone of the Passage may be enough to justify it. And, “presumed privileges” does match the selfishness of men described before the exclamation, so we should keep this option. Answer Choice B says the exclamation is used to, “show anger about the failure of men to provide women with useful educations.” Although this answer choice does match one of the central claims of the Passage and is certainly something the author would agree with, in its immediate context, the exclamation is not used to show anger towards men for their failure to give women useful educations, but instead to use them for increasing their own happiness without giving in return, so we can eliminate this option. Answer Choice C says the exclamation is used to, “emphasize disappointment about the fact that men always put their own needs ahead of those of their loved ones.” This answer choice is slightly more neutral than Answer Choice A, and because women certainly qualify as men’s loved ones, this answer choice could work, so we should keep this option as well. Answer Choice D says that the author uses the exclamation in order to “indicate frustration about the unwillingness of men to demonstrate openly their sensitivity.” Because nowhere in either the Passage or the Text does the author criticize men for unwillingness to demonstrate openly their sensitivity, we should eliminate this option.

This leaves us with either Answer Choice A or Answer Choice C for Question 19. To solve this, we have to focus carefully on the exclamation and surrounding text for Line 53. Ultimately, we can say that the exclamation justifies the strong claim “express contempt” in Answer Choice A

Page 37: SAT Full Reading Explanations (QAS 9-19) · SAT Full Reading Explanations (QAS 9-19) SAT #9 Reading 1 1.R.9 Answer Choice C is the correct answer because Lines 19-22 say, “Moths

while there’s less support for the claim that men’s selfishness is perpetual and applies to all of their loved ones, so we can eliminate Answer Choice C and choose Answer Choice A for our correct answer.

20.R.10 (medium)

Answer Choice C is the correct answer, because in Lines 66-69 the author writes, “So that every good which cements the bonds of civilized society, originates wholly in the forbearance, and conscientiousness of woman,” which most closely matches the claim that, compared to men, women act in ways that are “beneficial to the functioning of society.” Answer Choice A is incorrect because, although the claim might be true that the author suggests that women are dissatisfied with their political and social status, the author doesn’t focus on it as a point of comparison with men. Answer Choice B and D are also incorrect because the author doesn’t claim that women desire independence more than men, or that they are more focused on the achievement of their goals than men.

How to solve this? The Questions asks, “The Passage indicates that compared to men, women behave in ways that are typically more [...]?” To answer this question, we should first try to identify our Text. We can use the [Question Order Rule] to predict that, because this is the last question and because the previous question references Line 53, that it will probably come at some point towards the end of the Passage, after Line 53. From there, we should scan for any possible comparison she makes between men and women. In Line 60, we should notice the opening sentence to the paragraph, “Supposing women were to act upon the same principle of egotism,” which begins her making a comparison between men and women.

Reading further, we note that she asks what would happen if women behaved as men did and concludes that the consequence would be “the annihilation of all moral and religious order.” She finishes the Passage by saying, “So that every good which cements the bonds of civilized society, originates wholly in the forbearance, and conscientiousness of woman.” Thus, the author casts women in a positive light compared to men, making the latter associated with egotism and selfishness, and women associated with forbearance, conscientiousness and the good for society. So, we should expect our answer choice to reference these.

Answer Choice A says that the author characterizes women, compared to men, as behaving in ways that are “suggestive of general dissatisfaction.” This answer choice might seem plausible, because the author does suggest that women are dissatisfied with their current social position but because (1) this choice doesn’t match our Text and (2) the author never makes a clear comparison between men and women on this point, we should eliminate this option. Answer Choice B says that women behave in ways that are “enhanced by a desire for independence.” Again, this answer choice seems plausible but doesn’t match our Text or any clear comparison made between men and women on the same point. Answer Choice D suffers for the same reason. That leaves Answer Choice C, which says that women behave in ways that are typically more “beneficial to the functioning of society.” Because this answer choice does match our Text and a comparison the author makes between men and women on this point, we should choose Answer Choice C as our correct answer.

Page 38: SAT Full Reading Explanations (QAS 9-19) · SAT Full Reading Explanations (QAS 9-19) SAT #9 Reading 1 1.R.9 Answer Choice C is the correct answer because Lines 19-22 say, “Moths

21.R.10 (medium)

Answer Choice A is the correct answer because the Passage most clearly discusses “the use of a new astronomical technique,” as can be shown in the opening Paragraph, “The results [...] also show the power of an up-and-coming method of calculating the masses of alien worlds from the way they eclipse their stars,” and the beginning of later Paragraphs which begin with “The new technique,” or “The technique.” Answer Choice B is incorrect because, while the Passage does mention date about certain planets, it focuses more on the technique of TTV than these planets. Answer Choice C is incorrect because no controversial experiment is discussed in the Passage. Answer Choice D is incorrect because, while it might be true that TTV is an innovative procedure, it’s never set in contrast to an already “established procedure,” and so is “easier to disprove” than Answer Choice B.

How to solve this? The Questions asks, “The main purpose of the passage is to [...]?” Because this is a [Big Picture Question], we have to approach this question by (1) having a general understanding of the structure and content of the Passage and (2) looking for particular [high-density information] which can tell us more clearly about the purpose of the Passage. If we track the general purpose of each paragraph, we can outline it in the following way: (1) new exoplanets are found which shows diversity and the power of a new technique, (2) the new technique TTV and what it can do, (3) the origin and idea of the technique, (4) some more history of TTV, (5) when the technique became practical, (6) a finding that used the technique, (7) another finding that used the technique. Looking for particular sentences with [high-density information], we can first look at the title of the Passage, “Star-Crossing Planets Literally Strut Their Stuff.” Because this title doesn’t tell us too much about the content of the Passage we can go down to the introductory paragraph, which ends by stating how new findings “show the power of an up-and-coming method of calculating the masses of alien worlds from the way they eclipse their stars.” The next Paragraph begins with “The new technique,” indicating that the Passage will continue to discuss the technique of TTV. Taking these two together, both the general picture and specific texts, we can say the Passage primarily concerns this new technique of TTV and should expect our answer choice to relate to this.

Answer Choice A says the main purpose of the passage is to “discuss the use of a new astronomical technique.” This does match our understanding of the Passage, so we should keep this option. Answer Choice B says the purpose of the passage is to “provide preliminary data about certain planets.” Based on the opening Paragraph, this might seem plausible. But we should note how the first Paragraph shifts to focus on the technique of TTV and continues exploring it throughout the rest of Passage, so we should eliminate this option. Answer Choice C says the purpose of the passage is to, “argue in favor of a controversial experiment.” Because the Passage doesn’t mention any controversial experiment, we should eliminate this option. Answer Choice D says the purpose of the Passage is to “suggest an innovative alternative to an established scientific procedure.” Although this might seem to plausibly mention TTV as an “innovative alternative,” the Passage never presents it in contrast to an already established procedure, so we should eliminate this option as well. That leaves Answer Choice A, which we should choose as our correct answer.

Page 39: SAT Full Reading Explanations (QAS 9-19) · SAT Full Reading Explanations (QAS 9-19) SAT #9 Reading 1 1.R.9 Answer Choice C is the correct answer because Lines 19-22 say, “Moths

22.R.10 (medium)

Answer Choice A is the correct answer because in Lines 5-9 the author says that, “The results, announced at a meeting of the American Astronomical Society, also show the power of an up-and-coming method of calculating the masses of alien worlds from the way they eclipse their stars.” This most clearly matches Answer Choice A which says that the author’s central claim is that, “TTV has enabled astronomers to determine more accurately than before the mass of certain planets outside of our solar system.” Answer Choice B is incorrect because, although the Passage does tell us that NASA’s Kepler spacecraft provided scientists with a “surprising finding,” the Passage focuses more directly on TTV. Answer Choice C is incorrect because the Passage doesn’t give evidence that “there are more planets outside of our solar system with an atmosphere similar to that of Earth than had previously been hypothesized.” Answer Choice D is incorrect because, although the Passage does tell us that scientists’ “expertise has been growing,” it doesn’t focus on this development of their skill but instead focuses on TTV as a technique.

How to solve this? The Question asks, “The author’s central claim in the passage is that [...]?” To solve this question, we should (1) have a general understanding of the structure and content of the Passage and (2) look for specific, [high-density] information. [You can look at the previous question for a breakdown of this]. Once we’ve identified these two things, we can go to the answer choices and begin looking for a correct choice while eliminating wrong ones.

Answer Choice A says that the author’s central claim in the passage is that, “TTV has enabled astronomers to determine more accurately than before the mass of certain planets outside of our solar system.” [include things from above, previous question].

23.R.10 (medium)

Answer Choice C is the correct answer because Paragraphs 1-5 introduce and describe the method and history of TTV, while the remaining Paragraphs 6-8 discuss the application of TTV in two separate studies. This most clearly matches the breakdown offered by Answer Choice C that the Passage shifts from a “description of an innovative procedure to an account of some specific applications of that procedure.” Answer Choice A is incorrect because the Passage doesn’t begin with a “summary of the results of several experiments” but instead ends with it. Answer Choice B is incorrect because the Passage doesn’t outline the “traditional difficulties of a scientific problem” prior to TTV. Answer Choice D is incorrect because the Passage doesn’t discuss a controversial scientific practice.

How to solve this? The Question asks, “Over the course of the passage, the main focus shifts from a [...?” Because this is a [Shift in Focus Question], we should try to solve it by (1) tracking the general outline of the Passage and noting any possible shifts, (2) looking for specific transitions beginning with words like “but.” Although it’s impossible to perfectly predict in advance the shift in focus the SAT will highlight, we can note something like the following grouping and possible shifts between them:

● new findings about exoplanets (Paragraph 1) ● the usefulness and history of TTV (Paragraphs 1-4)

Page 40: SAT Full Reading Explanations (QAS 9-19) · SAT Full Reading Explanations (QAS 9-19) SAT #9 Reading 1 1.R.9 Answer Choice C is the correct answer because Lines 19-22 say, “Moths

● when TTV became practical (Paragraph 5) ● two studies that used TTV (Paragraphs 6-8)

No major [specific transitions / shifts show up in the Passage], so we can expect our answer choice to highlight one of the shifts between ideas above. At this point we can go the answer choices, looking for issues in any part of the options to leave us with our correct answer

Answer Choice A says the focus of the Passage shifts from a “summary of the results of experiments to a chronicle of the process used in of these experiments.” The “summary of the results of experiments,” could possibly refer to the first sentence of the opening paragraph, but this would be a very weak connection and the best summary comes at the end of the Passage, so we should eliminate this option. Answer Choice B says the focus of the Passage shifts from a “reflection regarding the traditional difficulties of a scientific problem to a consideration of a new technique rendering that problem obsolete.” The Passage does highlight a “new technique,” but because it doesn’t first describe the “traditional difficulties of a scientific procedure,” we should eliminate this option. Answer Choice C says the focus of the Passage shifts from a “description of an innovative procedure to an account of some specific applications of that procedure.” Because this does match one our possible breakdowns, between the discussion of TTV and its becoming practical (Paragraphs 1-5) and the the two studies that used TTV (Paragraphs 6-8), we should keep this option. Answer Choice D says the focus of the passage shifts from a “defense of a controversial scientific practice to a demonstration of that practice’s ultimate usefulness.” Because the Passage doesn’t discuss any “controversial scientific practice,” we can eliminate this option. This leaves us with Answer Choice C, which we should choose as our correct answer.

24.R.10 & 25.R.10 (medium)

Answer Choices A and B are the correct options here, because Lines 15-17 tell us, “To know whether [a planet] is rocky, gaseous, or some mixture of the two, astronomers also need its mass,” which most clearly provides evidence for a question which “astronomers [are] unable to answer unless they know a particular exoplanet’s mass.” Answer Choice A most clearly matches these Lines with the question, “How similar to Earth is that planet in its ratio of rock to gas?” None of the remaining texts for Question 25 tell us about a question astronomer’s couldn’t answer unless they know an exoplanet’s mass, and only none of the answer choices for Question 24 match Lines 15-17.

How to solve this? Question 24 asks, “Based on the passage, which question are astronomers unable to answer unless they know a particular exoplanet’s mass?” Because this is a Paired Question, to solve it we should begin by going through the answer choices for Question 25, looking for any text that tells us something astronomers couldn’t figure out unless they know an exoplanet’s mass. Moreover, using the [Question Order Rule] as a guide, we can predict that our line will come before Line 17, thus making Answer Choices A and B for Question 25 more likely.

Page 41: SAT Full Reading Explanations (QAS 9-19) · SAT Full Reading Explanations (QAS 9-19) SAT #9 Reading 1 1.R.9 Answer Choice C is the correct answer because Lines 19-22 say, “Moths

For Question 25, Answer Choice A says, “The results, announced at a meeting of the American Astronomical Society, also show the power of an up-and-coming method of calculating the masses of alien worlds from the way they eclipse their stars.” Because this answer choice only mentions the power of TTV, and not something astronomer’s couldn’t figure out without knowing the mass of an exoplanet, we can eliminate this option. Answer Choice B says, “To know whether it is rocky, gaseous, or some mixture of the two, astronomers also needs it mass.” Because this answer choice does clearly tell us something astronomers need the mass of an exoplanet to know, and because it comes before Line 17, we should keep this option, predicting that its most likely to be our answer choice. Answer Choice C says, “If both planets were transiting, astronomers could measure the perturbations in both their orbits and work out the planets’ masses.” Because this answer choice tells us how scientists use TTV to determine mass and not what they need mass for to know, we can eliminate this option. Answer Choice D says, “They found that the outer, KOI-314c, which orbits the star every 23 days, has the same mass as Earth, although it is about 60% larger than Earth in radius.” Because this answer choice also doesn’t tell us what scientists need the mass of an exoplanet to know, we can also eliminate this option. That leaves Answer Choice B as our correct answer for Question 25.

Looking at Lines 15-17 again, we can ask ourselves what they tell us about what scientists need the mass of an exoplanet to know. It reads, “To know whether it is rocky, gaseous, or some mixture of the two, astronomers also needs it mass.” Thus, we should predict that our answer choice will relate to scientists needing the mass to determine whether an exoplanet is “rocky, gaseous or some mixture of the two.”

For Question 24, Answer Choice A says that scientists need a planet’s mass to determine, “How similar to Earth is that planet in its ratio of rock to gas?” Because this answer choice does reference scientists needing to know an exoplanet’s mass to know whether its “rocky, gaseous, or some mixture of the two,” we should keep this option. Because Answer Choices B, C and D for Question 24 don’t mention a planet being “rocky or gaseous”, we can eliminate these options and choose Answer Choice A for our correct answer for Question 24.

26.R.10 (medium)

Answer Choice C is the correct answer because in Line 17, “traditionally” can best be replaced by “historically,” with its emphasis on what scientists had to do in the past to determine an exoplanet’s mass. Answer Choice A is incorrect, because the Line isn’t emphasizing the authenticity of the scientist’s method, as opposed to an inauthentic approach. Answer Choice B is incorrect, because the Line isn’t highlighting the appropriateness of the scientist’s method, as opposed to an inappropriate one.. And Answer Choice D is incorrect, because the author isn’t emphasizing the conservative nature of their approach, as opposed to a more liberal one.

How to solve this? [...]

Page 42: SAT Full Reading Explanations (QAS 9-19) · SAT Full Reading Explanations (QAS 9-19) SAT #9 Reading 1 1.R.9 Answer Choice C is the correct answer because Lines 19-22 say, “Moths

27.R.10 (hard ~)

Answer Choice A is the correct answer because Lines 38-44 tell us that “astronomers failed to detect transit timing variations because almost all known exoplanets were gas giants,” and that these gas giants “formed farther from the star and later barreled upward, clearing away any potential wobble-inducing companions.” This most clearly matches Answer Choice A, which says that, “TTV requires the existence of companion planets, and most known exoplanets prior to Kepler did not show evidence of companions.” Answer Choice B is incorrect, because the Passage doesn’t tell us that only a single set of data was available when TTV required multiple sets. Answer Choice C is incorrect because the Passage doesn’t tell us that “the location of parent stars of exoplanets was difficult to determine.” Answer Choice D is incorrect because TTV doesn’t require that the mass of a planet be known.

How to solve this? The Question asks, “According to the passage, why was the TTV technique difficult to implement before the data from the Kepler spacecraft became available?” To solve this, we first have to go back to the Passage to find a Text that will tell us why the TTV technique was difficult to implement before the data from the Kepler spacecraft became available. Using the [Question Order Rule], we can predict that our Text should probably come before Line 45. Moreover, we can find that Paragraph 5 discusses the data from the Kepler spacecraft and that it allowed the TTV technique to become practical. In Lines 38-44, we find, “For years afterward, however, astronomers failed to detect transit timing variations because almost all known exoplanets were gas giants spinning around their stars in tight orbits. Theorists think such planets formed farther from the star and later barreled upward, clearing away any potential wobble-inducing companions.” Because these Lines tells us a reason why TTV was initially difficult to determine, we can use it as our Text. It tells us that all known exoplanets were gas giants which had “cleared away any wobble-inducing companions,” which are an essential component for the technique of TTV. Thus, we should expect our answer choice to relate to this fact.

Answer Choice A says that the difficulty of implementing TTV involved the fact that, “TTV requires the existence of companion planets, and most known exoplanets prior to Kepler did not show evidence of companions.” Because this does match our Text, we should keep this option. Answer Choice B says that, “TTV requires several different sets of data for confirmation, and prior to Kepler only a single set of data was available.” Because this answer choice doesn’t match either our Text or our Passage, we should eliminate this option. Answer Choice B says that the “location of the parent stars of exoplanets was difficult to determine,” which doesn’t match our Text and isn’t supported by the Passage, so we should eliminate this option. Answer Choice D says that, “TTV requires that the mass of a planet be known, and prior to Kepler the masses of exoplanets were impossible to ascertain.” This answer choice contradicts the Passage, which tells us that TTV can be used to determine the mass, not the other way around, so we should eliminate this option. This leaves us with Answer Choice A, which we should choose as our correct answer.

Page 43: SAT Full Reading Explanations (QAS 9-19) · SAT Full Reading Explanations (QAS 9-19) SAT #9 Reading 1 1.R.9 Answer Choice C is the correct answer because Lines 19-22 say, “Moths

28.R.10 (medium)

Answer Choice B is the correct answer because in Line 45 “practical” can best be replaced by “workable,” with its emphasis on the fact that the technique of TTV became possible to use after the data from the Kepler spacecraft became available. Answer Choice A is incorrect, because the Line isn’t highlighting that the technique became ordinary as opposed to strange. Answer Choice C is incorrect because the Line isn’t emphasizing that the technique became systematic as opposed random or unstructured. And Answer Choice D is incorrect because the Line isn’t emphasizing that the technique became a qualified one as opposed to an unqualified one.

How to solve this? [...]

29.R.10 & 30.R.10 (medium)

Answer Choices D and D are the correct answers because Lines 86-89 tell us that, “They also found a pattern: as the planets grew bigger in radius, their density declined. ‘If you make something twice as big, it becomes four times less dense.’” Thus, if there were a “discovery of planet with a radius four times greater than Earth’s and a density similar to Earth’s,” this would contradict Lithwick’s findings. Only Answer Choice D captures this when it says, “Such a discovery would challenge Lithwick’s findings, because such a planet would not conform to his expectations.” For Question 30, Answer Choices A and B are incorrect because neither tell us about Lithwick’s findings and come before his initial mention in the Passage. Answer Choice C is incorrect because it only introduces Lithwick’s study without telling us any details of it concerning the relationship between density and radius. For Question 29, only Answer Choice D captures the contradiction between the premise of the Question and what Lines 86-89 tell us.

How to solve this? Question 29 asks, “What would be the effect on Lithwick’s findings of the discovery of a planet with a radius four times greater than Earth’s and a density similar to Earth’s?” Because this is a Paired Question, to solve this we should go through the answer choices for Question 30, looking for any test that tells us about Lithwick’s findings and what they tell us about the relationship between a planet’s density and radius. Using the [Question Order Rule], we should expect our Text come near the end of the Passage, and using the [Linear Pacing Rule] we should expect our Text to come after the first mention of Lithwick in the final paragraph, making Answer Choices A and B for Question 30 unlikely.

For Question 30, Answer Choice A says that “By simulating the dance on a computer, the researchers worked out the masses of the two planets.” Because this choice doesn’t tell us about Lithwick’s study, comes before his first mention, and doesn’t concern the relationship between a planet’s density and radius, we should eliminate this option. Answer Choice B says that, “Kipping and his colleagues infer that the planet — the lightest exoplanet so far discovered — has a rocky core and a thick, gaseous atmosphere.” This choice also doesn’t tell us about Lithwick’s study, comes before his first mention, and doesn’t concern the relationship between a planet’s density and radius, so we should also eliminate this option. Answer Choice C says, “Meanwhile, researchers led by Yoram Lithwick, an astronomer at Northwestern University in Evanston, Illinois, were looking at the TTV signatures of 163 exoplanets found by Kepler.” This answer choice does mention Lithwick, but only introduces him and his study and doesn’t give any evidence about his findings on the relationship between the density and radius of an

Page 44: SAT Full Reading Explanations (QAS 9-19) · SAT Full Reading Explanations (QAS 9-19) SAT #9 Reading 1 1.R.9 Answer Choice C is the correct answer because Lines 19-22 say, “Moths

exoplanet, so we should eliminate this option. Answer Choice D says, “They also found a pattern: as the planets grew bigger in radius, their density declined. ‘If you make something twice as big, it becomes four times less dense.’” Because this answer choice meets all of our criteria — comes after Lithwick’s first mention and describes his ideas on the relationship between a planet’s radius and density — we should keep this option and choose Answer Choice D as our correct answer for Question 30.

Looking more closely at Lines 86-89, we can ask ourselves what they tell us would happen to Lithwick’s findings if a “planet with a radius four times greater than Earth’s and a density similar to Earth’s” were discovered. Because Lithwick thinks that as radius increases, density decreases, we can conclude that this finding would contradict his conclusion and should expect our answer choice to relate to this fact.

For Question 29, Answer Choice A says the “discovery would have no effect on Lithwick’s findings.” Without even reading the rest of this option, we can eliminate it because it doesn’t tell us that this finding would contradict his theory. Answer Choices B and C begin by saying that such a discovery would “bolster,” or boost and support, Lithwick’s findings which are also obviously incorrect, so we can eliminate these as well. This leaves us with Answer Choice D, which says that “such a discovery would challenge Lithwick’s findings, because such a planet would not conform to his experiments.” This does match our expectation that the discovery would contradict Lithwick’s findings, so we can keep this option and should choose Answer Choice A as our correct answer for Question 29.

31.R.10 (medium)

Answer Choice A is the correct answer because, as Lines 3-4 tell us, the Passage primarily concerns “just how farming spread from [the Middle East] into Europe,” which most closely matches Answer Choice A which says the purpose of the passage is to “discuss research into the origins of ancient European farmers.”Answer Choice B is incorrect because the Passage primarily concerns the origins of farming in Europe and the way it spread from the Middle East, but not a debate about when it first appeared. Answer Choice C incorrect because the Passage doesn’t focus on farming methods of ancient Europeans. And Answer Choice D is incorrect because, while it does discuss a conflict between the genetic evidence for the paternal and maternal line, it doesn’t highlight a conflict between genetic and archaeological evidence.

How to solve this? The Question asks, “The main purpose of the passage is to [...]?” Because this is a [Big Picture Question], we have to first construct an overall picture of the Passage by looking both at its general structure and specific sentences in the introductions and conclusions which tell us a clear thesis. Generally, the Passage can be organized like this:

● the question about how farming spread into europe and new related study ● questions that scientists studying farming have asked ● genetic studies reveal different origins ● recent sequencing which resolves the genetic conflict ● a new supportive study of a mass grave ● different lineages for males and females, and additional routes ● more conclusions about the society

Page 45: SAT Full Reading Explanations (QAS 9-19) · SAT Full Reading Explanations (QAS 9-19) SAT #9 Reading 1 1.R.9 Answer Choice C is the correct answer because Lines 19-22 say, “Moths

● more genetic evidence about lactose intolerance

Looking more specifically, we can read the title of the Passage at the top — “Farming Conquered Europe at Least Twice.” We can also look at the introductory Paragraph for specific sentences that tell us about the purpose of the Passage. Lines 3-8 say, “But just how farming spread from there into Europe has been a matter of intense research. A new study of ancient DNA from 5000-year-old skeletons found in a French cave suggests that early farmers entered the European continent by at least two different routes [...].” Taking this general and specific picture together, we can say that the Passage concerns the origin and spread of farming into Europe as well as the conclusion that at least two paths were involved, and expect our answer choice to relate to this.

Answer Choice A says the main purpose of the passage is to “discuss research into the origins of ancient European farmers.” Because this answer choice matches our summary of the Passage, we should keep this option. Answer Choice B says the purpose of the passage is to “resolve a debate about when farming first appeared in Europe.” This might seem plausible at first, but we should note that it assumes that the Passage primarily concerns a debate and specifically a debate about the time that farming first appeared in Europe. Because the Passage doesn’t center on a specific debate and because it focuses on the origin of European farming and not its timeline, we should eliminate this option. Answer Choice C says the main purpose of the passage is to “consider a study of the farming methods of ancient Europeans.” Because the Passage doesn’t concern the farming methods of ancient Europeans, we should eliminate this option as well. Answer Choice D says the main purpose of the passage is to “explain the conflict between archaeological and genetic evidence about the first European farmers.” This answer choice might at first seem to reference the genetic conflict mentioned in Paragraphs 3 and 4, but we should note that the Passage doesn’t focus entirely on this conflict. Moreover, the conflict is between two conflicting pieces of genetic evidence, not between archaeological and genetic evidence, so we should eliminate this option as well. This leaves us with Answer Choice A, which we should choose as our correct answer.

Page 46: SAT Full Reading Explanations (QAS 9-19) · SAT Full Reading Explanations (QAS 9-19) SAT #9 Reading 1 1.R.9 Answer Choice C is the correct answer because Lines 19-22 say, “Moths

32.R.10 (medium)

Answer Choice C is the correct answer because the string of questions in Lines 12-18 set up the discussion of the following studies by Lacan and others which answer whether agriculture spread by the migration of farmers or through ideas, and whether there was one or multiple “waves of farming.” This best matches Answer Choice C which says that the main purpose of the Lines is to pose questions that “motivated the research of Lacan and other scientists.” Answer Choice A is incorrect because in the next paragraph we’re told that previous studies on these questions do exist. Answer Choice B is incorrect because the Passage suggests that these questions haven’t been resolved and that Lacan’s contributed to their solving them. Answer Choice D is incorrect because Lacan’s studies are presented as helping to solve these questions, not in making them more difficult to answer.

How to solve this? The Question asks, “The main purpose of Lines 12-18 is to pose questions that [...]?” To solve this, we should go back to the Lines in question and try to determine in advance how they work in the Passage and their immediate context. The Questions ask:

● Was agriculture brought in primarily by Middle Eastern farmers who replaced the resident hunter-gatherers?

● Or did agriculture advance through the spread of technology and ideas rather than people?

● And was there just one wave of farming into the continent or multiple waves and routes?

Based on our understanding the general structure of the Passage and the Paragraphs that immediately follow, we should see how these three questions set up the discussion of Lacan and how her study addressed them, and we should predict that our answer choice will relate to this.

[Answer Choice A says that… ]

Page 47: SAT Full Reading Explanations (QAS 9-19) · SAT Full Reading Explanations (QAS 9-19) SAT #9 Reading 1 1.R.9 Answer Choice C is the correct answer because Lines 19-22 say, “Moths

33.R.10 & 34.R.10 (medium)

Answer Choices C and A are the correct answers because Lines 21-25 tell us that, “Numerous such studies [looking at the genetic profiles of modern-day Europeans and Middle Easterners], especially of Y chromosomes, which are transmitted via the paternal line, suggest that actual farmers, not just their ideas, spread westward over the millennia, eventually reaching the British Isles.” This most closely matches Answer Choice C, which says that modern Europeans “descended at least in part from people who originated further east.” For Question 34, Answer Choice A is incorrect because it doesn’t concern modern Europeans. Answer Choice C is incorrect for the same reason and because it comes much later in the Passage. Answer Choice D is incorrect for the same reasons. For Question 33, Answer Choice A is incorrect because, while the mtDNA and Y chromosomes of modern Europeans show different lineages, the Passage never compares their diversity. Answer Choice B is incorrect because the Text tells us that these modern Europeans can trace their ancestry in part to the Middle East, not primarily from Southern Europe. Answer Choice D is incorrect because it was the farmers who migrated westward, and not the hunter-gatherers.

How to solve this? Question 33 asks, “DNA evidence discussed in the passage most strongly suggests that modern Europeans [...]?” Because this is a Paired Question, to solve this we should begin by going through the answer choices for Question 34, looking for any Text that tells us about modern Europeans and DNA evidence. Using the [Question Order Rule], we should predict that our Text is most likely to come between Lines 18 and 31, making Answer Choice A the most likely option, and Answer Choices C & D the most unlikely.

For Question 34, Answer Choice A says, “Numerous such studies, especially of Y chromosomes, which are transmitted via the paternal line, suggest that actual farmers, not just their ideas, spread westward over the millennia, eventually reaching the British Isles.” Remembering our [Pronoun Rule / Check Your References], we should look at the surrounding Passage to identify what “such studies” refer to, and find that they refer to studies that look at the “genetic profiles of modern-day Europeans and Middle Easterners.” Because this answer choice tells us about modern Europeans, mentions relevant DNA evidence and comes early in the Passage, we should keep this option as a strong candidate. Answer Choice B says, “Some of this research, most notably in Germany, suggests that male farmers entering Central Europe mated with local female hunter-gatherers — thus possibly resolving the contradiction between the Y chromosome and mtDNA results.” This answer choice might seem to suggest something about modern Europeans, given that it discusses their potential lineage. But, because it doesn’t mention modern Europeans explicitly, it’s a weaker option that Answer Choice A, so we should eliminate or discount it. Answer Choice C says that Lacan’s team “found that the female and male lineages seemed to have different origins,” when discussing her studies of a mass grave. Because this answer choice doesn’t reference modern Europeans, and because it comes so much later in the Passage, we should eliminate this option. Answer Choice D says of the same grave that, “Most of the skeletons were males, and many appeared to be very closely related.” Again, because this Text doesn’t mention modern Europeans directly and comes so much later in the Passage, we should eliminate this option. That leaves us with only Answer Choice A remaining, and we should choose it as our correct answer to Question 34.

Page 48: SAT Full Reading Explanations (QAS 9-19) · SAT Full Reading Explanations (QAS 9-19) SAT #9 Reading 1 1.R.9 Answer Choice C is the correct answer because Lines 19-22 say, “Moths

Lines 21-25 tell us that studies of the genetic profiles of modern-day Europeans and Middle Easterners show that the Y chromosomes of modern-day Europeans show evidence of farmers, not just their ideas, migrating into Europe. This suggests that modern-day Europeans in part descend, at least in their paternal line, from Middle Eastern farmers.

For Question 33, Answer Choice A says that DNA evidence suggests that modern Europeans “show more diversity in their mtDNA than in their Y chromosomes.” Although our Text doesn’t mention mtDNA, we may remember that later in the Passage we’re told that genetic evidence for Y chromosomes and mtDNA showed different lineages, making this answer choice plausible. However, the Passage doesn’t compare the diversity of mtDNA and Y chromosomes of modern Europeans. Moreover, our Text itself doesn’t discuss mtDNA, so we should eliminate this option. Answer Choice B says that modern Europeans “can trace their ancestry primarily to people from ancient southern Europe.” Because our Text tells us that modern Europeans show descension from Middle Easterners and not from southern Europe, we should eliminate this option as well. Answer Choice C says that modern Europeans “descended at least in part from people who originated further east.” This does match our Text, so we should keep this option as well. Answer Choice D says that modern Europeans “have hereditary links to hunter-gatherers who migrated westward across Europe.” Because our Text doesn’t discuss hunter-gatherers, and because it was the farmers who migrated westward across Europe, not the hunter-gatherers, we should eliminate this option. This leaves us with Answer Choice C, which we should choose as our correct option for Question 33.

35.R.10 & 36.R.10

Answer Choices A and B are the correct answers because Lines 30-32 tell us that, “In recent years, studies have begun to resolve these issues by sequencing the DNA of the prehistoric farmers themselves,” which best answers how “seemingly contradictory findings about the spread of farming in ancient Europe began to be reconciled [...].” This Text best matches Answer Choice A for Question 35, that these contradictory findings began to be reconciled once scientists “analyzed the genetic makeup of prehistoric farmers.” For Question [... wrong answers].

How to solve this? Question 35 asks, “According to the passage, seemingly contradictory findings about the spread of farming in ancient Europe began to be reconciled once scientists [...]?” Because this is a Paired Question, we should begin by going through the answer choices for Question 36, looking for any Texts that tell us how scientists began to resolve seemingly contradictory evidence. Using the [Question Order Rule], we should expect our Text to come between Lines 18 and 31, making Answer Choices A and B more likely than C or D.

For Question 36, Answer Choice A says, “Yet other studies, based on mitochondrial DNA (mtDNA), which is inherited maternally, have come to the opposite conclusion, suggesting that farmers had local European ancestry.” Because this answer choice tells us about the contradictory findings and not their resolution, we should eliminate it. Answer Choice B says, “In recent years, studies have begun to resolve these issues by sequencing the DNA of the prehistoric farmers themselves.” This answer choice does tell us about the resolution of seemingly contradictory findings, so we should keep this option. Answer Choice C says, “A team led my molecular anthropologist Marie Lacan reports work on ancient DNA — both

Page 49: SAT Full Reading Explanations (QAS 9-19) · SAT Full Reading Explanations (QAS 9-19) SAT #9 Reading 1 1.R.9 Answer Choice C is the correct answer because Lines 19-22 say, “Moths

mitochondrial and Y-chromosomal — from more than two dozen skeletons found in the 1930s in a cave called Treilles in southern France.” Because this answer choice only introduces a study and doesn’t tell us about any resolution of contradictory findings, we should eliminate it as an option. Answer Choice D says, “The mtDNA showed genetic markers previously identified as having deep roots in ancient European hunter-gatherer populations, but the Y chromosomes showed the closest affinities to Europeans currently living along the Mediterranean [...].” Because this answer choice also doesn’t tell us about how scientists reconciled contradictory findings, we should eliminate it as well. This leaves us with Answer Choice B, which we should choose as our correct answer for Question 36.

Lines 30-32 tell us that studies began to resolve issues about contradictory genetic findings when scientists began “sequencing the DNA of the prehistoric farmers themselves,” so we should expect our answer choice to relate to this.

For Question 35, Answer Choice A says that seemingly contradictory findings began to be reconciled when scientists “analyzed the genetic makeup of prehistoric farmers.” Because this matches our Text pretty closely, we should keep this option. Answer Choice B says that seemingly contradictory findings began to be reconciled when scientists “unearthed a large number of skeletons from the Treilles cave.” Because this doesn’t match our Text, we should eliminate this option. Answer Choice C says that seemingly contradictory findings began to be reconciled when scientists “sequenced Y-chromosomal DNA from modern Europeans.” This answer choice is incorrect because we’re told that scientists sequenced DNA from prehistoric farmers, not modern Europeans. Answer Choice D says that seemingly contradictory findings began to be reconciled when scientists “examined mtDNA apart from other genetic materials,” which doesn’t match our Text or Passage, so we should eliminate this option as well. This leaves us with Answer Choice A, which we should choose for our correct answer for Question 35.

37.R.10

Answer Choice C is the best answer because in Line 31 the word “resolve,” has the meaning of “to fix,” or “settle.” Answer Choice A is incorrect because the Line doesn’t mean that scientists “reduced” the issue, or made it smaller, but that they solved it. Answer Choice B is incorrect because the Line doesn’t mean that the scientists drove away or “dispelled” the issue. Answer Choice D is incorrect because the Line doesn’t mean that the scientists announced or “declared” the issue, but that they solved it.

38.R.10 (hard)

Answer Choice D is the correct answer, because in Line 63, “yielded” most nearly means “gave” or “furnished.” Answer Choice A is incorrect because the Line doesn’t mean that the findings let go of or “relinquished” intriguing details, as if they wanted to hold onto them in the first place. Answer Choice B is incorrect because the Line doesn’t mean that the findings “submitted” new intriguing details, which in this context would give a degree of agency to the findings that’s inappropriate. Answer Choice C is incorrect because the Line doesn’t mean that the findings nurtured, grew or “cultivated” intriguing details.

Page 50: SAT Full Reading Explanations (QAS 9-19) · SAT Full Reading Explanations (QAS 9-19) SAT #9 Reading 1 1.R.9 Answer Choice C is the correct answer because Lines 19-22 say, “Moths

39.R.10

Answer Choice D is the correct answer because the “archaeological evidence” in Line 78 is cited as support for a view that there was a second migratory pathway for the spread of farming through Southern Europe. This is a view that Lacan holds, making Answer Choice D — “bolsters a conclusion about the spread of farming in Europe that Lacan’s team members drew from their analysis of DNA” the best answer. Answer Choice A is incorrect because the evidence doesn’t introduce an unresolved complication in a theory but is instead used to support one. Answer Choice B is incorrect because the Passage doesn’t give us any earlier hypothesis about “the use of fermented milk by farmers living in Southern Europe.” Answer Choice C is incorrect because, although the evidence does highlight a specific genetic factor, it doesn’t highlight one that is likely to have influenced the settlement patterns of Middle Eastern immigrants in Europe.

How to solve this? Question 39 asks, “It can most reasonably be inferred that the ‘archaeological evidence’ referred to in line 78 [...]?” To solve this, we should first go back to the line referenced and decide what role the “archaeological evidence” cited plays in the Passage and its immediate context.

Lines 77-81 say, “That’s consistent with other archaeological evidence that central European farmers herded dairy cows, whereas Mediterranean farmers herded sheep and goats and drank fermented milk, which has much lower lactose levels.” In context, these Lines come just after a discussion of findings that the skeletons in Treilles show genetic signs that these people were unable to digest lactose, and the “archaeological evidence” is told to be consistent with this. What this means is that Lacan’s theory that farming spread into Europe not only through the Middle East but also through southern Europe (Lines 58-62) has evidence to support it. Because the people of Treilles were unable to digest lactose, it makes sense to say that they were descended from southern Europe or the Mediterranean, where farmers drank dairy products with much lower lactose levels.

Answer Choice A says that the evidence, “introduces an unresolved complication into an evolving theory about the spread of farming in ancient Europe.” Because our Text presents the “archaeological evidence” as supporting a theory, not introducing an unresolved complication into one, we should eliminate this option. Answer Choice B says that evidence “confirms an earlier hypothesis about the use of fermented milk by farmers living in southern Europe.” Because the Passage doesn’t contain any hypothesis about the use of fermented milk and doesn’t give any special focus to it, we should eliminate this option. Answer Choice C says that the evidence “highlights a genetic factor that likely influenced the settlement patterns of Middle Eastern immigrants in Europe.” Because our Text and the findings refer mainly to immigrants from southern Europe and not from the Middle East, and because it doesn’t make sense to say that lactose intolerance of those in the Mediterranean influenced the settlement patterns of Middle Eastern immigrants, we should eliminate this option as well. Answer Choice D says that the evidence “bolsters a conclusion about the spread of farming in Europe that Lacan’s team members drew from their analysis of DNA.” Because the “archaeological evidence” in our Text does support the conclusion about how farming spread in Europe and is consistent with Lacan and her team members’ findings, we should keep this option and choose it as our correct answer for Question 39.

Page 51: SAT Full Reading Explanations (QAS 9-19) · SAT Full Reading Explanations (QAS 9-19) SAT #9 Reading 1 1.R.9 Answer Choice C is the correct answer because Lines 19-22 say, “Moths

40.R.10 (easy)

Answer Choice A is the correct answer because the map shows the percent of shared Y-chromosome lineage betten Treilles individuals and other, current European populations. According to the legend on the right side of the map, the darker the gradient, the higher the percentage of shared Y-lineage with Treilles individuals. Because Southern Portugal shows the darkest shading, we should choose it as our answer. Answer Choice B is incorrect because the British Isles are not shaded at all, showing the lowest percentage of shared Y lineages. Answer Choices C and D are incorrect because they both show the same amount of light shading, both less than that of Southern Portugal.

41.R.10 (medium)

Answer Choice C is the correct answer because Lacan holds the view that some ancient European farmers descended in part via southern Europe. The map best supports this view because it shows that Treilles individuals share a high percentage of Y-chromosome lineage with southern European regions along the Mediterranean like Italy, Cyprus and Turkey. Answer Choice A is incorrect because Lacan doesn’t hold the view that ancient European farmers had local European ancestry and the map shows a high percentage of shared lineage with southern Europe. Answer Choice B is incorrect because Lacan doesn’t hold the view that ancient European farmers traveled from the British Isles and because that chart shows no shared Y lineage between Treilles individuals and those living in the British Isles. Answer Choice D is incorrect because, while Lacan does hold the view that ancient European farmers “established patrilocal societies in central Europe,” the map doesn’t show any evidence for this.

42.R.10

Answer Choice A is the correct answer because in Line 16 “gain” most nearly means “benefit” or “profit,” and can best be replaced by the word in the sentence.

● The plant species that benefit the most from high resource availability may also profit the most from escaping enemies upon moving to a new range.

● The plant species that benefit the most from high resource availability may also increase the most from escaping enemies upon moving to a new range.

● The plant species that benefit the most from high resource availability may also traverse the most from escaping enemies upon moving to a new range.

● The plant species that benefit the most from high resource availability may also reach the most from escaping enemies upon moving to a new range.

Answer Choice B is incorrect because the Line isn’t emphasizing that the plant species will increase, but that it will benefit from escaping enemies. Answer Choice C is incorrect because the Line isn’t emphasizing that the planet species will travel, or traverse the most. And Answer Choice D is incorrect because the Line isn’t emphasizing that the planets species will “reach” or extend the most.

Page 52: SAT Full Reading Explanations (QAS 9-19) · SAT Full Reading Explanations (QAS 9-19) SAT #9 Reading 1 1.R.9 Answer Choice C is the correct answer because Lines 19-22 say, “Moths

43.R.10 & 44.R.10

Answer Choices B and D are the correct answers because Lines 32-34 say that, “Plant communities with lots of disturbance, high resource availability, or reduced species diversity tend to be easily colonized,” so that an area unlikely to be colonized would have little disturbance, low resource availability and a large amount of species diversity. This best matches Answer Choice B which describes, “A forested area that has numerous species of plants and has received a nearly normal amount of rainfall over the last five years.” For Question 44, only Answer Choice D describes the features that would make an area likely or unlikely to be colonized by a fast growing invasive species. For Question 43, Answer Choice A is incorrect because it describes a wetland area that “was recently converted to farmland” and experiences “flooding and soil erosion,” which qualify as “lots of disturbance” and would make it likely to be colonized by an invasive species. Answer Choice C is incorrect because an area “that experienced a fire within the last year” and “has few species” of plants qualifies as having experienced “lots of disturbance” and having “reduced species diversity,” making it likely to be colonized by an invasive species. Answer Choice D is incorrect because an area that has “experienced drought over the last seven years” and that has “fewer species of plants,” qualifies as having experienced “low resource availability” and as having “reduced species diversity.”

How to solve this? Question 43 asks “Based on the information in Passage 1, which area would be LEAST likely to be colonized by a fast-growing invasive plant species.” Because this is a Paired Question, we should begin by going through the answer choices for Question 44, looking for any information that would tell us about the likelihood of an area being colonized by an invasive species. This could be information that tells us about areas that are likely or unlikely to be colonized in this way. // Using the [Question Order Rule] we should expect our Text to come earlier in the Passage, most likely before Line 36. Because all of our texts come before that Line, the Rule won’t help us too much. //

For Question 44, Answer Choice A says, “The problems caused by such invasive species are the direct result of their success in colonizing new habitats, and understanding why they are so successful is essential to controlling their spread.” Because these Lines only tell us about the success of invasive species and why we should understand that success, but not about the features that contribute to that success, we should eliminate this option. Answer Choice B says, “Although there are many competing ideas to explain invasion, it is possible that two of the most important are interrelated: The plant species that benefit the most from high resource availability may also gain the most from escaping enemies upon moving to a new range.” Because this answer choice only tell us about two hypotheses about the success of invasive species, and not about the likelihood of their success in particular areas, we should eliminate this option. Answer Choice C says “Hypotheses explaining the exceptional success of exotic species are based upon ways in which a species’ new range differs from its native range: fewer insects and diseases, less competitive environments, and competitors that are more susceptible to chemicals produced by the invader.” This answer choice goes further than the others in telling us about the factors that contribute to the success of invasive plant species, but because it doesn’t tell us about the specific environments or areas that produce these favorable factors, we should discount against it. Answer Choice D says that, “Plant communities with lots of disturbance, high resource availability, or reduced species diversity tend to be easily colonized.” Because this answer choice tells us clearly about the types of areas that would be likely to be

Page 53: SAT Full Reading Explanations (QAS 9-19) · SAT Full Reading Explanations (QAS 9-19) SAT #9 Reading 1 1.R.9 Answer Choice C is the correct answer because Lines 19-22 say, “Moths

easily colonized, we can draw conclusions about the likelihood of an area to be susceptible to invasive species, and we should thus choose it as our correct answer for Question 44.

This Text, from Lines 32-34, tell us that areas with “lots of disturbance, high resource availability, or reduced species diversity” are the most likely areas to be colonized by an invasive species. This means that areas with the opposite features, (1) little disturbance, (2) low resource availability and (3) plentiful species diversity, would be the least likely to be colonized by an invasive species.

For Question 43, Answer Choice A says that an invasive plant species would be unlikely to colonize “a wetland area that was recently converted to farmland but now commonly experiences flooding and soil erosion.” Because this area would have “lots of disturbance,” we should eliminate this option. Answer Choice B says that an invasive plant species would be unlikely to colonize “a forested area that has numerous species of plants and has received a nearly normal amount of rainfall over the last five years.” Because this area would show little disturbance, normal resource availability and lots of species diversity, it would be unlikely to be colonized, so we should keep it as an option. Answer Choice C says that an invasive plant species would be unlikely to colonize “a previously forested area that experienced a fire within the last year and currently has few species of grasses and herbaceous plants growing.” Because this area would have “lots of disturbance” and “reduced species diversity,” we should eliminate this option. Answer Choice D says that an invasive plant species would be unlikely to colonize “a plains area that has experienced drought over the last seven years and has fewer species of plants than before the drought began.” Because this area would have “low resource availability” and “reduced species diversity,” we should eliminate this option. This leaves us with Answer Choice B as our correct answer for Question 43.

45.R.10

Answer Choice D is the correct answer because in Line 36 the word “supported” most nearly means confirmed or “substantiated,” and can best be replaced by this word. Answer Choice A is incorrect because the studies aren’t “championing” these two mechanisms. Answer Choice B is incorrect because it wouldn’t make sense to say that the study “assisted” the two possible mechanisms. Answer Choice C is incorrect because the studies didn’t brace or physically reinforce the two mechanisms.

● Of primary interest are two mechanisms of invasion that are particularly well championed by existing studies of plant invasions: release from natural enemies and increased resource availability.

● Of primary interest are two mechanisms of invasion that are particularly well assisted by existing studies of plant invasions: release from natural enemies and increased resource availability.

● Of primary interest are two mechanisms of invasion that are particularly well braced by existing studies of plant invasions: release from natural enemies and increased resource availability.

● Of primary interest are two mechanisms of invasion that are particularly well substantiated by existing studies of plant invasions: release from natural enemies and increased resource availability.

Page 54: SAT Full Reading Explanations (QAS 9-19) · SAT Full Reading Explanations (QAS 9-19) SAT #9 Reading 1 1.R.9 Answer Choice C is the correct answer because Lines 19-22 say, “Moths

46.R.10

Answer Choice C is the best answer because Lines 75-79, which say that “the results were not significantly different between the control and experimental plots, indicating that insects were not a determining factor in pinweed’s mechanism of invasion.” This answer choice most clearly provides the “best evidence from Passage 2 that plant growth in Kimball’s experimental plots and control plots was similar over the growing season.” [these are incorrect because].

How to solve this? The Question asks, “Which choice provides the best evidence from Passage 2 that plant growth in Kimball’s experimental plots and control plots was similar over the growing season?” To solve this, we should go through each of the answer choices.

Answer Choice A says, “The experimental plots were sprayed weekly with insecticide to eliminate insects that feed on plants, while control plots were left unsprayed.” All this tells is the difference between the experimental and control plot, and not that their growth was similar, so we should eliminate this option. Answer Choice B says, “At the end of the growing season, Kimball determined, for each plot, the number of each species of plant, the number of fruits on each plant, and the mass of each plant.” Because this only tells us about what Kimball recorded at the end of the growing season and not the actual results being similar, we should eliminate this option as well. Answer Choice C says, “The results were not significantly different between the control and experimental plots, indicating that insects were not a determining factor in pinweed’s mechanism of invasion.” Because this does clearly tells us that the growth for the two plots were similar, or “not significantly different,” we should keep this option. Answer Choice D says that, “She found that the growth rates of the two species were nearly the same in the season (2007-2008) with close to average annual rainfall but that the invasive pinweed plants exhibited a greater growth rate than did the native heronbill plants in the season (2004-2005) when there was much more rainfall than in a typical year.” This answer might seem plausible at first, because it directly states that the “growth rates of the two species were nearly the same,” but we should note that here Kimball is comparing the growth rate of two species in the wild, and not between the experimental and control plots from the previous study. For this reason, then, we should eliminate this option. This leaves us with Answer Choice C, which we should choose as our correct option.

47.R.10

Answer Choice D is the correct answer because Lines 89-91 say, “She also found that the invasive plants lost less water each day through the pores in their leaves than the native plants did regardless of the growing season,” which supports Kimball’s hypothesis about the importance of water to invasive species’ success. Answer Choice A is incorrect because although the Lines do tell us something about the leaf structure in desert plants, the Passage doesn’t take this a topic of focus, therefore it doesn’t use these lines to provide background information about it. Answer Choice B is incorrect because the author of Passage 1 supports the resource hypothesis, as does Kimball, and her claim would support not refute that hypothesis. Answer Choice C is incorrect because the evidence given in Lines 89-91 would support the claim that pinweed plants are overtaking heronbill plants, not refute it.

Page 55: SAT Full Reading Explanations (QAS 9-19) · SAT Full Reading Explanations (QAS 9-19) SAT #9 Reading 1 1.R.9 Answer Choice C is the correct answer because Lines 19-22 say, “Moths

How to solve this? The Question asks, “In Passage 2, the main purpose of the information in lines 89-91 (“She...season”) is to [...]?” To solve this, we should first go back to Lines 89-91 and figure out their importance to the Passage and their immediate context. Lines 89-91 say, “She also found that the invasive plants lost less water each day through the pores in their leaves than the native plants did regardless of the growing season.” This comes in the context of the final paragraph, which describes additional studies Kimball performed that show that pinweed plants can outcompete native plants because of a higher growth rate when water is present and better water conservation. These studies all support her general hypothesis that what allows invasive species to thrive is the better access to and utilization of resources. So, we should say that Lines 89-91 work in favor of her conclusion and offer additional support, although we can’t be sure in advance what angle the SAT will take.

Answer Choice A says that the purpose of the information given is to “provide background information about leaf structure in desert plants.” Because the Passage doesn’t take leaves or leaf structure as a topic of focus, we can’t say that the Lines in question go to provide background information about them, so we should eliminate this option. Answer Choice B says that the purpose of the information is to “refute the claim made by the author of Passage 1 about the resource hypothesis.” Because the views presented in both Passages are in accord, and because the information in the Lines specifically doesn’t refute the author’s claim about the resource hypothesis, we should eliminate this option. Answer Choice C says that the purpose of the information is to “refute the claim presented in Passage 2 that pinweed plants are overtaking heronbill plants in the Sonoran desert.” Because the information given in the Lines supports the author’s claim that pinweed plants are overtaking heronbill plants, we should eliminate this option. Answer Choice D says the information is given to “support the conclusion that water availability is essential to pinweed’s mechanism of invasion,” which does match our understanding of the Text, so we should choose Answer Choice D as our correct answer for Question 47.

48.R.10 (easy)

Answer Choice D is the correct answer because the Question asks about the “relative growth rate in the 2007-2008 season” for the “heronbill plants in Kimball’s study.” Because the dark columns refer to the 2007-2008 season, and because the second set of columns describes the growth rates of the heronbill plants, we should choose the fourth column, or the dark column in the second set. This column goes up to between 0.06 and 0.07 mg of growth per day, which best matches Answer Choice D.

Page 56: SAT Full Reading Explanations (QAS 9-19) · SAT Full Reading Explanations (QAS 9-19) SAT #9 Reading 1 1.R.9 Answer Choice C is the correct answer because Lines 19-22 say, “Moths

49.R.10

Answer Choice A is the correct answer because both Passage 1 and Passage 2 describe hypotheses related to what factors allow invasive plant species to thrive. These discussions both depend on the belief or idea that “competition for the acquisition of space exists between native and nonnative plant species,” because one species will thrive at the expense of another. Answer Choice B is incorrect because, while it’s probably true that both authors for Passage 1 and Passage 2 believe this, neither Passage takes this as a point of concern or presents ideas that explicitly rely on this claim. Answer Choice C is incorrect because neither Passage gives us evidence about the success or unsuccess of “efforts to control the speed of invasive plants in North America have been unsuccessful.” Answer Choice D is incorrect because, while both authors probably agree with the claim, neither take natural disasters like fires or hurricanes as point of focus.

How to solve this? The Question asks, “An idea central to both Passage 1 and Passage 2 is that [...]?” To solve this, we should first have a general understanding of the purpose and content for both Passages, so that we can better eliminate or select answer choices. In general, Passage 1 concerns the fact (1) that invasive species have a large impact, (2) that many competing theories about the success of invasive species exists, (3) that the two most important theories, the resource hypothesis and enemy escape hypothesis, are related, and (5) an explanation of why these hypotheses are related and offer good explanations for the success of invasive species. In general, Passage 2 looks at (1) a biologist attempting to understand why a specific invasive species is taking over a native one, (2) an experiment the biologist performed which demonstrates that that the enemy hypothesis isn’t a factor, and (3) additional studies the biologist performed which shows that water conservation plays an essential role. Both of these Passages, then, concern different hypotheses about the success of invasive species. Now we can go back to the answer choices for the Question, looking in particular for errors in the answer choices.

Answer Choice A says that an idea central to both Passages is that “competition for the acquisition of space exists between native and nonnative plant species.” Because both Passages depend on this competition for land and resources between invasive and native plant species, this answer choice does provide a good example of a central idea relevant to both passages, and we should keep it as an option. Answer Choice B says that an idea central to both Passages is that “a hypothesis should not be tested without the proper use of experimental and control groups.” While this is probably a statement that both Passages would agree with, it doesn’t offer an idea that either explores in depth or on which their main claims depend, so we should eliminate this option. Answer Choice C says that an idea central to both Passages is that “efforts to control the spread of invasive plants in North America have been unsuccessful.” Only Passage 1 briefly suggests that efforts like these have failed, and even it doesn’t conclusively say so, only that its a problem that costs millions of dollars. So, because neither Passage offers strong evidence that this is true nor do they take these efforts as a focus, we should eliminate this option. Answer Choice D says that an idea central to both Passages is that “natural events such as fires and hurricanes can have a devastating effect on plant life.” Although, again, this is a claim that both Passages would likely agree with, neither take this fact as a primary focus or make conclusions that depend on it, so we should eliminate this option as well. This leaves us only with Answer Choice A, which we should choose for our correct answer for Question 49.

Page 57: SAT Full Reading Explanations (QAS 9-19) · SAT Full Reading Explanations (QAS 9-19) SAT #9 Reading 1 1.R.9 Answer Choice C is the correct answer because Lines 19-22 say, “Moths

50.R.10

Answer Choice D is the correct example because Passage 1 discusses generally two hypotheses that explain the success of invasive plant species: the resource hypothesis and the enemy escape hypothesis. Passage 2 discusses the findings about a specific invasive plant species which provides an example of the resource hypothesis at work. Answer Choice D best represents this dynamic, saying that “Passage 2 presents a specific example of the general topic discussed in Passage 1.” Answer Choice A is incorrect because Passage 1 doesn’t discuss a research study. Answer Choice B is incorrect because Passage 1 doesn't present a controversial policy. Answer Choice C is incorrect because Passage 2 supports and doesn’t question the conclusions drawn by Passage 1, when it shows a specific example of the resource hypothesis at work.

How to solve this? The Question asks, “Which choice best states the relationship between the two passages [...]?” To solve this, we should begin by getting a general understanding of the two Passages.

[General breakdown of the two passages, with bullet points. Conclusion that the second is a more specific analysis of issues in the first.]

51.R.10

Answer Choice B is the correct answer because the experiment described in Lines 65-79) tests the effect of insects on the success of invasive plant species by measuring the growth rates of an invasive and native plant species when insects were present versus when they weren’t. This best matches the claim that Passage 1 makes about the enemy release hypothesis, which claims that the success of invasive plant species depends on their “escape from diseases and herbivores,” like insects. This relationship best matches Answer Choice B, that the experiment tested the claim in Passage 1 that “an invasive species’ colonization of a new range is facilitated by having fewer insects that feed on it.” Answer Choice A is incorrect because the experiment doesn’t test the effect of chemicals produced by invasive species — the only chemicals mentioned are the insecticides used to kill insects. Answer Choice C is incorrect because the experiment discussed doesn’t measure the differential success of invasive species in areas with abundant resources — this analysis comes later, in the following paragraph. Answer Choice D is incorrect for the same reason, that the experiment doesn’t these the effect of abundant resources on plant growth.

How to solve this? The Questions ask, “Which claim from Passage 1 about an area colonized by an invasive species was directly tested in the experiment described in the second paragraph of Passage 2 (lines 65-79)?” To solve this, we should have both a general understanding of some of the claims from Passage 1 (see above / Text Analysis) and then a more specific understanding of what claim is tested in Lines 65-79.

Going back to these Lines, we should note that it describes an experiment where Kimball tested two plots of land, one with insects and one without (because it was sprayed with insecticide). She then tested how an invasive plant species and native plant species grew in each plot of land. The experiment ends with Kimball find little difference between the growth rates of either

Page 58: SAT Full Reading Explanations (QAS 9-19) · SAT Full Reading Explanations (QAS 9-19) SAT #9 Reading 1 1.R.9 Answer Choice C is the correct answer because Lines 19-22 say, “Moths

plant, leading her to confirm that insects don’t play a role in the success of this invasive plant species. This experiment matches the claims in Passage 1 about the enemy release hypothesis, so, we should expect our answer choice to relate these two things together.

Answer Choice A says that the experiment in Passage 2 tests the claim in Passage 1 that “native plants are susceptible to chemicals produced by an invasive species.” Because the only chemicals introduced here are the insecticides, and because the experiment doesn’t test the chemicals produced by invasive species, we should eliminate this option. Answer Choice B says that the experiment in Passage 2 tests the claim in Passage 1 that “an invasive species’ colonization of a new range is facilitated by having fewer insects that feed on it.” This does match our understanding of the Text, that the experiment tested the resource hypothesis discussed in Passage 1, so we should keep this option. Answer Choice C says that the experiment in Passage 2 tests the claim in Passage 1 that “fast-growing native plants can effectively colonize areas with abundant resources.” Because the experiment in Passage 2 doesn’t concern the effect of resources on plant growth — that comes in the following paragraph — we should eliminate this option. Answer Choice D says that the experiment in Passage 2 tests the claim in Passage 1 that “high resource availability benefits fast-growing invasive species.” Because, again, the experiment in Passage 2 doesn’t concern the effect of resources on plant growth, we should eliminate this option. That leaves us with Answer Choice B, which we should choose as our correct answer.

52.R.10 (medium)

Answer Choice C is the correct answer because the graph measures the difference in growth for the pinweed and heronbill plants for two separate intervals, from 2004-2005 and from 2007-2008, which we’re told in Lines 82-88 describe times when there was “much more rainfall that in a typical year” and “close to average annual rainfall,” respectively. Because these two years differ in terms of rainfall, it only makes sense that the graph could offer support of the resource hypothesis, since the effect of an “enemy” wouldn’t be measured. Moreover, the resource hypothesis suggests that invasive species when an abundance of resources is made available. The graph supports this hypothesis because in the interval of years when there was an abundance of rainfall, 2004-2005, the invasive pinweed plant species grew at a faster rate. This most closely matches Answer Choice C, which says that the graph supports the resource hypothesis because “the pinweed plants had a greater relative growth rate than the heronbill plants did in a season with greater-than-average rainfall.” Answer Choices A and B are incorrect because the graph only measures a difference in rainfall and so couldn’t support the enemy release hypothesis. Answer Choice D is incorrect because the graph doesn’t measure the mass of the fruits on either of the plants, and because the mass of the fruits on either plant wouldn’t support the resource hypothesis.

How to solve this? [...]

Page 59: SAT Full Reading Explanations (QAS 9-19) · SAT Full Reading Explanations (QAS 9-19) SAT #9 Reading 1 1.R.9 Answer Choice C is the correct answer because Lines 19-22 say, “Moths

SAT #11

Text by Paragraph / Things to Notice

● somebody arrives to the ink shop ● noticing everything in the shop, and its description ● father opens up the ink and ledger ● a memory of precious auntie and her advice ● back from the memory to the ink stick ● pride in her father’s work

● some names ● the setting, the scene, the process ● the attitude and tone (reverential, etc.)

1.R.11 (hard) ****lots of wrong things, it is the character visiting the shop and not old widow lau who is the customer***

Answer Choice D is the correct answer because the Passage depicts Old Widow Lau arriving to the family’s ink shop and the narrator observing her and her father, taking interest in the ink and her father’s words and concluding, “I was very proud to hear Father speak of our family’s ink this way.” This most closely matches Answer Choice D which says, “A character’s visit to her family’s ink shop deepens her appreciation of her family’s work.” Answer Choice A is incorrect because, although the character does remember her Precious Auntie during the visit, this memory doesn’t become the primary focus of the Passage. Answer Choice B is incorrect because, although there is a small reunion between the narrator and Big Uncle and Little Uncle, this reunion doesn’t take center stage in the Passage. Answer Choice C is incorrect because the Passage doesn’t talk about the narrator’s father’s interests.

How to solve this? The Question asks, “Which choice best summarizes the passage?” Because this is a [Big Picture] Question, we should try to have a general understanding of the Passage first, and then try to identify and specific sentences which frame the Passage as a whole. [For the basic breakdown, see above]. Looking for specific texts, we can look at the introductory blurb as well as the introductory paragraph. This Passage doesn’t offer too much in either, which is more typical of literary passages, except for setting up the scene of the Old Widow Lau’s arrival to the ink shop. The concluding sentence, however, says, “I was very proud to hear Father speak of our family’s ink this way,” which does provide something of a summation for the Passage. At this point, we should go through the answer choices for the question, focusing on looking for errors within them.

Answer Choice A says, “A character’s arrival at her family’s ink shop sparks fond memories of her favorite aunt.” The first part of this answer choice is correct, because Old Widow Lau does arrive at the family’s ink shop. The second part might also seem correct, because Lines 46-59 show the narrator remembering her Precious Auntie, so we can keep this option. Answer Choice B says that “a character’s surprise visit leads to a happy reunion at her family’s ink shop.” This answer choice might also seem correct at first, because it describes a visitor coming to the shop like the answer choices, and because something like a reunion occurs in the first paragraph,

Page 60: SAT Full Reading Explanations (QAS 9-19) · SAT Full Reading Explanations (QAS 9-19) SAT #9 Reading 1 1.R.9 Answer Choice C is the correct answer because Lines 19-22 say, “Moths

when the narrator reunites with Big and Little Uncle. However, we should note that Old Widow Lau’s arrival is never presented as a “surprise visit,” and that the reunion between the Uncles and the narrators doesn’t have a primary place in the Passage, so we can eliminate this option. Answer Choice C says that “a character comes to understand her father’s ambitions while visiting her family’s ink shop.” Because the Passage doesn’t discuss her father’s ambitions in the Passage, we should eliminate this option. Answer Choice D says that “a character’s visit to her family’s ink shop deepens her appreciate of her family’s work.” Because the Passage does end with a mention of the narrator’s appreciation for her family work following Old Widow Lau’s visit, we should keep this option.

That leaves us with Answer Choice A and Answer Choice D. Because the beginnings of these answers choices are identical, we should focus on the second part and decide if the Passage sparks fond memories of the narrator’s favorite aunt or deepens her appreciation of her family’s work. To solve this, we should note that although Paragraph 4 does show the narrator remembering her Precious Auntie, the Passage doesn’t make this its primary focus and presents it as an aside. In Lines 60-64, the Passage transitions away from this memory when the narrator says, “I remembered this, and yet that day in the ink shop, I listened to what Father was saying, and his words became far more important than anything Precious Auntie had thought.” After this transition, the Passage ends with, “I was very proud to hear Father speak of our family’s ink this way,” giving us a more central summary of the Passage. Thus, we should choose Answer Choice A for our correct answer. [Plus, a more general answer choice].

2.R.11

Answer Choice B is the correct answer because the Passage does present the theme that “quality is achieved through deliberate effort,” when, for example, in Lines 50-51, the narrator remember Precious Auntie saying, “You can never be an artist if your work comes without effort.” Answer Choice A is incorrect because, although the Passage does present family relationships, it never expressly focuses on the need to nurture these. Answer Choice C is incorrect because the Passage doesn’t discuss how hard work leads to material compensation (and because of rhetorical flair). Answer Choice D is incorrect because the Passage also doesn’t discuss how “creativity needs to be expressed concretely.”

How to solve this? The Question asks, “A main theme of the passage is that [...]?” To solve this, we should first have a general understanding and memory of the Passage, and then work through the answer choices, finding errors and eliminating those options.

Answer Choice A says that “family relationships should be nurtured.” Either by memory, or by searching through the Passage, we can look for evidence in the Passage for this idea. After doing so, we should note that no such evidence exists. Answer Choice B says that “quality is achieved through deliberate effort,” which is supported by Lines 50-51 and subsequent lines in the paragraph, so we should keep this option. Going through Answer Choices C and D, we should also look for supporting evidence either by memory or by scanning, and note that it doesn’t exist, so we should eliminate these options. That leaves Answer Choice B which we should choose as our correct answer.

Page 61: SAT Full Reading Explanations (QAS 9-19) · SAT Full Reading Explanations (QAS 9-19) SAT #9 Reading 1 1.R.9 Answer Choice C is the correct answer because Lines 19-22 say, “Moths

3.R.11

Answer Choice B is the correct answer because the Passage shows us multiple instances where the narrator shows that she’s “attuned to her immediate surroundings.” For example, in Lines 22-23 she says, “I tried to notice everything so I could later tell GaoLing what I had seen [...],” and in Lines 63-64, “‘Look here,’ Father said to his customer, and I looked.” Answer Choices A, B and C are incorrect because no evidence in the Passage shows the narrator being “reserved around unfamiliar people,” “sympathetic to the needs of others,” or “anxious about her responsibilities.”

How to solve this? The Question asks, “Throughout the passage, the narrator is portrayed as someone who is [...]?” To solve this, we should just work with our basic memory of the Passage and the narrator’s portrayal in it, and then focus on eliminating options and finding support for others.

Answer Choice A says that the narrator is portrayed as someone who is, “reserved around unfamiliar people.” We should recognize this option as unlikely because it has some [pejorative flair] and makes a strong claim that we should expect to have noticed. Moreover, if we go back to Passage, we find no evidence in support of this. Answer Choice B says that the narrator is portrayed as someone who is “attuned to her immediate surroundings.” Because this answer choice is neutral in both meaning and tone, and because sufficient evidence for this portrayal of the narrator runs throughout the Passage, we should keep this option. Answer Choice C says that the narrator is “sympathetic to the needs of others.” This answer choice is also fairly neutral, and therefore plausible. But, looking back over the Passage, we should note that no evidence exists for it, so we should eliminate this option. Answer Choice D says that the narrator is portrayed as someone who is, “anxious about her responsibilities.” This answer choice is both “loud” and somewhat critical, so we should consider it unlikely. Moreover, we should note that no evidence exists for the answer choice so we should eliminate this option.

4.R.11 & 5.R.11

Answer Choices A and C are the correct answers because Lines 15-18 say, “Old Widow Lau refused their invitation three times, exclaiming that my father and uncles must be too busy for visitors. She made weak efforts to leave,” which most closely matches Answer Choice A, that Old Widow Lau’s reluctance was “feigned, because she is not genuinely firm in her resolve.” For Question 5, Answer Choices A, B and D are all incorrect because neither give us evidence about Old Widow Lau’s reluctance to stay for tea. For Question 4, Answer Choices B and D are incorrect because the Passage doesn’t tell us that the family has been planning the visit or that Old Widow Lau is exhausted. Answer Choice C is incorrect, because the store only has one customer and isn’t described as being unusually busy.

How to solve this? Question 4 asks, “It can be most reasonably inferred from the passage that Old Widow Lau’s reluctance to stay for tea is [...]?” Because this is a Paired Question, to solve it we should first go through the answer choices for Question 5, looking for any Text that tells us about Old Widow Lau’s reluctance to stay for tea. Because this is an earlier question in the series of questions, we should expect our lines to come earlier in the Passage and, [using the

Page 62: SAT Full Reading Explanations (QAS 9-19) · SAT Full Reading Explanations (QAS 9-19) SAT #9 Reading 1 1.R.9 Answer Choice C is the correct answer because Lines 19-22 say, “Moths

Question Order Rule], before Lines 46. But since all of the texts fit these criteria, this won’t help us too much.

For Question 5, Answer Choice A says, “At last, Old Widow Lau was done haggling with the driver and we stepped inside Father’s shop. It was north-facing, quite dim inside, and perhaps this was why Father did not see us at first.” Because this answer choice doesn’t tell us anything about the Old Widow Lau’s reluctance to take tea, we should eliminate this option. Answer Choice B says, “So I called his name in a shy voice. And he squinted at me, then laughed and announced our arrival to Little Uncle, who apologized many times for not rushing over sooner to greet us.” This answer choice also doesn’t tell us about her reluctance, so we should eliminate this answer choice. Answer Choice C says, “Old Widow Lau refused their invitation three times, exclaiming that my father and uncles must be too busy for visitors. She made weak efforts to leave.” Because this answer choice does tell us about her reluctance to take tea, we should keep this option. Answer Choice D says, “Then Little Uncle brought us hot tea and sweet oranges, as well as bamboo latticework fans with which to cool ourselves.” Because this option comes after Old Widow Lau’s initial reluctance and shows her actually sitting down for tea, we should eliminate this option. This leaves us with Answer Choice C, which we should choose for our correct answer.

Lines 15-18 say, “Old Widow Lau refused their invitation three times, exclaiming that my father and uncles must be too busy for visitors. She made weak efforts to leave.” We should recognize that her exclamation that the “father and uncles must be too busy” doesn’t match her “weak efforts to leave,” and therefore that her reluctance is not entirely genuine. So, we should expect our answer choice to relate to this.

For Question 4, Answer Choice A says that her reluctance was, “feigned, because she is not genuinely firm in her resolve.” This does match our understanding of the Text, that Old Widow Lau didn’t mean what she said. Answer Choice B says that her reluctance was, “inconsiderate, because the family has been planning her visit.” Because this answer choice has some [pejorative flair], we shouldn’t expect it to be our answer. Moreover, the Passage gives us no evidence that the family “has been planning her visit,” nor does our specific Text present her as being inconsiderate, so we should eliminate this option. Answer Choice C says that her reluctance is “appropriate, because the shop is unusually busy.” Because the Passage doesn’t tell us that the shop is especially busy, but only that it has one customer, we should eliminate this option. Answer Choice D says that her reluctance is “ill-advised, because she is exhausted from the journey.” Because the Text doesn’t tell us that Old Widow Lau is exhausted, we should eliminate this option. That leaves only Answer Choice A, which we should choose as our correct answer.

Page 63: SAT Full Reading Explanations (QAS 9-19) · SAT Full Reading Explanations (QAS 9-19) SAT #9 Reading 1 1.R.9 Answer Choice C is the correct answer because Lines 19-22 say, “Moths

6.R.11

Answer Choice A is the correct answer because Paragraph 2 describes the physical details of the father’s shop, focusing specifically on the glass display cases, of which the narrator makes the comment, “They looked so much nicer than they had in the ink-making studio at Immortal Heart village.” This most closely matches Answer Choice A, which says that, compared to the ink-making studio at Immortal Heart village, the narrator’s family ink shop “displays the family’s ink more impressively.” Answer Choices B and C are incorrect because the Passage doesn’t describe the family’s ink shop as being “more conveniently located for the public” or as providing “greater individual attention to customers.” Answer Choice D is incorrect because the narrator comments on the quality and appearance of the display, not on the size of the space.

How to solve this? The Question asks, “The narrator indicates that the contrast between the ink-making studio at Immortal Heart village and her family’s ink shop is that the ink shop [...]?” To solve this, we should first go back to the Passage and look for what the narrator says about her family’s ink shop. The [Question Order Rule] tells us that we should expect our Text to come before Lines 46, and we can begin scanning in that area for “Immortal Heart Village.”

Lines 30-32 say, “They looked so much nicer than they had in the ink-making studio at Immortal Heart village.” Checking our references, we should note that the “they” mentioned here refers to the glass display cases that the narrator admires. Thus, we see that the narrator finds the display cases at her father’s ink shop to be look better than the ones from the studio at Immortal Heart village, and we should expect our answer choice to refer to this.

Answer Choice A says that thinks that her family’s ink shop “displays the family’s ink more impressively.” This does match our understanding of the Text, so we should keep this option. Answer Choice B says that the family’s ink shop “is more conveniently located for the public,” which neither the Passage nor our Text mention, so we should eliminate this option. Answer Choice C says that the family’s ink shop “provides greater individual attention to customers,” which we should also eliminate because neither our Text nor our Passage mentions it. Answer Choice D says that the family’s ink shop “offers a larger space for presenting products,” which is also incorrect because the narrator highlights the appearance of the display cases and not the size of the space. This leaves us with only Answer Choice A, which we should choose as our correct option.

Page 64: SAT Full Reading Explanations (QAS 9-19) · SAT Full Reading Explanations (QAS 9-19) SAT #9 Reading 1 1.R.9 Answer Choice C is the correct answer because Lines 19-22 say, “Moths

7.R.11 & 8.R.11

Answer Choices C and C are the correct answers because Lines 52-55 say, “You simply write what is swimming on the top of your brain. And the top is nothing but pond scum, dead leaves, and mosquito spawn,” which best answers how “Precious Auntie would consider a hastily written first draft of a story to be.” For Question 7, Answer Choice C most closely matches this Text when it says Precious Auntie would consider such a story, “essentially worthless in and of itself.” For Question 8, Answer Choice A is incorrect because it only tells us that Precious Auntie believes everything has a purpose, not how she would consider a hastily written first draft. Answer Choice B is incorrect because it only comments on the qualities of “good ink,” not how she would consider a first draft. Answer Choice D is incorrect because it details the importance, thought process and benefit of using an inkstick and inkstone, not how she would consider a first draft.

How to solve this? Question 7 asks, “Based on the artistic philosophy expressed in the fourth paragraph (lines 46-59), it is reasonable to infer that Precious Auntie would consider a hastily written first draft of a story to be [...]?” Because this is a Paired Question, we should begin by going through the answer choices for Question 8 and looking for any texts that can answer how Precious Auntie would consider such a draft. Our attention should be drawn especially to the words “hastily written,” which tell us that our answer will probably have something to do with a hurried, poorly completed work of art.

For Question 8, Answer Choice A says, “As he said this, Precious Auntie came back into mind. I was remembering how she taught me that everything, even ink, had a purpose and a meaning.” Because this answer choice doesn’t tell us anything about how Precious Auntie would consider a “hastily, written first draft,” we should eliminate this option. Answer Choice B says, “Good ink cannot be the quick kind, ready to pour out of a bottle.” Because this answer choice only comments on the quality of ink and not a quickly written draft, we should eliminate this option. Answer Choice C says, “You simply write what is swimming on the top of your brain. And the top is nothing but pond scum, dead leaves, and mosquito spawn.” Expanding the reference a little, we see that here Precious Auntie is discussing the “quick kind” of “modern ink,” and contrasting it the slower kind of ink. Because these lines reference a written first draft as well as the speed with which it’s written, we should keep this option. Answer Choice D says, “You push and you ask yourself, What are my intentions? What is in my heart that matches my mind?” Because this answer choice refers to the thoughts that follow the use of an old version of ink, involving the inkstick and inkstone, and the slowness of that ink, we should eliminate this option. That leaves us with Answer Choice C, which we should for our correct answer for Question 8.

Answer Choice C says, “You simply write what is swimming on the top of your brain. And the top is nothing but pond scum, dead leaves, and mosquito spawn.” Taking this as our evidence for how Precious Auntie would feel about a “hastily written first draft,” we can say that she would take a strongly negative stance towards one, considering it “pond scum, dead leaves and mosquito spawn.” For Question 7, Answer Choice A says that Precious Auntie would consider a draft like this to be, “emotionally raw and powerful.” The word “raw” might be the only plausible thing about this answer choice, but because the Text doesn’t tell us anything about a draft being emotional and powerful, and because it isn’t negative enough, we should eliminate this option. Answer Choice

Page 65: SAT Full Reading Explanations (QAS 9-19) · SAT Full Reading Explanations (QAS 9-19) SAT #9 Reading 1 1.R.9 Answer Choice C is the correct answer because Lines 19-22 say, “Moths

B says that she would consider such a draft to be, “creatively satisfying for the author.” This answer choice is also not negative enough, nor does our Text talk about the creative satisfaction of the author, so we should eliminate this option as well. Answer Choice C says she would consider such a draft to be “essentially worthless in and of itself.” Because this answer choice is both negative and captures the Precious Auntie’s view of the work as “worthless” or “nothing but pond scum,” we should keep this option. Answer Choice D says that Precious Auntie would consider such a draft to be “inappropriately analytical for a piece of art.” Because our Text doesn’t mention the analytical nature of such a draft, and because this option isn’t negative enough, we should eliminate it. This leaves us with answer Choice C, which we should choose as our correct answer for Question 8.

9.R.11

Answer Choice B is the correct answer because the word “matches” in Line 59 can be best replaced by “corresponds with,” emphasizing the [...] as opposed to [...].

● What is in my heart that matches my mind? ● What is in my heart that corresponds with my mind?

Answer Choice A is incorrect, because the line isn’t emphasizing that the heart and mind are competing with each other.

● What is in my heart that competes against my mind?

Answer Choice B is incorrect, because the line isn’t emphasizing that [...]

● What is in my heart that runs counter to my mind? ● What is in my heart that treats equally my mind?

10.R.11

Answer Choice C is correct because Line 68 emphasizes the quality of the ring of the bell, and especially the purity and distinctness of the tone, making the word “clean” best replaced by the word “distinct.”

● And I heard a sound as clean and pure as a small silver bell. ● And I heard a sound as distinct and pure as a small silver bell.

Answer Choice A is incorrect because it doesn’t make sense to describe the sound of the bell as “complete,” as opposed to “incomplete.” Answer Choice B is incorrect because it also doesn’t make sense to call the sound of the bell “skillful.” Answer Choice D is incorrect because the line isn’t emphasizing the “upright” or “proper” sound of the bell.

Page 66: SAT Full Reading Explanations (QAS 9-19) · SAT Full Reading Explanations (QAS 9-19) SAT #9 Reading 1 1.R.9 Answer Choice C is the correct answer because Lines 19-22 say, “Moths

Reading 2

Passage

● a study show how technology changes the way our memories function ● it shows how people remember fewer facts and less information ● internet has become part of our “transactive memory” source and part of daily habits ● the four experiments in the study ● questions about whether or not this is negative ● how memory has changed what we remember, and future studies of interdependence

between computers and humans

11.R.11 (hard)

Answer Choice D is the correct answer because the Passage can best be described as tracking a particular study that examines how the Internet affects memory, as the title itself suggests, “How the Web Affects Memory.” Answer Choice A is incorrect because the Passage looks at memory and not “critical thinking.” Answer Choice B is incorrect because, although the Passage does suggest some concern over the negative effects of humans’ dependence on computers, it never makes so strong a claim as to assert that they have become “overly dependent.” Answer Choice C is incorrect because the Passage doesn’t give enough evidence to support to strong claim that people’s capacity for memory is “much weaker” than it used to be and because it concludes with a statement that memory is changing, not necessarily becoming weaker.

How to solve this? The Questions asks, “The main purpose of the passage is to [...]?” To solve this, we should both have a general understanding of the Passage as a whole and also find specific sentences that tell us what it’s about. See [above / prior] for a general outline of the Passage. Specifically, we should begin by looking at the subtitle for the Passage which reads, “How the Web Affects Memory,” which gives us a pretty good indication of what the Passage is about. Next, we should look at the first paragraph, which introduces Wegner’s study and tells us that it is “changing the way our memories function.” Finally, we can look at the conclusion, which tells us that “even though we may not be taxing our memories to recall distinct fact, we are still using them to consider where the facts are located and how to access them.” Taking all these together, we can say that the Passage primarily concerns a single study that details how the Internet affects and changes our memories, without making strong claims about the negative nature of this impact, and should expect our answer choice to concern this.

Answer Choice A says that the purpose of the passage is to “describe a series of experiments on the way technology interferes with critical thinking.” This answer choice might seem initially plausible, but because the Passage concerns the effect of technology on memory and not “critical thinking,” we should eliminate this option. Answer Choice B says that the purpose of the passage is to “assert that people have become overly dependent on computers for storing information.” This answer choice might also seem plausible because, at first, the Passage seems to suggest that technology affects our memory negatively. But, we should note both the strong and judgmental nature of the phrase “overly dependent,” and look for more direct evidence for the claim. Towards the end of the Passage we see that the author of the study

Page 67: SAT Full Reading Explanations (QAS 9-19) · SAT Full Reading Explanations (QAS 9-19) SAT #9 Reading 1 1.R.9 Answer Choice C is the correct answer because Lines 19-22 say, “Moths

finds that technology has primarily caused memory to change and “concedes that questions remain about whether dependence on computers will affect memories negatively.” Because of this, we should eliminate this answer choice. Answer Choice C says that the purpose of the passage is to “discuss the idea that humans’ capacity for memory is much weaker than it once was.” This answer choice is somewhat similar to the previous choice, in that it both presents a strong claim about the effect of technology on humans’ capacity for memory, and because it presents that effect as explicitly negative. These claims however, don’t find support in the Passage, which never claims that human memory is “much weaker” or takes a negative stance towards the effect of technology on human memory, so we should eliminate this option. Answer Choice D says that the purpose of the passage is to “share the findings of a study examining the effect of computer use on memory recall.” Because this answer choice matches our understanding of the Passage and because it presents a neutral stance towards the effect of technology on memory (simply “examining the effect”), we should keep this option. That leaves us only with Answer Choice D, which we should choose as our correct answer.

12.R.11

Answer Choice D is the correct answer because Lines 72-75 say that “even though we may not be taxing our memories to recall distinct facts, we are still using them to consider where the facts are located and how to access them,” which most clearly tells us “that reliance on computers does not necessarily diminish human memory.” Answer Choices A, B and C are incorrect because the lines all present [negative instances] of the effect of computers on memory without adding the claim that, in some cases, computers don’t diminish human memory.

How to solve this? The Question asks, “Which choice best supports the idea that reliance on computers does not necessarily diminish human memory [...]?” [Because the Passage mostly takes a suggestively negative view towards the effect of technology on human memory, we should expect that any claim about computers not diminishing human memory should be presented as a qualification, or counter [Qualification Rule]].

Answer Choice A says, “But Harvard professor of psychology Daniel Wegner’s recent research proves that websites — and the Internet — are changing much more than technology itself.” Because this answer choice doesn’t tell us the [exception, qualification] of the effect of technology on memory, but only introduces the study that suggests that technology changes memory, we should eliminate this option. Answer Choice B says that, “Wegner points out that we never have to stretch our memories too far to remember the name of an obscure movie actor or the capital of Kyrgyzstan [...].” Because this answer choice only tells us a negative effect of technology on memory and not an [exception, qualification], we should eliminate this option as well. Answer Choice C says, “Students who have trouble remembering distinct facts, for example, may struggle to employ those facts in critical thinking.” This also presents a suggestively negative effect of technology on memory and not an [exception, qualification], so we should eliminate this option as well. Answer Choice D says that “even though we may not be taxing our memories to recall distinct facts, we are still using them to consider where the facts are located and how to access them.” This answer choice does give us something like an [exception, qualification], and tells us clearly that “reliance on computers does not necessarily

Page 68: SAT Full Reading Explanations (QAS 9-19) · SAT Full Reading Explanations (QAS 9-19) SAT #9 Reading 1 1.R.9 Answer Choice C is the correct answer because Lines 19-22 say, “Moths

diminish human memory,” so we should choose keep this option and choose Answer Choice D as our correct answer.

13.R.11

Answer Choice D is the correct answer because Lines 19-21 introduce the concept of “transactive memory” and give the example, “as when a husband relies on his wife to remember a relative’s birthday.” This most closely matches Answer Choice D which says, “illustrate the concept of a transactive memory source using a familiar situation.” Answer Choice A is incorrect because the lines don’t tell us that “people who are closely related tend to have shared memories.” Answer Choice B is incorrect because the lines don’t talk about the development of external source sof memory. And Answer Choice C is incorrect because the lines give an example of transactive memory sources, and don’t emphasize the effectiveness and accuracy of these sources.

How to solve this? The Question asks, “In context, the reference to remembering a relative’s birthday mainly serves to [...]?” To solve this, we should go back to the mention in the Passage of remembering a relative’s birthday and look at its function in its surrounding context. To find this Text, we can use the [Question Order Rule] and predict that it will come earlier in the Passage and probably before Line 26. Moreover we can scan for the words “relative” and “birthday.”

In Lines 19-21 we find, “First hypothesized by Wegner in 1985, transactive memory exists in many forms, as when a husband relies on his wife to remember a relative’s birthday.” The introduction of this mention of remembering a relative’s birthday, beginning with “as,” shows that the mention is used as an example of “transactive memory,” and we should expect our answer choice to relate to this. Answer Choice A says that the reference serves to “show that people who are closely related tend to have shared memories.” Because this doesn't’ capture our understanding of the Text, which doesn’t say that relatives have shared memories, we should eliminate this option. Answer Choice B says that the reference serves to “demonstrate how people initially developed external sources of memory,” which also doesn’t match our understanding of the Text, so we should eliminate this option. Answerr Choice C says that the reference serves to “emphasize the effectiveness and accuracy of transactive memory sources,” which also doesn’t match our understanding of the Text, so we should eliminate this choice as well. Answer Choice D says that the reference serves to “illustrate the concept of a transactive memory source using a familiar situation.” Because this does capture our understanding of the Text, that it serves as an example of the concept of a transactive memory source, we should keep this option. That leaves us only with Answer Choice D which we should choose as our answer choice.

Page 69: SAT Full Reading Explanations (QAS 9-19) · SAT Full Reading Explanations (QAS 9-19) SAT #9 Reading 1 1.R.9 Answer Choice C is the correct answer because Lines 19-22 say, “Moths

14.R.11

a shitty fuck question and you should ignore it (no good way to decide between B and C)

15.R.11

Answer Choice B is the correct answer because Lines 25-26 tell us that computers and technology have now become parts of our memory or “extensions of” it, which most closely matches “additions to” and can be replaced by it.

● Now computers and technology as well are becoming virtual extensions of our memory. ● Now computers and technology as well are becoming virtual additions to our memory.

Answer Choice A is incorrect because the Lines don’t tell us that computers and technology delay our memory. Answer Choice C is incorrect because it’s unclear how computers and technology could become “lengths of our memory,” and doesn’t preserve the meaning we want. Answer Choice D is incorrect because the Lines don’t mean that computers and technology develop our memory, just that they’re further parts of it.

16.R.11 & 17.R.11

Answer Choices C and A are the correct answers because Lines 42-45 say that, “In the first experiment, participants demonstrated that they were more likely to think of computer terms like ‘Yahoo’ or ‘Google’ after being asked a set of difficult trivia questions.” This most closely matches Answer Choice C, which says “people are inclined to think of specific information sources” when they are “asked to provide facts that are not already familiar to them.” For Question 17, Answer Choice B is incorrect because it only provides the context for an experiment, and doesn’t show people thinking of specific information sources. Answer Choice C is incorrect because it only provides a result of the experiment involving participants remembering statements, and not thinking of a specific information source. Answer Choice D is incorrect because the mentions of ‘Yahoo’ and ‘Google’ qualify as better instances of “specific information sources” than the “folder locations” mentioned in these lines. For Question 16, Answer Choice A is incorrect because the participants think of Yahoo and Google when being asked trivia questions, not told to “memorize details that will then be made inaccessible.” Answer Choice B is incorrect, because the participants in the Texts are not “directed to develop a system for organizing and saving content.” Answer Choice D is incorrect because the participants are also not “prompted to identify terms related to dependence on computers.”

Page 70: SAT Full Reading Explanations (QAS 9-19) · SAT Full Reading Explanations (QAS 9-19) SAT #9 Reading 1 1.R.9 Answer Choice C is the correct answer because Lines 19-22 say, “Moths

How to solve this? Question 16 asks, “The discussion of the experiments suggests that people are inclined to think of specific information sources in response to being [...]?” Because this is a Paired Question, to solve this we should go through the answer choices for Question 17, focusing on finding texts that show us people thinking of “specific information sources.”

Answer Choice A says, “In the first experiment, participants demonstrated that they were more likely to think of computer terms like ‘Yahoo’ or ‘Google’ after being asked a set of difficult trivia questions.” Because this answer choice does show people thinking of specific information sources, namely Yahoo and Google, we should keep this option. Answer Choice B says, “Half the subjects were told that their work would be saved to a computer; the other half were informed that the statements would be erased.” Because this only sets up the experiment to follow and doesn’t show participants thinking of specific information sources, we should eliminate this option. Answer Choice C says that, “In subsequent memory testing, participants who were told their work would not be saved were best at recalling the statements.” Because this answer choice shows individuals recalling statements themselves and not particular memory sources, we should eliminate this option. Answer Choice D says that, “the participants proved better able to recall the folder locations than the statements themselves.” This answer choice might seem plausible, because it shows people thinking of a particular information source, that of the folders. However, because this answer choice doesn’t mention very “specific” information sources, especially compared to the Yahoo and Google of Answer Choice A, we can eliminate this option. This leaves us with Answer Choice A, which we should choose as our correct answer for Question 17.

These lines say, “In the first experiment, participants demonstrated that they were more likely to think of computer terms like ‘Yahoo’ or ‘Google’ after being asked a set of difficult trivia questions.” So, we see people thinking of “specific information sources” in response to being asked a “set of difficult trivia questions,” or questions with answers unknown to the participants, so we should expect our answer choice to relate to this.

For Question 16, Answer Choice A says that the experiments show people thinking of specific information sources in response to being “required to memorize details that will then be made inaccessible.” Because our Text shows participants thinking of these sources in response to being asked trivia questions and not memorizing details, we should eliminate this answer choice. Answer Choice B says that the experiments show people thinking of specific information sources in response to being “directed to develop a system for organizing and saving content.” Because our Text doesn’t show participants being asked to develop an organizing system, we should eliminate this option. Answer Choice C says that the experiments show people thinking of specific information sources in response to being “asked to provide facts that are not already familiar to them.” Because this answer choice does match our Text, and “a set of difficult trivia questions,” we should keep this option. Answer Choice D says that the experiments show people thinking of specific information sources in response to being “prompted to identify terms related to dependence on computers.” Because the participants are asked to identify terms related to computers, but difficult trivia questions, we should eliminate this option as well. That leaves us with Answer Choice C, which we should choose as our correct answer for Question 16.

Page 71: SAT Full Reading Explanations (QAS 9-19) · SAT Full Reading Explanations (QAS 9-19) SAT #9 Reading 1 1.R.9 Answer Choice C is the correct answer because Lines 19-22 say, “Moths

18.R.11

Answer Choice A is the correct answer because Lines 66-68 say that students who can’t remember distinct facts will have difficulty using or employing those facts in critical thinking, and because “employ” can best be replaced by “utilize.”

● Students who have trouble remembering distinct facts, for example, may struggle to employ those facts in critical thinking.

● Students who have trouble remembering distinct facts, for example, may struggle to utilize those facts in critical thinking.

Answer Choice B is incorrect because the line doesn’t mean that students will “enroll” facts into critical thinking, like into a school. Answer Choice C is incorrect because the students aren’t said to “exert” facts in critical thinking, which implies a kind of inappropriate force or struggle. Answer Choice D is incorrect because it’s unclear what it would mean for students to “assign” facts in critical thinking.

19.R.11

Answer Choice C is the correct answer because the first column in the series of columns gives data for the participants who could remember “statements and folder locations,” or “both parts of the information given to them.” This column raises to between 15 and 20%, which most closely matches Answer Choice C which says 17%. Answer Choices A, B and D are incorrect because they don’t accurately measure the first column in the graph.

How to solve this? The Question asks, “According to the graph, approximately what percentage of participants remembered both parts of the information given to them during the fourth experiment?” To solve this, we should go back to graph and try to determine which part of it measures the percentage of participants who remembered both of the information given to them.

Along the bottom of the graph, we notice four labels for each of the four columns: (1) remember statements and folder locations, (2) remember statements not folder locations, (3) remember folder locations not statements and (4) remember nothing. Only the first column shows those who remembered “both parts of the information given to them,” so we should find the percentage for this column. It rises to about halfway between 15 and 20%, which most closely matches Answer Choice C, which we should choose as our correct answer.

Page 72: SAT Full Reading Explanations (QAS 9-19) · SAT Full Reading Explanations (QAS 9-19) SAT #9 Reading 1 1.R.9 Answer Choice C is the correct answer because Lines 19-22 say, “Moths

20.R.11

Answer Choice D is the correct answer because the largest single group of participants represented in the graph refer to those who “remember nothing,” and, in the discussion of the four experiments Wegner performed (Paragraph 5), there’s no mention of participants who remembered nothing. Because we have no evidence about this group, Answer Choice D, “there is not enough information to determine the cause of the results for those participants,” makes the most sense. Answer Choices A, B and C are incorrect because, while they may be plausible explanations for the group represented, we have no evidence in the Passage to support one over the other.

How to solve this? The Question asks, “Based on the description of Wegner’s fourth experiment, what is the most likely explanation for the findings for the largest single group of participants represented in the graph?” To solve this, we should first go back to the graph, find the largest group represented, and then determine what the Passage says about it.

Going back to the graph, we see that the column for the group who “remembered nothing” is the largest single group represented. Because the Question asks specifically about Wegner’s fourth experiment, we should go back to the Passage to the paragraph where these experiments are described (Paragraph 5). In this paragraph, however, and even the following discussion about conclusions, we see that nothing is mentioned about the group who “remembered nothing,” which means that we can’t make any claims about them and should expect our answer choice to reference this.

Answer Choice A says that the participants in the group “focused on remembering the folder locations,” which is incorrect because we have no evidence to support it. Answer Choice B is incorrect because we also have no evidence that the participants in the group “attempted to remember the statements and folder locations.” Answer Choice C is also incorrect because, although it might be plausible, there’s no evidence that the participants in the group “did not attempt to remember any specific pieces of information.” Answer Choice D says that “there is not enough information to determine the cause of the results for those participants,” which most closely matches our own answer, so we should choose Answer Choice D as our correct answer for this question.

Page 73: SAT Full Reading Explanations (QAS 9-19) · SAT Full Reading Explanations (QAS 9-19) SAT #9 Reading 1 1.R.9 Answer Choice C is the correct answer because Lines 19-22 say, “Moths

Reading 3

Passage Outline

● why guppies are good for studying evolution, and Reznick’s experiment (Paragraph One)

● two types of guppy environment: predatory and not (Paragraph Two) ● different traits between the guppies in each environment (Paragraph Three) ● the experiment to show how quickly these traits could have evolved (Paragraph Four) ● results of experiment and other mentions of quick evolution (Paragraph Five)

21.R.11

Answer Choice A is the correct answer because Lines 3-5, in the first paragraph, say that the rate of guppy reproduction “makes guppies ideally suited for studying the rate of evolution,” and because the Passage continues to discuss observational experiments of guppies. Answer Choice B is incorrect because the paragraph doesn’t mention the value of studying guppy offspring “shortly after birth,” and because the later Passage doesn’t emphasize it either. Answer Choice C is incorrect because the first paragraph doesn’t introduce a theory, and especially not one “at the center of an ongoing scientific debate.” Answer Choice D is incorrect because the first paragraph doesn’t talk about a “new field of scientific inquiry.”

How to solve this? The Question asks, “The first paragraph mainly serves to [...]?” To solve this, we should have both a general understanding of the Passage and mores specific understanding of the paragraph works in its immediate context. [See above for the general outline of the Passage].

The first paragraph mentions (1) the reproductive rate of guppies, (2) why this makes them “suited for studying the rate of evolution,” and (3) Reznick’s study which makes use of this fact. Because the Passage goes on to look at an experiment performed by Reznick which relies on guppies’ rapid rate of reproduction, we can say the first paragraph serves to give us a reason why these guppies make a good topic of study, and should expect our answer choice to reference this.

Answer Choice A says that the first paragraph serves to “establish the reason why a certain species was selected for scientific observation.” Because this matches our own understanding of the paragraph, that it gives us a reason why guppies were chosen for study, we should keep this option. Answer Choice B says that the first paragraph serves to “illustrate the value of studying the offspring of a particular animal shortly after birth.” This answer choice might seem plausible, because it mentions the value of studying a particular species. However, because the paragraph doesn’t mention the specific value of studying these animals shortly after birth and because the experiments don’t show Reznick doing so, we should eliminate this option. Answer Choice C says that the first paragraph serves to “introduce a theory at the center of an ongoing scientific debate.” Because the first paragraph doesn’t introduce a theory, and especially not one at the center of an ongoing scientific debate, we should eliminate this option. Answer Choice D says that the first paragraph serves to “offer a rationale for the prevalence of a new

Page 74: SAT Full Reading Explanations (QAS 9-19) · SAT Full Reading Explanations (QAS 9-19) SAT #9 Reading 1 1.R.9 Answer Choice C is the correct answer because Lines 19-22 say, “Moths

field of scientific inquiry.” However, because the first paragraph doesn’t mention a “new field of scientific inquiry,” or its prevalence, we should eliminate this option. That leaves us with Answer Choice A, which we should choose as our correct option.

22.R.11

Answer Choice B is the correct answer because Lines 14-16 describe a “lucky guppy” as being “born above a waterfall or a set of rapids, which keep out the predatory fish called pike cichlids found in calmer downstream waters.” This most closely matches Answer Choice B, which says that a “lucky guppy” is one that “inhabits an environment that provides natural protection from predators.” Answer Choice A is incorrect, because the lines don’t establish that a guppy is lucky because it’s born in a river with an “established guppy population.” Answer Choice C is incorrect because the lines emphasize that a guppy is lucky for living apart from predators, not for managing the risks of living near a waterfall. Answer Choice D is incorrect because, while it is true that a guppy is lucky for avoiding predators, these lucky guppies live above a waterfall and not in downstream waters, where in fact more predators live.

How to solve this? The Question asks, “In describing the living conditions of guppies, the author indicates that a ‘lucky guppy’ (line 14) is one that [...]?” To solve this, we should go back to Line 14 and look at its surrounding context to determine how the author describes a “lucky guppy.”

Lines 14-16 say, “A lucky guppy is born above a waterfall or a set of rapids, which keep out the predatory fish called pike cichlids found in calmer downstream waters.” This line tells us clearly that a guppy is luck because it avoids predators by living above a waterfall, as opposed to downstream in calmer waters, and we should expect our answer choice to relate to this.

Answer Choice A says that a lucky guppy “is born in a major river having an established guppy population.” Because our lines don’t mention the benefit of being born in such a river, we should eliminate this option. Answer Choice B says that a lucky guppy “inhabits an environment that provides natural protection from predators.” Because this answer choice does match our understanding of the Passage, we should keep this option. Answer Choice C says that a lucky guppy “manages to navigate the risks associated with living near a waterfall.” Because our Lines don’t mention guppies navigating the risks of living near a waterfall, but instead avoiding certain risks by living near one, we should eliminate this option. Answer Choice D says that a lucky guppy “avoids predatory fish by living in calmer downstream waters.” This answer choice might seem plausible because our Lines do mention avoiding predatory fish, but because these guppies avoid predatory fish by living upstream near a waterfall and not in the downstream where predators are more prevalent, we should eliminate this option. That leaves us with Answer Choice B, which we should choose as our correct answer.

Page 75: SAT Full Reading Explanations (QAS 9-19) · SAT Full Reading Explanations (QAS 9-19) SAT #9 Reading 1 1.R.9 Answer Choice C is the correct answer because Lines 19-22 say, “Moths

23.R.11

Answer Choice D says that “Although small predatory killifish occurred in these new sites new sites, these fish do not pose anything close to the danger of the cichlids,” which most clearly tells us that “the streams used by Reznick’s team in their real-world study were not entirely free of predators.” Answer Choice A is incorrect because these lines only describe what makes a “lucky guppy,” and doesn’t discuss Reznick’s experiment or its streams still containing predators. Answer Choice B is incorrect because these linesonly describes the difference in mortality rates for two guppy populations, and not Reznick’s experiment. Answer Choice C is incorrect because these lines only describe the type of of experiment Reznick performed, and doesn’t mention that the streams he worked in still have some predators.

How to solve this? The Question asks, “Which choice provides the best evidence for the conclusion that the streams used by Reznick’s team in their real-world study were not entirely free of predators?” To solve this, we should go through each of the answer choices given, looking for the one that best tells us about the streams used by Reznick. [...].

24.R.11

Answer Choice A is the correct answer because in Lines 43-44 Reznick describes treating streams “like giant test tubes” and performing experiments in them. This most closely matches Answer Choice A which says that such streams, “provide suitable experimental conditions.” Answer Choice B is incorrect because neither the Text nor our Passage talk about such streams being used to promote “cooperative behaviors in specimens.” Answer Choice C is incorrect because the streams don’t “expedite the rate of genetic changes,” but instead provide a place to manipulate and observe nature. Answer Choice D is incorrect because the streams aren’t used to “solve widespread environmental problems.”

How to solve this? The Question asks, “In lines 43-44, Reznick uses the phrase “giant test tubes” to suggest that certain streams can [...]?” To solve this, we should go to the lines referenced and understand what they say about the streams. Lines 42-46 say “Reznick realized that he could, as he put it in a 2008 paper, ‘treat streams like giant test tubes by introducing guppies or predators’ to place they had not originally occurred, and then watch as natural selection acted on the guppies.” Based on this, we see that the streams mentioned are used by Reznick as places to experiment and manipulate nature experimentally, and should expect our answer choice to refer to this.

Answer Choice A says that the phrase is used to suggest that certain streams can “provide suitable experimental conditions.” Because this answer choice does match our understanding of the Text, we should keep this option. Answer Choice B says that the phrase is used to suggest that certain streams can “promote cooperative behaviors in specimens.” Because the lines mentioned don’t discuss how streams promote cooperative behavior, we should eliminate this option. Answer Choice C says that the phrase is used to suggest that certain streams can “expedite the rate of genetic changes.” Because the lines don’t mention that streams expedite this rate of change, we should also eliminate this option. Answer Choice D says that the phrase is used to suggest that certain streams can “solve widespread environmental problems.” Because the lines don’t say that streams can affects environmental problems, we should also

Page 76: SAT Full Reading Explanations (QAS 9-19) · SAT Full Reading Explanations (QAS 9-19) SAT #9 Reading 1 1.R.9 Answer Choice C is the correct answer because Lines 19-22 say, “Moths

eliminate this option. That leaves us with Answer Choice A, which we should choose as our correct answer.

25.R.11

Answer Choice C is the correct answer, because Lines 46-52 tell us that the “real-world manipulation of nature,” called “experimental evolution,” has become increasingly popular, or commonly used, among scientists. “Widespread” can most nearly replaces the word “popular” here.

● This kind of real-world manipulating of nature is called ‘experimental evolution,’ and it is growing increasingly popular among scientists [...].

● This kind of real-world manipulating of nature is called ‘experimental evolution,’ and it is growing increasingly widespread among scientists [...].

Answer Choice A is incorrect because the Lines don’t emphasize that the method of “experimental evolution” is now “accessible” to scientists, whereas before it couldn’t be used. Answer Choice B is incorrect because the Lines don’t emphasize that the method of “experimental evolution” is growing increasingly “suitable” or workable, whereas before it didn’t work as well. Answer Choice D is incorrect because the Lines don’t emphasize that the method is increasingly “likable,” whereas before it was unliked.

26.R.11 (hard)

Answer Choice B is the correct answer because Reznick’s findings depend on the environment in which he releases the guppies containing little to no predators, so that if “the new site into which Reznick released the guppies is inhabited by fish that are found to be as predatory as the cichlids in the original sites,” this would undermine his finding that the lack of predators in the experimental site influenced the rapid evolution of new traits. Answer Choice A is incorrect because Reznick’s findings don’t depend on a uniform rate of genetic-shift for all guppies worldwide. Answer Choice C is incorrect because, although finding that experimental evolution is harmful to an environment might be concerning, it wouldn’t undermine Reznick’s findings which don’t depend on experimental evolution not being harmful to an environment. Answer Choice D is incorrect because if the transplanted guppies were found to mature later than the guppies that live downstream, this would reinforce not undermine Reznick’s findings.

How to solve this? The Question asks, “Which finding, if accurate, would undermine Reznick’s findings?” To solve this, we should first go back to the part of the Passage that most specifically mentions Reznick’s findings, and determine the basis on which they rest. Based on that we can determine what finding would most likely undermine his findings. Because this question focuses on Reznick’s findings or conclusions, we should focus on the later part of the Passage, that deals with his conclusions regarding the experiments. Moreover, we can use the [Question Order Rule] to predict that the most relevant Text for this question will come after Line 49. Paragraph 5, from Lines 53 to 72, most directly deals with Reznick’s findings, which show that when he takes fish from predatory environments and places them in non-predatory environments, they show a rapid shift in towards traits similar to guppies which exist in naturally non-predatory environments. Anything that would disrupt or disprove part of the logical

Page 77: SAT Full Reading Explanations (QAS 9-19) · SAT Full Reading Explanations (QAS 9-19) SAT #9 Reading 1 1.R.9 Answer Choice C is the correct answer because Lines 19-22 say, “Moths

foundation of his findings (the fish not actually showing any change, fish randomly changing in other environments, a confounding variable, etc.) could be our correct answer.

Answer Choice A says that it would undermine Reznick’s findings if “guppies examined in other parts of the globe exhibit genetic shifts in traits at a different rate from that exhibited by the guppies Reznick examined.” Because his findings don’t depend on the uniformity of the rate of genetic shifts across the globe, we should eliminate this option. Answer Choice B says that it would undermine Reznick’s findings if “the new site into which Reznick released the guppies is inhabited by fish that are found to be as predatory as the cichlids in the original sites.” Because Reznick’s findings do depend on the fact that his new site contains no predators and because he couldn’t then conclude that the change in traits exhibited by the transplanted guppies was caused by a predator free environment, we should keep this option. Answer Choice C says that it would undermine Reznick’s findings if “experimental evolution is shown to be harmful to the environments where studies like Reznick’s are conducted.” Because his findings don’t depend on evolutionary experiments not being harmful to the environments they’re performed in, they can’t undermine his findings, and we should eliminate this option. Answer Choice D says that it would undermine Reznick’s findings if “the descendants of Reznick’s transplanted fish are proven to mature later than the guppies living below the waterfall.” Because the guppies living below the waterfall live in relatively predator-free environments and mature later because of it, this finding would actually reinforce Reznick’s findings because the transplanted guppies exhibit traits like those in naturally predator-free environments, so we should eliminate this option as well. That leaves us with Answer Choice B, which we should choose as our correct option.

27.R.11 & 28.R.11

Answer Choices A and D are the correct answers because Lines 67-72 say that, “other studies of guppies in Trinidad have shown evolutionary change in as few as two and a half years [...],” which most clearly matches Answer Choice A, that “some genetic traits will evolve more readily than others.” For Question 28, Answer Choices A, B and C are all incorrect because none of the the choices tell us about experiments in Trinidad, but only the methodology and set up for Reznick’s experiment. For Question 27, Answer Choices B, C and D are all incorrect because our lines only mention the rapid change of shown in some experiments with guppies, and not the relative dangers of predatory fish, how guppies fare in different environments or the differences in preventing genetic shifts.

How to solve this? Question 27 asks, “It can most reasonably be inferred from the passage that the experiments in Trinidad have shown which of the following about guppies?” Because this is a Paired Question, to solve it we should go through the answer choices for Question 28, looking for any texts that tell us about the experiments in Trinidad. Using the [Question Order Rule], we should also expect that our Texts will probably come towards the end of the Passage, and most likely in the final paragraph.

For Question 28, Answer Choice A says, “How quickly, though, could these differences in how the two kinds of guppies lived their lives have evolved?” Because this only asks a question that sets up the experiment that Reznick performed, and doesn’t tell us anything about the experiments in Trinidad, we should eliminate this option. Answer Choice B describes the tributaries in Trinidad, and the fact that Reznick could treat them as “giant test tubes.” This

Page 78: SAT Full Reading Explanations (QAS 9-19) · SAT Full Reading Explanations (QAS 9-19) SAT #9 Reading 1 1.R.9 Answer Choice C is the correct answer because Lines 19-22 say, “Moths

answer choice does tell us something about Reznick’s experiment in Trinidad, but it’s too preliminary, only describing his reasons for choosing the location, and only sets up his experiment without giving some conclusion for multiple experiments. So, we should eliminate this option as well. Answer Choice C says that, “Along with his students and colleagues, Reznick removed groups of guppies from their predator-ridden lives below the waterfall and released them into previously guppy-free streams above the falls.” This answer choice, like the one before it, only gives us the set up for Reznick’s experiment and not any conclusion from multiple experiments, so we should eliminate this option as well. Answer Choice D says that, “Other studies of guppies in Trinidad have shown evolutionary change in as few as two and a half years, or a little over four generations, with more time required for genetic shifts in traits such as the ability to form schools and less time for changes in the colorful spots and stripes on a male’s body.” Because this answer choice does tell us about other experiments in Trinidad, and gives us some conclusions based on them, we should keep this option. This leaves only Answer Choice D which we should choose as our correct answer.

Lines 67-72 tell us that other experiments in Trinidad have shown how quickly evolutionary change can occur in a group of guppies, and more time can be required for some traits than for others, so we should expect our answer choice to refer to this.

Answer Choice A says that, “some genetic traits will evolve more readily than others.” Because this answer choice does match our understanding of the Text, that traits can evolve quickly and at different speeds than others, we should keep this option. [...]

29.R.11

Answer Choice D is the correct answer because the first column in the figure represents the “guppies living in the south slope high-predation environment,” and rises to about 6.5 on the graph, which most closely matches Answer Choice D which says “6 and 7.” Answer Choices A, B and C are incorrect because they don’t closely match the level of the first column.

How to solve this? The Question asks, “According to figure 1, guppies living in the south slope high-predation environment produced a mean number of offspring between [...]?” To solve this, we should go back to figure in question and locate the column that measures the mean number of offspring for “guppies living in the south slope high-predation environment.” Only the first column matches this, so we should refer to it and notice that it rises to a mean of about 6.5, which most closely matches Answer Choice D. Because none of the other answer choices are as close, we should eliminate them and choose Answer Choice D as our correct answer choice.

30.R.11

Answer Choice C is the correct answer because it says, “the predation level observed in each environment had more of an effect on mean embryo mass than did slope locations.” This most closely matches our figure which shows little to no variation between different slopes with the same level of predation (see Column 1 and Column 3), but does show a lot of variation between high-predation and low-predation (see Column 1 and Column 2). Answer Choice A is incorrect because the slope location isn’t as good an indicator of the mean embryo mass as the predation level, since little variation occurs between different slope locations with the same level of

Page 79: SAT Full Reading Explanations (QAS 9-19) · SAT Full Reading Explanations (QAS 9-19) SAT #9 Reading 1 1.R.9 Answer Choice C is the correct answer because Lines 19-22 say, “Moths

predation, but does occur between different predation levels with the same slope location. Answer Choice B is incorrect is incorrect because the columns that measure the north slope location exceed that of the south slope location, meaning their mean would be higher not smaller. Answer Choice D is incorrect because the guppies born in low-predation environments actually had a greater mean embryo mass than those born in high-predation environments.

How to solve this? The Question asks, “Which conclusion about the mean mass of guppy embryos is best supported by figure 2?” To solve this, we should go through each of the answer choices, noting which statements could plausibly describe the graph, and eliminating the others. Because only Answer Choice C matches the graph, we should keep that option. Because Answer Choices A, B and D don’t describe the graph accurately, we should eliminate those options and choose Answer Choice C as our correct answer.

31.R.11

Answer Choice B is the correct answer because Figure 1 shows guppies in low-predation environments tending to have fewer offspring and Figure 2 shows guppies in low-predation environments tending to have a greater mean embryo mass. That most closely matches the description in Answer Choice B, which says guppies from low-predation environments are likely to “be part of a smaller litter and have a greater mean embryo mass.” Answer Choice A is incorrect because, while the first part is true, the rate of maturity is not measured by either figure and therefore can’t be our answer. Answer Choice C is incorrect because neither figure measures the rate of survival and because guppies from low-predation environments have greater, not lesser embryo mass. Answer Choice D is incorrect because, while these guppies do have a greater mean embryo mass, they produce a lesser not greater number of offspring.

How to solve this? The Question asks, “The data presented in figures 1 and 2 best support the conclusion that compared with guppies from high-predation environments, guppies from low-predation environments were more likely to [...]?” To solve this, we should begin by going back to both figures and noticing any trends that either shows for guppies born in low-predation environments.

Figure 1, which measures the mean number of offspring for guppies, shows that those born in low-predation environments tend to have a much lower number of guppy-offspring than those born in high-predation environments. Figure 2 measures the mean embryo mass of guppy offspring, and show that those in low-predation environments tend to have a much greater mean embryo mass than those born in high-predation environments. So, we can say that both figures show that guppies born in low-predation environments to have a smaller mean number of offspring and a greater mean embryo mass, and we should expect our answer choice to relate to this. Because only Answer Choice B matches both of these, “smaller litter and greater mean embryo mass,” we should choose this option. We should be careful with Answer Choice A which more clearly than Answer Choice B says that these guppies “have fewer offspring,” but also measures maturity, something our figures don’t. Answer Choices C and D are incorrect because they misrepresent the information given in the figures.

Page 80: SAT Full Reading Explanations (QAS 9-19) · SAT Full Reading Explanations (QAS 9-19) SAT #9 Reading 1 1.R.9 Answer Choice C is the correct answer because Lines 19-22 say, “Moths

Reading 4

Passage Outline

● told that the question of slavery is a political question, inappropriate for women to ask. but it is also a question of humanity and morality. (paragraph one).

● shouldn’t women be allowed to participate in this debate. is it really only a “political question”? (paragraph two).

● given that the constitution would require the north to put down a slave insurrection, and cost the lives of men, shouldn’t this question concern women? a woman’s duty to herself, family and country should be enough to participate in the debate. (paragraph three).

● even if women do admit it is a political question, so what? shouldn’t they participate in such an important debate? their country matters as much to them as anyone.

32.R.11 (4)

Answer Choice D is the correct answer, because throughout the Passage, the author argues the right of women to participate in the cause of abolition and defends that right against various accusations. In Lines 51-57, she writes, “Surely this consideration is of itself sufficient to arouse the slumbering energies of women [...]; and she will not be deterred from the performance of her duty to herself, her family, and her country [...].” Or, in Lines 73-77 she says that ”the events of the last two years have cast their dark shadows before [...], shrouding the destinies of our country in more than midnight gloom, and we cannot remain inactive.” This most closely matches Answer Choice D, which says the purpose of the passage is to “encourage women to see their participation in the abolitionist cause as just and important.” Answer Choice A is incorrect because the author doesn’t accuse other abolitionists in the Passage, nor does she spend time defending the contributions of women, instead spending time justifying their position. Answer Choice B is incorrect because, while the author does defend and treat positively the cause of abolition and women’s participation in it, she never directly argues that these are explicit continuations of the spirit of the American Revolution, a very specific claim. Answer Choice C is incorrect because, while it presents the author as being sympathetic towards the cause of abolition, in the Passage the author never claims that women’s rights are meaningless while slavery exists, which is a strong and specific claim.

How to solve this? The Question asks, “Smith’s main purpose in the passage is to [...]?” Because this is a [Big Picture] Question, to solve it we should go back to the Passage to get both a general understanding of its structure and content as well as to find specific texts within it that will tell us about its main purpose. [See above for general purpose]. Specifically, we can begin by looking at the subtitle, introductory paragraph and conclusion, while also scanning for any [indicators] like “but” or “however.” The subtitle for this Passage doesn’t tell us much related to its purpose. The introduction shows the author arguing against the idea that women can’t participative in the discussion of the topic of slavery because it is a “political question,” arguing in fact that it is more than simply that. There are no strong claims in this introduction beyond the general idea that the author believes women should participate in the discussion on slavery. The conclusion begins with a series of questions, essentially making the point that even if women admit the topic to be a political one, they should be allowed to participate in it. It concludes with

Page 81: SAT Full Reading Explanations (QAS 9-19) · SAT Full Reading Explanations (QAS 9-19) SAT #9 Reading 1 1.R.9 Answer Choice C is the correct answer because Lines 19-22 say, “Moths

her stating that the country is as dear to women as anyone, and that women’s course, should still be “onward.” There’s no strong single statement in this conclusion either, but a general reinforcement of the author’s view that the topic of slavery is an important one in which women should be allowed to participate. So, we may not know in advance what our answer choice will look like, we can expect it to have something to do with the importance of women participating in the cause of abolition.

Answer Choice A says that the main purpose of the passage is to “accuse fellow abolitionists of overlooking the contributions that women have made to the movement.” Although this answer choice seems favorable towards women involved in abolition, its incorrect because the author never accuses fellow abolitionists nor does she highlight specific contributions from women in the movement. Answer Choice B says that the main purpose of the passage is to “argue that the causes of abolition and women’s rights are continuations of the spirit of the American Revolution.” This answer choice also seems plausible, since it relates women’s rights and abolition in a positive manner. It’s incorrect, however, because it makes the specific claim that these causes and rights are “continuations of the spirit of the American Revolution,” a point the author never makes explicitly. So, we should eliminate this option as well. Answer Choice C says that the main purpose of the passage is to “make the case that women’s rights are meaningless while slavery exists.” This answer choice is also sympathetic to the cause of abolition, but it fails because it makes the too strong and specific claim that women’s rights are “meaningless” while slavery exists, a point the author never clearly makes. Answer Choice D says that the main purpose of the passage is to “encourage women to see their participation in the abolitionist cause as just and important.” This answer choice has the virtue of being very general, and avoiding strong or specific claims. It also matches our understanding of the Passage, that the author argues that the participation of women in the cause of abolition is “just and important.” So, we should keep this option and choose Answer Choice D as our correct answer choice.

33.R.11 (3)

Answer Choice A is the correct answer because throughout the Passage, Smith asks a variety of rhetorical questions (“Will it be easy to convince them that it is no concern of theirs, that their homes are rendered desolate, and their habitations the abodes of wretchedness?” or “May we not permit a thought to stray beyond the narrow limits of our own family circle, and of the present hour?”), all of which the reader understands should be answered with “No.” This most closely matches Answer Choice A, which says that one technique Smith uses to advance her point is by presenting “claims in the form of rhetorical questions that mostly have implicit negative answers.” Answer Choice B is incorrect because the author never quotes any of her opponents. Answer Choice C is incorrect because the author doesn’t use any anecdotes or stories in the Passage. Answer Choice D is incorrect because, although the author certainly believes in her own claims, she never presents them as though they are universally held. In fact, she herself provides the claims of those who disagree with her.

How to solve this? The Question asks, “Which statement provides the best description of a technique that Smith uses throughout the passage to advance her main point?” To solve this, we should rely on both memory and going back to the Passage to find what [rhetorical device / literary technique] the author uses. A common technique used in SAT Passages, and one which

Page 82: SAT Full Reading Explanations (QAS 9-19) · SAT Full Reading Explanations (QAS 9-19) SAT #9 Reading 1 1.R.9 Answer Choice C is the correct answer because Lines 19-22 say, “Moths

should stand out to us in particular is the use of rhetorical questions, which appear frequently in this Passage. We can’t be sure that these will be a part of the correct answer choice, but we should have a reasonably high expectation that they will be.

Answer Choice A says that Smith uses the technique of presenting “her claims in the form of rhetorical questions that mostly have implicit negative answers.” This does match our prediction and understanding of the Passage, and, going back, we can check that the rhetorical questions asked do indeed have negative implicit answers and assure ourselves that they do, so we should keep this option. Answer Choice B says that Smith uses the technique of criticizing “her opponents by quoting self-contradictory remarks they have made.” Because nowhere in the Passage does the author criticize opponents directly, especially by quoting their self-contradictory remarks, we should eliminate this option. Answer Choice C says that Smith uses the technique of illustrating “each of her central ideas with an emotionally powerful anecdote.” Because nowhere in the Passage does the author give anecdotes or stories, we should eliminate this option as well. Answer Choice D says that Smith uses the technique of emphasizing “the reasonableness of her views by presenting them as though they are universally held.” Because the author doesn’t present her views this way, and, in fact, gives voice to those who do disagree with her, we should eliminate this option. That leaves us with Answer Choice A, which we should choose as our correct answer choice.

34.R.11 (4)

Answer Choice B is the correct answer because Smith discusses the topic of slavery as a “political question” at two points in the Passage. In the first, she says that “it is not true that it is merely a political question, it is likewise a question of justice, of humanity, of morality, of religion.” Here, she expands the scope of the question of slavery beyond that of a “political question” only. In the second place, she asks, “But admitting it to be a political question, have we no interest in the welfare of our country?” and makes her point that even if slavery is indeed a “political question,” it still has relevance to women as such. These two instances in the development of her point most closely match Answer Choice B, which says that “she dismisses the designation as too narrow but then demonstrates its relevance to her audience.” Answer Choice is incorrect because the author does dismiss the designation of “political question” in the beginning of the Passage, but never by explicitly calling it “outdated,” a very specific claim. She also doesn’t later present alternative definitions. Answer Choice C is incorrect because the author never calls the designation of “political question” trite, or cliche, but instead argues that its insufficient. She also never clearly asks her audience itself to revitalize the term. Answer Choice D is incorrect because she doesn’t present the meaning the designation has for men in the beginning of the Passage.

How to solve this? The Question asks, “How does Smith develop her argument about slavery as a ‘political question’ (line 3) over the course of the passage?” To solve this, we can approach the question in a number of ways. [Because this question requires a lot of back and forth, a linear approach won’t represent all of the correct steps perfectly, but should give a good enough approximation of some of the steps in the correct thought process and some of the things you should notice]. First, we can notice how each of the answer choices is split into two parts, showing us that the test understands the development of the author’s argument about slavery as a “political question” to have two parts. This means that we should go back to the Passage to

Page 83: SAT Full Reading Explanations (QAS 9-19) · SAT Full Reading Explanations (QAS 9-19) SAT #9 Reading 1 1.R.9 Answer Choice C is the correct answer because Lines 19-22 say, “Moths

look at these two specific moments, and then concentrate on proving or disproving either moment in the answer choices. In Lines 4-11, the author says of slavery that “it is not true that it is merely a political question, it is likewise a question of justice, of humanity, of morality, of religion [...].” This tells us that the author wants to expand the issue of slavery to concern more than just politics. In Lines 58-59, she returns to the topic by asking, “But admitting it to be a political question, have we no interest in the welfare of our country?” and continues to make the case that even as a “political question,” the topic concerns women. This tells us that she later embraces that designation, and we should expect our answer choice to relate to this and the first point.

Answer Choice A says of Smith’s argument over slavery as a “political question” that she “claims the designation is an outdated one and then offers alternative definitions.” This answer choice might seem plausible, since it presents the sort of negative-positive structure that we saw in the Passage. But, more specifically, we should note that the author doesn’t claim that the designation of “political question” is an outdated one, but that its insufficient. Answer Choice B says of Smith’s argument over slavery as a “political question” that she “dismisses the designation as too narrow but then demonstrates its relevance to her audience.” This answer choice does match our understanding of the Passage, where the author first states that there is more to question of slavery than it being a “political question” and then makes the point of its importance, so we should keep this option. Answer Choice C says of Smith’s argument over slavery as a “political question” that she “contends that the designation has become trite and then invites her audience to revitalize it.” This answer choice also captures the basic negative-positive structure we see in the Passage, but it makes the too-specific claim that Smith claims the designation is trite, or cliche, which she doesn’t do. Moreover, it says that Smith invites her audience to revitalize the claim, which she also doesn’t do, so we should eliminate this option as well. Answer Choice D says of Smith’s argument over slavery as a “political question” that she “describes the meaning the designation has for men and then challenges women to embrace it.” Because Smith doesn’t describe the meaning that the designation has for men in the beginning of the Passage, we should eliminate this option as well. That leaves us with Answer Choice A, which we should choose as our correct answer for the question.

35.R.11

Answer Choice B is the correct answer because, in the first paragraph, the author says of the topic of slavery that “it is not true that it is merely a political question, it is likewise a question of justice, of humanity, of morality, of religion [...],” and continues to elaborate on the reason why this is, concluding that ‘these considerations (of much more than politics) are all involved in the question of liberty and slavery.” This most closely matches Answer Choice B, which says that “Smith rejects a claim and elaborates on her reasons for doing so.” Answer Choice A is incorrect because Smith doesn’t support any conventional viewpoint, but, in fact, argues against one. Answer Choice C is incorrect because Smith doesn’t provide any historical background for her subject. Answer Choice D is incorrect because Smith doesn’t present any specific problem and then offer solutions, but rather argues about the designation of “political question.”

How to solve this? The Question asks, “Which choice best summarizes the first paragraph?” To solve this, we should go back to the first paragraph and try to identify its basic structure and content. Going back, we should note that we can split the question into two basic parts:

Page 84: SAT Full Reading Explanations (QAS 9-19) · SAT Full Reading Explanations (QAS 9-19) SAT #9 Reading 1 1.R.9 Answer Choice C is the correct answer because Lines 19-22 say, “Moths

● the subject of slavery is not merely a “political question” ● it’s also a question “of justice, of humanity, of morality, of religion,” and these and more

considerations are “all involved in the question of liberty or slavery.”

Answer Choice A says that, in the first paragraph, “Smith explains a conventional viewpoint and presents evidence supporting it.” Because Smith seems to take the opposite of a conventional viewpoint, arguing against the idea that slavery is merely a “political question,” we should eliminate this option. Answer Choice B says that, in the first paragraph, “Smith rejects a claim and elaborates on her reasons for doing so.” Because this answer choice does match our understanding of the Passage, that Smith rejects the designation of “political question” and then argues why, we should keep this option. Answer Choice C says that Smith “introduces her subject and provides historical background for understanding it.” Because nowhere in the first paragraph does Smith give historical background for anything, we should eliminate this option. Answer Choice D says that, in the first paragraph, “Smith identifies a problem and proposes steps to remedy it.” Because Smith doesn’t present her topic as a problem, and doesn’t proceed to give steps to remedy it later on, we should eliminate this option as well. That leaves us with Answer Choice B, which we should choose as our correct answer.

36.R.11 & 37.R.11

Answer Choices A and A are the correct answers because Lines 26-33 ask, “May not the ‘ornament of a meek and quiet spirit’ exist with an upright mind and enlightened intellect, and must women necessarily be less gentle because her heart is open to the claims of humanity, or less modest because she feels for the degradation of her enslaved sisters, and would stretch forth her hand for their rescue?” Here, Smith makes the case that woman can still be “gentle” and also “stretch forth her hand for their rescue.” This most closely matches Answer Choice A for Question 36, which says that it is possible for women to act “according to humanitarian principles while preserving their femininity.” For Question 37, Answer Choice B is incorrect because it only highlights that women would be affected by a war over slavery, but doesn’t claim that women are able to participate in two apparently contradictory activities. Answer Choice C is incorrect because it only claims that considering their homes, habitats and family member is itself sufficient to bring women to action, without presenting the possibility of woman performing two seemingly contradictory activities. Answer Choice D is incorrect because it only makes the claim that women feel that their country is as dear to them as anyone else. For Question 36, Answer Choice B is incorrect because our Text doesn’t present a reconciliation between adhering to a personal morality while still being neutral politically. Answer Choice C is incorrect because our Text doesn’t talk about women contributing to their family’s financial security. Answer Choice D is incorrect because neither our Text nor our Passage talk about women resisting calls for war.

How to solve this? Question 36 asks, “In the passage, Smith argues that it is possible for women to engage in which activity?” Because this is a Paired Question, to solve it we should go through the answer choices for Question 37, looking for any texts that tell us which activities it is possible for women to engage in. Using the Question Order Rule, we can predict that it’s more likely for our Text to come between Lines 3 and 40. [Some alert. This Question requires some back and forth to find the right answer. After first looking at the Text, we should note that no

Page 85: SAT Full Reading Explanations (QAS 9-19) · SAT Full Reading Explanations (QAS 9-19) SAT #9 Reading 1 1.R.9 Answer Choice C is the correct answer because Lines 19-22 say, “Moths

answer choice clearly tells us about some activity that it is possible for women to do. However, after going back to the answer choices for Question 36, we can start to see that they all take the structure of bringing together two possibly contradictory activities, and that the majority have to do with tensions between domestic activities. This helps calibrate our second search, which will really help us arrive at Answer Choice A.]

For Question 37, Answer Choice A says, “May not the ‘ornament of a meek and quiet spirit’ exist with an upright mind and enlightened intellect, and must women necessarily be less gentle because her heart is open to the claims of humanity, or less modest because she feels for the degradation of her enslaved sisters, and would stretch forth her hand for their rescue?” Because this answer choice does reconcile two possibly contradictory activities for women, we should keep this option. [None of the other answer choices do.]

[For Question 36, none of the other answer choices work except for Answer Choice A, where femininity and “gentleness” are matched, while acting according to humanitarian principles is matched with her stretching “forth her hand for their rescue.”]

38.R.11

Answer Choice C is the correct answer because Lines 34-42 say that, “by the Constitution of the United States, the whole physical power of the North is pledged for the suppression of domestic insurrections, and should the slaves, maddened by oppression, endeavor to shake off the yoke of the taskmaster, the men of the North are bound to make common cause with the tyrant, and put down, at the point of the bayonet, every effort on the part of the slave, for the attainment of his freedom.” This most clearly tell us that, if slaves were to revolt, the US Constitution would require the Northern States to side with the slave states and put down the insurrection. Answer Choice C most clearly matches this when it says that, “the Northern states would have to help the slave states fight the slaves’ rebellion.” Answer Choice A, B and D are incorrect because our Lines only tell us that the US Constitution would require Northern free states to “put down, at the point of the bayonet, every effort on the part of the slave,” not that they would have to “sever ties,” “give shelter to refugees,” or “provide financial assistance to rebelling slaves.”

How to solve this? The Question asks, “According to Smith, the US Constitution requires which action on the part of the Northern free states if slaves were to revolt?” To solve this, we should go back to the Passage and the find the Text which can answer this. Using the [Question Order Rule], we can predict that our Text is most likely to come before Lines 40-50 and scan for the words “US Constitution” and “Northern free states.”

We should finally end up on Lines 34-42, which say that, “by the Constitution of the United States, the whole physical power of the North is pledged for the suppression of domestic insurrections, and should the slaves, maddened by oppression, endeavor to shake off the yoke of the taskmaster, the men of the North are bound to make common cause with the tyrant, and put down, at the point of the bayonet, every effort on the part of the slave, for the attainment of his freedom.” These Lines tell us that the Constitution would require Northern states to partner with the South to put down a slave insurrection in the slave states, and we should expect our answer to relate to this.

Page 86: SAT Full Reading Explanations (QAS 9-19) · SAT Full Reading Explanations (QAS 9-19) SAT #9 Reading 1 1.R.9 Answer Choice C is the correct answer because Lines 19-22 say, “Moths

Answer Choice A says that, if slaves were to revolt, “The Northern states would have to sever ties with the slave states.” Because this answer choice doesn’t match either our Text or our Passage, we should eliminate this option. Answer Choice B says that, if slaves were to revolt, “The Northern states would have to give shelter to refugees from the slave states.” This answer choice also doesn’t match our Text, so we should eliminate this option as well. Answer Choice C says that, if slaves were to revolt, “The Northern states would have to help the slave states fight the slaves’ rebellion,” which does match our answer choice, so we should keep this option. Answer Choice D says that, if slaves were to revolt, “the Northern states would have to provide financial assistance to the rebelling slaves.” Because this answer choice doesn’t match our Text or Passage, we should eliminate this option. That leaves us with Answer Choice C, which we should choose as our correct option.

39.R.11

Answer Choice D is incorrect because in Lines 38-40 the author says that “the men of the North are bound to make common cause with the tyrant,” where “tyrant” refers to the slave states in the South. Later, in Lines 82-84, the author mentions “the foot of the tyrant,” referring again to those who own slaves. This most clearly matches Answer Choice D, which says the word “tyrant” in the passage “emphasizes the unjustness of slavery.” Answer Choice A is incorrect because the word doesn’t refer to a specific individual, but to many people. Answer Choice B is incorrect because the word refers to the slaves states, not a threat of aggression from abroad. And Answer Choice C is incorrect because the word is used to describe the slave states, not to critique the limited role for women in antislavery movements.

How to solve this? The Question asks, “In context, what is the main effect of Smith’s use of the word ‘tyrant’ in lines 40 and 83?” To solve this we should go back to both lines mentioned and determine how the word is being used in context, who it refers to and what effect this may have on the Passage. Lines 38-41 tell us that “the men of the North are bound to make common cause with the tyrant, and put down, at the point of the bayonet, every effort on the part of the slave.” Within its context, the word refers to the slave states, noting them as “tyrants.” In Lines 82-84, the Passage says “that the foot of the tyrant may no longer invade the domestic sanctuary,” again referring to slave owners. Based on these two, we can say that the word “tyrant” describes the “slave states,” obviously denoting them as evil and oppressive.

Answer Choice A says the word “identifies a specific individual as oppressive.” Because the word refers to a group and not a specific individual, we should eliminate this option. Answer Choice B says that the word “highlights the threat of aggression from abroad.” However, because the word refers to the domestic slave states, and not to a threat from abroad, we should eliminate this option as well. Answer Choice C says the word “critiques the limited roles for women in antislavery movements.” Because the word, however, refers to slave states and not to the limited role of women, we should eliminate this option as well. Answer Choice D, which has the virtue of generality, most clearly matches our understanding of the use of the words, so we should keep it. This leaves us with only Answer Choice D which we should choose as our correct answer for Question 39.

Page 87: SAT Full Reading Explanations (QAS 9-19) · SAT Full Reading Explanations (QAS 9-19) SAT #9 Reading 1 1.R.9 Answer Choice C is the correct answer because Lines 19-22 say, “Moths

40.R.11

Answer Choice C is the correct answer, because the word “slumbering” in Line 52 can be best replaced by the word “dormant,” referencing the unused or unengaged energies of woman. Answer Choice A is incorrect because it emphasizes an actual physical sleepiness, and too specifically describes an actual person not a general state. Answer Choice B suffers for the same reasons. Answer Choice D misses the point, because we want to emphasize the temporary lack of engagement, not a complete unconsciousness as well as personifying the the “energies of woman” in the same way as Answer Choices A and B. This leaves us with Answer Choice C, which we should choose as our correct answer.

41.R.11 & 42.R.11 (4)

Answer Choices A and C are the correct answers because Lines 68-73 describes how the US is “rendered a hissing and a reproach throughout the world, by this system which is already tolling the death-bell of her decease among the nations,” which most clearly matches Answer Choice A for Question 41 which says that slavery is “lowering the country’s reputation in the international community.” For Question 42, Answer Choice A is incorrect because, while it refers obliquely to the effect of slavery on the US, it’s less direct than C and doesn’t match any answer Choice in Question 41. Answer Choice B is incorrect because it only asks a rhetorical question about what’s appropriate for women to think. Answer Choice D is incorrect because no answer choice for Question 41 directly references it. For Question 41, Answer Choice B is incorrect because neither our Text nor the other given choices describe women disavowing their allegiance to the country. Answer Choice C is incorrect because, while one of the texts may seem to support this option, it does so weakly and hypothetically, not giving us enough support to choose this option. Answer Choice D is incorrect because neither our Text nor the other options say that the authority of the country’s government is weakened by slavery.

How to solve this? Question 41 asks, “In the passage, Smith most strongly suggests that slavery affects the United States by [...]?” Because this is a Paired Question, we should begin by going through the answer choices for Question 42, looking for any texts that tell us how slavery affects the United States.

For Question 42, Answer Choice A asks, “Will it be easy to convince them that it is no concern of theirs, that their homes are rendered desolate, and their habitations the abodes of wretchedness?” Although this question is only hypothetical, and doesn’t directly describe the effect of slavery on the country, it does discuss some kind of effect, that women’s “homes are rendered desolate, and their habitations the abodes of wretchedness,” so we should keep this option. Answer Choice B asks, “May we not permit a thought to stray beyond the narrow limits of our own family circle, and of the present hour?” Because this question only refers to what’s appropriate for women to consider, and not the effect of slavery on the country, we can eliminate this option. Answer Choice C asks, “Shall we silently behold the land which we love with all the heart-warm affection of children, rendered a hissing and a reproach throughout the world, by this system which is already tolling the death-bell of her decease among the nations?” Because this answer choice references the United States (“the land which we love”), slavery (“this system”), and the effect of slavery on the US (“rendered a hissing and a reproach throughout the world”), we should consider this a strong option and keep it as an answer choice. Answer

Page 88: SAT Full Reading Explanations (QAS 9-19) · SAT Full Reading Explanations (QAS 9-19) SAT #9 Reading 1 1.R.9 Answer Choice C is the correct answer because Lines 19-22 say, “Moths

Choice D ssays, “the events of the last two years have cast their dark shadows before, overclouding the bright prospects of the future, and shrouding the destinies of our country in more than midnight gloom, and we cannot remain inactive.” Because this answer choice also tells us about the effect of slavery on the United States, we should keep this option as well.

At this point, then, we’re unable to decide between the answer choices for Question 42 and have to use Question 41 to help us make a decision. Before going back, however, we should make a mental note of the meaning for the three remaining answer choices for Question 42.

● Answer Choice A — slavery could cause homes to be “rendered desolate and their habitations the abodes of wretchedness.”

● Answer Choice C — slavery causes the US to be a “hissing and a reproach throughout the world.”

● Answer Choice D — slavery causes a “dark shadow” to fall over the country, and clouds its “bright prospects” and destiny.

For Question 41, Answer Choice A says that slavery affects the United States by “lowering the country’s reputation in the international community.” Because this answer choice matches Answer Choice C for Question 42, a strong option, we should keep this answer choice. Answer Choice B says slavery affects the United States by “leading many women to disavow their allegiance to the country.” Because none of our possible texts suggest that women would disavow their loyalty, we should eliminate this option as well. Answer Choice C says slavery affects the United States by “causing violent conflicts in many areas of the country.” This answer choice could refer to Answer Choice A for Question 42, so we can keep this option. Answer Choice D says slavery affects the United States by “weakening the authority of the country’s government.” Because none of our texts say that slavery weakens the authority of the government, we should eliminate this option as well.

That leaves us with a choice between Answer Choice A and C, or between Answer Choice C and A. We should focus on eliminating one option, thinking about which could have an error or insufficient evidence. Ultimately, because Lines 48-50 refer to a possible state of affairs that slavery could cause, an insurrection and subsequent conflict, and not something that’s yet actual, it’s a weaker answer choice and we should Answer Choices A and C for our correct answers.

Page 89: SAT Full Reading Explanations (QAS 9-19) · SAT Full Reading Explanations (QAS 9-19) SAT #9 Reading 1 1.R.9 Answer Choice C is the correct answer because Lines 19-22 say, “Moths

Reading 5

Passage Outline

Passage One

● introduce the scientist Kim Lewis, who created new antibiotic with novel beneficial effects (paragraph one)

● how they approached and thought about growing these antibiotics (paragraph two) ● how they actually grew these antibiotics (paragraph three and four) ● a reason why the new antibiotic may be so effective (paragraph five)

Passage Two

● how Lewis figured out how to grow new kinds of antibiotics ● one of the antibiotics they found, and some of its positive features ● some issues and imperfections with the new antibiotic

43.R.11

Answer Choice C is the correct answer because the first paragraph says that “‘Pathogens are acquiring resistance faster than we can introduce new antibiotics,’” which introduces the problem that the Passage goes on to address. This most clearly matches Answer Choice C which says that the purpose of the Passage is to “identify a problem that the research discussed in the passage may help to address.” Answer Choice A is incorrect, because the remainder of Passage 1 doesn’t go on to support and develop the claim that “pathogens are acquiring resistance faster than new antibiotics can be introduced.” Answer Choice B is incorrect because the first paragraph doesn’t introduce a “controversy,” but only a problem to be addressed. Answer Choice D is incorrect because the first paragraph doesn’t offer any theory, and especially doesn’t offer one that is challenged by later findings.

How to solve this? The Question asks, “The first paragraph of Passage 1 primarily serves to [...]?” To solve this, we should have both a general understanding of the Passage, as well as a specific understanding of the first paragraph and what role it might play in the Passage as a whole. The general outline of Passage 1 looks like:

Passage One

● introduce the scientist Kim Lewis, who created new antibiotic with novel beneficial effects (paragraph one)

● how they approached and thought about growing these antibiotics (paragraph two) ● how they actually grew these antibiotics (paragraph three and four) ● a reason why the new antibiotic may be so effective (paragraph five)

Going back to the first paragraph we see it introduces a quote by Lewis, the scientist described throughout the Passage, that says, “Pathogens are acquiring resistance faster than we can introduce new antibiotics, and this is causing a human health crisis.” Taking these two things together, the general outline of the Passage and specific meaning of the paragraph, we can say

Page 90: SAT Full Reading Explanations (QAS 9-19) · SAT Full Reading Explanations (QAS 9-19) SAT #9 Reading 1 1.R.9 Answer Choice C is the correct answer because Lines 19-22 say, “Moths

that the first paragraph serves to set up or introduce the problem that motivated the creation of the drug explored in the Passage, and should expect our answer to have something to do with this.

Answer Choice A says that the first paragraph serves to “present a claim that is supported and developed over the course of the passage.” Because the claim in the first paragraph isn’t something that’s developed and supported throughout the passage, but the introduction of a problem that gets solved, we should eliminate this option. Answer Choice B says that the first paragraph serves to “introduce a controversy that the study described in the passage is intended to resolve.” This answer choice might at first seem plausible, but we should note the strong claim that the first paragraph introduces a “controversy,” or subject of debate, which is too strong of a claim for the lines given, so we should eliminate this option. Answer Choice C says that the first paragraph serves to “identify a problem that the research discussed in the passage may help to address.” Because this answer choice does match our understanding of the Text and Passage, we should keep this option. Answer Choice D says that the first paragraph serves to “offer a theory that is challenged by the findings presented in the passage.” Because the first paragraph doesn’t offer a theory, and especially not one that is later challenged in the passage, we should eliminate this option. That leaves us with Answer Choice C, which we should choose as our correct option.

44.R.11 & 45.R.11 (4)

Answer Choices D and A are the correct answer choices because Lines 17-21 say, “But only about one percent of these organisms can be grown in a lab. The rest in staggering numbers, have remained uncultured and of limited use to medical science, until now.” This tells us that most of the organisms that could produce antibiotics were unable to be grown in labs, until Lewis’s method was introduced. This most clearly Answer Choice D which says that the advantage of Lewis’s method is that it “allows researchers to make use of soil bacteria that they had previously been unable to exploit.” For Question 45, Answer Choice B is incorrect because, although it does provide a possible advantage that Lewis’s team exploits, no answer choice for the previous question matches it. Answer Choice C also presents a possible advantage of the method, but likewise doesn’t have an answer choice that matches it. Answer Choice D is incorrect because it talks about the advantage and effectiveness of the teixobactin, but the method of Lewis’s team. For Question 44, Answer Choice A is incorrect because the experiment doesn’t replicate specific features in “artificial soil,” but rather grows them “in their natural environment.” Answer Choice B is incorrect because neither our Text nor our Passage tell us that Lewis’s method allows microorganisms to take in more nutrients than in their natural settings. Answer Choice C is incorrect because nothing is said about Lewis’s method affecting the cell walls of bacteria rather than the protein they produce.

Page 91: SAT Full Reading Explanations (QAS 9-19) · SAT Full Reading Explanations (QAS 9-19) SAT #9 Reading 1 1.R.9 Answer Choice C is the correct answer because Lines 19-22 say, “Moths

How to solve this? Question 44 asks, “The author of Passage 1 suggests that an advantage of the method Lewis’s team used to grow microorganisms is that it [...]?” Because this is a Paired Question, to solve it we should begin by going through the answer choices for Question 45, looking for any texts that tell us about a possible advantage to Lewis’s team’s method.

For Question 45, Answer Choice A says that “only one percent of these organisms can be grown in a lab. The rest, in staggering numbers, have remained uncultured and of limited use to medical science, until now.” This answer choice doesn’t seem to tell us too directly the benefit of Lewis’s method, but because it does suggest that the method allows for new antibiotics to be grown in culture, a possible advantage, we should keep this option. Answer Choice B says, “to do this, the team designed a gadget that sandwiches a soil sample between two membranes, each perforated with pores that allow molecules like nutrients to diffuse through but don’t allow the passage of cells.” Because this answer choice tells us about a gadget that the team developed, a possible advantage, so we should keep this option. Answer Choice C says that the team use the gadget “to trick the bacteria into thinking that they are in their natural environment.” Because this answer choice elaborates on the first one, and suggests a possible advantage of the team, we should keep this option. Answer Choice D says that, “It’s likely that teixobactin is effective because of the way it targets disease: The drug breaks down bacterial cell walls by attacking the lipid molecules that the cell creates organically.” Because this answer choice refers more to an advantage of the teixobactin itself, and not the method of the team that cultured it, we should eliminate this option.

This leaves us with Answer Choices A, B and C, which we have to use Question 44 to decide between. Before looking back at that question, however, we should note briefly the point each text makes:

● Answer Choice A — previously, helpful organisms were unable to be grown in labs, but now they can be.

● Answer Choice B — the team designed a new gadget which allowed nutrients to pass through a membrane but not cells.

● Answer Choice C — the team “tricked” the bacteria into thinking that they were in their natural environment.

For Question 44, Answer Choice A says that an advantage of the method used by Lewis is that it “identifies the requirements for soil bacteria to thrive and replicates those features in artificial soil.” Because this answer choice does seem to match both Answer Choices B and C for Question 45, we can keep this option. Answer Choice B says that an advantage of the method used by Lewis is that it “enables soil bacteria to take in more nutrients than they typically consume in natural settings.” Because the Passage only says that it enables soil bacteria to absorb similar amounts of nutrients to what they absorb naturally, not more, we should eliminate this option. Answer Choice C says that an advantage of the method used by Lewis is that it “directly affects the cell walls of bacteria rather than the proteins those bacteria produce.” This answer choice seems to reference Answer Choice D from Question 45, but we should remember that Choice D referred to an advantage of teixobactin, not Lewis’s method, so we can eliminate this option. Answer Choice D says that an advantage of the method used by Lewis is that it “allows researchers to make use of soil bacteria that they had previously been unable to exploit.” Because this answer choice matches Answer Choice A, we should keep this option.

Page 92: SAT Full Reading Explanations (QAS 9-19) · SAT Full Reading Explanations (QAS 9-19) SAT #9 Reading 1 1.R.9 Answer Choice C is the correct answer because Lines 19-22 say, “Moths

That leaves us with either Answer Choice A for Question 44 and Answer Choices B or C for Question 45, or it leaves us with Answer Choice D for Question 44 and Answer Choice A for Question 45. We should notice two things. One, that Answer Choice D for Question 44 provides more general answer choice, which is usually an advantage. Two, that Answer Choice A says that the soil used by Lewis’s team is “artificial,” when elsewhere in the Passage we’re told that the bacteria are grown in their natural environment (Line 25). For that reason, we should eliminate that option, and choose Answer Choices D and A for our correct answers.

46.R.11

Answer Choice D is the correct answer because Lines 51-56 say that “Many good antibiotic families — penicillin, streptomycin, tetracycline — come from soil fungi and bacteria and it has long been suspected that, if we could grow more types of bacteria from soil — or from exotic environments, such as deep oceans — then we might find new natural antibiotics.” The phrase, “long been suspected,” makes these Lines most clearly match Answer Choice D, which says that the author of Passage 2 would agree with the statement that the development of teixobactin “confirms a long-held belief about a potential source of new antibiotics.” Answer Choice A is incorrect because the Passage says that teixobactin is ineffective against gram-negative bacteria, not effective against it. Answer Choice B is incorrect because the development of teixobactin used an unconventional method for its development. Answer Choice C is incorrect because there’s no evidence that the development of teixobactin should cast doubt on the practicality of searching new exotic environments, and, in fact, the development of teixobactin shows that there is practicality in this kind of search.

How to solve this? The Question asks, “The author of Passage 2 would most likely agree with which statement about the development of teixobactin?” To solve this, we should go back to the Passage first to try and identify what we already know about the author of Passage 2’s perspective on the development of teixobactin. Because this question is the first that concerns Passage 2, and because it references the “development” of teixobactin, we can expect our Text to come earlier in the Passage. Moreover, using the Question Order Rule, we can predict that our Text is likely to come before Line 79.

In the first paragraph, we see that the author makes a number of points about the development of teixobactin. One, he takes an overall positive stance towards the development, saying that, “this simple and elegant methodology is their most important finding to my mind.” Two, he mentions that “it has long been suspected” that antibiotics could have been found by cultivating different types of bacteria. At this point, we should go down to our answer choices, looking for any choice that matches our Passage. Answer Choice A says that the author of Passage 2 would agree that the development of teixobactin “reveals that some antibiotics are effective against gram-negative bacteria.” Because later in the Passage, the author says that these antibiotics are in fact ineffective against gram-negative bacteria, we should eliminate this option. Answer Choice B says that the author of Passage 2 would agree that the development of teixobactin “shows that conventional methods can still yield new types of antibiotics.” However, because the development of teixobactin used unconventional methods, not conventional ones, we should eliminate this option. Answer Choice C says that the author of Passage 2 would agree that the development of teixobactin “casts

Page 93: SAT Full Reading Explanations (QAS 9-19) · SAT Full Reading Explanations (QAS 9-19) SAT #9 Reading 1 1.R.9 Answer Choice C is the correct answer because Lines 19-22 say, “Moths

doubt on the practicality of searching for new antibiotics in exotic environments.” Because the discovery of teixobactin would actually support the practicality of this kind of search, we should eliminate this option as well. Answer Choice D says that the author of Passage 2 would agree that the development of teixobactin “confirms a long-held belief about a potential source of new antibiotics.” Because this answer choice does match our understanding of the Passage, that “it has long been suspected” that antibiotics could be found in the manner that they were with teixobactin, we should keep this option. That leaves us with Answer Choice D, which we should choose as our correct option.

47.R.11

Answer Choice C is the correct answer because the question, “So, what are my caveats?” most nearly means, “What issues do I see with teixobactin?” “Misgivings” most clearly replaces “caveats” here, meaning some point of apprehension or reluctance. Answer Choice A is incorrect because the author doesn’t go on to point out “exceptions” to teixobactin, but things he finds issue with. Answer Choice B is incorrect because it’s unclear and unidiomatic to say “So what are my restrictions?” Answer Choice D is incorrect because the author doesn’t go on to give “explanations,” but some reasons why he doubts the effectiveness of teixobactin.

48.R.11

Answer Choice A is the correct answer because Lines 90-91 say, “That’s going to take five years and 500 million,” with “that” referring to the “long haul of clinical trials” required to make teixobactin available as a drug. Because the author uses this phrase to express the process still required to make teixobactin available, Answer Choice A most clearly expresses the purposes of the lines when it says that they “emphasize the scale of the effort needed to make teixobactin available for consumer use.” Answer Choice B is incorrect because the $500 million mentioned are an estimated amount for the cost of teixobactin development, not an amount that the government has committed to. Answer Choice C is incorrect because the $500 million hasn’t yet been spent on teixobactin. Answer Choice D is incorrect because the author doesn’t compare the amount of money needed to develop teixobactin with the amount needed to develop other antibiotics.

How to solve this? The Question asks, “In the last sentence of Passage 2, the author uses the phrase ‘five years and $500 million’ primarily to [...]?” To solve this, we should go back to the phrase mentioned and determine its meaning and purpose in context. Lines 90-93 say, “That’s going to take five years and $500 million and these are numbers we must find ways to reduce (while not compromising safety) if we’re to keep ahead of bacteria [...],” with “that” referring to the “long haul of clinical trials” necessary to bring teixobactin to market. The phrase “five years and $500 million,” then, is used to detail his claim about the length of time and cost necessary to make teixobactin viable, and we should expect our answer choice to reference this.

Answer Choice A says that the author uses the phrase to “emphasize the scale of the effort needed to make teixobactin available for consumer use.” Because this answer choice does match our understanding of the Text, we should keep this option. Answer Choice B says that the author uses the phrase to “criticize the level of funding that the government has committed to teixobactin development.” Because the author doesn’t tell us that this level of money has been

Page 94: SAT Full Reading Explanations (QAS 9-19) · SAT Full Reading Explanations (QAS 9-19) SAT #9 Reading 1 1.R.9 Answer Choice C is the correct answer because Lines 19-22 say, “Moths

committed by the government to teixobactin development, but is a hypothetical cost, we should eliminate this option. Answer Choice C says that the author uses the phrase to “underscore the amount of time and money that has already been spent researching teixobactin.” Because the $500 million is an expected amount of money, and not an amount that has already been spent, we should eliminate this option. Answer Choice D says that the author uses the phrase to “compare the amount of money spent developing teixobactin with the amount spent developing other antibiotics.” Because the author doesn’t compare the cost of teixobactin development with any other antibiotic, we should eliminate this option. That leaves us with Answer Choice A, which we should choose as our correct answer.

49.R.11

Answer Choice A is the correct answer because Passage 2 can best be described as evaluating the findings from Passage 1, commenting on the positive (“this simple and elegant methodology is their most important finding to my mind”), netural (“teixobactin [...] is less exciting to my mind, though it doesn’t look bad”), and negative (“So, what are my caveats?”) aspects of the finding. Answer Choice B is incorrect because the author of Passage 2 doesn’t suggest a modification to the methodology described in Passage 1, and, in fact, simply praises it as an “important finding.” Answer Choice C is incorrect because the two Passages don’t take the structure of one presenting concepts and the other giving concrete examples, because both provide concrete examples. Answer Choice D is incorrect because it’s too strong to say that Passage 2 takes a “dismissive stance” towards the findings mentioned in Passage 1.

How to solve this? The Question asks, “Which choice best describes the relationship between Passage 1 and Passage?” To solve this, we should both have a general understanding of the structure and content of each passage and then determine the relationship between them in advance. Generally, the two passages break down like this:

Passage One

● introduce the scientist Kim Lewis, who created new antibiotic with novel beneficial effects (paragraph one)

● how they approached and thought about growing these antibiotics (paragraph two) ● how they actually grew these antibiotics (paragraph three and four) ● a reason why the new antibiotic may be so effective (paragraph five)

Passage Two

● how Lewis figured out how to grow new kinds of antibiotics ● one of the antibiotics they found, and some of its positive features ● some issues and imperfections with the new antibiotic

Moreover, we can look at specific moments in either Passage that will tell us about their relationship to one another. The titles of each proves useful for these two Passages, with Passage 1 being titled “A New Antibiotic Found in Dirt Can Kill Drug-Resistant Bacteria” and Passage 2 being titled, “This New Antibiotic Is Cause for Celebration — and Caution.” Taking

Page 95: SAT Full Reading Explanations (QAS 9-19) · SAT Full Reading Explanations (QAS 9-19) SAT #9 Reading 1 1.R.9 Answer Choice C is the correct answer because Lines 19-22 say, “Moths

both the general breakdown and title of each Passage, we can say in advance that Passage 1 details the finding of a new antibiotic, in generally positive terms, while Passage 2 discusses the good and bad of these findings, in more generally skeptical terms.

Answer Choice A says that “Passage 2 offers an evaluation of the significance of the research discussed in Passage 1.” Because this answer choice matches our understanding of the relationship between the two passages, and because it’s so general, we should keep this option. Answer Choice B says that “Passage 2 suggests a modification to the methodology described in Passage 1.” Because there’s no evidence that Passage 2 ever makes this suggestion, we should eliminate this option. Moreover, we see that the author of Passage 2 actually praises the methodology of Passage 1, saying that “this simple and elegant methodology is their most important finding to my mind [...].” Answer Choice C says that “Passage 2 uses concrete examples to illustrate concepts considered in Passage 1.” Because the two Passages don’t take this structure of concept and example, and because both passages contain concrete examples, we should eliminate this option. Answer Choice D says that “Passage 2 takes a dismissive stance regarding the findings mentioned in Passage 1.” This answer choice might seem plausible, because the author of Passage 2 does express some caution towards the findings of Passage 1, but “dismissive” is a strong claim with a large burden of proof. Because there’s insufficient evidence to prove that the author is dismissive, and because Answer Choice A is so general, essentially saying that “Passage 2 gives some opinions on Passage 1,” we should eliminate this option. That leaves us with Answer Choice A, which we should choose as our correct answer.

50.R.11

Answer Choice B is the correct answer because both passages point out that teixobactin is effective against infections that are particularly resistant to antibiotics. Lines 8-10 say, “In animal tests, teixobactin proved effective at killing off a wide variety of disease-causing bacteria — even those that have developed immunity to other drugs,” and Lines 74-77 say that teixobactin “killed the tuberculosis bacterium, which is important because there is a real problem with resistant tuberculosis in the developing world.” This most closely matches Answer Choice B, which says that both passages make the point that teixobactin could be useful in “combating infections that are no longer responding to treatment with other antibiotics.” Answer Choice A is incorrect because teixobactin itself would not prove useful in making the future development of antibiotics standard, even if the methodology that produced it would. Answer Choice C is incorrect because neither passage talks about teixobactin “controlling the spread of pathogenic soil fungi.” Answer Choice D is incorrect because, again, the methodology that produced teixobactin may be useful in “shaping a new method of studying the effectiveness of antibiotics,” but teixobactin itself wouldn’t be.

How to solve this? The Question asks, “Both passages make the point that teixobactin could be useful in [...]?” To solve this, we should go back to both passages and look for what they say about the usefulness of teixobactin. Passage 1 talks about the usefulness of the antibiotic in a number of places. In Lines 7-10 it says that “teixobactin proved effective at killing off a wide variety of disease-causing bacteria — even those that have developed immunity to other drugs.” In Lines 39-43 it describes how teixobactin cured mice of respiratory tract infections with “no noticeable toxic effects.” And, finally, in Lines 44-50 it describes how teixobactin works

Page 96: SAT Full Reading Explanations (QAS 9-19) · SAT Full Reading Explanations (QAS 9-19) SAT #9 Reading 1 1.R.9 Answer Choice C is the correct answer because Lines 19-22 say, “Moths

differently than other antibiotics, breaking down a bacteria’s cell walls as opposed to its proteins. We don’t have to identify all of these points to answer this question correctly, but should have the general impression that Passage 1 believes teixobactin is particularly useful in the way it treats infections, either because of how it targets disease or in its ability to treat infections previously resistant to antibiotics. Passage 2 talks about the usefulness of teixobactin in its second paragraph, saying that it “killed Gram-positive bacteria” as well as “the tuberculosis bacterium, which is important because there is a real problem with resistant tuberculosis in the developing world.” Both of these Passages, then, comment on the usefulness of teixobactin in treating infection. Moreover, both mention that teixobactin is effective against particularly resistant infections, so we can predict that our answer choice may deal with this point.

Answer Choice A says that teixobactin could be useful in “standardizing the future development of antibiotics produced in laboratory environments.” Because teixobactin itself wouldn’t be responsible for standardizing this future, even if the methodology that produced it would be, we can eliminate this option. Answer Choice B says that teixobactin could be useful in “combating infections that are no longer responding to treatment with other antibiotics.” Because this answer choice matches our understanding of the Texts for both passages, we should keep this option. Answer Choice C says that teixobactin could be useful in “controlling the spread of pathogenic soil fungi” which neither passage discusses, so we can eliminate this option. Answer Choice D says that teixobactin could be useful in “shaping a new method of studying the effectiveness of antibiotics.” Because, again, this is more true of the methodology that produced the discovery of teixobactin and not the drug itself, we should eliminate this option. That leaves us with Answer Choice B, which we should choose for our correct answer.

51.R.11 & 52.R.11

Answer Choices C and D are the correct answers because Lines 80-82 say that teixobactin “doesn’t kill the Gram-negative opportunists as it is too big to cross their complex cell wall,” while in Lines 39-43 Passage 1 says that “mice infected with bacteria that cause upper respiratory tract infections [...] were treated with teixobactin, and the drug knocked out the infection with no noticeable toxic effects.” Taking these two together, we can say that because the mice in Passage 1 were successfully cured of their infection, then “their upper respiratory tract infections were likely not caused by gram-negative bacteria.” For Question 52, Answer Choice A is incorrect because it only comments on a belief about possible sources of new antibiotics, and doesn’t relate to the mice in Passage 1. Answer Choice B is incorrect because it only comments on the methodology used by Lewis and her team, not the mice in Passage 1. Answer Choice C is incorrect because it only makes a brief, general comment on teixobactin without giving any information relevant to the mice from Passage 1. For Question 51, Answer Choice A is incorrect because neither Passage makes the case that teixobactin makes mice less susceptible to future infections. Answer Choice B is incorrect because it doesn’t make sense to say that Gram-positive bacteria “enhanced the effectiveness of teixobactin,” since that bacteria is what teixobactin targets. Answer Choice D is incorrect because Passage 1 tells us that teixobactin doesn’t target the proteins of bacteria, but rather their cell walls.

How to solve this? Question 51 asks, “Information in Passage 2 best supports which conclusion about the mice in the experiment described in Passage 1?” To solve this, we should first go back to Passage 1 to find the conclusion it gives about the mice. Lines 39-43 tell us that

Page 97: SAT Full Reading Explanations (QAS 9-19) · SAT Full Reading Explanations (QAS 9-19) SAT #9 Reading 1 1.R.9 Answer Choice C is the correct answer because Lines 19-22 say, “Moths

“mice infected with bacteria that cause upper respiratory tract infections [...] were treated with teixobactin, and the drug knocked out the infection with no noticeable toxic effects.” From here, because this is a Paired Question, we should go back through the answer choices for Question 52, looking for any texts that tell us about something relevant to these mice.

For Question 52, Answer Choice A says, “Many good antibiotic families — penicillin, streptomycin, tetracycline — come from soil fungi and bacteria and it has long been suspected that, if we could grow more types of bacteria from soil [...] then we might find new natural antibiotics.” Because this answer choice doesn’t tell us anything relevant to the mice mentioned in Passage 1, only about the potential sources for new antibiotics, we should eliminate this option. Answer Choice B says, “This simple and elegant methodology is their most important finding to my mind, for it opens a gateway to cultivating a wealth of potentially antibiotic-producing bacteria that have never been grown before.” This answer choice also doesn’t tell us anything about the mice in Passage 1, only the usefulness of the methodology used by Lew and her team, so we should eliminate this option. Answer Choice C says, “The first new antibiotic that they’ve found by this approach, teixobactin, from a bacterium called Eleftheria terrae, is less exciting to my mind, though it doesn’t look bad.” Because this answer choice only introduces teixobactin and makes a brief comment on it, we should consider it unlikely to be our answer choice and eliminate it. Answer Choice D says “It doesn’t kill the the Gram-negative opportunists as it is too big to cross their complex cell wall,” with “it” referring to teixobactin. At first, this answer choice might seem equally unhelpful to answering our question, in which case we can go back to Question 51 to help us adjust and find a possible match in the answer choices. Or, we could make the logical inference here that if teixobactin can’t kill Gram-negative opportunists, and it did kill the bacteria that caused the infection in the mice from Passage 1, then that infection was unlikely to be caused by Gram-negative bacteria.

For Question 51, Answer Choice A says that Passage 2 supports the conclusion that “exposure to teixobactin made them less susceptible to subsequent upper respiratory tract infections.” Because neither Passage, nor any of our possible texts make the point that exposure to teixobactin prevents against future infections, we can eliminate this option. Answer Choice B says that Passage 2 supports the conclusion that “Gram-positive bacteria enhanced the effectiveness of teixobactin against their upper respiratory tract infections.” Because teixobactin targets Gram-positive bacteria, it doesn’t make sense to say that this bacteria enhances its effectiveness, so we should eliminate this option. Answer Choice C says that Passage 2 supports the conclusion that the mice’s “upper respiratory tract infections were likely not caused by gram-negative bacteria.” Because this answer choices matches well with Answer Choice D and our understanding of that Text, we should keep this option. Answer Choice D says that Passage 2 supports the conclusion that “teixobactin attacked the proteins of the bacteria that caused their upper respiratory tract infections.” Because Passage 2 tells us that teixobactin works by targeting cell walls and not cellular proteins, we should eliminate this option. That leaves us with Answer Choice C, which we should choose as our correct answer.

Page 98: SAT Full Reading Explanations (QAS 9-19) · SAT Full Reading Explanations (QAS 9-19) SAT #9 Reading 1 1.R.9 Answer Choice C is the correct answer because Lines 19-22 say, “Moths

SAT #12

SAT #12, Reading 1

Passage Outline

● a scene of reading, page by page without many breaks ● the narrator will remember these moments in the future with a crushing, heartbreaking

nostalgia ● how he treats customers, and a particular customer ● the narrator slipping into characters, probably like his dad ● the stories his father invented ● the narrator performing characters in the Brothers Karamazov

1.R.12

Answer Choice C is the correct answer, because the Passage most clearly breaks down into two parts where the first focuses on the experience of reading between Naomi and the narrator, while the second focuses on the narrator’s past and the role reading played in it. Answer Choice C most clearly matches this, when it says that the main shift in focus in the Passage is from “a description of an emotionally significant activity to a reflection on the narrator’s early experiences with that activity.” Answer Choice A is incorrect because neither the first nor the last part of the Passage recounts an amusing event. Answer Choice B is incorrect because there’s no foreshadowing of the future demise of the relationship in the Passage. Answer Choice D is incorrect because the first part of the Passage doesn’t give a character sketch of Naomi or the narrator, and doesn’t show how either changed over time.

How to solve this? The Question asks, “Over the course of the passage, the main shift in focus is from [...]?” Because this is a [Big Picture] Question focusing on shifts in the Passage, to solve this we should both have a general understanding of the Passage and its structure, and go back to try and identify any possible logical breaks. Generally, the Passage can be organized like this:

● a scene of reading, page by page without many breaks ● the narrator will remember these moments in the future with a crushing, heartbreaking

nostalgia ● how he treats customers, and a particular customer ● the narrator slipping into characters, probably like his dad ● the stories his father invented ● the narrator performing characters in the Brothers Karamazov

Page 99: SAT Full Reading Explanations (QAS 9-19) · SAT Full Reading Explanations (QAS 9-19) SAT #9 Reading 1 1.R.9 Answer Choice C is the correct answer because Lines 19-22 say, “Moths

More specifically, if we try to identify shifts in focus, we might identify the following:

● the experience of reading with Naomi ● how the narrator treats customers while reading ● embodying characters like his father, and his father ● specifically embodying characters within the novel

We don’t have to identify all of these breaks in advance to solve this question, but noticing them helps. At this point, we should go back to the answer choices given.

Answer Choice A says the passage shifts in focus from “an anecdote about an amusing event to a recollection of similar events from the narrator’s childhood.” This answer choice might seem plausible, if we take the scene with the man and the toilet paper roll as an amusing anecdote. Although it’s plausible to read it that way, the test itself doesn’t give us enough evidence to call the scene amusing. Moreover, the second part of this answer choice fails because the Passage doesn’t recollect similar events from the narrator’s childhood, so we should eliminate this answer. Answer Choice B says the passage shifts in focus from “an account of the early stages of a friendship to a foreshadowing of that friendship’s ultimate demise.” This answer choice might also seem plausible, since the first part of the Passage does seem to discuss the friendship between Naomi and the narrator, and because in Lines 19-24, the narrator comments on how “years from now I would remember this with a crushing, heartbreaking nostalgia, because of course I knew even then that I would eventually find myself standing here alone,” which might qualify as that friendship’s ultimate demise. However, three elements within this answer choice: that it’s the “early stages” of a friendship, involves a “foreshadowing,” and leads to the friendship’s “ultimate demise,” make this answer choice both stronger and more specific, and therefore more liable to be incorrect. We can keep this answer choice, but will see that compared to one of the other options, it fails for these reasons. Answer Choice C says the passage shifts in focus from “a description of an emotionally significant activity to a reflection on the narrator’s early experiences with that activity.” Because the first part of the Passage which describes the experience of reading between the narrator and Naomi can correctly be called an emotionally significant activity, and because the later part of the Passage does reflect on the narrator and his father’s experience reading, we should keep this option. Moreover, because of how general the choice is, we should consider this choice a strong candidate. Answer Choice D says the passage shifts in focus from “a character sketch of an individual to a consideration of how that individual has changed the life of the narrator.” Because the first part of the Passage doesn’t focus especially on Naomi or the narrator and sketch their character, but instead focuses on their experience of reading together, we can eliminate this option as well.

That leaves us with Answer Choice B and C. We should note how general Answer Choice C is, especially compared to Choice B which has more detail, and should ultimately choose Answer Choice C as our correct answer.

Page 100: SAT Full Reading Explanations (QAS 9-19) · SAT Full Reading Explanations (QAS 9-19) SAT #9 Reading 1 1.R.9 Answer Choice C is the correct answer because Lines 19-22 say, “Moths

2.R.12

Answer Choice D is the correct answer because Lines 35-38 say that the narrator “kept one man, who came to the counter with a single roll of toilet paper under his arm, waiting for more than a minute while I finished reading a page I had just started.” After this, the man gets frustrated and leaves the store. These lines most clearly show that “reading to Naomi interferes with some of the narrator’s responsibilities.” Answer Choices A is incorrect because it only tells us how many pages the narrator read. Answer Choice B is incorrect because it shows how Naomi takes the narrator out of his negative thoughts of the future, not how reading interferes with his business. Answer Choice C is incorrect because it shows the narrator focusing on his responsibilities at the expense of his reading.

How to solve this? The Question asks, “Which choice best supports the conclusion that reading to Naomi interferes with some of the narrator’s responsibilities?” To solve this, we should go back through each of the answer choices, looking for any texts that show this kind of interference. Answer Choice A says [...].

● Break them down ● Show how only D most clearly talks about responsibilities ● Done.

3.R.12

Answer Choice B is the correct answer because Lines 10-12 say, “I looked up every couple of pages to see if Naomi was still paying attention, and of course she was. Her attention, in fact, never seemed to waver.” This most clearly matches Answer Choice B, that Naomi is notable for her “ability to concentrate and observe.” Answer Choice A is incorrect because the Passage doesn’t show enough interaction between Naomi and others for us to say that she has empathy towards others. Answer Choice C is incorrect because it is the narrator, not Naomi, who dramatizes fiction. Answer Choice is incorrect because we’re not told about any of the hardships that Naomi experiences.

How to solve this? The Question asks, “The narrator’s descriptions of Naomi suggest that she is notable for her [...]?” To solve this, we should go back to the Passage and try to identify a specific Text which gives us a description of Naomi and explains why she is notable. Using the [Question Order Rule], we can predict that our Text will most likely come towards the beginning of the Passage, before Line 34. For this Question, finding our specific Text can be hard and might require us to go back to our answer choices first before we can identify it. However, we should eventually settle on Lines 10-16, which most clearly give us a description of Naomi. The Lines tell us that her attention “never seemed to waver” and that she was absorbing the scene that her and the narrator had created together.

Answer Choice A says that the narrator describes Naomi as notable in her “empathy toward other people.” Because neither the Passage nor our Text show Naomi being empathetic towards others, we should eliminate this option. Answer Choice B says that the narrator describes Naomi as notable in her “ability to concentrate and observe.” Because our Text does describe Naomi’s focus, that her attention never wavered, as well as her being observational and taking

Page 101: SAT Full Reading Explanations (QAS 9-19) · SAT Full Reading Explanations (QAS 9-19) SAT #9 Reading 1 1.R.9 Answer Choice C is the correct answer because Lines 19-22 say, “Moths

the whole scene in, we should keep this answer choice. Answer Choice C says that the narrator describes Naomi as notable in her “talent for dramatizing fiction.” However, because it’s the narrator and not Naomi who dramatizes the stories told, we should eliminate this option. Answer Choice D says that the narrator describes Naomi as notable in her “optimism despite many hardships.” Because the Passage doesn’t tell us about any of the hardships that Naomi experienced, and therefore none of her optimism despite them, we should eliminate this option. That leaves us with Answer Choice B, which we should choose as our correct option.

4.R.12 & 5.R.12

Answer Choices D and B are the correct answers because Lines 17-23 say, “Every time I looked at her I became aware of just how seemingly perfect this time was. I thought about how years from now i would remember this with a crushing, heartbreaking nostalgia, because of course I knew even then that I would eventually find myself standing here alone.” These lines most clearly tell us about the narrator’s perspective on his reading to Naomi, and most closely match Answer Choice D for Question 4, which says that the narrator is “somewhat ambivalent, because he knows that his reading time with Naomi will inevitably end.” For Question 5, Answer Choice A is incorrect because it only describes Naomi, and not the narrator’s thoughts about his reading to her. Answer Choice C is incorrect because it describes a scene involving a man at the store, and doesn’t clearly tell us about the narrator’s feelings toward reading to Naomi. Answer Choice D is incorrect because they only describe the narrator performing as Fyodor Karamazov, and not his thoughts in general towards reading to Naomi. For Question 4, Answer Choice A is incorrect because it doesn’t match our Text and because the Passage doesn’t tell us that the narrator needed a creative outlet. Answer Choice B is incorrect, because neither the Text nor the Passage tell us that the narrator didn’t expect Naomi to be so enthusiastic. Answer Choice C is incorrect, because we’re never told that the narrator feels regret for not being as skilled a reader as his father.

How to solve this? The Question asks “Which choice best reflects the perspective of the narrator regarding his reading to Naomi?” Because this is a Paired Question, to solve it we should begin by going through the answer choices for Question 5, looking for any texts that tell us about the narrator’s perspective on his reading to Naomi. Using the [Question Order Rule] we can predict that the Text is most likely to come before Line 34.

For Question 5, Answer Choice A says, “I looked up every couple of pages to see if Naomi was still paying attention, and of course she was.” Because this answer choice only tells us about Naomi’s attention, and not how the narrator feels about reading to her, we should eliminate this option. Answer Choice B says “Every time I looked at her I became aware of just how seemingly perfect this time was. I thought about how years from now I would remember this with a crushing, heartbreaking nostalgia, because of course I knew even then that I would eventually find myself alone.” Because the “this” in the lines refers to their act of reading together, we should keep this option because it tells us clearly how the narrator feels about reading to Naomi. Answer Choice C says, “The charm wore off when I refused to acknowledge him. He responded by slamming the roll on the counter, inches from my face, and storming out. Naomi and I read on.” Because this answer choice doesn’t tell us directly how the narrator feels about reading to Naomi, and only shows us a scene between them and a customer, we should eliminate this option. Answer Choice D says, “When Fyodor Karamazov spoke, I waved my hands wildly in the

Page 102: SAT Full Reading Explanations (QAS 9-19) · SAT Full Reading Explanations (QAS 9-19) SAT #9 Reading 1 1.R.9 Answer Choice C is the correct answer because Lines 19-22 say, “Moths

air. I grumbled in a deep baritone and tried as hard as I could to do my father proud.” Because this answer choice only shows the narrator performing a certain character in the novel and not how he felt about reading to Naomi in general, we should eliminate this option. That leaves us with Answer Choice B which we should keep as our correct answer.

Lines 17-23 say, “Every time I looked at her I became aware of just how seemingly perfect this time was. I thought about how years from now I would remember this with a crushing, heartbreaking nostalgia, because of course I knew even then that I would eventually find myself alone.” These lines tell us that although the narrator feels intensely positive about the experience of reading to Naomi, he also feels sadness at the thought that one day the experience will go away, and we should expect our answer choice to correspond to this.

For Question 4, Answer Choice A says the narrator feels “unequivocally delighted, because reading to Naomi has provided him with a much-needed creative outlet.” Because this answer choice doesn’t match our Text, nor does the Passage mention the narrator’s need for a creative outlet, we should eliminate this option. Answer Choice B says that the narrator feels “pleasantly surprised, because he did not expect Naomi to be so enthusiastic about reading.” Because this answer choice also doesn’t match our Text, nor does the Passage ever say that the narrator expected Naomi to be unenthusiastic about reading, we should eliminate this option. Answer Choice D says that the narrator feels “somewhat ambivalent, because he knows that his reading time with Naomi will inevitably end.” Because this answer choice does match our Text, that the narrator feels both that their reading together is “perfect” and also that “one day the experience will go away,” we should eliminate this option.

6.R.12

Answer Choice B is the correct answer because the word “concluded” in Line 32-34 most nearly means “finished,” because the narrator explains that he has just read or finished a word or sentence.

● She would take the book out of my hand, put her finger on the exact word or sentence I had just concluded, and hold it there until I returned.

● She would take the book out of my hand, put her finger on the exact word or sentence I had just finished, and hold it there until I returned.

Answer Choice A is incorrect because it doesn’t make sense to say that the narrator had just “decided” a word. Answer Choice C is incorrect because it doesn’t make sense to say that the narrator had just “inferred” a word. Answer Choice A is incorrect because it doesn’t make sense to say that the narrator had just “dismissed” a word. ///

Page 103: SAT Full Reading Explanations (QAS 9-19) · SAT Full Reading Explanations (QAS 9-19) SAT #9 Reading 1 1.R.9 Answer Choice C is the correct answer because Lines 19-22 say, “Moths

7.R.12

Answer Choice B is the correct answer because in Lines 43-47 say, “I slipped into the characters as I read. I grumbled and bellowed, slammed my fist onto the counter, and threw my arms wide open. I knew this was exactly what my father would have done had he been the one reading.” These lines show us that the narrator’s storytelling style comes from and is influenced by his father, and most closely match Answer Choice C which says that the narrator’s father influenced him by “providing a model for the dramatic recounting of a story.” Answer Choice A is incorrect because the Passage doesn’t tell us that the narrator’s father created an environment conducive for young people to express themselves. Answer Choice B is incorrect because the Passage doesn’t tell us directly that the narrator’s father tried to emphasize “what was most important in a life well lived.” Answer Choice D is incorrect because the Passage doesn’t tell us that the narrator’s father exposed him to classic novels written by renowned authors.

How to solve this? The Question asks, “It can reasonably be inferred from the passage that the narrator’s father influenced him by [...]?” To solve this, we should go back to the Passage and look for a Text that tells us how the narrator’s father influenced him. Using the [Question Order Rule] we can expect that our Text is most likely to come between Line 34 and 53. Going back to the Text, we find the first instance of the narrator’s father being mentioned in Lines 43-47, which read, “I slipped into the characters as I read. I grumbled and bellowed, slammed my fist onto the counter, and threw my arms wide open. I knew this was exactly what my father would have done had he been the one reading.” The Passage continues to describe the narrator’s father on him for the next several lines, although always in the context of the father as a storyteller. It can be difficult to tell in advance which Text in particular if any will best answer our question, but we should note that everything mentioned about the father concerns his storytelling as well as what the Passage doesn’t say about him.

Answer Choice A says that the narrator’s father influenced him by “creating an environment that encouraged young people to express themselves.” Because the Passage doesn’t describe the narrator’s father creating such an environment, we should eliminate this option. [...].

● Continue to describe from here.

8.R.12

Answer Choice D is the correct answer because Lines 48-55 say “He must have told me hundreds, perhaps even thousands, of stories, not just at night, but throughout the course of any given day [...]. There was no wrong time with him, or if there was, he didn’t live long enough for me to see it.” These lines tell us that the narrator’s father clearly viewed storytelling as something that “was part of the fabric of the family’s everyday life.” Answer Choice A is incorrect because the Passage doesn’t tell us that the narrator’s father was continuing a long-standing family tradition. Answer Choice B is incorrect because the stories the narrator’s father tells are primarily fictional. Answer Choice C is incorrect because the Passage doesn’t specify how the narrator’s father viewed storytelling as something that “helped children to distinguish good choices from bad ones.” How to solve this? The Question asks, “The passage indicates that the narrator’s father viewed storytelling as something that [...]?” To solve this we should go back to the Passage and

Page 104: SAT Full Reading Explanations (QAS 9-19) · SAT Full Reading Explanations (QAS 9-19) SAT #9 Reading 1 1.R.9 Answer Choice C is the correct answer because Lines 19-22 say, “Moths

try to identify any texts that will tell us clearly how the narrator’s father viewed storytelling. Using the [Question Order Rule] we can predict that our Text is most likely to come before Line 53. Because the part of the Passage that discusses the narrator’s father is rather long, it can be hard to identify a specific text that tells us how the father viewed storytelling, and we may not be able to do so until looking at the answer choices for guidance. However, if we read carefully, we can note in advance that the only lines which tell us about the father’s views on storytelling come in lines 48-55, where the narrator explains how his father told stories “throughout the course of any given day” and that “there was no wrong time with him.’

Answer Choice A says the narrator’s father viewed storytelling as something that “continued a long-standing family tradition. [...], etc.

● continue it from here.

9.R.12

Answer Choice A is the correct answer because Lines 52-55 mean that, for the narrator’s father, every time was a good time to tell a story, or that there was no “inappropriate” time. Answer Choices B, C and D are incorrect because [...].

● There was no wrong time with him, or if there was, he didn’t live long enough for me to see it.

● There was no inappropriate time with him, or if there was, he didn’t live long enough for me to see it.

● There was no unjust time with him, or if there was, he didn’t live long enough for me to see it.

● There was no immoral time with him, or if there was, he didn’t live long enough for me to see it.

● There was no inaccurate time with him, or if there was, he didn’t live long enough for me to see it.

10.R.12

Answer Choice C is the correct answer because Lines 66-68 describe how if the narrator’s father begins telling him a story he had heard before, he lets him tell it again. “His performance was that good, his love of a story that obvious,” the narrator says. This describes a scenario where a person likes a performance or story so much that they’ll rewatch it happily, which most clearly matches Answer Choice C, which says “A moviegoer happily sees a particularly memorable film for a second time.” Answer Choice A is incorrect because it describes a viewer eager to see a new episode of a favorite television show, not rewatch one that they’ve seen before. Answer Choice B is incorrect because a painter who paints at different times a day wants to capture a moment under different conditions, not recreate or consume the same thing over. Answer Choice D is incorrect because it only describes an actress rehearsing lines in order to memorize them, not consume a favorite story for the enjoyment of it.

Page 105: SAT Full Reading Explanations (QAS 9-19) · SAT Full Reading Explanations (QAS 9-19) SAT #9 Reading 1 1.R.9 Answer Choice C is the correct answer because Lines 19-22 say, “Moths

How to solve this? The Question asks “Which situation is most similar to the one described in lines 66-68?” To solve this, we should first go back to the situation described and try to characterize it generally. Lines 66-68 read, “If I had heard the story before, I let him tell it to me again. His performance was that good, his love of a story that obvious.” These lines describe a scene where someone loves a performance or show so much, that they’ll happily rewatch it, and we should go back to our answer choices looking for a choice that most clearly matches this.

[...]. Keep going from here.

SAT #12, Reading 2

Passage

● introduce a study that shows that humans perform better when being watched (paragraph one)

● the details of the study (paragraphs two and three) ● another study that contradicts the first (paragraph four) ● another study that found a solution between the first contradictory experiments ● the details of this last study

11.R.12

Answer Choice C is the correct answer because the Passage looks at three separate studies that examine the effect that an audience has on human performance. This most closely matches Answer Choice C, which says that the main purpose of the passage is to “show how various experiments helped establish and refine the understanding of an audience’s effect on performance.” Answer Choice A is incorrect because the Passage doesn’t focus on human learning, and especially not whether people learn better in groups or not. Answer Choice B is incorrect because the studies concern human performance and not learning. Answer Choice D is incorrect because, compared with C, it’s too narrow. The Passage doesn’t focus on broad theories of social interaction, but more specifically on how experiments about the effect of an audience on human performance. It also only shows three experiments over time, and not general historical development.

How to solve this? The Question asks, “The main purpose of the passage is to [...]?” For these types of questions, we should have both a general understanding of the passage based on our first reading, as well as a more specific understanding based on information in the beginning and the end of the Passage. From there, we should just go directly into the answer choices and try to eliminate and select our answer. In general, the Passage is organized like this:

● introduce a study that shows that humans perform better when being watched (paragraph one)

● the details of the study (paragraphs two and three) ● another study that contradicts the first (paragraph four) ● another study that found a solution between the first contradictory experiments ● the details of this last study

Page 106: SAT Full Reading Explanations (QAS 9-19) · SAT Full Reading Explanations (QAS 9-19) SAT #9 Reading 1 1.R.9 Answer Choice C is the correct answer because Lines 19-22 say, “Moths

More specifically, we can pay attention to the title, “Drunk Tank Pink: And Other Unexpected Forces That Shape How We Think, Feel, and Behave,” which doesn’t tell us much, except that the passage concerns forces that influence human behavior and thought. The introduction tells us about the first experiment, which “suggests that humans are often faster and stronger when they test their speed and strength in the company of other people, rather than alone.” The conclusion explores a final experiment that looks at the effect of a “cockroach audience” on the performance of cockroaches on certain tasks. Taking all these together, we can say generally that the Passage focuses on several experiments that explore this question about the effect of audiences on performance.

Answer Choice A says that the main purpose of the Passage is to “assert that the majority of people learn more effectively in a group than they do alone.” Because the Passage doesn’t make or center on this claim, and because it focuses on the effect of audiences on human performance and not learning, we should eliminate this option. Answer Choice B says that the main purpose of the Passage is to “present the contradictory results of two studies about learning that have each been considered groundbreaking.” Because, again, the Passage doesn’t focus on learning but on the effect of an audience on human performance, we should eliminate this option. Answer Choice C says that the main purpose of the Passage is to “show how various experiments helped establish and refine the understanding of an audience’s effect on performance.” Because the Passage does focus on several experiments, which do look more and more closely at the question of how audiences affect performance, so we should keep this option. Answer Choice D says that the main purpose of the Passage is to “chronicle the historical development of competing theories of social interaction.” This answer choice might seem plausible since it talks about “development” and “social interaction,” both things which the passage concerns. It’s also general, which is usually an asset for an answer choice. In this case, however, the answer choice is too general, since the Passage doesn’t quite focus on historical development but only looks at three experiments, and doesn’t focus on general “theories of social interaction,” but instead looks at the specific effect of an audience on human performance, so we should eliminate this option as well. That leaves us with Answer Choice B, which we should choose as our correct answer.

12.R.12

Answer Choice A is the correct answer because “easy” and “straightforward” best replace the word “simple” in Lines 27 and 36. In Line 27, the sentence “The task was simple but novel, and none of the children had played with fishing rods before the experiment,” emphasizes the simplicity or ease of the task. In line 36, the sentence, “Science doesn’t always tell simple stories, and other researchers challenged Triplett’s groundbreaking results well into the twentieth century,” makes the point that science often tells a complex, or multi-faceted story

Answer Choice B is incorrect because the first task given isn’t “mindless,” but simply easy to do, and the second sentence doesn’t make the point that science is insincere. Answer Choice C is incorrect because the first task isn’t described as “effortless,” and the second sentence doesn’t make the case that science is sometimes not humble. Answer Choice D is incorrect because it doesn’t make sense to describe the first task as “innocent.”

Page 107: SAT Full Reading Explanations (QAS 9-19) · SAT Full Reading Explanations (QAS 9-19) SAT #9 Reading 1 1.R.9 Answer Choice C is the correct answer because Lines 19-22 say, “Moths

13.R.12 & 14.R.12

Answer Choices B and C are the correct answers because Lines 27-29 say “The task was simple but novel, and none of the children had played with fishing rods before the experiment.” This sentence tells us that Triplett designed his experiment so that effects from the children’s prior experience with the rods would be eliminated. This most closely matches Answer Choice B which says that the design of the study eliminated the potential objection that “Variations in performance among the subjects under either audience condition may be attributed to variations in the subjects’ ages and physical development.” For Question 14, Answer Choice A is incorrect because it appears before Triplett’s fishing rod experiment is mentioned. Answer Choice B, while plausible because it matches Answer Choice C from Question 13, less directly states something Triplett did in the design of his study. Answer Choice D is incorrect because it explains the actual performance of the task in the study and doesn’t tell us anything about its design that would preclude objections. For Question 13, only Answer Choice B matches Lines 27-29, which tell us that none of the subjects had experience with the fishing rods.

How to solve this? Question 13 asks, “Based on the passage, the design of Triplett’s fishing-reel experiment most likely ruled out which potential objection to his findings?” Because this is a Paired Question, to solve it we should go first the answer choices for Question 14, looking for any texts that tell us about what objection the design of Triplett’s experiment would have ruled out. Because we’re talking about a scientific experiment, we should expect our answer choice to have to do something with how the experiment controlled for certain variables by making things standard (like age, weight, experience, etc.). We should also expect our answer choice to come earlier in the Passage, where the methodology of the experiment is discussed.

For Question 14, Answer Choice A says, “Triplett acknowledged that his observations were far from rigorous, so he conducted an experiment to show that the effect persisted in a tightly controlled lab study.” Because this answer choice comes before the mention of his fishing-reel experiment, which begins in the following lines, we should eliminate this option. Answer Choice B says, “Triplett recruited forty children, ages eight to thirteen, to complete his study in 1897.” Because this answer choice does tell us that Triplett selected children in a certain specific age range, it may work as a text about how Triplett designed his study with proper controls, so we should keep this option. Answer Choice C says, “The task was simple but novel, and none of the children had played with fishing rods before the experiment.” Because this answer choice also tells us about a possible control in the design Triplett’s experiment, that none of the children had played with fishing rods before, we should keep this option. Answer Choice D says, “They performed the task both alone and in the presence of other children, and Triplett noticed that they wound the reels faster in the presence of others.” Because this conclusion tells us more about the actual performance of the experiment, and not its design, we should eliminate this option.

That leaves us between Answer Choices B and C. If we look back at Question 13, we notice that each has a corresponding answer choice that could work, making it even harder to decide between the two. Ultimately, however, we should Answer Choice C, because it more directly highlights and focuses on a feature of the experiment that controlled for the results. It tells us directly that “none of the children had played with fishing rods before the experiment,” while the

Page 108: SAT Full Reading Explanations (QAS 9-19) · SAT Full Reading Explanations (QAS 9-19) SAT #9 Reading 1 1.R.9 Answer Choice C is the correct answer because Lines 19-22 say, “Moths

Choice B only mentions as a kind of aside the ages of the children. So, we should Answer Choice C for Question 14.

For Question 13 [...]. Go on to match it.

15.R.12 & 16.R.12

Answer Choices D and B are the correct answers because Lines 32-35 say that Triplett “concluded that an audience enables people to ‘liberate latent energy’ not normally available when they perform alone,” which most closely matches Answer Choice D from Question 15 which says that Triplett would agree that human beings “possess abilities that they are not always able to exploit.” For Question 16, Answer Choice A is incorrect because it doesn’t tell us anything that Triplett believes about human beings, only about the types of experiments he performed. Answer Choice C is incorrect because it only tells us about science in general and how other researchers challenged Triplett, not any thoughts that Triplett has about human nature. Answer Choice D is incorrect because it introduces the fact that researchers performed experiments that suggested the opposite of Triplett’s conclusions, not any thought that Triplett has about human nature in general. For Question 15, only Answer Choice D matches our Text.

How to solve this? Question 15 asks, “As presented in the passage, Triplett would most likely agree that human beings [...]?” Because this is a Paired Question, to solve ti we should begin by going through the answer choices for Question 16, looking for any texts that tell us something about how Triplett views human beings.

For Question 16, Answer Choice A says, “In dozens of experiments he pushed cyclists to ride as fast as they could on stationary bikes.” Because this answer choice only tells us about the experiment Triplett performed, and not any thoughts he has on human beings, we should eliminate this option. Answer Choice B says, “He concluded that an audience enables people to ‘liberate latent energy’ not normally available when they perform alone.” Because this answer choice does tell us a possible general view that Triplett has about people, we should keep this option. Answer Choice C says, “Science doesn’t always tell simple stories, and other researchers challenged Triplett’s groundbreaking results well into the twentieth century.” Because this answer choice also doesn’t tell us anything that Triplett believes about human beings, we should eliminate this option as well. Answer Choice D says, “While some researchers replicated Triplett’s effect — now known as the social facilitation effect — others found the opposite effect, known as social inhibition.” Because this answer choice only tells us about other researcher’s findings, and not Triplett’s views, we should eliminate this option as well.

[For Question 15, Answer Choice A says …]

Page 109: SAT Full Reading Explanations (QAS 9-19) · SAT Full Reading Explanations (QAS 9-19) SAT #9 Reading 1 1.R.9 Answer Choice C is the correct answer because Lines 19-22 say, “Moths

17.R.12

Answer Choice B is the correct answer because Lines 86-89 say, “But the cockroach athletes responded very differently to an audience when they were faced with the complex maze, reaching the goal seventy-six seconds more quickly when they were alone.” These lines tell us that “when the cockroaches in Zajonc’s experiment attempted to complete the complex maze in front of a cockroach audience” they performed it slower than when they did without. This most closely matches Answer Choice B which says that the “cockroaches completed the maze more slowly than they had without an audience.” Answer Choice A is incorrect because the Passage doesn’t tell us that the cockroaches became less hesitant in making choices and because, although we don’t this for sure, if anything the audience made the cockroaches more hesitant, since they finished the maze more slowly. Answer Choice C is incorrect because the Passage doesn’t tell us that the cockroaches appeared to communicate with their audience. Answer Choice D is incorrect because the Passage only tells us that the cockroaches performed slower in the complex maze, and not that they sought out the simpler one.

How to solve this? The Question asks, “Based on the passage, which choice best describes what happened when the cockroaches in Zajonc’s experiment attempted to complete the complex maze in front of a cockroach audience?” To solve this, we should go back to the Passage and look for where Zajonc’s experiment occurs and how the cockroaches in the complex maze performed.

Looking in the final paragraph of the Passage we should notice that Zajonc runs two experiments, one with a complex maze and the other with a simple one. So, we should make sure we double check which experiment the question is asking about, which, in this case, is the one involving the complex maze. Lines 86-89 tell us that, “[...].” Answer Choice A says [...].

18.R.12

Answer Choice D is the correct answer because it makes the most sense relative to the other answer choices. Answer Choice A is incorrect because using the terms “athletic cockroaches” and “roach spectators” doesn’t directly show how Zajonc’s two experiments are similar. This answer choice might seem plausible if it’s taken to mean that the terms show the similarity between Zajnoc’s experiments and Triplett’s experiment with the children. However, this choice fails in strength relative to some of the others. Answer Choice B says that the author most likely uses the terms to “stress the skepticism with which the author views the design of the experiment.” Because these terms don’t show any skepticism, we should eliminate this option. Answer Choice C says that the author most likely uses the terms to “create a casual tone that offsets the seriousness of the work being done.” This answer choice might seem plausible, since these words describe the cockroaches in human terms, which might be said to create a casual tone, however it fails in strength relative to Answer Choice D.

How to solve this? The Question asks, “The author uses the terms ‘athletic cockroaches,’ ‘roach spectators,’ and ‘cockroach athletes’ in the last paragraph of the passage most likely to [...]?” To solve this, we should go back to the Passage where the terms are used and try to determine what role they play in context and why the author might have included them. Going back, we find [...]. [Not too much in fact.]

Page 110: SAT Full Reading Explanations (QAS 9-19) · SAT Full Reading Explanations (QAS 9-19) SAT #9 Reading 1 1.R.9 Answer Choice C is the correct answer because Lines 19-22 say, “Moths

19.R.12

Answer Choice B is the correct answer because the graph shows the first two columns demonstrating the average time that participants the task while alone, and the first, darker column shows their performance on the easy task. It rises to about 16 seconds, which falls between the 15-20 seconds described by Answer Choice B.

20.R.12

Answer Choice D is the correct answer because the graph describes results for both attentive and inattentive audiences, which the Passage doesn’t. Answer Choice A is incorrect because the Passage does talk about the different times in which participants completed tasks. Answer Choice B is incorrect because the Passage does compare the effects of an audience on participants performing complex, or difficult tasks. Answer Choice C is incorrect because the Passage does compare how individuals performed under conditions where they were alone compared to when they were being watched by an audience.

How to solve this? The Question asks, “Information about which of the following is presented in the graph but NOT discussed in the passage?” To solve this, we should go back to the graph and look at the answer choices given, and try to find which items were and were not discussed in the graph. We can either rely on memory or search through the Passage to determine this.

Answer Choice A says that “the time needed to complete tasks” is not discussed in the Passage, but throughout it we find mentions of the differences in time that individuals took to finish tasks. For example, Lines 49 to 51 describe how some participants performed “more quickly” when they were alone than being watched by an audience, so we should eliminate this option. Answer Choice B says that information about “difficult tasks” is not discussed in the Passage, but most of the experiments in the Passage depend on the distinction between easy and complex, or difficult tasks, so we should eliminate this option as well. Answer Choice C says the Passage doesn’t provide information about “tasks performed alone,” which is also a central feature of the experiments in the Passage since they focus on differentials in performance time when participants are alone versus being watched, so we should eliminate this option as well. Answer Choice D says that the Passage doesn’t provide information about an “inattentive audience.” Going back, we should find that indeed it doesn’t, but only compares differences between results with and without an audience, without focusing on the degree of attention for these audiences. So, we should Answer Choice D as our correct answer.

Page 111: SAT Full Reading Explanations (QAS 9-19) · SAT Full Reading Explanations (QAS 9-19) SAT #9 Reading 1 1.R.9 Answer Choice C is the correct answer because Lines 19-22 say, “Moths

SAT #12, Reading 3

Passage Outline

Passage One

“Why Do Zebras Have Stripes? New Study Offers Strong Evidence.”

● the question about why zebras have stripes, and the five possible hypotheses that may explain it (paragraph one)

● scientists played these out in a statistical model, finding that only one factor consistently emerged — “to ban biting flies.” (paragraphs two and three)

● how the team conducted their study (paragraphs four to seven) ● the results, which show that stripes emerge where biting flies are most active (paragraph

eight) ● another biologist’s critique of the study for being too broad and belief that the idea will be

developed (paragraph nine and ten)

Passage Two

“Why Do Zebras Have Stripes? It’s Not for Camouflage.”

● another group of researchers conducted a large study about zebra stripes (paragraph one)

● found that stripes have more to do with temperature than with flies (paragraph two) ● one of the motivating questions for the study (paragraph three) ● more about why the researchers believe zebras need to regulate temperature in

particular (paragraphs four and five) ● further things to research to make their conclusion stronger (paragraphs six and seven)

21.R.12

Answer Choice D is the correct answer because Lines 31-34 say, “The results showed that the range of striped species overlaps with where biting flies are most active — regardless of species and where the stripes occur on the body, according to the study.” This point about striped species that holds “regardless of species,” most clearly “supports the idea that Caro’s team’s study may be relevant to animals other than zebras.” Answer Choice A is incorrect because it focuses specifically on zebras, introducing the question of why they have stripes to begin with and noting the five main hypotheses offered in answer. Answer Choice B only offers the conclusion of the study that focused specifically on zebras, and doesn’t offer anything relevant to other species.. Answer Choice C is incorrect because it only tells us about the methodology of Caro’s study of zebras, and not anything relevant to other species. [...]. How to solve this? The Question asks, “Which choice best supports the idea that Caro’s team’s study may be relevant to animals other than zebras?” To solve this [...].

Page 112: SAT Full Reading Explanations (QAS 9-19) · SAT Full Reading Explanations (QAS 9-19) SAT #9 Reading 1 1.R.9 Answer Choice C is the correct answer because Lines 19-22 say, “Moths

22.R.12

Answer Choice B is the correct answer because Lines 35-39 show a scientist saying of Caro’s study that its “approach is ‘broad brush,’ and that more specific research may be needed,” where “broad brush” most nearly means “too general.” This most closely matches Answer Choice B, which says that the term “revealed that further research will require a specialized focus.” It especially matches the latter part about “specialized focus,” a point which the Text itself highlights. Answer Choice A is incorrect because, although the term “broad brush” does mean “general,” the term isn’t used in the lines to say that the results will be relevant to other scientific disciplines. Answer Choice C is incorrect because the Lines don’t tell us that unanswerable theoretical questions were raised. Answer Choice D is incorrect because the Text doesn’t tell us that more data than can be analyzed was collected.

How to solve this? The Question asks, “In describing the new study’s approach as ‘broad brush’ (line 38), Larison suggests that the study [...]?” To solve this, we should go back to the Lines mentioned, and try to determine the use of the term in context. The Line reads, “Brenda Larison [...] said the new study’s approach is ‘broad brush,’ and that more specific research may be needed.” Because Larison claims that more specific research is needed, we can take her point to mean that the Caro’s study was too general and unfocused, and we should expect our answer choice to have something to do with that.

Answer Choice A says that Larison suggests that the study “produced results that are relevant to an array of scientific disciplines.” Although this roughly matches the designation of “general,” it too specifically says that she thinks the findings are applicable to other scientific disciplines which the Text or Passage never say, so we should eliminate this option. Answer Choice B says that Larison suggests that the study “revealed that further research will require a specialized focus.” Because this answer choice does match our understanding of the term, and because the Text does emphasize the need for more “specific research” or “specialized focus,” we should keep this option. Answer Choice C says that Larison suggests that the study “raised theoretical questions that proved unanswerable.” Because this is both a strong and specific claim that neither the Passage nor the Text tell us, we should eliminate this option. Answer Choice D says that Larison suggests that the study “collected far more data than could be analyzed properly.” Because this answer choice is also specific, and because neither the Passage or Text tell us that there was too much data to be analyzed properly, we should eliminate this option. That leaves us with Answer Choice B, which we should choose as our correct option.

23.R.12

Answer Choice A is the correct answer because Lines 65-69 say that “zebras may especially benefit from an extra cooling system because they digest food much less efficiently than other grazers in Africa. As such, zebras need to spend longer periods of time out in the heat of the midday sun, eating more food.” These Lines most clearly tell us why strips are “particularly beneficial” to zebras, and most nearly match Answer Choice A which says that zebras “endure greater exposure to the midday sun than other grazers do.” Answer Choice B is incorrect because the Passage doesn’t tell us that zebras can’t escape predators as easily as other grazers can, and, moreover, because the Passage doesn’t tell us that stripes would benefit animals who need to escape predators. Answer Choice C is incorrect because the Passage

Page 113: SAT Full Reading Explanations (QAS 9-19) · SAT Full Reading Explanations (QAS 9-19) SAT #9 Reading 1 1.R.9 Answer Choice C is the correct answer because Lines 19-22 say, “Moths

doesn’t tell us that hot, dry climates that zebras live in doesn’t provide an adequate food supply, but that they digest food less efficiently and need to spend more time in the sun. Answer Choice D is incorrect because the Passage doesn’t tell us anything about how zebras defend themselves against predator attacks.

How to solve this? The Question asks, “According to Larison in Passage 2, the reason stripes are particularly beneficial to zebras is probably because zebras [...]?” We should note that this question asks about why stripes are “particularly beneficial” to zebras, and not why they may be beneficial in general. To solve this question, we should go back to the Passage and look for a Text that tells us why stripes are particularly beneficial to zebras.

Paragraph Four in Passage Two explains why zebras “may especially benefit from an extra cooling system” like stripes. It says that, “because they digest food much less efficiently than other grazers in Africa. As such, zebras need to spend longer periods of time out in the heat of the midday sun, eating more food.” So, the reason why stripes are particularly beneficial to zebras is because they have to spend longer hours out in the sun, and we should expect our answer choice to have something to do with this.

Answer Choice A says [...].

24.R.12

Answer Choice B is the correct answer because the fourth paragraph begins by saying, “Other animals also need to regulate body temperature, or thermoregulate, Larison pointed out, but zebras may especially benefit from an extra cooling system [...]” and continues to explain why zebras in particular may need stripes. Because zebras “digest food much less efficiently than other grazers in Africa,” they “need to spend longer periods of time out in the heat of the midday sun, eating more food.” This tells us that zebras in particular benefit from stripes because their digestive systems function less efficiently, and they need to spend more time in the sun. This most closely matches Answer Choice B which says that the fourth paragraph mainly serves to “suggest that a weakness in zebra physiology might be mitigated by stripes.” Answer Choice A is incorrect because the Paragraph doesn’t compare zebras to other animals with stripes, but only discusses why zebras in particular may need stripes. Answer Choice C is incorrect because the Paragraph doesn’t tell us that Caro’s research is based on a false premise about grazing behavior. Answer Choice D is incorrect because the Passage doesn't mention zebras in warmer and cooler climates in this paragraph, but mentions them later. How to solve this? The Question asks, “In Passage 2, the fourth paragraph (lines 63-69) mainly serves to [...]?” To solve this, we should go back to fourth paragraph and try to identify its function as a whole in its context. The first sentence of the paragraph begins by explaining that other animals also need to regulate body temperature, but that only zebras have an inefficient digestive system that requires them to spend long periods of time in the sun. This would explain why zebras have stripes in the first place, because of their inefficient digestive systems, and we should expect our answer choice to relate to this. Answer Choice A says [...].

Page 114: SAT Full Reading Explanations (QAS 9-19) · SAT Full Reading Explanations (QAS 9-19) SAT #9 Reading 1 1.R.9 Answer Choice C is the correct answer because Lines 19-22 say, “Moths

25.R.12 & 26.R.12

Answer Choices D and D are the correct answers because Lines 77-81 say, “Still, the researchers have not experimentally tested the theory that black and white stripes may generate small-scale breezes over a zebra’s body, and some researchers don’t think stripes can actually create this effect,” which most clearly tells us why Larison’s study “falls short of being definitive.” These Lines most clearly match Answer Choice D for Question 25, which says that the study falls short of being definite because Larison “has yet to confirm a key assumption made in the study,” that stripes actually do allow for cooling. For Question 25, Answer Choice A is incorrect because Lines 49-53 only tell us about the conclusions of Larison’s study and not a reason why it they would fail to be definitive. Answer Choice B is incorrect because Lines 54-56 continue to elaborate on the findings of the study without offering a reason why they fail to be definitive. Answer Choice C is incorrect because Lines 70-76 tell us about the differences between the stripes of zebras living in cooler climates and those living in warmer ones, a point that supports the thermoregulation explanation and not a reason for doubting its conclusiveness. For Question 25, only Answer Choice D matches our Text that Larison’s team has failed to make the connection between stripes and thermoregulation, a central assumption behind their conclusion.

How to solve this? Question 25 asks, “Passage 2 implies that Larison’s team’s study falls short of being definitive because Larison [...]?” Because this is a Paired Question, to solve it we should begin by going through the answer choices for Question 26, looking for any texts that tell us why Larison’s findings may fall short of being definitive. Also, because this is a scientific passage, and we’re asked to look for a reason why a conclusion may be incomplete, we should expect to find a Text that highlights [an unfounded assumption, an uncontrolled variable, etc.].

For Question 26, Answer Choice A says [...].

Lines 77-81 tell us that, “Still, the researchers have not experimentally tested the theory that black and white stripes may generate small-scale breezes over a zebra’s body, and some researchers don’t think stripes can actually create this effect.” So, Larison’s conclusions fail to be definitive because they depend on the fact that stripes provide cooling effects, but that’s a fact that hasn’t yet been established, so we should expect our answer choice to have something to do with this.

For Question 25, Answer Choice A says that Larision “disregarded facts that did not support her conclusions,” a point that the Passage doesn’t make, so we should eliminate this option. Answer Choice B says that Larision used “research methods that have not proved effective,” which is also a point that neither our Text nor our Passage makes, so we should eliminate this option. Answer Choice C says that Larison “did not build on the achievements of prior research on the subject,” which again is not a point that either or Passage or Text makes, so we should eliminate this option. Answer Choice D says that Larison “has yet to confirm a key assumption made in the study,” which does match our Text that tells us that Larison doesn’t yet know if stripes provide cooling effects, so we should keep this option. That leaves us only with Answer Choice D which we should choose as our correct answer.

Page 115: SAT Full Reading Explanations (QAS 9-19) · SAT Full Reading Explanations (QAS 9-19) SAT #9 Reading 1 1.R.9 Answer Choice C is the correct answer because Lines 19-22 say, “Moths

27.R.12

Answer Choice B is the correct answer because Lines 82-88 provide a quote by Caro that saus, “‘I don’t think that you would want to have a lot of black hairs along the top of your back if you wanted to try to keep cool [...]. It’s kind of the last color that you would want.’” This most clearly tells us what Caro thinks about black stripes and matches Answer Choice B, which says that such stripes “might hinder thermoregulation in animals.” Answer Choice A is incorrect because Caro doesn’t describe black stripes as having an unknown genetic basis in the Passage. Answer Choice C is incorrect because Caro doesn’t say that black stripes occur in nature only rarely. Answer D is incorrect because Caro doesn’t say that black stripes are “aesthetically unappealing.”

How to solve this? The Question asks, “Based on Passage 2, Caro would most likely agree with which of the following statements about coloration patterns containing black stripes [...]?” To solve this, we should go back to the Passage and look for any texts from Caro’s point of view that describe his opinion on coloration patterns with black stripes. Using the Question Order Rule, we should expect our Answer Choice to come before Line 90.

28.R.12

Answer Choice B is the correct answer because the final paragraph of Passage Two says, “Caro said regions with warmer, wetter climates are particularly susceptible to several species of disease-carrying flies other than the tsetse flies that the team considered in their study, and that the relationship the researchers found may actually be a function of fly avoidance, not thermoregulation.” This Paragraph, along with what we already know about Caro’s ideas, tells us that Caro most likely uses the phrase to point out that the hot environment where zebras live also have high rates of flies. This most closely matches Answer Choice B which says that the phrase “particularly susceptible” serves to “emphasize the abundance of pests where some zebras live.” Answer Choice A is incorrect because Caro doesn’t challenge Larison’s conception of zebra’s geographic range. Answer Choice C is incorrect because Caro doesn’t make any point about the relationship between the terrain which zebras inhabit and how it impacts their avoidance of predators. Answer Choice D is incorrect because Caro doesn’t make a point about the heat of zebra’s environment affects them more negatively than other grazers.

How to solve this? The Question asks, “In Passage 2, the phrase ‘particularly susceptible’ (line 90) mainly serves to [...]?” To solve this question, we should go back to the lines and phrase referenced, looking for the role it plays in its immediate context. Lines 89-94 say that, “Caro said regions with warmer, wetter climates are particularly susceptible to several species of disease-carrying flies other than the tsetse flies that the team considered in their study, and that the relationship the researchers found may actually be a function of fly avoidance, not thermoregulation.” In context, then, the phrase “particularly susceptible” serves to point out that warmer regions also happen to have a high rate of disease-carrying flies, favoring the hypothesis that Caro supports, and we should expect our answer choice to have something to do with this.

Answer Choice A says [...].

Page 116: SAT Full Reading Explanations (QAS 9-19) · SAT Full Reading Explanations (QAS 9-19) SAT #9 Reading 1 1.R.9 Answer Choice C is the correct answer because Lines 19-22 say, “Moths

29.R.12

Answer Choice C is the correct answer because both Passages look closely at a theory which explains why zebras have stripes, with the first favoring a theory about the benefits stripes provide in avoiding flies and the second favoring a theory about the cooling benefits stripes provide. This most clearly matches Answer Choice C, which says that the purpose of both passages is to “explore the likely benefit that zebras derive from their stripes.” Answer Choice A is incorrect because neither Passage explores the evolutionary process that caused stripes to become widespread among zebra species, but simply takes it as a given fact. Answer Choice B is incorrect because neither Passage disputes a misconception about zebra stripes, instead simply offering explanatory theories themselves. Answer Choice D is incorrect because neither Passage focuses on the specific coloration of zebra species, nor do they compare these coloration patterns to other species.

How to solve this? The Question asks, “The primary purpose of both passages is to consider studies that [...]?” To solve this, we should have both a general and specific understanding of each of the Passages. Generally, both Passages can be outlined like this:

Passage One

“Why Do Zebras Have Stripes? New Study Offers Strong Evidence.”

● the question about why zebras have stripes, and the five possible hypotheses that may explain it (paragraph one)

● scientists played these out in a statistical model, finding that only one factor consistently emerged — “to ban biting flies.” (paragraphs two and three)

● how the team conducted their study (paragraphs four to seven) 7187048174

● the results, which show that stripes emerge where biting flies are most active (paragraph eight)

● another biologist’s critique of the study for being too broad and belief that the idea will be developed (paragraph nine and ten)

Passage Two

“Why Do Zebras Have Stripes? It’s Not for Camouflage.”

● another group of researchers conducted a large study about zebra stripes (paragraph one)

● found that stripes have more to do with temperature than with flies (paragraph two) ● one of the motivating questions for the study (paragraph three) ● more about why the researchers believe zebras need to regulate temperature in

particular (paragraphs four and five) ● further things to research to make their conclusion stronger (paragraphs six and seven)

Specifically, Passage One [...].

Page 117: SAT Full Reading Explanations (QAS 9-19) · SAT Full Reading Explanations (QAS 9-19) SAT #9 Reading 1 1.R.9 Answer Choice C is the correct answer because Lines 19-22 say, “Moths

Answer Choice A says [...].

30.R.12

Answer Choice A is the correct answer because both researchers in Passage 1 and Passage 2 believe that zebra stripes developed for a purpose other than camouflage, with Caro believing they evolved to deter flies and Larison believing they evolved to regulate temperature. Answer Choice B is incorrect because both researchers believe that they have new statistical evidence that explains the role of stripes, and that the question is less elusive than it was in the past. Answer Choice C is incorrect because neither researcher believes that stripes developed in response to predation. Answer Choice D is incorrect because both researchers do believe that zebra’s geographic location influences stripe pattern, with Caro believing that the environments in which zebras live have a high prevalence of disease-carrying flies and thus leading to the evolution of stripes and Larison believing that the temperature of zebra’s environment plays a role.

How to solve this? The Question asks, “Based on the description of Caro’s study in Passage 1 and that of Larison’s study in Passage 2, with which claim regarding zebras would both researchers most likely agree?” To solve this quickly we should have a decent understanding of the claims of both researchers, both what they claim and don’t claim, and then eliminate and select answers based on that. If we’re uncertain about one of the points, then we should go back to the Passage to look for evidence for or against a claim.

Answer Choice A says that both researchers would agree that “stripes in zebras are used primarily for a different purpose than to provide camouflage.” Because both researchers present hypotheses about the purpose of zebra stripes different from camouflage, fly deterrence and temperature control respectively, and therefore think that the stripes evolved for a purpose other than camouflage, we should keep this option. Answer Choice B says that both researchers would agree that “the role of stripes in zebras remains as elusive today as in the past.” Because both authors believe that they have new evidence about the purpose that stripes serve, and therefore that they don’t remain as elusive today as in the past, we should eliminate this option. Answer Choice C says that [...].

Page 118: SAT Full Reading Explanations (QAS 9-19) · SAT Full Reading Explanations (QAS 9-19) SAT #9 Reading 1 1.R.9 Answer Choice C is the correct answer because Lines 19-22 say, “Moths

31.R.12

Answer Choice D is the correct answer because both Caro and Larison build their arguments based on statistical analysis of multiple variables. Lines 7-9 say, in reference to Caro’s study, “scientists played all of these theories against each other in a statistical model” and Paragraph 7 of Passage 1 describes the variables Caro considered, ending by stating that “the data was then entered into a statistical model to find out which variable best explains striping.” Lines 45-48 describe how Larison came to her conclusions by “examining how 29 different-environmental variables influence the stripe styles of plains zebras at 16 different sites from south to central Africa.” Answer Choice A is incorrect because neither researcher is described as relying on experimental data. Answer Choice B is incorrect because neither researcher is said to have conducted field observations. Answer Choice C is incorrect because, although Passage 1 does mention that Caro used data from museums and old maps to make his conclusion, Passage 2 doesn’t describe Larison as doing the same. Moreover, even for Caro, the statistical correlations are the central evidence behind his conclusions and not the specific data from the maps.

How to solve this? The Question asks, “In the passages, Caro and Larison defend their conclusions by relying on [...]?” To solve this question, we can rely on memory, but it’s more efficient to go back to the Passages first to identify the evidence or data their each Passage relies on to build their conclusion. The second paragraph tells us in Paragraph 7 that Caro looked at a variety of variables, like “where biting flies are found, the ranges of predators like lions and hyenas, distribution of forests, and other environmental factors that could influence the evolution of stripes,” and then entered these into a “statistical model to find out which variable best explains striping.” For Passage 2, Paragraph 1 describes how Larison examined how “29 different environmental variables influence the stripe styles of plains zebras at 16 different sites from south to central Africa.” Both researchers, then, relied on statistical analysis of multiple variables to find out what patterns and correlations they could find in the data, and we should expect our answer to have something to do with that.

Answer Choice A says [...].

Page 119: SAT Full Reading Explanations (QAS 9-19) · SAT Full Reading Explanations (QAS 9-19) SAT #9 Reading 1 1.R.9 Answer Choice C is the correct answer because Lines 19-22 say, “Moths

SAT #12, Reading 4

Passage Outline

● the author wants to express his opinion on the American people (paragraph 1) ● they have many great qualities, especially when they are cultivated and refined. they

make great friends. (paragraph 2) ● but there are other influences at work (paragraph 3) ● suspicion or “Universal Distrust” is a negative quality of Americans, but one they take

pride in. (paragraph 4) ● an outsider would say that this distrust has negative consequences for the people. any

person in a high position is brought low. Americans are eager to believe negative rumors. other negative consequences. (paragraph 5)

● how the Americans would answer — that every man thinks for himself. (paragraph 6)

32.R.12

Answer Choice A is the correct answer because Lines 39-43 tell Americans that, “by repelling worthy men from your legislative assemblies, it has bred up a class of candidates for the suffrage, who, in their very act, disgrace your Institutions and your people’s choice.” These Lines most clearly match Answer Choice A , which says that Americans’ “wariness of their elected representatives has led to poor judgment on the part of citizens and to inferior candidates for public office.” Answer Choice B is incorrect because it is a strong and negative claim, which is not entirely supported, and because in Line 9 Dickens describes some American as being “refined.” Answer Choice C is incorrect because Dickens say that Americans’ independence and suspicion has actually led to poorly qualified elected officials. Answer Choice D is incorrect because it is a strong, specific and negative claim that doesn’t find enough support in the Passage — Dickens doesn’t go so far as to say the government is “undermined” or that America has “widespread instability.”

How to solve this? The Question asks, “One central idea of the passage is that Americans’?” To solve this question effectively, we should already have a general understanding of the Passage, so that we can quickly identify errors or correct assertions in the answer choices. We can also go back to the Passage to try and identify “central ideas,” but because this question asks about one of the important ideas and not the main idea, we might have trouble finding the right text in advance.

Answer Choice A says that the Passage says that Amercians’ “wariness of their elected representatives has led to poor judgment on the part of citizens and to inferior candidates for public office.” Because this answer choice generally matches the spirit of the Text, a critical stance towards Americans’ suspicion or wariness, and because we might remember a specific Text that supports it, we should keep this option. Answer Choice B says that the Passage says that Americans’ “distrust of both their fellow citizens and foreigners means that Americans will remain insular and unrefined.” Because this answer choice makes a strong and negative claim, we should be skeptical of it. Moreover, going back to the Passage, we should notice that in Line 9 Dickens describes some Americans as “refined,” so we should eliminate this option. Answer Choice C says that the Passage says that Americans’ “perceptiveness and independence have

Page 120: SAT Full Reading Explanations (QAS 9-19) · SAT Full Reading Explanations (QAS 9-19) SAT #9 Reading 1 1.R.9 Answer Choice C is the correct answer because Lines 19-22 say, “Moths

enabled Americans to be astute judges of their elected officials.” Because this answer choice contradicts a point that Dickens makes in Lines 39-43, that Americans’ independence has actually led to a poor choice of candidates, we should eliminate this option. Answer Choice D says that the Passage says that Americans’ “suspiciousness of their politicians has undermined the government and contributed to widespread instability.” This answer choice is also a fairly strong and negative claim. If we ask ourselves what could be wrong about this answer choice, and compare it to Answer Choice A, which is supported by Lines 39-43, we should ultimately eliminate this option. That leaves us with Answer Choice A, which we should choose as our correct answer.

33.R.12

Answer Choice D is the correct answer because Dickens spends Paragraphs 1-3 detailing some of the positive qualities of Americans, before turning in Paragraphs 4-6 to discussing some of the negative qualities of Americans, in particular their suspicion. This most closely matches Answer Choice D, which says that Dickens’s focus shifts from “praising admirable qualities identified with Americans to expressing concern about certain objectionable American traits” and has the favorable quality of generality. Answer Choice A is incorrect because Dickens doesn’t take a defensive stance towards Americans’ refinement, and because he doesn’t make the specific point that Americans would be better off being more refined or present it as a kind of concession. Answer Choice B is incorrect because, although Dickens does move towards questioning why Americans are suspicious in the second half of the Passage, he doesn’t describe “particular kindnesses shown to him” but only describes positive American qualities in general terms. Answer Choice C is incorrect because, although Dickens does mention the tendency of nations to take pride in their flaws, he doesn’t focus on this point and only makes it to support his general claim about Americans. Moreover, he never makes any clear recommendations towards Americans to view themselves more critically, even if it seems plausible that he might.

How to solve this? The Question asks, “Over the course of the passage, Dickens’s main focus shifts from [...]?” To solve this, we should have a general comprehension of the Passage and also try to identify specific places which could qualify as a shift in focus. Reading over the Passage, and looking closer to the middle of the text, we can identify two possible logical shifts. Between Paragraphs 3 and 4, the Passage shifts from positive qualities of Americans to negative ones. And between Paragraphs 4 and 5, the Passage shifts from the narrator’s perspective to the perspective of the “stranger” or responding “American.” We can’t know for sure, but we should expect our answer choice to have something to do with this.

Answer Choice A says [...].

Page 121: SAT Full Reading Explanations (QAS 9-19) · SAT Full Reading Explanations (QAS 9-19) SAT #9 Reading 1 1.R.9 Answer Choice C is the correct answer because Lines 19-22 say, “Moths

34.R.12

Answer Choice C is the correct answer because the word “character” in Lines 2 and 3 can best be replaced by the word “nature,” since it refers to quality or characteristics of the American people and their social system.

● But I may be pardoned, if on such a theme as the general character of the American people, and the general character of their social system [...] I desire to express my own opinions [...].

● But I may be pardoned, if on such a theme as the general nature of the American people, and the general nature of their social system [...] I desire to express my own opinions [...].

Answer Choice A is incorrect because the Lines aren’t referring to any symbol of the American people. Answer Choice B is incorrect because the Lines aren’t referring to the rank or status of the American people. And Answer Choice D is incorrect because the Lines aren’t discussing the portrayal, or representation of the American people.

[Wow, very descriptive.]

35.R.12

Answer Choice C is the correct answer because Lines 52-57 say that “any man who attains a high place among you, from the President downwards, may date his downfall from that moment; for any printed lie that any notorious villain pens, although it militate directly against the character and conduct of a life, appeals at once to your distrust, and is believed.” These Lines most clearly express the idea that “Americans too readily accept unfounded criticism of their elected leaders.” Answer Choice A is incorrect because it only describes some of the positive qualities of Americans, and not that they too easily believe criticism about their leaders. Answer Choice B is incorrect because it only continues to describe the positive qualities of American’s and the connections Dickens made with them, without discussing the idea that Americans accept unfounded criticism of their leaders. Answer Choice D is incorrect because it only asks if Americans’ suspicion of their leaders is justified, but less directly states how readily they accept unfounded criticism of their leaders.

How to solve this? The Question asks, “Which choice provides the best evidence for the idea that Americans too readily accept unfounded criticism of their elected leaders?” To solve this, we should go through each of the answer choices, looking for any texts that tell us that Americans “too readily accept unfounded criticism of their elected leaders.” Answer Choice A says [...].

Page 122: SAT Full Reading Explanations (QAS 9-19) · SAT Full Reading Explanations (QAS 9-19) SAT #9 Reading 1 1.R.9 Answer Choice C is the correct answer because Lines 19-22 say, “Moths

36.R.12

Answer Choice C is the correct answer because the word “regard” in Line 18 can be best replaced by the word “appreciation.” The line most clearly means [...]

● I never was so won upon, as by this class; never yielded up my full confidence and esteem so readily and pleasurably, as to them; never can make again, in half a year, so many friends for whom I seem to entertain the regard of half a life.

● I never was so won upon, as by this class; never yielded up my full confidence and esteem so readily and pleasurably, as to them; never can make again, in half a year, so many friends for whom I seem to entertain the appreciation of half a life.

Answer Choice A is incorrect because

Answer Choice B is incorrect

Answer Choice D is incorrect

[...?]

37.R.12

Answer Choice D is the correct answer because Lines 32-37 say that the “Universal Distrust” or skepticism of the American citizen is something with which he “plumes himself [...] even when he is sufficiently dispassionate to perceive the ruin it works; and will often adduce it, in spite of his own reason, as an instance of the great sagacity and acuteness of the people, and their superior shrewdness and independence.” This tell us that Dickens believes that the American tendency toward suspicion works against him, “in spite of his own reason.” Answer Choice A is incorrect because the Passage doesn’t say Americans’ suspicion causes them to be distrustful of their neighbors or friends, but instead focuses on their distrust of public figures like politicians. Answer Choice B is incorrect because the Passage doesn’t mention that Americans’ suspicion emboldens them to challenge each other. Answer Choice C is incorrect because, while Dickens certainly mentions Americans’ independence and suggests that its problematic, he never directly qualifies it by saying that its “too great.”

How to solve this? The Question asks, “Dickens suggests that the tendency toward suspicion exhibited by many Americans is especially problematic because it [...]?” Despite Americans’ suspicion being a general theme throughout the Passage, to solve this question we should go back to the Passage to look for a specific text that will tell us why the American tendency towards suspicion is especially problematic. Using the [Question Order Rule] we can expect that our Text will come between Lines 18 and 35.

Lines 32-37 say that [...].

Page 123: SAT Full Reading Explanations (QAS 9-19) · SAT Full Reading Explanations (QAS 9-19) SAT #9 Reading 1 1.R.9 Answer Choice C is the correct answer because Lines 19-22 say, “Moths

38.R.12

Answer Choice A is the correct answer because the word “reason” in Line 35 can best be replaced by “judgment,” emphasizing that Americans exercise their suspicion in spite of it begin a good decision, or against their better judgment.

● [The American citizen] will often adduce it, in spite of his own reason, as an instance of the great sagacity and acuteness of the people, and their superior shrewdness and independence.

● [The American citizen] will often adduce it, in spite of his own judgment, as an instance of the great sagacity and acuteness of the people, and their superior shrewdness and independence.

Answer Choices B, C and D are incorrect because the Lines don’t mean that Americans will be suspicious despite their own “explanation,” “cause,” or “defense.”

39.R.12

Answer Choice B is the correct answer because Dickens adopts the perspective of “the stranger” in order to explore the suspicion he finds characteristic of Americans. This most closely matches Answer Choice B, which says that he adopts the point of view in order to “dramatize a strong view of a certain American intellectual tendency in an imaginary conversation.” Answer Choice A is incorrect because it too strongly characterizes the view of the stranger as “harsh,” and misrepresents Dickens who is sympathetic with the point of view of the stranger. Answer Choice C is incorrect because the Passage doesn’t tell us that Dickens has abandoned his perspective on Americans’ suspicion. Answer Choice D is incorrect because the exchange between the stranger and the American is an imaginary, dramatization of an idea, not an actual incident.

How to solve this? The Question asks, “Dickens most likely adopts the point of view of ‘the stranger’ (line 38) in order to [...]?” To answer this, we should have a general understanding of the part of the passage where “the stranger” is mentioned, and then work through the answer choices to eliminate and select from the answers. The stranger is introduced in Line 38, where the text says, “‘You carry,’ says the stranger, ‘this jealousy and distrust into every transaction of public life,’” and continues to discuss the character and consequences of American suspicion. Line 65 seems to present an American answer to the claims the stranger makes. It can be hard in advance to predict what our answer choice will look like, but we can say that this scene involving the stranger doesn’t appear to be an actual incident, but seems to be created to illustrate Dickens’ points, and we might expect our answer choice to refer to that.

Answer Choice A says Dickens adopts this point of view in order to “expose harsh criticisms of Americans that he believes to be unfounded.” Because this answer choice makes a strong and negative claim without the requisite support, and because Dickens seems to be sympathetic and not opposed to the point of view of the stranger, we should eliminate this option. Answer Choice B says that Dickens adopts this point of view in order to “dramatize a strong view of a certain American intellectual tendency in an imaginary conversation.” Because this answer choice does match our understanding of the Text, that Dickens creates an imaginary

Page 124: SAT Full Reading Explanations (QAS 9-19) · SAT Full Reading Explanations (QAS 9-19) SAT #9 Reading 1 1.R.9 Answer Choice C is the correct answer because Lines 19-22 say, “Moths

conversation to illustrate the quality of American suspicion, we should keep this option. Answer Choice C says that Dickens adopts this point of view in order to “articulate a view of Americans that he previously held and has since abandoned.” Because the Passage doesn’t tell us that Dickens has changed his mind on the issue of American suspicion, we should eliminate this option. Answer Choice D says that Dickens adopts this point of view in order to “report on a conversation that he had with a particular American citizen.” Because the scene involving the stranger isn’t presented as an actual incident, but as an imaginary scene, we should eliminate this option. That leaves us with Answer Choice B which we should choose as our correct answer.

40.R.12 & 41.R.12

Answer Choices B and A are the correct options because Lines 39-43 say that, “By repelling worthy men from your legislative assemblies, it has bred up a class of candidates for the suffrage, who, in their very act, disgrace your Institutions and your people’s choice,” which tells us clearly something that Dickens thinks about American political leaders. This text, especially the claim that “worthy men” are repelled, most closely matches Answer Choice B from Question 40. For Question 41, Answer Choice B is incorrect because it tells us more about the fickleness of American citizens than something about their leaders. Answer Choice C is incorrect because no answer choice matches the claim that American political leaders are always brought down after being established. Answer Choice D is incorrect because it has no answer choice to match it and because it more clearly talks about American citizens than their leadership. For Question 40, Answer Choice A is incorrect because neither our Text nor any part of the Passage specifically claim that Amercina political leaders are “unresponsive,” even if it does characterize them as unworthy. Answer Choice C is incorrect because neither our Text nor our Passage tell us that American political leaders are “too changeable in their judgments.” Answer Choice D is incorrect because neither the Text nor the Passage takes a positive stance towards American political leaders, or says specifically that they’re “well positioned to wield their authority.”

How to solve this? Question 40 asks, “In the passage, Dickens implies that American political leaders are often [...]?” Because this is a Paired Question, to solve this we should begin by going through the answer choices for Question 41 and looking for a text that tells us something about American political leaders.

For Question 41, Answer Choice A says, “By repelling worthy men from your legislative assemblies, it has bred up a class of candidates for the suffrage, who, in their very act, disgrace your Institutions and your people’s choice.” Because this answer choice does tell us something about American political leaders, we should keep this option. Answer Choice B says, “It has rendered you so fickle, and so given to change, that your inconstancy has passed into a proverb.” Because this answer choice more clearly tells us about the American citizens than the political leaders they choose, we should discount against this option unless no other choices work. Answer Choice C says, “because you directly reward a benefactor, or a public servant, you distrust him, merely because he is rewarded; and immediately apply yourselves to find out, either that you have been too bountiful in your acknowledgment, or he remiss in his deserts.” Because this answer choice does tell us something about American political leaders, we should keep this option. Answer Choice D says, “You will strain at a gnat in the way of trustfulness and confidence, however fairly won and well deserved; but you will swallow a whole caravan of

Page 125: SAT Full Reading Explanations (QAS 9-19) · SAT Full Reading Explanations (QAS 9-19) SAT #9 Reading 1 1.R.9 Answer Choice C is the correct answer because Lines 19-22 say, “Moths

camels, if they be laden with unworthy doubts and mean suspicions.” This obliquely tells us something about American political leaders, but only indirectly so we should eliminate this option unless none of the other choices work.

At this point, we have two possible Texts, either from Answer Choice A or Answer Choice C. We should make a mental note of the point of either Text, and then bring those with us to Question 40 to help us decide. Answer Choice A, again, tells us that American political leaders are unworthy men, who disgrace America’s political institutions, so we might expect an answer related to that. Answer Choice C tells us that American political leaders, once elevated, are quickly torn down by their citizens, so we might expect an answer choice to relate to this.

For Question 40, Answer Choice A says that Dickens implies that American leaders are “unresponsive to their constituents.” Because neither of our Texts or Passage tell us that political leaders are “unresponsive to their constituents,” we should eliminate this option. Answer Choice B says that Dickens implies that American leaders are “undeserving of the positions they hold.” Because this answer choice does match one of our Texts, we should keep this option. Answer Choice C says that Dickens implies that American leaders are “too changeable in their judgments.” Because none of our Texts match this option, we should eliminate it. Answer Choice D says that Dickens implies that American leaders are “well-positioned to wield their authority.” [We should eliminate as well…]

42.R.12

Answer Choice A is incorrect because Lines 47-49 say that, “because directly you reward a benefactor, or a public servant, you distrust him, merely because he is rewarded,” which most clearly tells us why Americans quickly turn against the elected leaders. This Text most clearly matches Answer Choice A which says that Americans turn against their elected leaders because Americans “are suspicious that anyone who is elected might not be worthy of the honor.” Answer Choice B is incorrect because the Passage doesn’t tell us that Americans have learned that elected representatives are corrupt, just that they believe that they are. Answer Choice C is incorrect because the Passage doesn’t say that elected officials abandon their political platforms after being elected, or that Americans believe that they do. Answer Choice D is incorrect because, although the Passage does tell us very clearly that American are distrustful, it doesn’t specifically say that they’re distrustful of the institutions of American government, but that they’re distrustful of the specific political leaders.

How to solve this? The Question asks, “According to the passage, Amecians quickly turn against the elected leaders they had recently idolized because Americans [...]?” To solve this question, we should go back to the Passage to find any texts that tell us why Americans turn against their elected leaders. Because this is the last question for the Passage, we should expect our text to come later in the Passage.

Lines 45-59 tell us about the American tendency to go against their elected leaders, while Lines 47-49 tell us precisely why they do, saying “because directly you reward a benefactor, or a public servant, you distrust him, merely because he is rewarded.” This tells us that Americans turn against their political leaders simply because they distrust anybody who’s been reward, and we should expect our answer choice to have something to do with this option.

Page 126: SAT Full Reading Explanations (QAS 9-19) · SAT Full Reading Explanations (QAS 9-19) SAT #9 Reading 1 1.R.9 Answer Choice C is the correct answer because Lines 19-22 say, “Moths

Answer Choice A tell us that [...].

SAT #12, Reading 5

Passage Outline

● “Gut Bugs May Boost Flu Shot’s Effects” ● new study shows that immune response of vaccines depend on gut bacteria (paragraph

1) ● how this evidence was first found (paragraph 2) ● questions about how this effect works and some hypotheses (paragraph 3) ● another study to determine which hypothesis is true (paragraphs 4 and 5) ● another study which supports the finding (paragraph 6) ● further experiments and questions to answer (paragraph 7)

43.R.12

Answer Choice A is the correct answer because the Passage discusses the experiments behind and questions surrounding the finding that gut bacteria affects a vaccine’s effectiveness. Answer Choice A most clearly matches this and says that the purpose of the passage is to, “discuss research on a factor that influences vaccine effectiveness.” Answer Choice B is incorrect because the passage doesn’t discuss focus on the development of vaccines themselves or their safety and efficacy. Answer Choice C is incorrect because the Passage looks at the effect of bacteria on a vaccine’s effectiveness and not its role in causing illness. Answer Choice D is incorrect because the Passage doesn’t discuss the genetic components of the flu virus.

How to solve this? Question 43 asks, “the primary purpose of the passage is to [...]?” To solve this, we should have both a general understanding of the Passage as well as identify any specific sentences that tell us the purpose of the passage. In general, the Passage can be broken down like this:

● new study shows that immune response of vaccines depend on gut bacteria (paragraph 1)

● how this evidence was first found (paragraph 2) ● questions about how this effect works and some hypotheses (paragraph 3) ● another study to determine which hypothesis is true (paragraphs 4 and 5) ● another study which supports the finding (paragraph 6) ● further experiments and questions to answer (paragraph 7)

Specifically, we can look at the title of the Passage, “Gut Bugs May Boost Flu Shot’s Effects,” as well as the introduction which ends by telling us that, “the findings could help explain variation in the response to the vaccine and suggest ways to maximize its effectiveness.” Taking all these together, then, we can say in advance that the purpose of the Passage is to look at findings that describe how gut bacteria affects the effectiveness of vaccines, and should expect our answer choices to have something to do with this.

Page 127: SAT Full Reading Explanations (QAS 9-19) · SAT Full Reading Explanations (QAS 9-19) SAT #9 Reading 1 1.R.9 Answer Choice C is the correct answer because Lines 19-22 say, “Moths

Answer Choice A says that, [...].

44.R.12

Answer Choice C is the correct answer because the words “curious” and “unexpected” in the second paragraph tell us that the scientists were surprised by their findings, and specifically were surprised by the fact that, “in a long list of genes associated with strong vaccine response, the researchers found an unexpected one: the gene that codes for a protein called toll-like receptor 5 (TLR5).” This most clearly matches Answer Choice C which says that Pulendran and his colleagues were “surprised to find a link between the expression of the gene coding for TLR5 and relatively high counts of trivalent vaccine-specific antibodies.” Answer Choice A is incorrect because, although it is true that Pulendran and his colleagues didn’t anticipate their findings, it wasn’t because they were surprised by the prevalence of gene coding for those who simply received the vaccine, but instead by the prevalence in those who had a strong immune response. Answer Choice B is incorrect because the Passage doesn’t say that they had any assumption about the gene-coding for TLR5 and an immune response, and especially not about a weak response for those who have the gene. Answer Choice D is incorrect because the Passage doesn’t tell us that they had previously overlooked this connection.

How to solve this? The Question asks, “In the second paragraph (lines 10-26), the author uses the words ‘curious’ and ‘unexpected’ primarily to suggest that Pulendran and his colleagues [...]?” To solve this, we should go back to the Passage to look for how those words operate in context and the point that they help the second paragraph to make. Going back we see [...].

45.R.12 & 46.R.12

Answer Choices B and C are the correct answers because Lines 23-26 say that, “In a long list of genes associated with strong vaccine response, the researchers found an unexpected one: the gene that codes for a protein called toll-like receptor 5 (TLR5),” which tells us something clearly about the subjects “who had strong immune responses to the trivalent vaccine.” This Text most closely matches Answer Choice B from Question 45, which says that these subjects “had a similar pattern of gene expression that was not found in subjects with weak responses to the vaccine.” For Question 46, Answer Choice A is incorrect because it only tells us the preliminary findings of the research study, without focusing clearly on the human subjects who exhibited a strong immune response. Answer Choice B is incorrect because it only tells us about what the researchers were originally looking for in their study and not something about the subjects. Answer Choice D is incorrect because it only describes what TLR5 is, and not about subjects with a strong immune response. For Question 45, Answer Choice A is incorrect because, while the Passage does elsewhere imply that subjects with a strong immune response have higher counts of white blood cells in response to the trivalent vaccine than other subjects, it doesn’t compare their reaction to reactions from other vaccines. Answer Choice C is incorrect because neither the Passage nor our Text tell us that these subjects showed immunity to other flu strains. Answer Choice D is incorrect because, while the Passage does suggest that the effect of bacteria on immune response occurs in both mice and humans, it never tells us that the human subjects showed bacteria that had only previously been seen in mice.

Page 128: SAT Full Reading Explanations (QAS 9-19) · SAT Full Reading Explanations (QAS 9-19) SAT #9 Reading 1 1.R.9 Answer Choice C is the correct answer because Lines 19-22 say, “Moths

How to solve this? Question 45 asks, “Which statement regarding subjects who had strong immune responses to the trivalent vaccine can be most reasonably inferred from the passage?” Because this is a Paired Question, to solve this we should first go through the answer choices for Question 46, looking for any texts that tell us about the “subjects who had strong immune responses to the trivalent vaccine.”

For Question 46, Answer Choice A says [...].

47.R.12

Answer Choice D is the correct answer because Lines 32-35 say, “Maybe, the group thought, B cells — the white blood cells that produce antibodies — receive a signal from bacteria that boosts their activity,” and the following Lines tell us that, “to explore that possibility, the researchers designed a new study using mice.” This most clearly tells us “the hypothesis that Pulendran’s group tested in their experiment with mice” and matches Answer Choice D which says that the group tested whether “the strength of the immune response following vaccination is related to the white blood cells’ detection of signals from intestinal bacteria.” Answer Choice A is incorrect because the Passage tells us that the scientists wanted to test whether gut bacteria boosted B cells’ production of antibodies, not whether they required them. Answer Choice B is incorrect because neither the Passage nor our Text tell us that the scientists wanted to find out whether “vaccines containing active viruses must be accompanied by adjuvants” to activate a strong response. Answer Choice C is incorrect because the mechanism the Passage describes and which the scientists wanted to test doesn’t say that vaccines depend on TLR5 to stimulate flagellin activity, but that vaccines are boosted when TLR5 can detect already existent flagellin activity.

How to solve this? The Question asks, “According to the passage, which statement best explains the hypothesis that Pulendran’s group tested in their experiment with mice?” To solve this, we should go back to the Passage to find a text that tells us the hypothesis they wanted to test with the mice. Using the [Question Order Rule], we should expect our Text to come before Line 63 and probably somewhere after Line 20. Lines 36 tell us, “to explore that possibility, the researchers designed a new study using mice,” which tells us what the scientists wanted to test in their study with mice. Looking back at the previous sentence to find out what “that possibility” refers to, we find Lines 32-35 which say, “Maybe, the group thought, B cells — the white blood cells that produce antibodies — receive a signal from bacteria that boosts their activity,” and the following Lines tell us that, “to explore that possibility, the researchers designed a new study using mice.” This tells us that [...].

Page 129: SAT Full Reading Explanations (QAS 9-19) · SAT Full Reading Explanations (QAS 9-19) SAT #9 Reading 1 1.R.9 Answer Choice C is the correct answer because Lines 19-22 say, “Moths

48.R.12

Answer Choice C is the correct answer because the last paragraph makes the point that uncertainties around the mechanism talked about remain, and the question about “where and how the interaction happens ‘is a huge mystery [...].’” Answer Choice C most clearly matches this and says that the main purpose of the last paragraph is to “emphasize the fact that further research into the nature of the vaccine response is necessary.” Answer Choice A is incorrect because no new discoveries are mentioned in the last paragraph and because the Passage doesn’t ever question the importance of the vaccine response. Answer Choice B is incorrect because the scientists express uncertainty about the exact mechanism at work, but not about their data at large. Answer Choice D is incorrect because, although the last paragraph does mention the need for further research, it never explicitly urges other scientists to continue the work.

How to solve this? The Question asks, “The main purpose of the last paragraph is to [...]?” To solve this, we should go back to the last paragraph, understand its meaning and then think about its purpose in the Passage as a whole.

The last paragraph begins by making a claim about the uncertainty of the bacteria involved in the vaccine response, and a possible explanation. Then Pulendran offers his own explanation, until finally concluding that “where and how the interaction happens ‘is a huge mystery,’ he says. ‘We don’t have the full answer.’” This paragraph then, raises some questions about the topic of the study and makes the case that further research is required. This is common for the conclusion of science-based Passages, and we should predict that our answer will have something to do with the uncertainties that remain and the necessity for further study.

Answer Choice A says that the purpose of the last paragraph is to “question the overall importance of the vaccine response in light of new discoveries.” However, because the question in the last paragraph is about the mechanism behind the bacteria and immune response and not about the overall importance of the vaccine, we should eliminate this option. Answer Choice B says that the main purpose of the paragraph is to “suggest that the researchers who studied the vaccine response were uncertain about their data.” However, because the uncertainty in the paragraph concerns the mechanism and not the overall data, we should eliminate this option. Answer Choice C says that the purpose of the paragraph is to “emphasize the fact that further research into the nature of the vaccine response is necessary.” Because this answer choice is general, and tells us that the paragraph demonstrates the need for further research, we should keep this option. Answer Choice D says that the purpose of the paragraph is to “urge other scientists to consider furthering the existing work regarding the vaccine response.” Because the paragraph never explicitly urges other scientists to continue the work, we should eliminate this option as well. That leaves us with Answer Choice C which we should as our correct answer.

Page 130: SAT Full Reading Explanations (QAS 9-19) · SAT Full Reading Explanations (QAS 9-19) SAT #9 Reading 1 1.R.9 Answer Choice C is the correct answer because Lines 19-22 say, “Moths

49.R.12

Answer Choice A is the correct answer because the word “prompting” in Line 63 means starting, or “inciting,” and emphasizes which bacteria begins the vaccine response.

● No specific type of bacteria seemed more important than another in prompting the vaccine response.

● No specific type of bacteria seemed more important than another in inciting the vaccine response.

Answer Choice B is incorrect because it doesn’t make sense to say that bacteria are “suggesting” a vaccine response. Answer Choice C is incorrect because the line doesn’t say that the bacteria is “shortening” the vaccine response. And Answer Choice D is incorrect because the line doesn’t say that the bacteria are “refreshing” the response.

50.R.12

Answer Choice A is the correct answer because the column which corresponds to “germ-free mice 7 days after vaccination” is the fourth column, which rises to about .20.

51.R.12

Answer Choice A is incorrect because both the dotted and solid lines in Figure 2, which refer to the mice without Tlr5 and control mice, respectively, show an increase in antibodies between 7 and 28 days. Answer Choice B is incorrect because the mice without Tlr5 showed a relative decrease from their highest point at 85 days. Answer Choice C is incorrect because the control mice continued increasing after 28 days. And Answer Choice D is incorrect because neither type of mouse reached their lowest point 85 days after vaccination.

52.R.12

Answer Choice D is the correct answer because Figure 1 shows the relative concentration of antibodies for different types of mice 7 days after vaccination. Figure 2 shows that from 7 days onwards, both control mice and mice without Tlr5 continued to grow in their relative concentration up to and past 14 days. This makes it most likely that the concentration levels for all four mice would also be higher and most closely matches Answer Choice D which says, “The bars for all four groups of mice would be higher.” Answer Choice A is incorrect because Figure 1 shows mice with Tlr5 increasing in their relative concentration of antibodies from 7 to 14 days, not decreasing. Answer Choice B is incorrect because Figure 1 never shows mice with Tlr5 exceeding the control mice in terms of their relative concentration of antibodies. Answer Choice C is incorrect because, while it is true that bar for the control mice would be higher, it’s not true that it alone would be higher.

Page 131: SAT Full Reading Explanations (QAS 9-19) · SAT Full Reading Explanations (QAS 9-19) SAT #9 Reading 1 1.R.9 Answer Choice C is the correct answer because Lines 19-22 say, “Moths

SAT #13 Reading, Full

SAT #13, Reading 1

Passage Outline

● introduction to Tchartkoff (paragraph one) ● quotation from his art professor — a warning to be patient and avoid being too

fashionable as an artist (paragraph two) ● Tchartkoff’s reflection on this — his own development as an artist, and his thoughts

about fashionable artists who make a fortune while he has no food (paragraph three) ● quotation from Tchartkoff in response to the professor about the purpose of being

patient, avoiding being fashionable, not having money, etc. (paragraph four)

1.R.13

Answer Choice B is the correct answer because the Passage primarily focuses on the reflections of an artist on the value of being patient in art, avoiding commercial incentives, rejecting popular fashion, and putting the quality of the art first. This most closely matches Answer Choice B which says that the Passage is primarily focused on the “struggles of a young artist conflicted about his values.” Answer Choice A is incorrect because, while the professor does play a role in the Passage, his influence doesn’t take central focus and he is more used as a catalyst for the artist’s own thoughts about art, which differ from that of his professor. Answer Choice C is incorrect because it makes a strong and specific claim that isn’t supported by the Passage. Answer Choice D is incorrect because the Passage focuses less on the personal life of the artist, including relationships and hobbies and his biography, and instead focuses more narrowly on his concerns about art and his values.

How to solve this? The Question asks, “The passage is primarily focused on the [...]?” To solve this, we should have a general grasp of the entire Passage and its content. Typically, we should also go back to the Passage to look for more specific evidence about its focus, but, because this is a literature Passage, we’ll have more trouble identifying these types of specific claims, and should focus more on overall comprehension. Generally, we can look at the Passage like this,

● introduction to Tchartkoff (paragraph one) ● quotation from his art professor — a warning to be patient and avoid being too

fashionable as an artist (paragraph two) ● Tchartkoff’s reflection on this — his own development as an artist, and his thoughts

about fashionable artists who make a fortune while he has no food (paragraph three) ● quotation from Tchartkoff in response to the professor about the purpose of being

patient, avoiding being fashionable, not having money, etc. (paragraph four)

We can say, then, that the Passage primarily focuses on the questions and reflections of Tchartkoff about his art and the values he holds in relation to it.

Answer Choice A says that the Passage is primarily focused on the “influence of a professor on one of his students.” Although the Professor does play an important role, the Passage doesn’t

Page 132: SAT Full Reading Explanations (QAS 9-19) · SAT Full Reading Explanations (QAS 9-19) SAT #9 Reading 1 1.R.9 Answer Choice C is the correct answer because Lines 19-22 say, “Moths

primarily focus on his influence, but instead on the artist’s consideration of art and how to approach it. Answer Choice B says the Passage is primarily focused on the “struggles of a young artist conflicted about his values.” Because this answer choice matches our understanding of the Passage, and because the Passage does focus on an artist who struggles with considerations about artistic integrity and financial incentives, we should keep this option. Answer Choice C says the Passage is primarily focused on the “descent of a character into hopelessness and madness.” Because the Passage doesn’t give any evidence about this character descending into madness, a very strong and specific claim, we should eliminate this option. Answer Choice D says the Passage is primarily focused on the “personal life of a young painter in relation to his art.” This answer choice might seem plausible at first, but, because the Passage focuses more narrowly on artist questioning his artistic values and not on the details of his personal life, we should eliminate this option.

2.R.13

Answer Choice C is the correct answer because the first paragraph says, “Young Tchartkoff was an artist of talent, which promised great things: his work gave evidence of observation, thought, and a strong inclination to approach nearer to nature.” This paragraph serves to introduce the reader to Tchartkoff and describe some of his artistic characteristics. This most closely matches Answer Choice C which says that the paragraph serves to establish the “main character’s defining artistic traits.” Answer Choice A is incorrect because there’s no irony in the first paragraph of the Passage. Answer Choice B is incorrect because it doesn’t yet introduce the central conflict depicted in the passage, between artistic integrity and financial need. Answer Choice D is incorrect because only Tchartkoff is discussed in the first paragraph, and we’re not shown any relationship between two characters.

How to solve this? The Question asks, “the first paragraph serves mainly to establish the [...]?” To solve this, we should go back to the first paragraph to get a clearer grasp of its meaning and purpose in the Passage as a whole. It reads [...].

3.R.13 & 4.R.13

Answer Choices A and B are the correct answers because Lines 11-15 say, “At present your colouring begins to assert itself too loudly; and your drawing is at times quite weak; you are already striving after the fashionable style, because it strikes the eye at once,” which tells us most clearly what the professor thinks of “fashionable art” and, by contrast, what he thinks about “great art.” This Text most clearly matches Answer Choice A which says that the professor thinks great art should be “technically accomplished and not garish,” where “technically accomplished” references the “drawing” in the Text, and “garish” references the loud and assertive “colouring” in the Text. For Question 4, Answer Choice A is incorrect because it tells us more about the professor’s perspective on Tchartkoff and not art in general. Answer Choice C is incorrect because it also focuses primarily on Tchartkoff and doesn’t tell us anything about how the professor feels about great art. Answer Choice D is incorrect because it comes from the perspective of Tchartkoff and doesn’t give us any evidence for the professor’s view of great art. For Question 3, Answer Choice B is incorrect because, while the professor does criticize “fashionable art,” he never says that great art shouldn’t be overly popular or that is has to be pleasing to the eye. Answer Choice C is incorrect because neither the Passage nor the Text

Page 133: SAT Full Reading Explanations (QAS 9-19) · SAT Full Reading Explanations (QAS 9-19) SAT #9 Reading 1 1.R.9 Answer Choice C is the correct answer because Lines 19-22 say, “Moths

describe the professor’s claim that great art has to be original or spontaneous. Answer Choice D is incorrect for the same reason; neither the Passage nor the Text tell us that the professor believes great art has to be representative of the artist’s beliefs or morals.

How to solve this? Question 3 asks, “The passage suggests that Tchartoff’s professor believes that great art should be [...]?” Because this is a Paired Question, to solve it we should begin by going through the answer choices for Question 4, looking for any Text that tells us about Tchartkoff’s professor’s view on great art.

For Question 4, Answer Choice A says [...].

5.R.13

Answer Choice B is the correct answer, because the word “fashionable” in the lines mentioned most clearly means “trendy,” emphasizing the current and popular nature of the word. Answer Choice A seems plausible, but is incorrect because it fails to emphasize the recent or current quality of the word fashionable, and because the professor doesn’t criticize art for being stylish, but for following current fashions. Answer Choice C is incorrect because, while it does seem to emphasize the current time, it fails to capture the arbitrary and changing sense of the word “fashionable.” Answer Choice D is incorrect because the lines don’t mean conventional.

● See to it that you do not become a fashionable artist. ● Your drawing is at times quite weak; you are already striving after the fashionable style,

because it strikes the eye at once. ● It is seductive to paint fashionable little pictures and portraits for money; but talent is

ruined, not developed, by that means.

● See to it that you do not become a trendy artist. ● Your drawing is at times quite weak; you are already striving after the trendy style,

because it strikes the eye at once. ● It is seductive to paint trendy little pictures and portraits for money; but talent is ruined,

not developed, by that means.

6.R.13

Answer Choice C is the correct answer because Lines 36-39 say, “he already saw something in [the ancient pictures], though in private he did not agree with the professor that the secrets of the old masters are irremediably lost to us.” This Text most clearly tells us about “one point of disagreement” between Tchartkoff and his professor, and most clearly matches Answer Choice C which says that the two disaggree over whether “nineteenth-century painters had been able to expand on the insights of the old masters.” Answer Choice A is incorrect because neither character in the Passage makes the claim that “making money from selling paintings destroys artistic integrity.” Answer Choice B is incorrect because both characters would agree with the claim that “fashionable artists are capable of making enough money from their art to support themselves.” Answer Choice D is incorrect because neither character in the Passage expresses a view about whether “nonprofessional painters are capable of producing serious artworks.”

Page 134: SAT Full Reading Explanations (QAS 9-19) · SAT Full Reading Explanations (QAS 9-19) SAT #9 Reading 1 1.R.9 Answer Choice C is the correct answer because Lines 19-22 say, “Moths

How to solve this? The Question asks, “According to the passage, one point of disagreement between Tchartkoff and his professor concerns whether [...]?” To solve this, we should go back to the Passage and find a text that tells us about a disagreement between Tchartkoff and his professor. Using the [Question Order Rule], we should expect our Text to come between Lines 18 and 50.

Going back to the Passage, we should notice Lines 36-39 which tell us clearly something about their disagreement and says “he already saw something in [the ancient pictures], though in private he did not agree with the professor that the secrets of the old masters are irremediably lost to us.” Although this point doesn’t seem to be central for the Passage overall, because it so clearly tells us about a point of disagreement between the two characters, we should expect that it will be our Text and see if any answer choice corresponds to it.

Answer Choice A says [...].

7.R.13

Answer Choice A is the correct answer because the word “want” in Line 50 most clearly means “need,” emphasizing [...]. Answer Choice B is incorrect because “absence” wouldn’t make sense if replaced in the sense, because it would need to specify the absence of something (maybe?). Answer Choice C is incorrect because the Lines aren’t emphasizing when [...].

● But when dire want arrived, when he had no money wherewith to buy brushes and colours, when his implacable landlord came ten times a day to demand the rent for his rooms, then did the luck of the wealthy artists recur to his hungry imagination.

● But when dire need arrived, when he had no money wherewith to buy brushes and colours, when his implacable landlord came ten times a day to demand the rent for his rooms, then did the luck of the wealthy artists recur to his hungry imagination.

8.R.13 & 9.R.13

Answer Choices D and D are the correct answers because Lines 74-76 say, “Why do I worry, and toil like a learner over the alphabet, when I might shine as brightly as the rest, and have money, too, like them?” This Text most clearly tells us about how Tchartkoff “finds maintaining his high artistic standards,” and matches Answer Choice D which says that he finds it to be a “laborious undertaking that does not provide suitable compensation,” where “laborious undertaking” matches “worry and toil” and “suitable compensation” matches “have money, too.” For Question 9, Answer Choice A is incorrect because it only describes Tchartkoff’s opinion on maintaining patience and doesn’t tell us about how he feels about maintaining his high artistic standard. Answer Choice B is incorrect because it only tells us that Tchartkoff feels that if he sold his paintings, he would only make a little money. Answer Choice C is incorrect because it only shows Tchartkoff asking a question about who would buy his work, and not about his feelings on maintaining his artistic integrity. For Question 8, Answer Choice A is incorrect because Tchartkoff doesn’t feel like maintaining his high artistic standards will be a “means of attaining short-lived fame,” but instead feels the opposite. Answer Choice B is incorrect because the Passage doesn’t tell us exactly how Tchartkoff views the importance of the goal of maintaining his high artistic standards, but, based on his questioning, it would seem that he

Page 135: SAT Full Reading Explanations (QAS 9-19) · SAT Full Reading Explanations (QAS 9-19) SAT #9 Reading 1 1.R.9 Answer Choice C is the correct answer because Lines 19-22 say, “Moths

thinks that it’s a less important goal for him than it is for his professor. Answer Choice C is incorrect because he never says that he views these standards as a necessary pathway to a goal he wants to accomplish.

How to solve this? Question 8 asks, “The passage suggests that to some extent, Tchartkoff finds maintaining his high artistic standards to be a [...]?” Because this is a Paired Question, to solve it we should first go back to the answer choices for Question 9, looking for any text that tells us how Tchartkoff finds maintaining these high standards to be.

For Question 9, Answer Choice A says [...].

10.R.13

Answer Choice C is the correct answer because the last paragraph primarily shows Tchartkoff questioning the patience recommended by his professor and the priority placed on high artistic standards. He asks questions like, “Be patient! but what money have I to buy a dinner with to-morrow?” and “Why do I worry, and toil like a learner over the alphabet, when I might shine as brightly as the rest, and have money, too, like them?” Answer Choice C best describes this and says that the last paragraph serves to “catalog Tchartkoff’s frustrations with his situation.” Answer Choice A is incorrect, because no contradictions are suggested in Tchartkoff’s argument. Answer Choice B is incorrect because Tchartkoff appears to be pretty unified in thought-process and doesn’t show many hypocrisies. Answer Choice D is incorrect because, while the subject matter of some of Tchartkoff’s paintings are mentioned, they aren’t the central focus of the paragraph and are never examined in depth.

How to solve this? The Question asks, “The last paragraph primarily serves to [...]?” To solve this, we should go back to the last paragraph to find its overall meaning and purpose in context. In the paragraph, we find a series of questions and statements from Tchartkoff, all towards the end of questioning the value of his patience and artistic integrity, especially in light of his financial situation,where he can’t pay rent or buy food. “Why do I worry, and toil like a learner over the alphabet, when I might shine as brightly as the rest, and have money, too, like them?” he asks at the end. This paragraph, then, shows Tchartkoff questioning certain principles and going through some of the problems his upholding of them has caused.

Answer Choice A says [...].

Page 136: SAT Full Reading Explanations (QAS 9-19) · SAT Full Reading Explanations (QAS 9-19) SAT #9 Reading 1 1.R.9 Answer Choice C is the correct answer because Lines 19-22 say, “Moths

SAT #13, Reading 2

Passage Outline

● introduction to false memories. how common they are and the consequences they have. (paragraph 1)

● the question of whether some people are more susceptible to false memories than others. a new study that answers no. (paragraph 2)

● the methodology of the study. choosing subjects with superior memory. (paragraph 3) ● how they screen patients, and how many were chosen. (paragraphs 4 and 5) ● results of the experiment, with most subjects falling for “lures.” (paragraph 6) ● commentary on the positives and negatives of the experiment, and the new studies that

have to be completed. (paragraphs 7-9)

11.R.13

Answer Choice C is the correct answer, because the word “curiosity” can best be replaced by the word “oddity” in Line 7, emphasizing the relatively unimportant but interesting phenomenon of false memories. Answer Choice A is incorrect because the phrase “mere concern” cancels out and changes the meaning. Answer Choice B is incorrect because the lines aren’t emphasizing that “false memories” can be a “mere” or pure question. Answer Choice D is incorrect because the lines don’t mean that false memories can be a “wonder.”

● False memories can sometimes be a mere curiosity, but other times they have real implications.

● False memories can sometimes be a mere oddity, but other times they have real implications.

12.R.13 & 13.R.13

Answer Choices D and B are the correct answers because Lines 6-8 say that “false memories can sometimes be a mere curiosity, but other times they have real implications,” which most closely matches Answer Choice D for Question 12, which says that false memories “can have consequences that are genuinely damaging.” For Question 12, Answer Choice A is incorrect because neither the Passage nor our Texts tell us that false memories can interfere with people’s general reasoning ability. Answer Choice B is incorrect because, although it might seem plausible, the Passage doesn’t tell us that anywhere that trying to remember large amounts of information and false memories are correlated. Answer Choice C is incorrect because the Passage doesn’t tell us that false memories are more commonly associated with events in the distant past, as opposed to more current ones. For Question 13, Answer Choices A, C and D are incorrect because they don’t support the claim from Question 12 that false memories “can have consequences that are genuinely damaging.”

How to solve this? Question 12 asks, “Which statement about false memories can reasonably be inferred from the passage?” Because this is a Paired Question, we should typically try to first go through the answer choices for Question 13, looking for a text that could answer Question 12. However, because Question 12 asks about something the passage says about false

Page 137: SAT Full Reading Explanations (QAS 9-19) · SAT Full Reading Explanations (QAS 9-19) SAT #9 Reading 1 1.R.9 Answer Choice C is the correct answer because Lines 19-22 say, “Moths

memories in general, and all of our texts could plausibly answer that, we should adjust our strategy and go through the answer choices for Question 12 first. If we want, we can go through each of the answer choices for Question 13 and try to get a sense of the point each of them makes, because one of them will contain our answer. Or, we can try to go from memory and go back and forth between choices until we find the right match.

Ultimately, then, we should find that Answer Choice D from Question 12, which says that false memories “can have consequences that are genuinely damaging.” matches Answer Choice B from Question 13, which says “false memories can sometimes be a mere curiosity, but other times they have real implications,” and choose Answer Choices D and B for our correct answers.

14.R.13

Answer Choice A is the correct answer because the word “exact” in line 28 is used to emphasize how accurate the memory of these participants is, and can best be replaced by “precise.” Answer Choice B is incorrect because it doesn’t make sense to describe the date as “rigorous,” as if it could be an “unrigorous” date. Answer Choice C is incorrect because the lines are emphasizing the “honesty of the date, which also wouldn’t make sense. Answer Choice D is incorrect because, although it does capture the sense of specificity we’re looking for, it seems to suggest the participants [memory for dates that can be distinguished characteristically] and is less idiomatic than “precise.”

● They’re the ones who can tell you the exact date on which particular events happened — whether in their own lives or in the news [...].

● They’re the ones who can tell you the precise date on which particular events happened — whether in their own lives or in the news [...].

15.R.13

Answer Choice C is the correct answer because the Passage describes the word lists in the Passage as containing “so-called lures — words that would make subjects think of other related ones,” which most closely matches Answer Choice C, which says that each list “included words related to a central theme or topic.” Answer Choice A is incorrect because the Passage doesn’t tell us anywhere that the words appeared alongside pictures. Answer Choice B is incorrect because the Passage doesn’t tell us anything about the emotional content of the words given. And Answer Choice D is incorrect because the Passage doesn’t describe the difficulty of the words, or whether they were straightforward or not.

How to solve this? The Question asks, “According to the passage, one characteristic of the word lists used in the study was that each list [...]?” To solve this question, we should go back to the Passage and look for where the word lists are described. If we follow the track of the study, this should be relatively easy to find, but we can also use the [Question Order Rule] and predict that our answer choice is likely to come after Line 28.

Lines 49-53 describes the lists and say, “The scientists showed participants word lists, then removed the lists and tested the subjects on words that had and hadn’t been included. The lists

Page 138: SAT Full Reading Explanations (QAS 9-19) · SAT Full Reading Explanations (QAS 9-19) SAT #9 Reading 1 1.R.9 Answer Choice C is the correct answer because Lines 19-22 say, “Moths

all contained so-called lures — words that would make subjects think of other related ones.” Because this is the only time that the word lists are mentioned in the Passage, our correct answer will have to reference these lines somehow, even if we don’t know in advance which point they’ll make about them.

Answer Choice A says that these words “appeared in conjunction with related visual images.” However, the Passage doesn’t tell us anything about these words being accompanied by visuals, a very specific claim, so we should eliminate this option. Answer Choice B says that these words “consisted of words intended to evoke emotional reactions.” Because the text doesn’t tell us anything about the emotional content of the words presented, we should eliminate this option as well. Answer Choice C says that the list “included words related to a central theme or topic.” Admittedly, we have to draw an inference from the given text to support this answer choice, but it does plausibly match because the Passage tells us that “lures” were included in each list, and suggests that they were in the same category. [The major confusion here is that the question seems to imply that there are two lists, the original words and then the test words, where each contained words that were similar and meant to confuse participants. But the Text doesn’t tell us that the second test consisted of another list. If the question is only asking about similar words within a single list, then we have to draw an inference to get to our answer because a “lure” could exist for a single word and wouldn’t necessarily require groups of related words within the list, etc.] Answer Choice D says that the word lists “made use of relatively straightforward words.” This answer choice is also incorrect, because the Passage doesn’t tell us whether the words were straightforward or not.

16.R.13 & 17.R.13

Answer Choices B and A are the correct answers because Lines 57-61 say “‘What I love about the study is how it communicates something that memory-distortion researchers have suspected for some time, that perhaps no one is immune to memory distortion.” This comment, that the fact that memory-distortion afflicts us all is something that researchers have “suspected for some time,” most clearly matches Answer Choice B, which says that the study’s “main finding was not a surprise to certain scientists.” For Question 16, Answer Choice A is incorrect because Patihis never makes the statement that the “overall goal has been questioned by several researchers.” Answer Choice C is incorrect because Patihis doesn’t make any positive claim about the methodology of the research, only its importance. Answer Choice D is incorrect because Patihis doesn’t present any sort of debate, and shows that questions still remain about some features of the study, not that things have been definitively decided.

How to solve this? Question 16 asks, “Which statement about the study led by Patihis can reasonably be inferred from the passage?” Because this is a Paired Question, to solve it we should first go through the answer choices for Question 17, looking for any texts that tell us something that Patihis says about the study. However, we should find that all of the answer choices for Question 17 show a perspective of Patihis on the study, and flip our strategy so that we go through Question 16 first.

For Question 16, Answer Choice A says [...].

Page 139: SAT Full Reading Explanations (QAS 9-19) · SAT Full Reading Explanations (QAS 9-19) SAT #9 Reading 1 1.R.9 Answer Choice C is the correct answer because Lines 19-22 say, “Moths

18.R.13

Answer Choice B is the correct answer because the bar graph in Figure 1, which measures the “Mean proportion of indications of recognition of included words,” shows the HSAM group just exceeding 0.7 and the ordinary memory group falling between 0.6 and 0.7. This matches Answer Choice B, which says that “the mean proportion of indications of recognition of included words was over 0.7 in the HSAM group and between 0.6 and 0.7 in the ordinary memory group.” Answer Choice A is incorrect because the HSAM group, in Figure 1, doesn’t exceed 0.8 and because the ordinary memory group fell below 0.7. Answer Choice C is incorrect because both bar graphs showed a difference between the HSAM and ordinary memory group. Answer Choice D is incorrect because the first bar graph shows the HSAM group remembering more of the included words than the ordinary memory group.

How to solve this? [...].

19.R.13

Answer Choice C is the correct answer because Figure 1 shows the ordinary memory group remembering a mean proportion of about .65 of included words and a mean proportion .71 of critical lures, showing that they remember more lures than included words. This most closely matches Answer Choice C which says that, “they recalled a greater proportion of critical lures than included words, on average.” Answer Choice A is incorrect because neither Figure measures words recalled that weren’t on either the list or from the set of critical lures. Answer Choice B is incorrect because neither Figure tells us about the time afforded to each group. Answer Choice D is incorrect because the group confused critical lures for included words more than 50% of the time.

How to solve this? [...].

20.R.13

Answer Choice D is incorrect because the Passage makes the point clearly that “‘perhaps no one is immune to memory distortion,’” including HSAM people, and Figure 2 shows that subjects from both the HSAM and ordinary memory group showed a high rate of recognition of critical lures not included in the list. This most closely matches Answer Choice D which says that people with HSAM “are about as susceptible to memory distortion as are people with ordinary memory.” Answer Choice A is incorrect because, although the claim is true about HSAM, Figure 2 doesn’t give us any evidence related to this. Answer Choice B is incorrect because neither the Passage nor the Figure mention visual lures or compares them to verbal lures. Answer Choice C is incorrect because the Passage says that HSAM individuals aren’t more skilled than people with ordinary memory at distinguishing false from true memories, and Figure 2 supports this.

Page 140: SAT Full Reading Explanations (QAS 9-19) · SAT Full Reading Explanations (QAS 9-19) SAT #9 Reading 1 1.R.9 Answer Choice C is the correct answer because Lines 19-22 say, “Moths

SAT #13, Reading 3

Passage Outline

● introduction of a new study — shows a kind of “internet” of plants, communication using fungi. (paragraph 1)

● the origin of the experiment — Dr. Johnson wanting to know whether messages spread from plant to plant (paragraph 2)

● the setup of the experiment (paragraphs 3-5) ● the data collection process (paragraph 6) ● the main finding of the study — (paragraph 7) ● another important finding of the study (paragraph 8) ● a final conclusion (paragraph 9)

21.R.13

Answer Choice B is the correct answer because the Passage primarily concerns the process and conclusion of a single study meant to support the idea mentioned in the introduction, that “plants have developed a subterranean internet, which they use to raise the alarm when danger threatens.” This most closely matches Answer Choice B, which says that the main purpose of the passage is to “describe an experiment whose results support a particular conclusion.” Answer Choice A is incorrect because the finding of the study isn’t presented as addressing an ongoing problem. Answer Choice C is incorrect because the Passage doesn’t tell us that the methodology of the research study “revolutionizes a process,” and doesn’t give it much attention in general. Answer Choice D is incorrect because the scientific study outlined doesn’t “undermine a popular theory.”

How to solve this? The Question asks, “The main purpose of the passage is to [...]?” To solve this, we should have a general understanding of the Passage and identify specific sentences that tell us about the main point of the passage. In general, the Passage can outlined like this:

{...}

Specifically, we can look at the title of the Passage, which only says, “Beans’ Talk,” and doesn’t provide us with much. The introduction, however, introduces the idea of an “internet” of plants, saying, “The idea that plants have developed a subterranean internet, which they use to raise the alarm when danger threatens, sounds like science fiction.” The following sentence begins with the word “But,” an important indicator on the Reading Test, and introduces a study that reinforces the first claim about an internet of plants. The Passage then proceeds to outline the process and conclusions of that study. At this point, we should look at the answer choices.

Answer Choice A says that the purpose of the passage is to “discuss a finding that addresses an ongoing problem.” [...].

Page 141: SAT Full Reading Explanations (QAS 9-19) · SAT Full Reading Explanations (QAS 9-19) SAT #9 Reading 1 1.R.9 Answer Choice C is the correct answer because Lines 19-22 say, “Moths

22.R.13

Answer Choice C is correct because the first paragraph describes the process of plant communication as an internet, “with fungal hyphae [the branching filaments that make up a fungus’s body] standing in for local Wi-Fi.” This most closely matches Answer Choice C, which says that the first paragraph introduces the rest of the Passage by “using an analogy to show how communication among plants might occur.” Answer Choice A is incorrect because the first paragraph only mentions plant communication in a single instance, and doesn’t say that it’s more widespread than is recognized. Answer Choice B is incorrect because, although it might be plausible to describe plant and parasite interactions as complex, the Passage never directly tells us this in the first paragraph. Answer Choice D is incorrect because the research in the Passage isn’t described as unorthodox, and the paragraph doesn’t take the structure of giving a rationale or providing a defense for the study.

How to solve this? The Question asks, “The first-paragraph of the passage introduces the subsequent discussion mainly by [...]?” To solve this, we should look back at the first paragraph and try to understand its structure, content, and purpose in the Passage at large. It can be divided like this:

● The idea that plants have developed a subterranean internet, which they use to raise the alarm when danger threatens, sounds like science fiction.

● But David Johnson of the University of Aberdeen believes he has shown that just such an internet with fungal hyphae [the branching filaments that make up a fungus’s body] standing in for local Wi-Fi, alerts beanstalks to danger if one of their neighbours is attacked by aphids.

Here, the first sentence introduces the general idea of plant communication, while the second introduces a study meant to support this claim, comparing plant communication to “internet” and “Wifi.”

{After a cursory glance through the answer choices, our attention should be drawn to Answer Choice C, which references the interesting use of a comparison to “Wifi” or the “internet” that the Passage uses, etc.}

Answer Choice A says that [...].

Page 142: SAT Full Reading Explanations (QAS 9-19) · SAT Full Reading Explanations (QAS 9-19) SAT #9 Reading 1 1.R.9 Answer Choice C is the correct answer because Lines 19-22 say, “Moths

23.R.13

Answer Choice C is the correct answer because Lines 10-13 say, “Dr. Johnson knew from his own past work that when broad-bean plants are attacked by aphids they respond with volatile chemicals that both irritate the parasites and attract aphid-hunting wasps.” This tells us a fact on which Johnson’s study depends, and most closely matches Answer Choice C which says that, “broad bean plants release noxious chemicals to ward off infestation.” Answer Choice A is incorrect because Dr. Johnson doesn’t know in the beginning of his study if “fungal hyphae warn beanstalks of danger if a nearby plant is attacked,” since this is what he wants to test in the first place. Answer Choice B is incorrect because the Passage tells us that wasps are helpful to broad bean plants since they kill off aphids, and that the plants actually send out signals to attract the wasps. Answer Choice D is incorrect because the Passage doesn’t tell us anything about the actual interaction between the aphids and wasps, but does suggest that the wasps are at least somewhat successful in preying upon the aphids.

How to solve this? The Question asks, “The passage suggests that in designing the experiment, Johnson relied on the fact that [...]?” To solve this, we should go back to the Passage, looking for a Text that will tell us something that Johnson relied on in his study. Using the [Linear Pacing Rule] as well as our understanding of the Passage, we should expect our Text to come after the introduction and right when the methodology of the experiment begins to be described. {We can glance over the answer choices, without really analyzing them, to get a sense of the kind of thing that the Text should answer for us. In this case all of the answer choices tell us something about the relationship between bean-stalk plants, aphids and wasps, so we should look for a Text that tells us a fact about the interactions between all three of these.}

Lines 10-13 say, “Dr. Johnson knew from his own past work that when broad-bean plants are attacked by aphids they respond with volatile chemicals that both irritate the parasites and attract aphid-hunting wasps.” This clearly tells us a fact that Dr. Johnson relies on in setting up the design of his study, and we should [...].

Answer Choice A says [...].

24.R.13 & 25.R.13

Answer Choices C and A are the correct answers because Lines 13-17 say, “He did not know, though, whether the message could spread from plant to plant. So he set out to find out — and to do so in a way which would show if fungi were the messengers.” These lines most clearly tell us what research question the experiment was attempting to answer and most closely match Answer Choice C which says that the question the experiment wanted to answer was, “Do broad bean plants use fungal hyphae to help convey information?” For Question 25, Answer Choice B is incorrect because it only describes a feature of the design of the study and not a question it was intended to answer. Answer Choice C is incorrect because it also describes a feature of the methodology. Answer Choice D is incorrect because it also describes a step in the process of the study and not a question it was intended to answer. For Question 24, Answer Choice A is incorrect because neither the Text nor the Passage tell us that the research wanted to understand how wasps protect plants against aphids. Answer Choice B is incorrect because it

Page 143: SAT Full Reading Explanations (QAS 9-19) · SAT Full Reading Explanations (QAS 9-19) SAT #9 Reading 1 1.R.9 Answer Choice C is the correct answer because Lines 19-22 say, “Moths

doesn’t match either the Text or the Passage. Answer Choice D is incorrect because, although the study does concern plant communication, it doesn’t look at plant communication specifically when roots are restricted from growing.

How to solve this? Question 24 asks, “Based on the passage, what research question was the experiment mainly attempting to answer?” Because this is a Paired Question, to solve this we should begin by going through the answer choices for Question 25, looking for a text that tells us about the research question that the experiment was designed to answer.

For Question 25, Answer Choice A says [...].

26.R.13

Answer Choice D is the correct answer because the third and fourth paragraphs of the Passage primarily detail the methodology of the experiment, explaining how the researchers designed it and its various controls. This most closely matches Answer Choice D, which says that the two paragraphs “explain the experiment’s conditions.” Answer Choice A is incorrect because, although the passages do discuss and outline the “methods” of the experiment, they don’t make an “evaluation” or judgment about them. Answer Choice B is incorrect because these paragraphs simply outline the methodology of the study, and don’t formulate its goal, which comes earlier in the Passage. Answer Choice C is incorrect because the paragraphs don’t discuss the findings of the experiment, which come later.

How to solve this? The Question asks, “The third and fourth paragraphs (lines 18-33) primarily serve to [...]?” To solve this, we should go back to those two paragraphs and look at what they have in common, and their general structure and content. They can be outlined like this:

● the researchers set up eight different zones or environments. the plants grew for four months and could interact with fungi in the soil.

● but each of their relationships to the fungi was different. some could communicate via chemicals and water, some via hyphae and some via roots.

We see then, that these two paragraphs set up the methodology of the study, and we should expect our answer choice to reference this.

Answer Choice A says that the paragraphs serve to “evaluate the experiment’s methods.” This answer choice seems plausible, because the paragraphs do discuss the experiments methodology, so we can keep this option. Answer Choice B says that the paragraphs serve to “formulate the experiment’s goal.” Because this formulation came earlier in the Passage, during the {hypothesis} stage of the research, and isn’t given in these paragraphs, we should eliminate this option. Answer Choice C says that the paragraphs serve to “document the experiment’s findings.” Because the findings of the research come later in the Passage and aren’t given in the paragraphs, we should eliminate this option as well. Answer Choice D says that the paragraphs serve to “explain the experiment’s conditions.” Because this answer choice also says that the two paragraphs concern the conditions, or setup of the experiment, we should keep this option.

Page 144: SAT Full Reading Explanations (QAS 9-19) · SAT Full Reading Explanations (QAS 9-19) SAT #9 Reading 1 1.R.9 Answer Choice C is the correct answer because Lines 19-22 say, “Moths

So, we’re left to decide between Answer Choices A and D. To decide between them, we need to focus clearly on their distinctions, and {pull them apart}. We should notice that Answer Choice D uses the neutral verb “explain,” while Answer Choice A uses the more “opinionated” verb “evaluate,” which suggests that the paragraphs talk about whether the methods were good and bad. Because the methodology is simply given, and not “evaluated,” we should eliminate Choice A and choose Answer Choice D as our correct answer.

27.R.13

Answer Choice C is the correct answer because the word “control” in Line 50 is being used in the sense of an experimental control, which is used as a baseline against which to measure the effect of something. This most closely matches “comparative element,” as strange as it sounds, which captures the sense of something to compare against. Answer Choice A is incorrect because a control isn’t a regulating force. Answer Choice B is incorrect because a control isn’t a mechanism that restricts. Answer Choice D is incorrect because the control isn’t a “supervising factor, etc.”

● Each insect was placed for five minutes in an apparatus that had two chambers, one of which contained a sample of the volatiles and the other an odorless control.

● Each insect was placed for five minutes in an apparatus that had two chambers, one of which contained a sample of the volatiles and the other an odorless comparative element.

28.R.13

Answer Choice A is the correct answer because Lines 61-65 tell us that “If both hyphae and roots had been blocked throughout the experiment, though, the volatiles from uninfested plants actually attracted aphids (they spent 3½ minutes in the volatiles’ chamber), while the wasps were indifferent.” These lines most clearly tell us “which factor is most likely responsible for aphids’ attraction to some of the uninfested plants in the experiment,” and best match Answer Choice A which says that, “the plant were unable to receive distress signals from infested plants through hyphal contact.” Answer Choice B is incorrect because the Passage, in Paragraph 7, doesn’t tell us that the plants emitted a chemical that “repelled the wasps,” even if it does describe them as indifferent. Answer Choice C is incorrect because the Passage tells us that the hyphal connections were severed between plants, not that they had developed connections. Answer Choice D is incorrect because the Passage doesn’t tell us that the root systems were compromised by the aphids, but instead blocked by the researchers.

How to solve this? The Question asks, “Based on the passage, which factor is most likely responsible for aphids’ attraction to some of the uninfested plants in the experiment?” To solve this, we should go back to the Passage and look for a Text that describes aphids being attracted to some of the uninfested plants in the experiment. Using the [Question Order Rule], we can predict that ourText is likely to come after Line 50. In Lines 61-65, we’re told about one of the scenarios where aphids were attracted to the unifested bean-stalk plants. The Lines read, “If both hyphae and roots had been blocked throughout the experiment, though, the volatiles from uninfested plants actually attracted aphids (they spent 3½ minutes in the volatiles’ chamber), while the wasps were indifferent.” This tells

Page 145: SAT Full Reading Explanations (QAS 9-19) · SAT Full Reading Explanations (QAS 9-19) SAT #9 Reading 1 1.R.9 Answer Choice C is the correct answer because Lines 19-22 say, “Moths

us that the significant factor in the aphids’ attraction to the uninfested bean-stalk plants was that their “hyphae and roots had been blocked,” and we should expect our answer choice to match this.

29.R.13 & 30.R.13

Answer Choices A and D are the correct answers because Lines 72-76 say that “Such a general response no doubt helps the plant first attacked by attracting yet more wasps to the area, and it helps the fungal messengers by preserving their leguminous hosts,” which most clearly tells us about the general nature of the relationship between broad bean plants and the fungi discussed in the passage and best match Answer Choice A, which describes the relationship as “mutually beneficial, since both organisms profit from the arrangement.” For Question 30, Answer Choices A, B and C are incorrect because each only further explains the conclusions of the study, without offering a general picture of the relationship between the fungi and the bean-stalk plants. For Question 29, Answer Choice B is incorrect because the Text and Passage describes how both the bean-stalk plants and the fungi are helped, and not that the relationship is unbalanced. Answer Choice C is also incorrect, because the Passage describes how both organisms help one another, and don’t tell us that one is harmed by the exchange. Answer Choice D is incorrect because it makes too specific a claim in comparison to Choice A, and because the Passage describes how an independent bean-stalk plant can produce chemicals on its own (Lines 10-13) and that fungi are the messengers, not that neither organism “can produce chemicals independently of the other.”

How to solve this? Question 29 asks, “Which choice best describes the nature of the relationship between the broad bean plants and fungi discussed in the passage?” Because this is a Paired Question, to solve it we should begin by going through the answer choices for Question 30, looking for any texts that tell us generally about the relationship between the broad bean plants and fungi.

For Question 30, Answer Choice A says [...].

Page 146: SAT Full Reading Explanations (QAS 9-19) · SAT Full Reading Explanations (QAS 9-19) SAT #9 Reading 1 1.R.9 Answer Choice C is the correct answer because Lines 19-22 say, “Moths

SAT #13, Reading 4

Passage Outline

Passage 1 — “What the Black Man Wants”

● the danger of the forced labor policy. not being able to choose work is slavery. it defeats the point of freedom from slavery. (paragraph 1)

● supported only one idea: the “immediate, unconditional, and universal enfranchisement of the black man.” without this, his liberty is a mockery and not real. (paragraph 2)

● the counterargument that this argument is too premature. let labor be organized, and then later it can be free. but this is the time for the argument. if you lose it now, it will be lost forever. (paragraph 3)

Passage 2 — “To Consider the Subject of Re-organization of the Rebel States”

● is it enough to have emancipation simply a law? in other times and places yes, but not now, because it is a slavery of race.

● many of the South’s laws to not allow a free negro to live in their States. ● a choice has to be made. either four million disenfranchised and degrade men, or four

million “land-holding, industrious, arms-bearing and voting population.” and it has to be decided soon.

31.R.13

Answer Choice B is the correct answer because Lines 3-4 say of Banks’s policy that it “makes the Emancipation Proclamation of 1863 a mockery and delusion” and, in Lines 15, that “it defeats the beneficent intention of the Government, if it has beneficent intentions, in regards to the freedom of our people.” These lines tell us clearly how Douglass characterizes Banks’s policy and matches Answer Choice B which says that it is, “contrary to the purpose of the government’s abolition of slavery.”Answer Choice A is incorrect because Banks’s opposition towards slavery is never mentioned, and Douglass never makes the case that his policy is incosistent with such a stance. Answer Choice C is incorrect because, while Douglass does describe the policy as “absolute slavery,” he never makes the point that it is “worse in many respects.” Answer Choice D is incorrect because Douglass never makes the point that the policy is an imporvement on slavery, even if he does say that it isn’t ideal.

How to solve this? The Question asks, “In Passage 1, Douglass characterizes Banks’s labor policy in Louisiana as [...]?” To solve this, we should go back to the Passage and look for any texts that tell us about his attitude towards the policy. In the first paragraph, Douglass makes two specific claims about the policy. In Lines 1-4 he describes it as the “chief danger at the present moment; that it practically enslaves the Negro, and makes the Emancipation Proclamation of 1863 a mockery and delusion.” In Lines 15-18, he says that the policy “defeats the beneficent intention of the Government, if it has beneficent intentions, in regards to the freedom of our people.” So, we see that Douglass views the policy as negative, as nearly identical to slavery, and as working counter to the Emancipation Proclamation, and we should expect our answer choice to relate to this.

Page 147: SAT Full Reading Explanations (QAS 9-19) · SAT Full Reading Explanations (QAS 9-19) SAT #9 Reading 1 1.R.9 Answer Choice C is the correct answer because Lines 19-22 say, “Moths

Answer Choice A says [...].

32.R.13

Answer Choice A is the correct answer because the word “practically” in Line 10 means almost, or just about but not officially. “Effectively,” most nearly has the same meaning and best replaces the word in the sentence. Answer Choice B is incorrect because Douglass isn’t highlighting the reasonability of the policy. Answer Choice C is incorrect because Douglass isn’t highlighting that the policy cleverly accomplishes this. And Answer Choice D is incorrect because Douglass doesn’t mean that the policy only partially reduces the black man to slavery.

● “When any individual or combination of individuals undertakes to decide for any man [...] at what he shall work, and for what he shall work, he or they practically reduce him to slavery.”

● “When any individual or combination of individuals undertakes to decide for any man [...] at what he shall work, and for what he shall work, he or they effectively reduce him to slavery.”

33.R.13

Answer Choice B is the correct answer because in Paragraph 3, Douglass mentions a possible counterargument that his “pressing of the Negro’s right to suffrage is premature” and that “in the natural course of events, the right of suffrage will be extended to the Negro.” He responds by saying, “I do not agree with this. The constitution of the human mind is such, that if it once disregards the conviction forced upon it by a revelation of truth, it requires the exercise of a higher power to produce the same conviction afterwards.” This most clearly matches Answer Choice B which says that Douglass responds to the counterargument by “citing a universal characteristic [the constitution of the human mind] that makes it unlikely that the sequence of events suggested in the counterargument [the natural course of events] would actually occur.” Answer Choice A is incorrect because Douglass doesn’t describe the suffering that has resulted from the policies directly in response to the counterargument (the streets in mourning and falling tears aren’t used to directly address the counterargument and don’t strongly describe the actual suffering caused by the policy). Answer Choice C is incorrect because Douglass doesn’t make the point in the last paragraph that the proponents of the counterargument are making their case out of self-interest. Answer Choice D is incorrect because Douglass doesn’t present the counterargument as a good short-term solution but one requiring a permanent solution, since he thinks things need to be resolved in the now.

How to solve this? The Question asks, “In the last paragraph of Passage 1, Douglass rejects a counterargument to one of his claims by [...]?” To solve this, we should look back to the last paragraph given, find the counterargument and see how Douglass directly responds to it. Lines 31-36 say, “It may be objected, however, that this pressing of the Negro’s right to suffrage is premature. Let us have slavery abolished, it may be said, let us have labor organized, and then, in the natural course of events, the right of suffrage will be extended to the Negro.” [...].

Page 148: SAT Full Reading Explanations (QAS 9-19) · SAT Full Reading Explanations (QAS 9-19) SAT #9 Reading 1 1.R.9 Answer Choice C is the correct answer because Lines 19-22 say, “Moths

34.R.13

Answer Choice D is the correct answer, because the word “disposition” in Line 46 is meant to emphasize that Douglass doesn’t think that same sentiments or feelings will persist, and that they should be taken advantage of while they do. This most closely matches the word “attitude.” Answer Choice A is incorrect because Douglass isn’t highlighting a specific “habit” that exists at the moment and should be taken advantage of. Answer Choice B is incorrect because Douglass isn’t making a point about a certain location or “position.” Answer Choice C is incorrect because [...].

● I fear that if we fail to do it now, … we may not see, for centuries to come, the same disposition that exists at this moment.

● I fear that if we fail to do it now, … we may not see, for centuries to come, the same disposition that exists at this moment.

35.R.13

Answer Choice B is the correct answer because Lines 50-57 say, “In ancient times when the slaves were of the same race with their masters, when the slaves were poets, orators, scholars, ministers of state, merchants, and the mothers of kings — if they were emancipated, nature came to their aid, and they reached an equality with their masters. [...] But, my friends, this is a slavery of race.” These Lines most clearly draw a comparison between slavery in antiquity and slavery in the United States, and match Answer Choice B which says that, “slavery was not based on race and thus former slaves could achieve equality with slaveholders, while in the United States race-based slavery leads some people to view former slaves as inferior.” Answer Choice A is incorrect because it doesn’t closely match our Text and doesn’t capture the essential difference of race that the Passage highlights, and because the Passage doesn’t say that freed slaves in the United States were forced into agricultural and domestic labor. Answer Choice C is incorrect because Dana doesn’t make the case that sometimes in antiquity individuals were enslaved on the basis of race. Answer Choice D is incorrect because Dana doesn’t draw a distinction between the legal and social ranking of slaves in either point in history.

How to solve this? The Question asks, “In Passage 2, the main contrast Dana draws between slavery in antiquity and slavery in the United States is that in antiquity [...]?” To solve this, we should look back to Passage 2 to find where Dana makes this distinction. In Lines 50-57, we find that, “In ancient times when the slaves were of the same race with their masters, when the slaves were poets, orators, scholars, ministers of state, merchants, and the mothers of kings — if they were emancipated, nature came to their aid, and they reached an equality with their masters. [...] But, my friends, this is a slavery of race.” These Lines tell us clearly that the major distinction between slavery in antiquity and slavery in the United States is that one was not based on race, while the other was. So, we should expect our answer choice to have something to do with this distinction.

Answer Choice A says [...].

Page 149: SAT Full Reading Explanations (QAS 9-19) · SAT Full Reading Explanations (QAS 9-19) SAT #9 Reading 1 1.R.9 Answer Choice C is the correct answer because Lines 19-22 say, “Moths

36.R.13

Answer Choice D is the correct answer because Lines 81-85 say that, “It has got to be decided pretty soon, which you will have.” These Lines most clearly give evidence that “Dana believes that the conditions of Southern black men must be improved quickly to avoid negative long-term consequences.” Answer Choice A is incorrect because Lines 56-59 only tell us that slavery as it exists in the United States is a matter of race. Answer Choice B is incorrect because Lines 68-70 don’t tell us about the urgency of the decision to be made but asks rhetorical questions about slavery. Answer Choice C is incorrect because Lines 70-72 also don’t tell us about any of the urgency of the decision to be made.

How to solve this? [...].

37.R.13

Answer Choice D is the correct answer because the word “fired” in Line 60 is used to emphasize how the Southern heart has been stirred up or stimulated with certain ideas, which most clearly matches “roused.” Answer Choice A is incorrect because it’s unclear what it would mean for the Southern heart to be “dismissed” for thirty years and [...].” Answer Choice B is incorrect because Dana doesn’t mean that the Southern heart has been “illuminated” or enlightened. Answer Choice C is incorrect because Dana doesn’t quite mean that the Southern heart has been “propelled” or forcibly motivated.

● I ask you, has the Southern heart been fired for thirty years for nothing? ● I ask you, has the Southern heart been roused for thirty years for nothing?

38.R.13

Answer Choice B is the correct answer because both Passages make the point the the abolition of slavery alone is “insufficient to ensure true freedom and equality for black men.” Lines 24-29 say that, “Without this [the “immediate, unconditional, and universal” enfranchisement of black men], his liberty is a mockery; without this, you might as well almsot retain the old name of slavery for his condition; for, in fact, if he is not the slave of the individual master, he is the slave of society, and holds his liberty as a privilege, not as a right.” These lines tell us that even with the abolition of slavery, black men can still be the “slave of society,” under a policy like the forced labor policy. In Passage 2, Dana asks, “Is it enough that we have emancipation and abolition upon the statute books?” He answers that, “In some states of society, I should say yes,” but continues to make the case that in the United States, where slavery is a slavery of race, this isn’t possible. These Lines support Answer Choice B. Answer Choice A is incorrect because, while Dana may hint that civil unrest would follow having “four millions of disenfranchised, disarmed, untaught, landless, degraded men, neither author makes the clear case that additional civil conflicts would arise. Answer Choice C is incorrect because, while both authors are critical of the forced labor policies and believe that they’re counter in spirit to the abolition of slavery, neither makes the case that that abolition is completely undermined by the policies. Answer Choice D is incorrect because, while both authors might agree with the claim, neither author makes the clear connection between the abolition of slavery and the “founding ideals of the United States.”

Page 150: SAT Full Reading Explanations (QAS 9-19) · SAT Full Reading Explanations (QAS 9-19) SAT #9 Reading 1 1.R.9 Answer Choice C is the correct answer because Lines 19-22 say, “Moths

How to solve this? The Question asks, “Both Douglass and Dana make the point that the abolition of slavery in the United States was [...]?” To solve this, we can either go back to the Passage and look for clear statements that either author makes about the abolition of slavery in the United States or begin with the answer choices, and look for evidence for some while eliminating others. Because so much of either Passage could be evidence about the authors’ views on the abolition of slavery, it’s probably more efficient to do the latter.

Answer Choice A says [...].

39.R.13

Answer Choice A is the correct answer because Lines 40-44, which say, “This is the hour. Our streets are in mourning, tears are falling at every fireside, and under the chastisement of this Rebellion we have almost come up to the point of conceding this great, this all-important right of suffrage,” tell us that Douglass believes that the Civil War has allowed the prospect of extending black men’s rights to be more plausible. Lines 64-65 say that, “Besides all that, [Southerners] look upon the negro as the cause of their defeat and humiliation…,” which tells us that Dana believes that the South blames black men for their loss inthe Civil War. These Lines most closely match Answer Choice A which says that Douglass believes that the war “created a political climate in which the extension of black men’s rights seems more feasible, while Dana believes that such an extension faces opposition from those who blame black men for the South’s defeat.” Answer Choice B is incorrect because Douglass doesn’t make a point about the employment prospects of black men and Dana doesn’t suggest that Southerners now think that black workers can play an important role in the economy, but rather that they’re the cause of their loss. Answer Choice C is incorrect because Dana believes that white Southerners still see black men as inferior and not as their equals. Answer Choice D is incorrect because Dana doesn’t make any claim about the necessity of war memories fading for the expansion of black men’s rights to go forward.

How to solve this? The Question asks, “Based on the two passages, Douglass and Dana differ in their views of the effect of the Civil War in that Douglass believes that the war has [...]?” To solve this Question, we should first go back to either Passage and look for evidence that tells us clearly the view that either author has on the Civil War.

Going back to Passage 1 first, we may have difficulty identifying any clear point that Douglass makes about the war. However, if we jump to Passage 2, we should find Lines 64-65 which say, “Besides all that, [Southerners] look upon the negro as the cause of their defeat and humiliation.” [From here, we can go down to the answer choices to look for those that will match this claim that Dana makes, and then use that choice to help us identify texts in Passage 1.] [...].

Page 151: SAT Full Reading Explanations (QAS 9-19) · SAT Full Reading Explanations (QAS 9-19) SAT #9 Reading 1 1.R.9 Answer Choice C is the correct answer because Lines 19-22 say, “Moths

40.R.13 & 41.R.13

Answer Choices B and D are the correct answers because Dana, in Lines 72-74, says that, “By their laws, a black man cannot testify in court; by their laws he cannot hold land; by their laws he cannot vote.” These Lines make a point that we can expect Douglass to find harmful, and consistent with his own negative view of the forced labor policies. Answer Choice B for Question 40 most clearly expresses this and a point which Douglass makes throughout the Passage and says, “the conditions that Dana points out that black men experience constitute a form of slavery.” Answer Choice D for Question 41 most clearly matches this, and says, “Without this, his liberty is a mockery; without this, you might as well almost retain the old name of slavery of his condition; for, in fact, if he is not the slave of the individual master, he is the slave of society, and holds his liberty as a privilege, not as a right.”

For Question 40, Answer Choice A is incorrect because we shouldn’t expect Douglass to find Dana’s comments “highly insensitive,” because the authors are mostly in agreement and Douglass makes his own similar comments. Answer Choice C is incorrect because Dana doesn’t make the case, either in the Lines or in the Passage, that black men will continue to be satisfied with limited rights. Answer Choice D is incorrect the comments Dana makes don’t concern the legal status of slaveruy in formal states and because neither author mentions the question of whether slavery will remain legal or not in former slave states. For Question 41, Answer Choice A is inocrrect because it doesn’t match our Answer Choice and make the case that the legal conditions of black men are a kind of slavery, but only that they make the Emancipation Proclamation a mockery and a delusion. Answer Choice B is incorrect because, while the Lines do say, “It is absolute slavery,” they focus more clearly on Banks’s employment policies and not the general conditions of black men’s political status. Answer Choice C is incorrect because these Lines don’t make the case that the conditions described by Dana are a kind of slavery, but states the principles that Douglass upholds.

How to solve this? Question 40 asks, “Based on Passage 1, Douglass would most likely respond to Dana’s comments in lines 72-74, Passage 2, by stating that [...]?” To solve this, we should begin by going to the Lines cited in Passage 2, which say, “By their laws, a black man cannot testify in court; by their laws he cannot hold land; by their laws he cannot vote.” Because this is a Paired Question, we might try to go first through the answer choices for Question 41. However, to help us find a Text, we should try to predict in advance how Douglass might respond to Dana’s comments. We may not be able to establish something specific, but we should predict that, in general, he should have a sympathetic stance to the validity of the claims and find that they confirm that the current state of black men is an injustice. Going through the answer choices for Question 41 we should find that nearly all of the Lines given could potentially tell us something relevant to the Question, so we should [switch gears] and look through the answer choices for Question 40 first, seeing if we can decide on one of those.

Page 152: SAT Full Reading Explanations (QAS 9-19) · SAT Full Reading Explanations (QAS 9-19) SAT #9 Reading 1 1.R.9 Answer Choice C is the correct answer because Lines 19-22 say, “Moths

{ ● basically choose answer choice B, because it’s the only that matches the spirit of the

text and doesn’t say something wrong. ● then we have to decide on the Text. ● the choice between Answer Choices B and D for Question 41 is hard, because both say

that something is slavery. but if we’re looking for something different between them, we should ulimately find that one talks specifically about the effect of Banks’ policies while the other talks more generally about the conditions black men face, and speaks more broadly and specifically...matching answer choice b from Question 40. }

SAT #13, Reading 5

Passage Outline

“Gouldian finches’ head colour reflects their personality.”

● introduction to gouldian finches and different head colors. (paragraph one) ● predictions about head color and personality. (paragraphs two and three) ● how they tested their first prediction. (paragraph four) ● the results of that test. (paragraph five) ● how they tested their second prediction. (paragraph six) ● the results of that test. (paragraph 7) ● a final experiment, on the birds’ interest in novel objects. (paragraph 8) ● the results of that experiment. (paragraph 9) ● reflections on what’s interesting about the findings. (paragraphs 10 and 11)

42.R.13

Answer Choice B is the correct answer because the Passage outlines a series of studies that focus on a particular species of bird, the Gouldian finches. This is almost certain to “expand our knowledge of the behavior of a bird species,” and so matches Answer Choice B. Answer Choice A is incorrect because the mention of the finches’ predators only takes place in one of the experiments and doesn’t take a central place in the Passage, and because the Passage doesn’t discuss how the birds defend against these attacks. Answer Choice C is incorrect because there’s no mention of the research topic of the Passage being dismissed, a quality we should expect to see in the introductory paragraph. Answer Choice D is incorrect because the Passage doesn’t attempt to make a connection between the bird behavior and other animal behaviors, or trace out a common ancestry.

How to solve this? The Question asks, “The main purpose of the passage is to [...]?” To solve, we should have both a general understanding of the Passage as a whole and more specific comprehension of the title and introductory paragraph. Generally, the Passage can be organized like this:

● introduction to gouldian finches and different head colors. (paragraph one) ● predictions about head color and personality. (paragraphs two and three) ● how they tested their first prediction. (paragraph four)

Page 153: SAT Full Reading Explanations (QAS 9-19) · SAT Full Reading Explanations (QAS 9-19) SAT #9 Reading 1 1.R.9 Answer Choice C is the correct answer because Lines 19-22 say, “Moths

● the results of that test. (paragraph five) ● how they tested their second prediction. (paragraph six) ● the results of that test. (paragraph 7) ● a final experiment, on the birds’ interest in novel objects. (paragraph 8) ● the results of that experiment. (paragraph 9) ● reflections on what’s interesting about the findings. (paragraphs 10 and 11)

More specifically, we can look at the tile of the Passage, which reads, “Gouldian finches’ head colour reflects their personality,” as well as the introductory paragraph. Lines 1-4 introduce the finch and its two different head colorings, while the second paragraph continues and introduces the study of the rest of the Passage. So, we see that the Passage primarily focuses on Gouldian finches and a series of experiments that test predictions about their head colorings.

43.R.13

Answer Choice C is the correct answer because Lines 13-15 say that, “this hypothesis (the researcher’s second prediction) is based on previous studies of other species that have shown a correlation between aggression and these behavioural characteristics.” This tells us that researchers believe that because they see a correlation between aggression and certain characteristics in one species, they’re justified in predicting that that same correlation will hold for the finches. This most closely matches Answer Choice C which says, “Observed correlations between certain behaviors in one species translate to other species.” Answer Choice A is incorrect because the researchers never suggest that their prediction rests on this assumption, and, in fact, takes the opposite stance, claiming that red-headed birds should be more risk-taking (and thus more likely to receive a benefit from being so). Answer Choice B is incorrect because it’s the researcher’s first prediction that concerns aggression, while the second concerns risk taking. Answer Choice D is incorrect because the researchers never mention this tension in distinguishing between such features, a sort of nature vs. nurture argument.

How to solve this? The Question asks, “It can reasonably be inferred that the second prediction tested by Williams and her colleagues reflects which assumption?” To solve this, we should go back to the Passage and look at where they make their second prediction. Because this is a question asking about [research assumptions], we should be paying attention to a conclusion the researchers hold that isn’t perfectly founded, or a kind of educated guess they make. Lines 13-15 say that, “this hypothesis (the researcher’s second prediction) is based on previous studies of other species that have shown a correlation between aggression and these behavioural characteristics.” Because these lines tell us clearly something on which the researcher’s prediction depends, and because it requires a logical leap from the behavior of other species to the behavior of gouldian finches, we should feel confident that this is our Text, and expect our answer to have something to do with it.

Answer Choice A says [...].

Page 154: SAT Full Reading Explanations (QAS 9-19) · SAT Full Reading Explanations (QAS 9-19) SAT #9 Reading 1 1.R.9 Answer Choice C is the correct answer because Lines 19-22 say, “Moths

44.R.13

Answer Choice D is the correct answer because Lines 34-37 say, “Aggressive interactions such as threat displays and displacements were then counted over a 30-minute period,” in describing how the researchers tested their first prediction, where birds have to fight for access to a feeder. Displacements then, make the most sense as one bird moving aside the other so that it can feed, which most closely matches Answer Choice D which says that the word suggests that one bird “jostles the other aside to access the food supply.” Answer Choice A is incorrect because the Passage never directly mentions a scale of social dominance, and the first experiment is measuring the birds’ struggle for access to food, not their ranking in an abstract hierarchy. Answer Choice C is incorrect because, while it may be true, it contains too many unfounded details for us to be sure.

How to solve this? The Question asks, “The author uses the word ‘displacements’ in Line 36 most likely to suggest that one bird [...]?” To solve this, we should go back to the Lines cited, and try to understand how the word is being used in context. Lines 34-37 say, “Aggressive interactions such as threat displays and displacements were then counted over a 30-minute period.” This takes place in the discussion of the researcher’s first experiment, where birds are trapped in a cage without food, and then given a feeder which only one bird could access at a time. Although the word “displacement” is never defined for the reader, given the context of the experiment and the meaning of the word, it makes sense to say that it most likely refers to one bird moving aside, replacing or displacing another bird so that it can get access to the feeder, and we should expect our answer choice to refer to this.

Answer Choice A says [...].”

45.R.13

Answer Choice B is the correct answer because Lines 48-50 describe how the researchers moved a “silhouette of an avian predator [...] up and down in front of the cage to scare the birds from the feeder,” strongly implying that the predator was a fabrication of the researchers, and not real. So, the use of quotations around the word “predator” in Line 56 is most likely used to suggest that it isn’t a real predator that’s being used, and matches Answer Choice B which say the quotations “indicate that the predator was actually a simulation.” Answer Choice A is incorrect because nothing in the Passage suggests that this is real predator or a dangerous predator, and it’s unclear how the quotations would work to imply danger. Answer Choice C is incorrect because the Passage doesn’t say anything about the predator being used in other experiments. Answer Choice D is incorrect because the Passage suggests that the predator isn’t real and never mentions or hints at it being a disruptive presence.

How to solve this? The Question asks, “Information in the passage indicates that the purpose of the quotation marks around the word “predator” in line 56 is to [...]?” To solve this, we should either rely on memory or go back to the Passage to see what it says about the predator. Lines 48-50 say that the researchers moved a “silhouette of an avian predator [...] up and down in front of the cage to scare the birds from the feeder,” suggesting that [...].

Page 155: SAT Full Reading Explanations (QAS 9-19) · SAT Full Reading Explanations (QAS 9-19) SAT #9 Reading 1 1.R.9 Answer Choice C is the correct answer because Lines 19-22 say, “Moths

46.R.13 & 47.R.13

Answer Choices B and C are the correct answers because Lines 64-67 say, “These experiments were repeated after a two-month interval and showed that different birds differed in their responses but the responses of individual birds were consistent over time,” telling us that it’s unlikely for a single bird to show inconsistent behaviors. Answer Choice B best matches this, saying that it would be atypical for a single red-headed finch to “approach novel objects without hesitation one week but entirely avoid them the next.” For Question 46, Answer Choice A is incorrect because the Passage suggests that, one, the behavior of an individual finch remains consistent over time and, two, that red-headed finches return to feeding relatively slowly after a predator display. Answer Choice C is incorrect because it only concerns the relative difference in speed for two red-headed finches, a point that the Passage claims is normal. Answer Choice D is incorrect because the Passage tells us that the behavior of individual finches is consistent over time and that red-headed finches are likely to display aggression. For Question 47, Answer Choice A is incorrect because it only describes a feature of the researcher’s methodology, and not something to suggest the atypical behavior of a finch. Answer Choice B is incorrect because it only says that red-headed and black-headed birds differed in their response to novel objects, and doesn’t match any of our answer choices from Question 46. Answer Choice D is incorrect because it recapitulates the differences between red-headed and black-headed birds, but doesn’t match any of our answer choices for Question 46.

How to solve this? Question 46 asks, “It can reasonably be inferred from the passage that is would be atypical for an individual red-headed finch to [...]?” Because this is a Paired Question, we should typically begin by going through the answer choices for Question 47, looking for a Text that can tell us something about what would be atypical of a red-headed finch. However, as we go through the answer choices, we should notice that none of them say explicitly something that is atypical of a single red-headed finch, although a couple do suggest some of the general behavioral differences between red-headed and black-headed finches. But, we should realize that we can’t say in advance which point the Question is going to highlight, so we should [switch gears] and try to approach the Paired Questions from Question 46.

Along the way, we should / may also notice that the Question asks about an “individual” red-headed finch and that one of the answer choices for Question 46 makes a point about individual finches, saying, “These experiments were repeated after a two-month interval and showed that different birds differed in their responses but the responses of individual birds were consistent over time,” so we might expect our answer choice to relate to this.

For Question 45, Answer Choice A says [...].

Page 156: SAT Full Reading Explanations (QAS 9-19) · SAT Full Reading Explanations (QAS 9-19) SAT #9 Reading 1 1.R.9 Answer Choice C is the correct answer because Lines 19-22 say, “Moths

48.R.13 & 49.R.13

Answer Choices A and C are the correct answers because Lines 81-84 say that, “Interestingly boldness and risk taking behaviours were found to be strongly correlated, regardless of head colour they always occurred together, forming a ‘behavioural syndrome.’” This tells us that it would be unlikely for an individual finch to show the risk-taking and bold behaviour of quickly feeding after a predator display, and not show the behaviour of approaching a novel object, or vice versa, which most closely matches Answer Choice A, which says that it would be unlikely for an individual finch to be seen, “returning quickly to feeding after a predator display and failing to approach a novel object.” For Question 48, Answer Choice B is incorrect because returning slowly to feed after a predator display and failing to approach a new object are characteristic of red-headed finches, and wouldn’t be surprising to see in an individual finch. Answer Choice C is incorrect because black-headed finches are described as failing to display aggression and also readily approaching novel objects, so it wouldn’t be surprising to see these two behaviours in an individual finch. Answer Choice D is incorrect because red-headed finches are described as displaying aggression and also being uninterested in exploring new surroundings, so it wouldn’t be surprising to see these two characteristics in a single finch. For Question 46, Answer Choice A is incorrect, because neither our answer choice for Question 45 nor any of the choices violate this finding that aggression does not correlate with risk taking behaviour. Answer Choice B is incorrect because it only comments on why natural selection could have favored red-headed birds taking less risks. Answer Choice D is incorrect because it only explains the evolutionary logic between the behaviors that the birds exhibit.

How to solve this? Question 48 asks, “Based on the passage, which choice reflects behaviors UNLIKELY to be exhibited by an individual finch [...]?” Because this is a Paired Question, we should attempt to answer this Question by going through the answer choices for Question 49 first, looking for any Texts that tell us the behaviors we would be unlikely to see in an individual finch.

Going through the choices, we may find ourselves able to eliminate a few of the options, because they only explain the logic behind some of the conclusions and not the behavior we would be unlikely to see. However, we should find it difficult to effectively choose a single answer, and adjust our strategy so that we work through Question 48 first. Going through the options for Question 48, we should notice that they just list different combinations of behavioural characteristics, like aggression and object approach or feeding after predator display and aggression. Because we’re looking for an unlikely set of behaviors, we simply have to look at what’s expected for black-headed finches and what’s expected for red-headed finches, and then choose the option that violates those correlations. Here are the characteristics we should expect for each type of finch:

Red-Headed Finches

● more aggressive ● slower to return after predator, less risk taking ● less interest in novel objects

Page 157: SAT Full Reading Explanations (QAS 9-19) · SAT Full Reading Explanations (QAS 9-19) SAT #9 Reading 1 1.R.9 Answer Choice C is the correct answer because Lines 19-22 say, “Moths

Black-Headed Finches

● less aggressive ● quicker to return after predator, more risk taking ● more interest in novel objects

Now, we can go through the answer choices for Question 48 and choose the one that violates these correlations. Answer Choice A says [...]. [Eventually choose Answer Choice A.]

[And then from here, going through the answer choices for Question 49 to look for the choice that best supports this].

50.R.13

Answer Choice D is the correct answer because Lines 89-93 say, “This makes sense when you consider the high risk of predation faced by red-headed birds if they take too many risks and the need for black-headed birds to find food away from the dominant red heads which occupy the safest foraging locations.” This tells us that black-headed birds have to take more risks because all of the food in the safe-locations are protected by red-headed birds, and most closely matches Answer Choice D which says that black-headed finches “struggle to obtain food at the safer locations favored by red-headed finches.” Answer Choice A is incorrect because the Passage describes black-headed finches as being more interested in novelty than red-headed finches, and because it wouldn’t make sense as an explanation for their risk taking behavior. Answer Choice B is incorrect because the black-headed finches don’t have conspicuous coloring, unlike the red-headed finches, and because the Passage doesn’t talk about their boldness towards prey. Answer Choice C is incorrect because the Passage never connects aggression and predator attraction, and doesn’t cite this as a reason why black-headed finches take more risks.

How to solve this? The Question asks, “the author indicated that a possible reason for black-headed finches’ risk-taking behavior is that [...]?”

51.R.13

Answer Choice D is the correct answer because [...].

52.R.13

Answer Choice A is the correct answer because [...].

Page 158: SAT Full Reading Explanations (QAS 9-19) · SAT Full Reading Explanations (QAS 9-19) SAT #9 Reading 1 1.R.9 Answer Choice C is the correct answer because Lines 19-22 say, “Moths

SAT #14, Reading Full

SAT #14, Reading 1

Passage Outline

● introduction of the scene at the play. main character notices the surrounding wealth and elegance of the other women. she questions herself — that she hasn’t achieved what she wanted. (paragraph one).

● the scene of the play. the main character wants to be a part of that, the suffering of that world. still wondering at her surroundings and other people. (paragraph two).

● going out into the city. still wondering at the other women, spending so much money. she thinks she hasn’t lived. (paragraph three).

● her own apartment. its drabness. reflections on one character in the play, and her feeling like she could do the role. (paragraph four).

1.R.14

Answer Choice D is the correct answer, because the Passage follows Carrie and her perceptions of a theater performance (Lines 1-47) and then her experience out in the city (Line 48-60), both of which force her to recognize “her own solitude,” “her indifference to the fact that she had never achieved what she had expected,” and the “sharp lesson” that she hadn’t “seen it all.” Answer Choice A is incorrect because while the Passage does tell us that Carrie feels of one actress’s performance that “it was done as she was sure she could do better,” it doesn’t make a close comparison between Carrie and that character, nor does it spend too much focus on the scene. Answer Choice B is incorrect because while Carrie does provide the reader with some of her own perceptions of herself, the Passage doesn’t take the perspective of any other characters, and give us their perception of Carrie. Answer Choice C is incorrect because the Passage doesn’t focus on Carrie’s appearance, nor does it create a contrast between her actual and perceived appearance.

How to solve this? The Question asks, “Throughout the passage, the narrator provides insight into Carrie’s character mainly by [...]?” Because the Question asks about a technique the narrator uses, we might begin by scanning the Passage for any obvious rhetorical elements, like repetition, questions, dramatizations, metaphor, etc. — features which usually stand out of a Passage and can help choose an answer choice more quickly. However, nothing in this Passage stands out, so we should make sure we have a decent general understanding of the content and structure of the Passage, and then go down through the answer choices where we can eliminate and select. Generally, the Passage looks like this:

● introduction of the scene at the play. main character notices the surrounding wealth and elegance of the other women. she questions herself — that she hasn’t achieved what she wanted. (paragraph one).

● the scene of the play. the main character wants to be a part of that, the suffering of that world. still wondering at her surroundings and other people. (paragraph two).

● going out into the city. still wondering at the other women, spending so much money. she thinks she hasn’t lived. (paragraph three).

Page 159: SAT Full Reading Explanations (QAS 9-19) · SAT Full Reading Explanations (QAS 9-19) SAT #9 Reading 1 1.R.9 Answer Choice C is the correct answer because Lines 19-22 say, “Moths

● her own apartment. its drabness. reflections on one character in the play, and her feeling like she could do the role. (paragraph four).

Going through the Answer Choices, we can begin to look for more obviously wrong answers and try to eventually settle on a correct one. Answer Choice A says that the narrator provides insight into Carrie by “explaining how Carrie is similar to a character she sees in a play.” Because this [...].

2.R.14

Answer Choice B is the correct answer because the Passage shows Carrie fantasizing about the wealth and lifestyle of other women (“Whence came the rich, elegant dresses, the astonishingly coloured buttons, the knick-knacks of silver and gold? Where were these lovely creatures housed? Amid what elegancies of carved furniture, decorated walls, elaborate tapestries did they move?”), and concluding that her own life is drab in comparison (“It was not what the rest of the world was enjoying”). This most closely matches Answer Choice B which says that one theme of the Passage is that, “imagined pleasures have the power to make everyday life seem unacceptable.” Answer Choice A is incorrect because the Passage doesn’t focus on Carrie’s “natural talent” or its undernourishment, only suggesting that she feels like she could perform the role of the actress as well as the actress. Answer Choice C is incorrect because the Passage doesn’t represent any goal that a character persistently pursues. Answer Choice D is incorrect because the Passage doesn’t talk about relationships, or how they determine happiness more than money.

How to solve this? The Question asks, “One important theme of the passage developed through the narrator’s presentation of Carrie is that [...]?” To solve this, we should have a general enough understanding of the Passage to be able to eliminate incorrect options, and help us narrow down to a correct one. Answer Choice A says [...].

3.R.14

Answer Choice A is the correct answer because the word “stirred’ in Line 2 means the same as “moved,” emphasizing the emotional arousal of the character. Answer Choice B is incorrect because the character isn’t represented as angry. Answer Choice C is incorrect because it doesn’t accurately replace the word, since “prodded” means to be motivated from an outside force and not emotionally aroused. Answer Choice D is incorrect because “encouraged” is too specific an emotion.

● It was not often that she came to the play stirred to her heart’s core by actualities. ● It was not often that she came to the play moved to her heart’s core by actualities.

Page 160: SAT Full Reading Explanations (QAS 9-19) · SAT Full Reading Explanations (QAS 9-19) SAT #9 Reading 1 1.R.9 Answer Choice C is the correct answer because Lines 19-22 say, “Moths

4.R.14

Answer Choice C is the correct answer because “knick-knacks” is a word that emphasizes the triviality of an object and “concoctions” is one that emphasizes the obviously invented, or fictional quality of a story. Since these are both words that downplay the significance or meaning of something, we can choose Answer Choice C which says that the words “emphasize how little value Carrie actually attaches to the things she appears to covet.” Answer Choice A is incorrect because these words don’t suggest the popularity of something and they’re not used in the Passage for that purpose. Answer Choice B is incorrect because these words don’t offer any special insight into Carrie or her materialism, but only concern the relative value of the objects discussed. Answer Choice D is incorrect because, although Carrie herself describes these objects in diminutive terms, neither she nor the Passage make the additional judgment that they “ought to be regarded as trivial.”

How to solve this? The Question asks, “What main effect do the words ‘knick-knacks’ (line 8) and ‘concoctions’ (line 26) have on the Passage?” To solve this, we should first notice both the casual and slightly negative tone of the words given, and expect that our answer choice may highlight these qualities. [Often, in questions like these, the answer has something to do with the casual quality of the words.] Going back to the Passage we see that in Line 8 Carrie is describing [...]. And in Line 26, [...].

5.R.14

Answer Choice D is the correct answer because the repeated questions in the first paragraph are used rhetorically, that is to communicate a point stylistically rather than ask an actual question. And all of the questions (i.e. “Whence came the rich, elegant dresses, the astonishingly coloured buttons, the knick-knacks of silver and gold? Where were these lovely creatures housed? Amid what elegancies of carved furniture, decorated walls, elaborate tapestries did they move?”) all serve to illustrate Carrie’s near obsession with the women, and her appreciation of them. This most closely matches Answer Choice D which says that the questions “develop the narrator’s portrayal of Carrie’s intense admiration of privileged women.” Answer Choice A is incorrect because the questions are used rhetorically, to make a point, and don’t suggest real ignorance about or interest in the answers to the questions and especially the women’s private struggles. Answer Choice B is incorrect because, while the questions do provide the reader with detail about the women described, they more directly tell us about Carrie and her interest than the women themselves. Answer Choice C is incorrect because it makes a strong and negative a claim (“contempt” is about as strong a word as you can use), that isn’t supported by the Passage.

How to solve this? The Question asks, “The main purpose of the series of questions in the first paragraph is to [...]?” To solve this, we should go back to that series of questions and determine in advance how they function in the paragraph and Passage as a whole. In that paragraph, we see [...].

Page 161: SAT Full Reading Explanations (QAS 9-19) · SAT Full Reading Explanations (QAS 9-19) SAT #9 Reading 1 1.R.9 Answer Choice C is the correct answer because Lines 19-22 say, “Moths

6.R.14

Answer Choice B is the correct answer because Lines 28-31 say, “Such bon-mots are ever enticing to those who have all their days longed for such material surroundings and have never had them gratified.” In this case, “such bon-mots” refers to one of those “drawing-room concoctions” mentioned in the previous sentence, and the narrator describes these plays dismissively, as “concoctions,” but notes that those who have always longed for material goods, like Carrie, find them “ever enticing,” which Carrie eventually does. So, these Lines best tell us that “the narrator does not share Carrie’s view of the type of play that Carrie attends.” Answer Choice A is incorrect because these Lines only tell us an opinion of Carrie and not one of the play. Answer Choice C is incorrect because it only tells us an opinion of Carrie and not of the narrator. Answer Choice D is incorrect because it doesn’t give an opinion of the play, from either Carrie or the narrator.

How to solve this? The Question asks, “Which choice provides the best evidence that the narrator does not share Carrie’s view of the type of play that Carrie attends?” To solve this, we should simply go back through the answer choices given and look for the one which [...].

7.R.14 & 8.R.14

Answer Choices A and A are the correct answers because Lines 22-24 say, “She wondered at her own solitude these two years past — her indifference to the fact that she had never achieved what she had expected.” These Lines best tell us how Carrie “regards her day-to-day lifestyle,” and match Answer Choice A which says that she thinks her lifestyle has “not been conducive to helping her achieve her ambitions.” For Question 8, Answer Choice B is incorrect it only tells us that Carrie wants to be a part of the world represented in the play but not directly what she feels about her current lifestyle. Answer Choice C is incorrect because it only describes the scene that Carrie sees on Broadway not what she feels about her lifestyle. Answer Choice D is incorrect because it only tells us about the “elegant dames” that Carrie observes and not her own lifestyle. For Question 7, Answer Choice B is incorrect because the Text doesn’t describe her lifestyle as repetitive or productive, but about her solitude and failure to achieve her goals. Answer Choice C is incorrect because the Text doesn’t describe her interacting with a range of interesting people. Answer Choice D is incorrect because neither the Text nor the Passage describe Carrie’s ambition to be an actress.

How to solve this? Question 7 asks, “Based on the passage, Carrie regards her day-to-day lifestyle as one that has [...]?” Because this is a Paired Question, to solve this we should begin by going through the answer choices for Question 8, looking for any text that tells us about how Carrie feels about her day-to-day lifestyle. Answer Choice A says [...].

Page 162: SAT Full Reading Explanations (QAS 9-19) · SAT Full Reading Explanations (QAS 9-19) SAT #9 Reading 1 1.R.9 Answer Choice C is the correct answer because Lines 19-22 say, “Moths

9.R.14

Answer Choice C is the correct answer because Lines 31--36 tell us that, “They have the charm of showing suffering under ideal conditions. Who would not grieve upon a gilded chair? Who would not suffer amid perfumed tapestries, cushioned furniture, and liveried servants? Grief under such circumstances becomes an enticing thing.” These Lines most clearly match Answer Choice C, which says that Carrie envies the character’s “opportunity to bear their sufferings in fortunate circumstances.” Answer Choice A is incorrect because the Passage does tell us that the characters in the play have everyday problems, but just that they still have them “under ideal conditions.” Answer Choice B is incorrect because neither the Passage nor our Text tell us that these characters forget the past easily and focus on the present. Answer Choice D is incorrect because the Passage suggests that the characters have an attachment to material possessions, and never tells us of their open indifference to them.

How to solve this? The Question asks, “According to the passage, as Carrie views the play, she envies which aspect of the characters’ lives?” To solve this, we should go back to the Passage where Carrie describes the play, and find a Text that tells us something that Carrie envies. Going back, we should find [...].

10.R.14

Answer Choice A is the correct answer because Lines 70-73 say, “The anguish which [the actress] had portrayed Carrie could feel. It was done as she was sure she could do it. There were places in which she could even do better.” These Lines most closely match Answer Choice A which says that the sweetheart’s performance was, “inferior in certain respects to the performance that she believes herself to be capable of giving.” Answer Choice B is incorrect because the Passage never tells us that Carrie finds the performance dissatisfying, and doesn’t suggest that the actress made a tradeoff between physical grace and emotional authenticity and actually describes her as being especially graceful. Answer Choice C is incorrect because Carrie never describes the actress as clumsy or melodramatic, although she does say that her performance stood out to her above the other actors’ performances. Answer Choice D is incorrect because the Passage never says that Carrie found the play “difficult to watch,” but only that it made her feel.

How to solve this? The Question asks, “According to the passage, Carrie judges the performance of the actress who played the hero’s sweetheart as [...]?” To solve this, we should first go back to the Passage and find where Carrie talks about the performance of this actress [...].

Page 163: SAT Full Reading Explanations (QAS 9-19) · SAT Full Reading Explanations (QAS 9-19) SAT #9 Reading 1 1.R.9 Answer Choice C is the correct answer because Lines 19-22 say, “Moths

SAT #14, Reading 2

Passage Outline

● Dr. Abraham wants to know if the brain responds differently to real and fictional situations, and designed an experiment to find out. (paragraph one).

● the design of the experiment. (paragraph two). ● the results of the experiment. (paragraphs three to five). ● an explanation for the findings. (paragraph six).

11.R.14

Answer Choice B is the correct answer because the Passage primarily focuses on two experiments that explore brain function and response to fictional and real situations. This most closely matches the more general Answer Choice B, which says that the purpose of the passage is to “inform the reader about the results of experiments on brain function.” Answer Choice A is incorrect because, while the Passage does explore an experiment relevant to how the brain differentially processes fictional and real events, it doesn’t advocate for more research on the topic. Answer Choice C is incorrect because the Passage explores how the brain processes real and fictional scenarios, not simply factual information alone. Answer Choice D is incorrect because the Passage never provides a frame of reference for how the way the brain processes fictional situations is widely understood, nor does it make the case that it is misunderstood.

How to solve this? The Question asks, “The main purpose of the passage is to?” To solve this, we should have both a general understanding of the Passage and also go back to specific moments in the title and introduction to determine what it’s about. Generally, the Passage can be organized like this:

● Dr. Abraham wants to know if the brain responds differently to real and fictional situations, and designed an experiment to find out. (paragraph one).

● the design of the experiment. (paragraph two). ● the results of the experiment. (paragraphs three to five). ● an explanation for the findings. (paragraph six).

Specifically, we can look at the title which says, “Joy, Guilt, Anger, Love: What Neuroscience Can — and Can’t — Tell Us about How We Feel,” which doesn’t tell us too much about this specific Passage beyond a very general topic. In the introduction, however, we’re introduced to Dr. Anna Abraham who “wanted to find out whether the brain operates by different mechanisms when it is exposed to a situation that is real as opposed to one that is entirely fiction,” and “designed an interesting fMRI-based experiment that explored the brain’s reactions to situations that involved either [...].” This tells us that the Passage primarily concerns experiments that aim to answer how the brain differentially processes real and fictional scenarios, which our general understanding of the remainder of the Passage confirms.

Page 164: SAT Full Reading Explanations (QAS 9-19) · SAT Full Reading Explanations (QAS 9-19) SAT #9 Reading 1 1.R.9 Answer Choice C is the correct answer because Lines 19-22 say, “Moths

12.R.14

Answer Choice C is the correct answer because the word “operate” in Line 19 is used to mean work or “function.” Answer Choice A is incorrect because it’s unclear to ask “How does the brain manipulate?” without providing something for the brain to be manipulating. Answer Choice B is incorrect because “exercise” is unclear in this context without saying what the brain would be exercising, otherwise it takes on the normal meaning of the word which doesn’t make sense. Answer Choice D is incorrect for similar reasons since it doesn't provide an object for “conduct” and by default would refer to something like musical conduction.

● How does the brain operate when assessing these two different types of scenarios? ● How does the brain function when assessing these two different types of scenarios?

13.R.14

Answer Choice A is the correct answer because Lines 21-24 say, “Common to both types of situation was some level of mental activity in parts of the brain, such as the hippocampus, that are at work when in general recall facts or events,” which best tell us that “there are important similarities between how the brain responds to scenarios involving real people and how it responds to those involving fictional people.” Answer Choice B is incorrect because these lines introduce distinction between the two brain responses and don’t highlight similarities. Answer Choice C is incorrect because it only explains a fact about real characters in our lives, and not a similarity between brain response. Answer Choice D is incorrect because it continues to explain the researcher’s explanations about their findings, and doesn’t compare brain reactions.

How to solve this? The Question asks, “Which choice best supports the claim that there are important similarities between how the brain responds to scenarios involving real people and how it responds to those involving fictional people?” To solve this question, we should go through each of the answer choices and [...].

14.R.14

Answer Choice B is the correct answer because Lines 21-24 say that “common to both types of situation was some level of mental activity in parts of the brain, such as the hippocampus, that are at work when in general recall facts or events,” which tells us clearly that in Abraham’s experiments “both interactive and informative scenarios required memory recall.” Answer Choice A is incorrect because nowhere in the Passage does it talk about “finer distinctions in brain activity” between interactive and informative scenarios. Answer Choice C is incorrect because, although the experiment did rely on this question of possibility to stimulate brain activity, it never gives us the results of those questions nor do the researcher’s findings actually depend on the question. Answer Choice D is incorrect because the Passage doesn’t say that either or all scenarios yielded “unclassifiable brain activity.”

How to solve this? The Question asks, “It can reasonably be inferred from the passage that the results of Abraham’s experiments showed that [...]?” To solve this, we should go back to the Passage and look for what’s said about the results of Abraham’s experiments. Because the Passage looks at two studies, this question could concern the results from either and we should

Page 165: SAT Full Reading Explanations (QAS 9-19) · SAT Full Reading Explanations (QAS 9-19) SAT #9 Reading 1 1.R.9 Answer Choice C is the correct answer because Lines 19-22 say, “Moths

look at both. However, given the position of this question and relying on the Question Order Rule, we should predict that this question will deal with the first experiment. The results begin in Line 21, where the similarities between the studies are introduced and continue to about Line 45. Because this Question is an inference question, and a broad one at that, it may be hard to decide a Text in advance, so, after having reread through the results, we can go down to the answer choices.

Answer Choice A says [...].

15.R.14 & 16.R.14

Answer Choices B and C are the correct answers because Lines 62-64 say, “As predicted, the activation in the amPFC and PCC was indeed proportionally modulated by the degree of relevance of the characters described.” These lines best tell us that scenarios involving characters relevant to the subject’s life are the most likely to stimulate “the greatest increase in activity in the amPFC of a research subject’s brain [...].” For Question 15, Answer Choice B, which describes an interaction “with real people who were childhood friends of the subject’s,” best tells us the scenario that involves character’s that would be most relevant to a subject’s life and thus most likely to increase activity in the am PFC.

For Question 16, Answer Choice A is incorrect because it only describes a feature of the experiment’s design and not anything about amPFC. Answer Choice B is incorrect because it only describes a feature of the second experiment’s design and doesn’t mention amPFC. Answer Choice D is incorrect because it only describes how people understand real characters and doesn’t mention amPFC. For Question 15, Answer Choice A is incorrect because a real unknown person would be less relevant to a subject’s life than people who were childhood friends. Answer Choice C is incorrect because the Passage tells us that real people are more relevant, and thus more likely to stimulate the amPFC, than fictional characters. Answer Choice D is incorrect because childhood friends would more personally relevant to an individual’s life than a person who the subject has simply “previously met.”

How to solve this? Question 15 asks, “The greatest increase in activity in the amPFC of a research subject’s brain would most likely be observed in scenarios in which a real protagonist [...]?” Because this is a Paired Question, to solve this we should begin by going through the answer choices for Question 16, looking for any texts that tell us about what would most stimulate activity in the amPFC. For Question 16, Answer Choice A says [...].

Page 166: SAT Full Reading Explanations (QAS 9-19) · SAT Full Reading Explanations (QAS 9-19) SAT #9 Reading 1 1.R.9 Answer Choice C is the correct answer because Lines 19-22 say, “Moths

17.R.14

Answer Choice D is the correct answer because Lines 81-88 say that, “You may have read all the books about a fictional character, but the amount of information you have gathered about that character is still definitely limited compared with the wealth of information that is available to you about members of your family, friends, or famous real people who are part of your immediate and past experience.” These lines best tell us that the brain stores information about famous real people “in a network of memories broader in scope than memories about fictional people.” Answer Choice A is incorrect because the Passage never makes a distinction about famous real people that involves the brain’s ability to “allow efficient retrieval later.” Answer Choice B is incorrect because, although the Passage does make a comparison between how the brain stores information about famous real people and family and friends, it only does so in drawing a distinction between that group and fictional people, and doesn’t say that they’re stored identically. Answer Choice C is incorrect because the Passage doesn’t make any distinction between how the brain stores information about famous real people according to differential contact with them.

How to solve this? The Question asks, “According to the passage, the brain stores information about famous real people [...]?” To solve this, we should go back to the Passage to find a Text that will tell us about how the brain stores this kind of information. Using the Question Order Rule, we can predict that our Text is more likely to come towards the end of the Passage, around Lines 70-80 or later. From there, we can begin scanning for the phrase “famous real people.” Lines 81-88 say that [...].

● negative evidence ● too strong and specific evidence

18.R.14

Answer Choice B is the correct answer because the word “alien” in Line 80 most clearly means “foreign,” emphasizing the disconnect and difference between us and fictional characters. Answer Choice A is incorrect because the Lines don’t mean that fictional characters are “inconsistent,” but just different from us. Answer Choice C is incorrect because the word isn’t being used to mean out of this world, or “extraterrestrial.” And Answer Choice D is incorrect because the Lines aren’t emphasizing the complexity of these characters, just their difference from us.

● No matter how much we know about the world of a fictional character there will still be something alien and inscrutable to us about that world.

● No matter how much we know about the world of a fictional character there will still be something foreign and inscrutable to us about that world.

Page 167: SAT Full Reading Explanations (QAS 9-19) · SAT Full Reading Explanations (QAS 9-19) SAT #9 Reading 1 1.R.9 Answer Choice C is the correct answer because Lines 19-22 say, “Moths

19.R.14

Answer Choice C is the correct answer because the black, second column in Figure 1 measures the percent change in PCC activity for real people interacting rises tells us about the “subject’s reaction to a scenario in which someone has a conversation with a real person” and rises to about .13%, which matches Answer Choice C. Answer Choice A is incorrect because [...].

20.R.14

Answer Choice B is the correct answer because the highest percent change measured in either graph is the fourth, gray column in Figure 2 which measures the “Percent Change in Activity in the Inferior Frontal Gyrus in Experimental Scenarios” for fictional characters engaging in an interactive scenario. This matches Answer Choice B which says the highest percent change was in the “IFG when the subjects were exposed to an interactive scenario involving fictional people.” Answer Choice A is incorrect because [...].

SAT #14, Reading 3

Passage Outline

● the difference between a housecat and wildcat. scientists have found the genetic change behind this transformation. (paragraph one)

● a quote from scientist about discovering the genetics behind domestication. (paragraph two)

● history of domestication of cats. (paragraph three) ● the scientific study and some findings. (paragraphs four and five) ● the most intriguing findings, and specific genes they located. (paragraph six) ● a quote from scientist about important of this last finding. (paragraph seven) ● other genes the scientists found. (paragraph eight) ● comparing domestic cats to dogs. (paragraph nine)

21.R.14

Answer Choice C is the correct answer because the Passage primarily concerns research into the genetics behind the process of domestication of cats, which most closely matches Answer Choice C which says that the main purpose of the passage is to “discuss research into the genetic changes that cats underwent as part of domestication.” Answer Choice A is incorrect because the Passage doesn’t look at a study that compares different breeds of domestic cats. Answer Choice B is incorrect because the Passage doesn’t discuss the benefits that domestic cats provided to humans. Answer Choice D is incorrect because, although the Passage does highlight some differences between wildcats and domestic cats, it doesn’t take this difference as a central focus or present is as experimental evidence.

Page 168: SAT Full Reading Explanations (QAS 9-19) · SAT Full Reading Explanations (QAS 9-19) SAT #9 Reading 1 1.R.9 Answer Choice C is the correct answer because Lines 19-22 say, “Moths

How to solve this? The Question asks, “The main purpose of the passage is to [...]?” To solve this, we should have both a general understanding of the Passage as well as a more specific understanding of information in the title and introductory paragraph. Generally, the Passage can be outlined like this:

● the difference between a housecat and wildcat. scientists have found the genetic change behind this transformation. (paragraph one)

● a quote from scientist about discovering the genetics behind domestication. (paragraph two)

● history of domestication of cats. (paragraph three) ● the scientific study and some findings. (paragraphs four and five) ● the most intriguing findings, and specific genes they located. (paragraph six) ● a quote from scientist about important of this last finding. (paragraph seven) ● other genes the scientists found. (paragraph eight) ● comparing domestic cats to dogs. (paragraph nine)

More specifically, we can look at the title and introductory paragraph. The title of the Passage is, “The Genes That Turned Wildcats into Kitty Cats,” which does tell us quite clearly what it’s about and that it focuses on the genetic changes behind the domestication of wildcats. The introductory paragraph begins by comparing wildcats and domestic cats, and finishes by saying that, “Now scientists have begun to pinpoint the genetic changes that drove this remarkable transformation.” Taking these together, we can say that the Passage primarily concerns research into the underlying genetic changes that occurred when wildcats became domestic cats, and we should expect our answer choice to reference this.

Answer Choice A says [...].

22.R.14

Answer Choice D is the correct answer because the use of “and, well” in line 4 is used in the context of the author describing the similarity between domestic cats and wildcats, and is a casual phrase meant to underscore that the comparisons are obvious. This most closely matches Answer Choice D which says that the author includes the phrase to “acknowledge the obviousness of a similarity between wildcats and domestic cats.” Answer Choice A is incorrect because neither the Passage, Line or phrase suggests that the relationship between the two types of cats is shifting. Answer Choice B is incorrect because the phrase doesn’t precede a discussion about the physical attributes of domestic cats. Answer Choice C is incorrect because no hypothesis about wildcats is given in these lines.

How to solve this? The Question asks, “The author includes the phrase ‘and, well’ (line 4) most likely to [...]?” To solve this, we should go back to the lines in question to determine how they’re used in context. Lines 3-4 say [...].

Page 169: SAT Full Reading Explanations (QAS 9-19) · SAT Full Reading Explanations (QAS 9-19) SAT #9 Reading 1 1.R.9 Answer Choice C is the correct answer because Lines 19-22 say, “Moths

23.R.14 & 24.R.14

Answer Choices C and A are the correct answers because Lines 25-28 say, “There, many scientists suspect, they mostly domesticated themselves, with the friendliest ones able to take advantage of human table scraps and protection,” emphasizing the lack of human involvement in the domestication process. This most closely matches Answer Choice C from Question 23, which says that scientists would agree that the domestication of cats “was not intentionally undertaken by humans.” For Question 24, Answer Choice B is incorrect because it less clearly makes a claim about something “scientists would agree with” and doesn’t match any of the answer choices for Question 23. Answer Choice C is incorrect because it concerns the domestication of other animals, not just that of cats. Answer Choice D is incorrect because it concerns the finding of the research study and not what scientists would think about the domestication of cats. For Question 23, Answer Choice A is incorrect none of the texts compare the relative timeline of domestication for different animals. Answer Choice B is incorrect because the Passage says that cats “shrunk” slightly in size, not that they increased. Answer Choice D is incorrect because the Passage tells us that cats “domesticated themselves” and not that humans needed them as pets.

How to solve this? Question 23 asks, “The passage suggests that some scientists would agree with which statement about the domestication of cats?” Because this is a Paired Question, to solve it we should begin by going through the answer choices for Question 24, looking for any texts that tell us about a view of scientists on the domestication of cats. Answer Choice A says [...].

25.R.14

Answer Choice C is the correct answer because the sixth paragraph describes the order of Montague’s research as (1) sequencing the genomes of 22 domestic cats, (2) comparing them to the genomes of “two Near Eastern and two European wildcats,” (3) and uncovering “at least 13 genes that changed as cats morphed from feral to friendly.” This most closely matches Answer Choice C which says, “The team sequenced the genomes of several breeds of geographically diverse domestic cats, compared those genomes to the genomes of four wildcats from two locations, and identified the genes that appear to have changed due to domestication.” Answer Choice A is incorrect because the scientists didn’t compare the genomes of the domestic cats “to one another” but to the other wildcats, and didn’t identify genes that changed as a result of breeding but those that changed as a result of domestication. Answer Choice B is incorrect because the researchers sequenced the genomes themselves, used the genomes from 22 cats and not from one, and didn’t look only at the genes associated with hearing and vision. Answer Choice D is incorrect because the team sequenced the genomes from cats at may locations and didn’t look at genes which are geographically specific.

How to solve this? The Question asks, “Which choice best describes how Montague’s team achieved the results discussed in the sixth paragraph (line 52-67)?” To solve this, we should go back to the sixth paragraph and track the order of the research team’s process, and then compare that to the answer choices given.

Page 170: SAT Full Reading Explanations (QAS 9-19) · SAT Full Reading Explanations (QAS 9-19) SAT #9 Reading 1 1.R.9 Answer Choice C is the correct answer because Lines 19-22 say, “Moths

26.R.14 & 27.R.14

Answer Choices A and C are the correct answers because Lines 59-63 say, “Some of these, based on previous studies of knockout mice [genetically engineered mice], seem to play a role in cognition, including fear responses and the ability to learn new behaviors when given food rewards.” In these Lines, we see that the scientists make the assumption that the genes which code for certain behaviors in mice will code for the same behavior in cats, which most closely matches Answer Choice A which says that the scientists make the assumption that “genes associated with particular characteristics in one species are associated with similar characteristics in other species.” For Question 27, Answer Choice A is incorrect because it only describes part of the methodology of the scientists without suggesting a clear assumption they made or one that would match the answer choices for Question 26, and isn’t a part of Montague’s explanation. Answer Choice B is incorrect because it only gives the results of the experiment without suggesting a clear assumption that was made or explaining the gene changes in domestic cats. Answer Choice D is incorrect because it gives a quote from a scientist besides Montague and doesn’t suggest an assumption that was made. For Question 26, Answer Choice B is incorrect because none of the texts describe the scientists replacing missing sequences in the genome of one breed of cat with another. Answer Choice C is incorrect because none of the Texts compare the different rate of genes affected by domestication. Answer Choice D is incorrect because none of the Texts make the claim that the genome change associated with domestication occurred at the same time for different species.

How to solve this? Question 26 asks, “As presented in the passage, Montague’s explanation for the gene changes in domestic cats is based on the assumption that [...]?” Because this is a Paired Question, to solve this we should first go through the answer choices for Question 27, looking for any text that makes a scientific assumption, comes from the perspective of Montague and concerns an explanation for the gene changes in domestic cats.

For Question 27, Answer Choice A says [...].

28.R.14

Answer Choice C is the correct answer because Lines 80-84 say of the finding that certain genes influence the migration of neural crest cells that “this supports a recent proposal that such cells may be the master control switches of domestication, explaining why domestic animals share common traits, such as smaller brains and certain pigmentation patterns.” This designation of the neural crest cells as the “master control switches of domestication” tell us that their importance lies in their being foundational to the overall process which most closely matches Answer Choice C, which says that the cells are “thought to be fundamental to the overall process of the domestication of animals.” Answer Choice A is incorrect because the Passage doesn’t explain that the neural crest cells are “likely to be overlooked.” Answer Choice B is incorrect because the Passage doesn’t explain that neural crest cells help domestic animals thrive in harsh environments. Answer Choice D is incorrect because the Passage doesn’t connect neural crest cells to the domestic animals’ reproduction.

How to solve this? The Question asks, “According to the passage, the discovery that some genes in domestic cats influence the migration of neural crest cells is important because neural

Page 171: SAT Full Reading Explanations (QAS 9-19) · SAT Full Reading Explanations (QAS 9-19) SAT #9 Reading 1 1.R.9 Answer Choice C is the correct answer because Lines 19-22 say, “Moths

crest cells are [...]?” To solve this, we should go back to the Passage and look for the Text that will tell us about the importance of crest cells. Given the location of this question in the series of questions about the Passage, we should expect our Text come later in the Passage, probably in the 60-80 range.

Lines 80-84 say [...].

29.R.14

Answer Choice D is the correct answer because the words “common traits” in Line 83 are used to refer to physical features like “smaller brains and certain pigmentation patterns,” and so can best be replaced by “similar features.” Answer Choice A is incorrect because the traits referred to are physical and not “widespread mannerisms,” which are too anthropomorphic regardless. Answer Choice B is incorrect because “common traits” doesn’t refer to “inferior qualities,” which adds the extra dimension of inferiority. Answer Choice C is incorrect because “common traits” refers to physical features, not behavioral habits.

● This supports a recent proposal that such cells may be the master control switches of domestication, explaining why domestic animals share common traits, such as smaller brains and certain pigmentation patterns.

● This supports a recent proposal that such cells may be the master control switches of domestication, explaining why domestic animals share similar features, such as smaller brains and certain pigmentation patterns.

30.R.14

Answer Choice B is the correct answer because the quotation given in the last sentence of the Passage is used to offer an explanation for why cats are “still a bit wilder than our other favorite domesticate, the dog.” This most closely matches Answer Choice B which says that the quotation is used to “suggest a potential explanation for the distinct way in which domestic cats developed.” Answer Choice A is incorrect because the quotation given doesn’t offer an explanation about the domestication of cats, and especially not one that is an alternative to a generally accepted belief. Answer Choice C is incorrect because the quotation doesn’t concern a continual change in the cat genome. Answer Choice D is incorrect because the quote more specifically addresses the differences between the domestication of cats and dogs, and doesn’t generally concern findings about domestic animals at large.

How to solve this? The Question asks, “The author most likely includes the quotation in the last sentence of the passage to [...]?” To solve this, we should go back to the quotation given at the end of the Passage and determine its use in context.

The quote reads [...].

Page 172: SAT Full Reading Explanations (QAS 9-19) · SAT Full Reading Explanations (QAS 9-19) SAT #9 Reading 1 1.R.9 Answer Choice C is the correct answer because Lines 19-22 say, “Moths

SAT #14, Reading 4

Passage Outline

Passage 1

“Defense of the Kansas-Nebraska Bill”

● “Douglas, a senator from Illiniois, proposed a bill allowing voters in the new territories of Kansas and Nebraska to decide whether slavery should be permitted there.”

● Douglas’s opinion on the argument of his opponents — that they assume the founding fathers wanted to prohibit slavery in an all new states. (paragraph one).

● Douglas’s argument — that the early Presidents thought slavery would be allowed in some states and not in others, depending on the geography and economy of those places. (paragraph two).

● Douglas’s opinion that the question is simple — will people and states be allowed to decide for themselves on the issue of slavery? (paragraph three).

Passage 2

“The Crime against Kansas”

● Sumner’s opinion of the Nebraska bill — that it was a swindle, or theft. (paragraph one). ● This characteristic of theft shows itself even in the bill’s structure — it claimed to allow

for the people in a territory to decide things themselves, but they had all those privileges denied to them. (paragraph two).

● There’s not enough time to show all the defects of this bill. It’s enough to say that slavery is against human rights, and “no valid claim for slavery” can be set up in nation. (paragraph three).

31.R.14

Answer Choice A is the correct answer because the word “recognized” is used in Line 12 to show that the early Presidents understood, or acknowledged, the right of certain territories to hold slaves. Answer Choice B is incorrect because the Lines only go so far as to say that the early Presidents made an acknowledgement of the right, not that they approved or “commended” it. Answer Choice C is incorrect because the Lines don’t say that the early Presidents “confessed” this right, which has an extra element of contrition implied in it. Answer Choice D is incorrect because the Lines don’t mean that the early Presidents recalled or remembered the right.

● “You find upon the statute books under Washington and the early Presidents provisions of law showing that in the southwestern territories the right to hold slaves was clearly implied or recognized, while in the northwest territories it was prohibited.”

● “You find upon the statute books under Washington and the early Presidents provisions of law showing that in the southwestern territories the right to hold slaves was clearly implied or formally acknowledged, while in the northwest territories it was prohibited.”

Page 173: SAT Full Reading Explanations (QAS 9-19) · SAT Full Reading Explanations (QAS 9-19) SAT #9 Reading 1 1.R.9 Answer Choice C is the correct answer because Lines 19-22 say, “Moths

32.R.14 & 33.R.14

Answer Choices C and D are the correct answers because Lines 33-36 say, “If it is wrong, let it be repudiated. Let all this quibbling about the Missouri Compromise, about the territory acquired from France about the act of 1820, be cast behind you [...],” which provides the clearest instance of an address towards the legislators and what he feels about their consideration of the bill he has proposed. The Lines show Douglas minimizing some of the discussion surrounding the issue, calling it “quibbling,” and telling them to put those questions behind them to focus on the central question. This most closely matches Answer Choice C, which says that Douglas thinks his fellow legislators’ consideration of the bill has been clouded by the “discussion of issues that are fundamentally beside the point.”

For Question 33, Answer Choice A is incorrect because it doesn’t provide a clear instance of Douglas’s thoughts on the legislators’ consideration of his proposal. Answer Choice B is incorrect because it only gives Douglas’s opinion of the bill, and not of his fellow legislators. Answer Choice C is incorrect because it also only gives Douglas’s opinion of his bill, and not of the other legislators. For Question 32, Answer Choice A is incorrect because nowhere in the Passage or Texts does Douglas make the point that the legislators have been clouded in their judgment by the “moral dillemmas inherent in the issue of slavery.” Answer Choice B is incorrect is incorrect because in neither the Texts nor the Passage does Douglas talk about the distracting nature of the “emphatic language that characterizes the text of the bill.” Answer Choice D is incorrect because nowhere does Douglas talk about preexisting loyalties or rivalries among the legislators.

How to solve this? Question 32 asks, “In Passage 1, Douglas implies that legislators’ consideration of the bill he has proposed has been clouded by the [...]?” Because this is a Paired Question, to solve it we should go through the answer choices for Question 33, looking for ant texts that tell us something about how Douglas thinks of the “legislators’ consideration of the bill he has proposed.” [...]

● The influence of implication here. ● Answer Choice D says, “If it is wrong, let it be repudiated. Let all this quibbling about the

Missouri Compromise, about the territory acquired from France about the act of 1820, be cast behind you [...].” Although subtle, this Text most clearly tells us an opinion on the legislators’ consideration of his proposed bill, advising them to cast aside “all this quibbling” and consider the importance of the question at hand.

Page 174: SAT Full Reading Explanations (QAS 9-19) · SAT Full Reading Explanations (QAS 9-19) SAT #9 Reading 1 1.R.9 Answer Choice C is the correct answer because Lines 19-22 say, “Moths

34.R.14

Answer Choice B is the correct answer because the repetition of the word “swindle” in the first paragraph is used to express Sumner’s strong, negative opinion of the bill, with him saying that “no other word will adequately express the mingled meanness and wickedness of the cheat.” This most closely matches Answer Choice B which says that the word creates “an outraged tone that expresses Sumner’s indignation over the Kansas-Nebraska Bill.” Answer Choice A is incorrect because the Passage doesn’t give us any indication of Sumner having distrust for his fellow senators. Answer Choice C is incorrect because nowhere in the Passage does Sumner express a “conviction that the Kansas-Nebraska Act cannot be repealed.” Answer Choice D is incorrect because the Passage doesn’t indicate that Sumner feels a sense of betrayal over the founding father’s support of slavery, which he actually later suggests was a position they did not hold.

How to solve this? The Question asks, “The repetition of ‘swindle’ in the first paragraph of Passage 2 has the main effect of creating [...]?” To solve this, we should go back to the first paragraph of Passage 2 and look at the use of the word in context. [...].

35.R.14 & 36.R.14

Answer Choices D and B are the correct answers because Lines 58-61 say that, “Amid overflowing professions of regard for the sovereignty of the People in the Territory, they were despoiled of every essential privilege of sovereignty,” which tells us clearly an effect of the Kansas-Nebraska Bill. These Lines tell us that while the bill made the claim that the rights and authority of citizens would be respected, in reality they weren’t given that privilege. This most closely matches Answer Choice D which says that the Bill had the effect of “giving the impression that the rights of citizens in certain territories were being expanded when they were in fact being curtailed.”

For Question 36, Answer Choice A is incorrect because it gives more of a description of the Bill itself than its effect, and because it doesn’t clearly match any of the answer choices given. Answer Choice C is incorrect because it only gives Sumner’s opinion of slavery and not of the effect of the Kansas-Nebraska Bill. Answer Choice D is incorrect because it gives an address of Sumner to his but doesn’t comment on the effect of the Kansas-Nebraska Bill. For Question 35, Answer Choice A is incorrect because none of the Texts discusses the balance of authority between the two types of territorial governments. Answer Choice B is incorrect because the Passage doesn’t clearly describe a federal policy that has widespread popularity of its legitimacy or how the Nebraska Bill deprived it of its legitimacy. Answer Choice C is incorrect because it less clearly matches the Texts given than Answer Choice D, and because the Passage doesn’t mention a territory where the Compromise yielded no obvious benefit or how the citizens there were disenfranchised.

How to solve this? Question 35 asks, “One of Sumner’s central claims in Passage 2 is that the Kansas-Nebraska Bill had the effect of [...]?” Because this is a Paired Question, to solve it we should begin by going through the answer choices for Question 36, looking for any texts that tell us about an effect the Bill.

Page 175: SAT Full Reading Explanations (QAS 9-19) · SAT Full Reading Explanations (QAS 9-19) SAT #9 Reading 1 1.R.9 Answer Choice C is the correct answer because Lines 19-22 say, “Moths

For Question 36, Answer Choice A says [...].

37.R.14

Answer Choice D is the correct answer because Lines 61-64 say that, “They were not allowed to choose their governor, secretary, chief justice, associate justices, attorney, or marshal — all of whom are sent from Washington [...],” which most clearly tell us that the national government had the authority to “appoint officials to territorial posts.” Answer Choice A is incorrect because nowhere in the Passage is the right of the national government to regulate commerce mentioned. Answer Choice B is incorrect because the Passage, while it does talk about federal laws regarding slavery, never mentions the abilitiy or inability of that government to enforce laws concerning slavery. Answer Choice C is incorrect because the Passage also doesn’t discuss the national government mediating any disputes between governments.

How to solve this? The Question asks, “In Passage 2, Sumner indicates that under the Kansas-Nebraska Bill, the national government had the authority to [...]?” To solve this, we should go back to the Passage first and look for any texts that talk about the national government or any of its authority to do certain things. In the second paragraph for Passage 2 we see [...].

38.R.14

Answer Choice D is the correct answer because Lines 76-77 say “In refusing [slavery] such a place, I do not deny popular rights, but uphold them [...],” which continues Sumners’s argument that the claim that supporters of slavery make about popular sovereignty or the authority of citizens is unfounded, and that his view, rather, upholds actual support of popular sovereingty. This most clearly matches Answer Choice D which says that Sumner makes his argument to “argue that his position on slavery constitutes authentic popular sovereignty.” Answer Choice A is incorrect because Sumner is here arguing against supporters of slavery and not making the case that he shares certain fundamental values with them. Answer Choice B is incorrect because Sumner doesn’t make claims about the historical legacy of slavery, nor does he provide an explanation for them in these lines. Answer Choice C is incorrect because Sumner isn’t expressing an opinion on abolition in these lines.

How to solve this? The Question asks, “In the context of Sumner’s argument, lines 76-77 (“In refusing...them”) serve primarily to [...]?” To solve this, we should go back to the lines mentioned and determine how they work in the Passage both in terms of content and rhetoric.

Page 176: SAT Full Reading Explanations (QAS 9-19) · SAT Full Reading Explanations (QAS 9-19) SAT #9 Reading 1 1.R.9 Answer Choice C is the correct answer because Lines 19-22 say, “Moths

39.R.14

Answer Choice A is the correct answer because, in the Passage, Douglas characterizes the Kansas-Nebraska Bill as simple and direct, saying in Lines 25-31, “Is not the question involved the simple one, whether the people of the territories shall be allowed to do as they please upon the question of slavery, subject only to the limitations of the Constitution?. That is all the bill provides; and it does so in clear, explicit, and unequivocal terms.” Sumner, on the other hand, characterizes the bill negatively, calling it “in every respect a swindle” because “no other word will adequately express the mingled meanness and wickedness of the cheat.” These characterizations most cleary match Answer Choice A which says of the Kansas-Nebraska Bill that “Douglass characterizes it as straightforward in its intent, while Sumner characterizes it as fundamentally deceptive.” Answer Choice B is incorrect because Douglas thinks that the Bill protects popular sovereignty not the interests of slaveholding territories, and Sumner thinks that it unfairly benefits slave territories, not that it burdens them. Answer Choice C is incorrect because Douglas doesn’t clearly make the case that the bill is progressive in its treatment of slavery and Sumner doesn’t make the case that it upholds the status quo. Answer Choice D is incorrect because both authors think that the Bill is relevant to national concerns.

How to solve this? The Question asks “Which choice best identifies a distinction in how Douglas and Sumner characterize the Kansas-Nebraska Bill?” To solve this, we should have a general understanding of both authors’ point of view as well as go back to both Passages to try and find specific evidence for each of their views on the Bill. Generally [...]

40.R.14

Answer Choice D is the correct answer because the “simple question” Douglas mentions in Lines 36-37 refers to his question, “Will you allow the people to legislate for themselves upon the subject of slavery?” which Sumner would certainly find “immoral,” given what he says in Lines 72-76, “Suffice it to say that slavery is in itself an arrogant denial of human rights, and by no human reason can the power to establish such a wrong be placed among the attributes of any just sovereignty.” Answer Choice A is incorrect because Douglas doesn’t make the case of putting the interests of territories above those of established states and Sumner never acknowledges that kind of tension. Answer Choice B is incorrect because Sumner believes that territorial citizens don’t have the “freedom to legislate on local matters,” claiming that “they were despoiled of every essential privilege of sovereignty.” Answer Choice C, although it might seem plausible, is incorrect because Sumner’s point in Passage 2 is not that the Bill has other provisions beyond territorial sovereignty, but that violating human rights through the allowance of slavery is incompatible with popular sovereignty.

How to solve this? The Question asks, “Sumner would most likely fault the “simple question” (lines 36-37) proposed by Douglas in Passage 1 for being [...]?” To solve this, we should first go back to the “simple” question mentioned in Passage 1 and then decide, based on our general understanding of Passage 2, how Sumner would be most likely to respond to it. Lines 36-37 say [...].

Page 177: SAT Full Reading Explanations (QAS 9-19) · SAT Full Reading Explanations (QAS 9-19) SAT #9 Reading 1 1.R.9 Answer Choice C is the correct answer because Lines 19-22 say, “Moths

41.R.14

Answer Choice D is the correct answer because Douglas makes the claim that the Constitution allows for slavery in the Southwestern states, saying in Lines 8-12, “You find upon the statute books under Washington and the early Presidents provisions of law showing that in the southwestern territories the right to hold slaves was clearly implied or recognized, [...],” while Sumner clearly believes the opposite, saying in Lines 80-83, “but also to the genius of our own Constitution, under which, when properly interpreted, no valid claim for slavery can be set up anywhere in the national territory.” These views most clearly match Answer Choice D which says that, “Douglas suggests that slavery may be permitted in the territories without violating the Constitution, whereas Sumner insists that slavery is not compatible with the ideals of the Constitution.” Answer Choice A is incorrect because Douglas and Sumner don’t directly differ in their opinion on the flexibility or rigidness of the Constitution, but on its provisions for slavery. Answer Choice B is incorrect because Douglas and Sumner don’t differ in their opinion on the clarity or ambiguity of the Constitution and its interpretation, but more directly on what it says about slavery. Answer Choice C is incorrect because Sumner doesn’t directly argue that Constitution treats states and territories equally, but simply that the Constitution doesn’t allow for slavery under its correct interreptation.

How to solve this? The Question asks, “Which choice best identifies a key difference in how Douglas and Sumner make use of the Constitution?” To solve this, we should go back to both Passages and scan for what either thinks about the Constitution.

Lines [...].

SAT #14, Reading 5

Passage Outline

“Staring Back to Cosmic Dawn”

● The Hubble Space Telescope, and its ability to “see back in time.” wondering if it could see back to the beginning of the universe. (paragraph one).

● Two scientists tried this using Hubble Ultra Deep Field and saw some of the oldest galaxies, but it was only a small part of the universe. (paragraph two).

● So they designed a new program, CANDELS, to look at the early galaxy. (paragraph three).

● This program became the largest ever done by Hubble and recovered a lot of information for five “patches” of the universe. (paragraph four).

● The scientists chose these patches because they’re representative of the universe as a whole. (paragraph five).

● Even though the area the telescope searched was small, it revealed a lot of data. (paragraph six).

● The data will continue to be analyzed and has already helped scientists. (paragraph seven).

Page 178: SAT Full Reading Explanations (QAS 9-19) · SAT Full Reading Explanations (QAS 9-19) SAT #9 Reading 1 1.R.9 Answer Choice C is the correct answer because Lines 19-22 say, “Moths

42.R.14

Answer Choice D is the correct answer because the Passage begins by introducing the Hubble Space Telescope in the first paragraph, describing its “extraordinarily sensitive detectors.” It then looks at a series of research projects that used the Hubble Telescope, like the one that looked at the HUDF or CANDELS. This organization of the Passage best matches Answer Choice D which says that the focus of the passage shifts from “an overview of the capabilities of the HST to a discussion of research undertaken that utilized those capabilities.” Answer Choice A is incorrect because the Passage doesn’t begin with data that the HST collected and because the Passage doesn’t describe any design flaws of the telescope. Answer Choice B is incorrect because no “new theory of star formation” is mentioned in the Passage or tested. Answer Choice C is incorrect because the Passage doesn’t discuss the authors or their accomplishments.

How to solve this? The Question asks, “Over the course of the passage, the main focus shifts from [...]?” To solve this Question, we should have a general understanding of the overall Passage and then go down to the answer choices to eliminate and select options. We can try to predict in advance where the Passage might present a clear shift in focus, but except for more obvious instances, it’s hard to know which break of several possible breaks the SAT will highlight. Generally, the Passage can be broken down like this:

“Staring Back to Cosmic Dawn”

● The Hubble Space Telescope, and its ability to “see back in time.” wondering if it could see back to the beginning of the universe. (paragraph one).

● Two scientists tried this using Hubble Ultra Deep Field and saw some of the oldest galaxies, but it was only a small part of the universe. (paragraph two).

● So they designed a new program, CANDELS, to look at the early galaxy. (paragraph three).

● This program became the largest ever done by Hubble and recovered a lot of information for five “patches” of the universe. (paragraph four).

● The scientists chose these patches because they’re representative of the universe as a whole. (paragraph five).

● Even though the area the telescope searched was small, it revealed a lot of data. (paragraph six).

● The data will continue to be analyzed and has already helped scientists. (paragraph seven).

Page 179: SAT Full Reading Explanations (QAS 9-19) · SAT Full Reading Explanations (QAS 9-19) SAT #9 Reading 1 1.R.9 Answer Choice C is the correct answer because Lines 19-22 say, “Moths

43.R.14

Answer Choice C is the correct answer because Lines 8-12 say, “And at the outer limits of its capabilities, we wondered if it could detect the faintest candles of creation: the earliest galaxies made of the earliest stars at cosmic dawn, when the universe was less than a billion years old.” These lines best tell us that the “first galaxies formed less than 1 billion years after the big bang.” Answer Choice A is incorrect because it only describes the HST and its capabilities, but not about the formation of the first galaxies. Answer Choice B is incorrect because it describes the star-formation that occurred at “cosmic high noon” and ended 5 billion years ago, not about the formation of the first galaxies. Answer Choice D is incorrect because it describes how the HUDF detected distant galaxies, but not about the formation of the first galaxies.

How to solve this? The Question asks, “Which choice best supports the conclusion that the first galaxies formed less than 1 billion years after the big bang?” To solve this question, we should go back through each of the answer choices given, looking for any text that tells us about these early galaxies. Lines 1-4 say [...].

44.R.14

Answer Choice A is the correct answer because Lines 22-24 say, “Those deep HUDF images revealed some of the most distant galaxies ever found, which look very different than nearby galaxies.” These “most distant galaxies” refer to galaxies from billions of years ago, and these lines most clearly match Answer Choice A which says that galaxies from billions of years ago “looked very different from galaxies today.” Answer Choice B is incorrect because the Passage only mentions that Type Ia supernovae are a “valid yardstick for the early universe,” not that they dominated galaxies billions of years ago. Answer Choice C is incorrect because the Passage doesn’t tell us that galaxies from billions of years ago were grouped together in one location, and actually implies the opposite when it says that the HUDF “represented just a pinprick poke at the universe.” Answer Choice D is incorrect because the Passage doesn’t tell us that these early galaxies were larger than astronomers had expected.

How to solve this? The Question asks, “Based on the passage, what can reasonably be inferred about galaxies billions of years ago?” To solve this question, we should go back to the Passage and look for a Text that tells us about these old galaxies. Using the Question Order Rule, we can predict that our text is likely to come before Line 31. Lines 22-24 tell us [...].

Page 180: SAT Full Reading Explanations (QAS 9-19) · SAT Full Reading Explanations (QAS 9-19) SAT #9 Reading 1 1.R.9 Answer Choice C is the correct answer because Lines 19-22 say, “Moths

45.R.14

Answer Choice C is the correct answer because the word “document” in Line 31 most closely means “record,” in the sense of “take note of.” Answer Choice A is incorrect because it doesn’t make sense for the Passage to say that CANDELS will “produce” the first one-third of galaxy evolution. Answer Choice B is incorrect because it’s unclear what it would mean for the program to “define” the first one-third of galaxy evolution and isn’t expressed in the Passage. Answer Choice D is incorrect because the Lines don’t mean that the program will “construct” or create the first one-third of galaxy evolution.

● We designed CANDELS primarily to document the first one-third of galaxy evolution. ● We designed CANDELS primarily to record the first one-third of galaxy evolution.

46.R.14

Answer Choice A is the correct answer because Lines 32-38 say that, “The program also would enable astronomers to search for the most distant Type Ia supernovae — exploding whitte dwarf stars that are the best-known standard candles for measuring the universe’s recent expansion rate. CANDELS could thus test whether Type Ia supernovae are also a valid yardstick for the early universe.” These lines tell us that scientists use Type Ia supernovae to measure the expansion rate for the relatively recent universe, and that CANDELS could tell them if they could use the same supernovae to measure expansion in distant past. This most closely matches Answer Choice A, which says that Type Ia supernovae may be useful because they may help astronomers to “determine the expansion rate of the universe soon after the big bang.” Answer Choice B is incorrect because the Passage doesn’t mention other methods of measuring distance nor does it talk about how Type Ia supernovae are used to measure distance. Answer Choice C is incorrect because the Passage doesn’t mention astronomers interest in discovering what causes white dwarf stars to explode or how Type Ia supernovae could help them determine this. Answer Choice D is incorrect because the Passage doesn’t mention scientists wanting to develop a catalog of earliest known galaxies by physical properties or how Type Ia supernovae could help them get there.

How to solve this? The Question asks, “In the context of the research described in the passage, Type Ia supernovae may be useful because they could allow astronomers to [...]?” To solve this, we should go back to the Passage to find what it says about these kinds of supernovae and how astronomers may want to use them. Using the Question Order Rule, we can predict that our Text is likely to come before Line 48. Lines 32-38 say, [...].

Page 181: SAT Full Reading Explanations (QAS 9-19) · SAT Full Reading Explanations (QAS 9-19) SAT #9 Reading 1 1.R.9 Answer Choice C is the correct answer because Lines 19-22 say, “Moths

48.R.14 & 49.R.14

Answer Choices B and C are the correct answers because Lines 53-58 say, “Much as pollsters and medical researchers learn about the human population as a whole by studying carefully selected samples of a small number of individuals, we chose the five target areas because they’re physically representative of the universe at large.” These Lines best tell us how the authors would likely think about deep all-sky surveys. Just as pollsters and medical researchers look at representative samples and not the entire population, these researchers would be best looking at representative samples of the sky as opposed to analyzing it entirely. Thus, we can say that the authors would most likely call such large surveys “unnecessary,” matching Answer Choice B. For Question 49, Answer Choice A is incorrect because it describes CANDELS and and doesn’t give us an indication of how scientists would think about such surveys mentioned. Answer Choice B is incorrect because it describes the five target areas surveyed by the study and not what the authors are likely to think about conducting all-sky surveys. Answer Choice D is incorrect because it only comments on the rewarding nature of the new data gathered from the study and not about the author’s thoughts on conducting those kinds of surveys. For Question 48, Answer Choice A is incorrect because our Text describes these scientists as taking a sample of the entire sky, and are likely to think that taking all-sky surveys are actually impractical. Answer Choice C is incorrect because our Text would seem to suggest that the authors of the Passage would find deep all-sky surveys to be unjustified, since similar data can be gathered from taking representative surveys of the sky. Answer Choice D is incorrect because it wouldn’t make sense to describe these kinds of surveys as “incomplete,” since, if anything, they’re too complete in their taking such a comprehensive look at the sky when a survey would do just as well.

How to solve this? Question 48 asks, “Based on the passage, the authors would most likely say that conducting deep all-sky surveys to obtain information about ancient galaxies is [...]? Because this is a Paired Question, to solve this we should begin by going through the answer choices for Question 49, looking for any texts that are likely to tell us how the authors would think about conducting deep, all-sky surveys.

50.R.14

Answer Choice A is the correct answer because Lines 62-63 say, “Although CANDELS surveyed a total area only about that of the full Moon, [...],” which describes how the researchers looked at an area of the sky equivalent to the area of the full Moon. This most closely matches Answer Choice A which says that the references gives “readers a sense of the size of the area investigated by CANDELS.” Answer Choice B is incorrect because the Lines don’t tell us that the researchers pointed the telescope at the Moon, which would “identify the direction the HST was directed while collecting data for the HUDF,” but instead describes the area that was surveyed .Answer Choice C is incorrect because the reference to the Moon isn’t used to describe the size of the image processed, but the size of the actual space that was surveyed. Answer Choice D is incorrect because the reference to the Moon isn’t used to compare relative distances.

Page 182: SAT Full Reading Explanations (QAS 9-19) · SAT Full Reading Explanations (QAS 9-19) SAT #9 Reading 1 1.R.9 Answer Choice C is the correct answer because Lines 19-22 say, “Moths

How to solve this? The Question asks, “The reference to the full Moon line 63 primarily serves to [...]?” To solve this, we should go back to the lines cited and determine how the reference functions both in terms of content as well as rhetoric. Lines 62-63 say, [...].

51.R.14

Answer Choice D is the correct answer because the top of the Figure gives the redshift measurement, and the gray shaded area represents star formation rates during the cosmic dawn. Since the graph only shows redshifts between 6-9 for star-formation rates during the cosmic dawn, we can only choose Answer Choice D which says 9.0. Answer Choices A, B and C are incorrect because they all give redshifts below 6.0.

How to solve this? [...].

52.R.14

Answer Choice C is the correct answer because the bottom axis of the graph gives the “Time since big bang” and the left y-axis gives the “Star-formation rate per year). The Sun, which was formed 8.5 billion years after the big bang, corresponds most closely to a star-formation rate of about 4, which most closely matches Answer Choice C.

How to solve this? [...].

Page 183: SAT Full Reading Explanations (QAS 9-19) · SAT Full Reading Explanations (QAS 9-19) SAT #9 Reading 1 1.R.9 Answer Choice C is the correct answer because Lines 19-22 say, “Moths

SAT #15 Reading, Full Test

SAT #15, Reading 1

Passage Outline

[x]

1.R.15

Answer Choice A is the correct answer because the subtitle of the Passage describes the narrator as “Gladys Cailiff” who is “eleven years old in 1938 when a new, well-traveled young schoolteacher, Miss Grace Spivey, turns a small Georgia town upside down.” Moreover, the Passage describes the narrator as being a part of Miss Spivey’s classroom (“She told us all that at school the first day”). This most clearly matches Answer Choice A, which describes the narrator of the passage as “one of Miss Spivey’s former students.” Answer Choice B is incorrect because the Passage is presented from the perspective of a student and not Miss Spivey’s predecessor, who we’re told in Lines 65 is actually Miss Chandler. Answer Choice C is incorrect because the narrator isn’t anonymous, but clearly identified in the subtitle of the Passage. Answer Choice D is incorrect because the Passage isn’t taken from the perspective of Miss Spivey herself.

How to solve this? The Question asks, “The narrator of the passage can best be described as [...]?” To solve this, we should have a general sense of the overall passage and perspective of the narrator presented, and then move down to the answer choices to begin eliminating and selecting answer choices. We should also pay attention to the subtitle of this Passage, which says that “Gladys Cailiff is eleven years old in 1938 [...].” Moreover, we should notice that the Passage is written in first person and seems to take the perspective of someone within Ms. Spivey’s classroom and the Threestep community. [...].

2.R.15

Answer Choice B is the correct answer because, in the subtitle, Threestep is described as “a small Georgia town,” which is consistent with how the rest of the Passage describes Threestep, as in contrast to the Miss Spivey’s cosmopolitanism and suggestively as “the poorest, darkest, most remote and forgotten corner of America.” Answer Choices A, C and D are incorrect because the Passage never describes Threestep as a “summer retreat,” a town that houses a prominent university or as a “comfortable suburb.”

How to solve this? The Question asks, “In the passage, Threestep is mainly presented as a [...]?” To solve this, we should go back to the passage to find a Text or piece of evidence that describes Threestep. Because this is the second question [...].

Page 184: SAT Full Reading Explanations (QAS 9-19) · SAT Full Reading Explanations (QAS 9-19) SAT #9 Reading 1 1.R.9 Answer Choice C is the correct answer because Lines 19-22 say, “Moths

3.R.15 & 4.R.15

Answer Choices D and B are the correct answers because Lines 10-14 say, “I believe her remark irritated some of the people gathered to welcome her on the burned grass alongside the tracks.” The following sentences elaborate on this, saying, “When folks are sweating through their shorts, they don’t like to hear that this is nothing compared to someplace else.” Both of these Lines tell us that Miss Spivey’s remarks annoyed the people of Threestep because she made the heat they experienced insignificant. This most closely matches Answer Choice D which says that the people reacted with “resentment, because they feel that she is minimizing their discomfort.” For Question 4, Answer Choice A is incorrect because it only describes Miss Spivey getting off the train at Threestep and comes before her comment about the Georgia heat. Answer Choice C is incorrect because, although it does mention the people’s reaction to Miss Spivey’s comment, it focuses primarily on their intention to remain positive towards Miss Spivey and not on their immediate reaction. Answer Choice D is incorrect because it describes Miss Spivey’s background, and not the people’s reaction to her comment. For Question 3, Answer Choice A is incorrect because the people don’t react with sympathy or assume that she is experiencing heat like this the first time, since her comment is that she’s experienced worse heat elsewhere. Answer Choice B is incorrect because the people don’t react with disappointment or doubt that Miss Spivey won’t be in Threestep for long. Answer Choice C is incorrect because the Passage describes them as responding with irritance, and not embarrassment.

How to solve this? Question 3 asks, “It can reasonably be inferred from the passage that some of the people at the train station regard Miss Spivey’s comment about the Georgia heat with [...]?” Because this is a Paired Question, to solve this we should begin by going through the answer choices for Question 4, looking for any texts that tell us how the people at the train station reacted to Miss Spivey’s remark.

For Question 4, Answer Choice A says [...].

5.R.15

Answer Choice A is the correct answer because the Passage describes Miss Spivey’s “fruitful intermission” as a period in which she traveled with a medical doctor through the Near East and Africa. Both her actual activities and the description of “fruitful,” tell us that Miss Spivey’s travels during her break were rewarding and worthwhile. This most closely matches Answer Choice A which says that the phrase was used to indicate that Miss Spivey “benefited from taking time off from her studies in order to travel.” Answer Choice B is incorrect because the Passage doesn’t describe Janet Miller as encouraging Miss Spiey to become a doctor, and it’s not clear how the actual phrase “fruitful intermission” could contribute to that. Answer Choice C is incorrect because the phrase doesn’t make a connection between her early years at boarding school and the unanticipated rewards they brought. Answer Choice D is incorrect because the Passage doesn’t suggest that Miss Spivey believed the break would be short or that it lasted several years.

How to solve this? The Question asks, “Miss Spivey uses the phrase “fruitful intermssion” (line 26) to indicate that […].

Page 185: SAT Full Reading Explanations (QAS 9-19) · SAT Full Reading Explanations (QAS 9-19) SAT #9 Reading 1 1.R.9 Answer Choice C is the correct answer because Lines 19-22 say, “Moths

6.R.15

Answer Choice A is the correct answer because in the interaction between Miss Spivey and Ralphord (Lines 33-37), the narrator tells us that “Miss Spivey explained that Barnard, [...] was the sister school of Columbia University, of which, she expected, we all had heard.” This last part of the Text most clearly matches Answer Choice A which says that the interactions suggests, “that Miss Spivey has an exaggerated view of what information should be considered common knowledge.” Answer Choice B is incorrect because the interaction, while not hostile, doesn’t establish a “friendly dynamic” and it’s too strong to describe Miss Spivey in this scene as an “indulgent and doting new instructor.” Answer Choice C is incorrect because Ralphord’s question doesn’t establish him as a precocious young student and it’s too strong and negative to describe Miss Spivey as being dismissive and disinterested. Answer Choice D is incorrect because Ralphord’s question appears genuine and not intended to amuse Miss Spivey.

How to solve this? The Question asks, “The interaction between Miss Spivey and Ralphord serves mainly to [...]?” To solve this, we should go back to the scene where the two interact and decide how it works in terms of content and rhetoric [...].

7.R.15

Answer Choice D is the correct answer because the word “wandered” carries the connotation of indirection and accident, and the Passage uses it in the context of Miss Spivey “trying to find her true mission in life,” and because “marched” carries the connotation of direction and purpose, and the Passage uses it when describing Miss Spivey’s decision to teach. This most closely matches Answer Choice D which says that the words suggest that, “Miss Spivey’s initial encounter with Dewey’s ideas was somewhat accidental but ultimately motivated her to decisive action.” Answer Choice A is incorrect because the word “wandered” is used to describe Miss Spiey before her decision to teach, so it wouldn’t suggest her lack of confidence in her ability to do so. Answer Choice B is incorrect because the Passage describes Miss Spivey actually making her decision to teach over coffee with Dewey, not becoming excited about her previous decision to teach. Answer Choice C is incorrect because the Passage shows Miss Spivey going from indecision to decisiveness over pursuing teaching and going to Teacher’s College to learn, and not being anxious to start teaching.

How to solve this? The Question asks, “In the third paragraph, what is the narrator most likely suggesting by describing Miss Spivey as having ‘wandered’ (line 39) in one situation and ‘marched’ (line 47) in another situation?” To solve this we should back to the Passage where these lines are mentioned and determine how they work in terms of both content and structure. Lines 39 say [...].

Page 186: SAT Full Reading Explanations (QAS 9-19) · SAT Full Reading Explanations (QAS 9-19) SAT #9 Reading 1 1.R.9 Answer Choice C is the correct answer because Lines 19-22 say, “Moths

8.R.15

Answer Choice C is the correct answer because Lines 49-52 say, “Two years later, she told a cheery blue-suited woman from the WPA that she wanted to bring democracy and education to the poorest, darkest, most remote and forgotten corner of America. They sent her to Threestep, Georgia.” These lines most clearly tell us that Miss Spivey ended up in Threestep directly after meeting with this woman from the WPA and being sent there. This most closely matches Answer Choice C which says that Miss Spivey ended up in Threestep as a direct result of “talking with a woman at the WPA.” Answer Choice A is incorrect because Janet Miller was the medical doctor who Miss Spivey travelled with, and isn’t cited as a primary reason for why Miss Spiey ended up in Threestep. Answer Choice B is incorrect because the interaction with the woman from the WPA more directly precedes Miss Spivey leaving for Threestep then her attending college in New York City. Answer Choice D is incorrect because the Passage doesn’t draw any connection between Miss Chandler’s retirement from teaching and her ending up in Threestep.

How to solve this? Question 8 asks, “According to the passage, Miss Spivey ended up in Threestep as a direct result of [...]?” To solve this, we should go back to the Passage to find the Text that most directly precedes Miss Spivey’s going to Threestep. Given the question order rule, we should expect that [...].

9.R.15 & 10.R.15

Answer Choices C and B are the correct answers because Lines 82-83 say, “We all hung there for a minute, thinking hard, until Mavis Davis spoke up.” This directly follows Miss Spivey’s mention of the camel and shows the students’ confused reaction. This most closely matches Answer Choice C for Question 9 which describes the students’ reaction as “baffled.” For Question 10, Answer Choice A is incorrect because it less directly describes the students’ reaction to Miss Spivey and more directly describes Miss Spivey’s anticipation of a response. Answer Choice C is incorrect because it describes one of the student’s explaining what Miss Spivey means, but less directly shows the group of students’ response. Answer Choice D is incorrect because it describes Miss Spivey’s response to Mavis and not the students’ reaction to Miss Spivey’s first comment on “camels.” For Question 9, Answer Choice A is incorrect because the Passage doesn’t describe the student’s as being delighted, but more confused. Answer Choice B is incorrect because the Passage doesn’t suggest that the students are fascinated, but moreso that they don’t understand her. Answer Choice D is incorrect because the PAssage gives no indication that the students worried.

How to solve this? Question 9 asks, “In the passage, when Miss Spivey announces that she had seen camels, the students’ reaction suggests that they are [...].” Because this is a Paired Question, [...].

Page 187: SAT Full Reading Explanations (QAS 9-19) · SAT Full Reading Explanations (QAS 9-19) SAT #9 Reading 1 1.R.9 Answer Choice C is the correct answer because Lines 19-22 say, “Moths

SAT #15, Reading 2

Passage Outline

Traffic Congestion Is Not an Environmental Problem

● building good public transportation can backfire, if it doesn’t replace people driving on roads. it has to be combined with efforts to make driving less easy and pleasurable. these aren’t popular ideas but are necessary. (paragraph one).

● one way to stop people from moving to suburbs and commuting a long way to the city, is traffic congestion. if public transportation makes driving easier, than more people will move farther from the city and commute more. (paragraph two).

● traffic congestion isn’t the problem, but the actual driving. so the goal should be to make roads and highways to encourage people to give up driving for public transportation. (paragraph three).

● it would be environmentally right to make a traffic plan that makes driving and traveling less convenient. (paragraph four).

11.R.15

Answer Choice D is the correct answer because Lines 8-10 say, “That means that a new transit system has to be backed up by something that impels complementary reductions in car use [...],” and “you can’t make people drive less, in the long run, by taking steps that make driving more pleasant, economical, and productive.” These Lines, in the introductory paragraph, tell us that the Passage will primarily focus on the unpopular idea that making driving less enjoyable or easy will ultimately be the way to reduce traffic. This most closely matches Answer Choice D which says that the main purpose of the passage is to “argue that one way to reduce the negative environmental effects of traffic is to make driving less agreeable.” Answer Choice A is incorrect because it doesn’t accurately capture the argument of the Passage, which holds that efforts to reduce traffic could increase traffic but don’t necessarily do so. Answer Choice B is incorrect because, while the Passage does make the point that efforts to build mass transit systems may not be helpful to the environment by themselves, it doesn’t make the claim that this is necessarily true. Answer Choice C is incorrect because, while the Passage does make this claim, it doesn’t take it as the primary focus.

How to solve this? The Question asks, “The main purpose of the passage is to [...]?” Because this is a [Big Picture / Main Purpose] Passage we should solve it by having both a general understanding of the Passage and by going back to read the title and introductory paragraph. Generally, the Passage can be organized like this:

Page 188: SAT Full Reading Explanations (QAS 9-19) · SAT Full Reading Explanations (QAS 9-19) SAT #9 Reading 1 1.R.9 Answer Choice C is the correct answer because Lines 19-22 say, “Moths

Passage Outline

● building good public transportation can backfire, if it doesn’t replace people driving on roads. it has to be combined with efforts to make driving less easy and pleasurable. these aren’t popular ideas but are necessary. (paragraph one).

● one way to stop people from moving to suburbs and commuting a long way to the city, is traffic congestion. if public transportation makes driving easier, than more people will move farther from the city and commute more. (paragraph two).

● traffic congestion isn’t the problem, but the actual driving. so the goal should be to make roads and highways to encourage people to give up driving for public transportation. (paragraph three).

● it would be environmentally right to make a traffic plan that makes driving and traveling less convenient. (paragraph four).

More specifically, the title of the Passage reads, Traffic Congestion Is Not an Environmental Problem, which doesn’t on its own clearly tell us much about the Passage. The introductory paragraph, however, says in Lines 14-17, “But they’re necessary, because you can’t make people drive less, in the long run, by taking steps that make driving more pleasant, economical, and productive.” This keyword, “but,” typically signifies important information and [...].

12.R.15

Answer Choice A is the correct answer because Lines 1-5 say, “Building good transit isn’t a bad idea, but it can actually backfire if the new trains and buses merely clear space on highway lanes for those who would prefer to drive — a group that, historically, has included almost everyone with access to a car.” These Lines most clearly tell us “that the author assumes that, all things being equal, people would rather drive than take mass transit.” Answer Choice B is incorrect because these Lines only explain what qualities makes new transit effective, and not that people would on average prefer to drive. Answer Choice C is incorrect because it less directly makes the case that most people would prefer to drive, but instead gives the what’s necessary to make people drive less in the long run. Answer Choice D is incorrect because it describes how commutes make “suburban sprawl” less common, not about people’s preference for driving.

How to solve this? The Question asks, “Which choice best supports the idea that the author assumes that, all things being equal, people would rather drive than take mass transit?” To solve this [...].

Page 189: SAT Full Reading Explanations (QAS 9-19) · SAT Full Reading Explanations (QAS 9-19) SAT #9 Reading 1 1.R.9 Answer Choice C is the correct answer because Lines 19-22 say, “Moths

13.R.15

Answer Choice A is the correct answer because the phrase “backed up” in lines 8-9 most clearly mean complimented, or “supported,” emphasizing that the system and “something that impels [...] reductions in car use” need to work together to solve traffic problems. Answer Choice B is incorrect because the Lines don’t mean that the transit system has to be copied. Answer Choice C is incorrect because the Lines mean that the two interventions need to work together, not replace or “substitute” one another. Answer Choice D is incorrect because [...].

● That means that a new transit system has to be backed up by something that impels complementary reductions in car use — [...].

● That means that a new transit system has to be supported by something that impels complementary reductions in car use — [...].

14.R.15

Answer Choice B is the correct answer because Lines 13-15 say, “Needless to say, those ideas are not popular.” This most clearly tells us that the author concedes that his recommendations are “not widely supported.” Answer Choice A is incorrect because the author of the Passage never makes the case his recommendations are “costly to implement.” Answer Choice C is incorrect because the author doesn’t mention experts or their opposition to his ideas. And Answer Choice D is incorrect because the author doesn’t draw a distinction between the short term and long term consequences of his recommendations, nor does he make the case that his recommendations are environmentally harmful.

How to solve this? The Question asks, “In the first paragraph, the author concedes that his recommendations are [...]?”

15.R.15 & 16.R.15

Answer Choices C and C are the correct answers because Lines 29-38 say, “If, in a misguided effort to do something of environmental value, municipalities take steps that make long-distance car commuting faster or more convenient — by addinglanes, building bypasses, employing traffic-control measures that make it possible for existing roads to accommodate more cars with fewer delays, replacing tollbooths with radio-based systems that don’t require drivers even to slow down — we actually make the sprawl problem worse [...].” These lines most clearly tell us how “the author most likely characterizes many attempts to improve traffic,” and tell us that the author thinks that municipalities may want to benefit the environment, but are “misguided” and can actually make the sprawl problem and negative environmental impacts worse. This most closely match Answer Choice C for Question 15, which says that these attempts “are well intentioned but ultimately lead to environmental harm.” For Question 16, Answer Choice A is incorrect because it explains what a new transit system has to be combined with but not what the author thinks about attempts to improve traffic. Answer Choice B is incorrect because it only comments on the tolerance for commuting, not about attempts to eliminate traffic. Answer Choice D is incorrect because it less directly mentions different attempts to eliminate traffic and their effect, and only talks about the broad consequences of “moving drivers out of cars and into other forms of transportation.” For Question 15, Answer Choice A is incorrect because it makes

Page 190: SAT Full Reading Explanations (QAS 9-19) · SAT Full Reading Explanations (QAS 9-19) SAT #9 Reading 1 1.R.9 Answer Choice C is the correct answer because Lines 19-22 say, “Moths

too strong and negative a claim, and because the author does think that certain interventions can be effective. Answer Choice B is incorrect because, while the author does mention people’s tolerance for long commutes, he doesn’t make the case that certain interventions overestimate this tolerance, but, if anything, that they underestimate it. Answer Choice D is incorrect because the author believes that interventions will only work if they make driving less convenient and enjoyable, not “more economical and productive.”

How to solve this? Question 15 asks, “Based on the passage, how would the author most likely characterize many attempts to improve traffic?” Because this is a Paired Question, to solve this we should begin by going through the answer choices for Question 16, looking for any texts that tell us what the author thinks about “attempts to improve traffic.”

For Question 16, Answer Choice A say [...].

17.R.15

Answer Choice D is the correct answer because Lines 42-45 say, “If you cut commuting time by 10%, people who now drive fifty miles each way to work can justify moving five miles farther out, because their travel time won’t change.” These Lines most clearly tell us what effects “reducing commuting time for drivers” could have, and most clearly match Answer Choice D which says that, “Drivers become more willing to live farther from their places of employment than they previously were.” Answer Choice A is incorrect because the Passage doesn’t say that reducing commuting time would make drivers more productive. Answer Choice B is incorrect because the Passage makes the case that drivers would be willing to move further from the city, not that mass transit would be extended further. Answer Choice C is incorrect because the Passage doesn’t make any connection between cutting commuting time and the rate at which mass transit carries passengers or receives funding.

How to solve this? The Question asks, “According to the passage, reducing commuting time for drivers can have which of the following effects?” To solve this, we should go back to the Passage and look for a Text that will tell us what the author tells us about reducing commuting time for drivers. [...].

18.R.15

Answer Choice C is the correct answer because the word “promotes” in Line 66 is used to make the point that almost nobody argues or makes the case for a certain kind of transit scheme. This most closely matches “advocates,” which captures this sense of making the case for. Answer Choice A is incorrect because it doesn’t make sense to say that people could “upgrade” or raise in status a transit scheme. Answer Choice B is incorrect because the Line more closely talks about arguing for a transit scheme than serving it. Answer Choice D is incorrect because the Lines don’t mean (developing, etc.).

How to solve this? The Question asks, “As used in line 66, ‘promotes’ most nearly means [...]?”

Page 191: SAT Full Reading Explanations (QAS 9-19) · SAT Full Reading Explanations (QAS 9-19) SAT #9 Reading 1 1.R.9 Answer Choice C is the correct answer because Lines 19-22 say, “Moths

● No one ever promotes a transit scheme by arguing that it would make traveling less convenient — even though, from an environmental perspective, inconvenient travel is a worthy goal.

● No one ever advocates a transit scheme by arguing that it would make traveling less convenient — even though, from an environmental perspective, inconvenient travel is a worthy goal.

19.R.15

Answer Choice B is the correct answer because the first column and second row gives the amount of vehicles per day on a road before it was altered in the Southampton city center, and reads 5,316. Answer Choice A is incorrect because [...].

How to solve this? The Question asks, “According to figure 1, how many vehicles traveled on the altered road through the Southampton city center per day before the route was altered?” To solve this [...].

20.R.15

Answer Choice B is the correct answer because Figure 1 measures the effect of “Route Capacity Reduction,” or altering roads to allow for less traffic, for different regions both before and after the alteration. The final column, which measures “Change in traffic,” shows a significant decline in traffic for all regions, thus supporting the authors claim that reducing traffic capacity or making travel less convenient contributes overall towards having less vehicles on the road. This most closely matches Answer Choice B, which says that the figure supports the argument of the Passage “because the data show that reducing road capacity can lead to a net reduction in traffic.” Answer Choice A is incorrect because the figure doesn’t measure the effect of “moving drivers out of cars” or show that it “induces traffic.” Answer Choice C is incorrect because the figure supports the author’s argument and because the figure doesn’t show any region where road alterations increase traffic. Answer Choice D is incorrect because the figure supports the author’s argument and doesn’t show that reductions in traffic tend to be brief.

How to solve this? The Question asks, “Do the data in figure 1 support or weaken the argument of the author of the passage and why?” To solve this [...].

21.R.15

Answer Choice D is the correct answer because figure 2 measures engineer’s predictions about driving behavior following a significant road space reallocation, and gives answers in terms of yes, yes (in exceptional circumstances), no and don’t know. To find the claim towards which they express the greatest degree of skepticism, we should look for the row with the smallest yes claim (gray shaded box) and the largest no claim (black shaded box). The bottom two behaviors, “their driving style” and “whether they car-share” are the two rows with these features, and because only “their driving style” is given as an answer choice, we should choose it. Answer Choice A is incorrect because [...].

Page 192: SAT Full Reading Explanations (QAS 9-19) · SAT Full Reading Explanations (QAS 9-19) SAT #9 Reading 1 1.R.9 Answer Choice C is the correct answer because Lines 19-22 say, “Moths

How to solve this? The Question asks, “Based on figure 2, the engineers surveyed were most skeptical of the idea that in the event of a reallocation of road space, drivers would change [...]?”

SAT #15, Reading 3

Passage Outline

Pleasant to the Touch

● scientists have known that humans have slow-conducting nerves, but think they only respond to pain and temperature. and their tests supported this. (paragraph one)

● two scientists wanted to know if slow-fibers also responded to pressure. they tested this, and showed that slow fibers do respond in this way. (paragraph two)

● they named these slow fibers “low-threshold” or CT fibers. (paragraph three) ● but why do humans have these slow fibers in certain parts of body? (paragraph four) ● some questions the scientists asked. can people distinguish where the brush touches?

what kind of brush touches them? could it convey pleasure? (paragraph five) ● to test this, they found a woman who only had sensitivity to slow fibers. (paragraph six) ● the subject reported a pleasurable sensation. (paragraph seven) ● MRI studies confirmed their thoughts, and showed that CT fibers specifically convey

pleasurable sensation. (paragraph eight)

22.R.15 & 23.R.15

Answer Choices D and B are the correct answers because Lines 4-7 say, “Sensations of pressure and vibration were believed to travel only along myelinated, fast-signaling nerve fibers, which also give information about location,” which clearly tell us what “textbook authors in the early 1990’s” thought about fast fibers. Based on these lines, we can say that these authors would most likely expect sensations of pressure and vibration to go away if fast fibers were blocked. This most closely matches Answer Choice D, which says that these authors would most likely expect “the ability to perceive vibrations would be impaired.” For Question 23, Answer Choice A is incorrect because these lines talk about slow-conducting nerves and not fast fibers. Answer Choice C is incorrect because these lines talk about “blocking slow fibers” and not about fast fibers. Answer Choice D is incorrect because it [separates from the pack] and because, although it concerns fast-conducting fibers, it doesn’t mention a likely perspective of textbook authors in the 1990’s. For Question 22, Answer Choice A is incorrect because neither the Passage nor our Texts mention that blocking fast fibers would cause other nerve fibers to fire more rapidly. Answer Choice B is incorrect because neither the Passage nor our Text talk about subjects perceiving gentle stimuli as painful if fast fibers were blocked. Answer Choice C is incorrect because the Passage never quite says that slow fibers would compensate for blocked fast fibers by sensing pressure, and because slow fibers ability to detect pressure came from Olausson and his colleagues and textbook authors in the 1990’s.

How to solve this? Question 22 asks, “Based on the passage, textbook authors in the early 1990’s would most likely have expected which condition to result from the blocking of fast fibers?” Because this is a Paired Question, to solve it we should begin by going through the

Page 193: SAT Full Reading Explanations (QAS 9-19) · SAT Full Reading Explanations (QAS 9-19) SAT #9 Reading 1 1.R.9 Answer Choice C is the correct answer because Lines 19-22 say, “Moths

answer choices for Question 23, looking for any texts that talk about how textbook authors in the 1990’s view fast fibers.

24.R.15

Answer Choice A is the correct answer because the word “active” in Line 15 most nearly means “present,” emphasizing their actual existence or presence in humans and mamals. Answer Choice B is incorrect because the lines don’t emphasize the “attentiveness” of the slow fibers, as if they could be attentive. Answer Choice C is incorrect because the Line doesn’t mention the mobility or possibility of “movement” for these slow fibers. Answer Choice D is incorrect because it doesn’t make sense to describe these slow fibers as being “restless.”

● Hakan Olausson and his Gothenburg University colleagues Ake Vallbo and Johan Wessberg wondered if slow fibers responsive to gentle pressure might be active in humans as well as in other mammals.

● Hakan Olausson and his Gothenburg University colleagues Ake Vallbo and Johan Wessberg wondered if slow fibers responsive to gentle pressure might be present in humans as well as in other mammals.

How to solve this? The Question asks, “As used in line 15, “active” most nearly means [...]?”

25.R.15

Answer Choice C is the correct answer because the word “capture” in Line 20 is used to mean “record” or “detect,” emphasizing how the fine filament senses electrical impulses. Answer Choice A is incorrect because the word “capture” isn’t used to emphasize an occupation, as if the filament occupied a nerve’s electrical impulses. Answer Choice B is incorrect because the line isn’t used to mean “seize,” as if [...]. Answer Choice D is incorrect because the lines aren’t saying that the filaments are influencing the nerve’s electrical impulses.

● Using a technique called microneurography, in which a fine filament is inserted into a single nerve to capture its electrical impulses, the scientists were able to measure how quickly — or slowly — the nerves fired.

● Using a technique called microneurography, in which a fine filament is inserted into a single nerve to record its electrical impulses, the scientists were able to measure how quickly — or slowly — the nerves fired.

How to solve this? The Question asks, “As used in line 20, “capture” most nearly means [...]?”

26.R.15 & 27.R.15

Answer Choices C and B are the correct answers because Lines 23-25 say, “They showed that soft stroking prompted two different signals, one immediate and one delayed,” and tell us most directly about the “findings of Olausson’s 1993 experiment.” These Lines tell us that Olausson found that soft touch caused two responses, one from the fast fibers and one from the slow fibers, and best match Answer Choice C which says that Olausson found that, “Gentle pressure is sensed not only by fast fibers but also by slow fibers.” For Question 27, Answer

Page 194: SAT Full Reading Explanations (QAS 9-19) · SAT Full Reading Explanations (QAS 9-19) SAT #9 Reading 1 1.R.9 Answer Choice C is the correct answer because Lines 19-22 say, “Moths

Choice A is incorrect because it only describes the methodology of Olausson’s 1993 experiment and not its findings. Answer Choice C is incorrect because it doesn’t match any of the Answer Choices for Question 26. Answer Choice D is incorrect because it concerns experiments from 1999 and not from 1993. For Question 26, Answer Choice A is incorrect because neither the Passage nor our Texts talk about the relative speeds of stimulation at the body’s extremities and parts closer to the brain. Answer Choice B is incorrect because neither the Passage nor our Texts talk about how the presence of hairs in human skin influences the speed of nerve signals. Answer Choice D is incorrect because the Passage talks about how two different nerve signals are activated by soft stroking, but not that the speed of either nerve fiber vares based on the strength of the applied pressure.

How to solve this? Question 26 asks, “Which conclusion is best supported by the findings of Olausson’s 1993 experiment?” Because this is a Paired Question, to solve this we should begin by going through the answer choices for Question 27, looking for any texts that tell us about Olausson’s 1993 experiment.

● For Question 27, both Answer Choices B and C are possible. Choice B talks more generally about the findings, while Choice C elaborates on them. Either could work, depending on the Choices for 26.

● For Question 26, Answer Choice C is the most general, neutral option and matches Choice B from 27 more closely.

28.R.15

Answer Choice D is the correct answer because Lines 40-42, which read, “But why exactly humans might have such fibers, which respond only to a narrow range of rather subtle stimuli, was initially mystifying,” present us with a question that Olausson and his colleagues wanted to solve. This most closely matches Answer Choice D which says that the lines serve to “show a problem from the perspective of Olausson’s team.” Answer Choice A is incorrect because the Lines only present a question, not a set of factors that Olausson had failed to consider. Answer Choice B is incorrect because the Lines present a question and not a solution to Olausson’s dilemma. Answer Choice C is incorrect because the Lines don’t anticipate any potential criticism.

How to solve this? The Question asks, “The sentence in lines 40-42 (“But … mystifying”) serves mainly to [...]?” To solve this, we should go back to the lines mentioned to identify how they function in the Text in terms of both content and structure. Lines 40-42 read [...].

Page 195: SAT Full Reading Explanations (QAS 9-19) · SAT Full Reading Explanations (QAS 9-19) SAT #9 Reading 1 1.R.9 Answer Choice C is the correct answer because Lines 19-22 say, “Moths

29.R.15

Answer Choice A is the correct answer because Lines 34-35 say, “Then, in 1999, the group looked more closely at the characteristics of the slow fibers,” and Lines 37-39 say that these slow fibers are sensitive to “slow, gentle tactile stimulation, but unresponsiveness to noxious stimuli like pinpricks.” These Lines show us that the 1999 studies intended to look more precisely at the nature of the slow fibers and, in particular, at the types of sensations or signals these slow fibers could process. This most closely matches Answer Choice C which says that the 1999 experiments were meant to determine the “precise nature of sensations that CT fibers can convey.” Answer Choice B is incorrect because the Passage doesn’t mention how body hair relates to CT fiber function, or how the 1999 studies intended to understand this relationship. Answer Choice C is incorrect because the Passage doesn’t specify that the 1999 studies were intended to look specifically at how CT fibers perceive pain, but suggest that they looked more broadly at the different types of sensations the fibers could perceive. Answer Choice D is incorrect because the Passage doesn’t specify how the 1999 studies looked at the effects of microneurography on CT fibers.

How to solve this? The Question asks, “It can reasonably be inferred that one of the intended goals of the 1999 experiment was to determine the [...]?” To solve this Question, we should first go back to where the 1999 experiment is discussed and try to determine what the Passage says about its goals, etc. [...].

30.R.15

Answer Choice D is the correct answer because the sixth paragraph primarily discusses why G.L’s unique condition, being unable to sense things via her fast conducting fibers, contributed to solving Olausson’s group’s question. The first sentence of the paragraph begins with, “To address the question [...].” This most closely matches Answer Choice D which says that the main purpose of the paragraph is to “indicate why G.L.’s medical condition was of value to Olausson’s experiment.” Answer Choice A is incorrect because the sixth paragraph doesn’t mention how Olausson’s group intended to relieve G.L. of some of her symptoms through the experiment. Answer Choice B is incorrect because the sixth paragraph more directly details why G.L.’s condition was useful to Olausson’s experiment and doesn’t directly compare her to other adults. Answer Choice C is incorrect because none of G.L.’s previous procedures are discussed in the paragraph.

How to solve this? The Question asks, “The main purpose of the sixth paragraph (lines 54-63) is to [...]?” To solve this, we should go back to the sixth paragraph and determine [...].

31.R.15

Answer Choice A is the correct answer because Lines 77-80 read, “In normal subjects, both the somatosensory and insular cortices were activated, but only the insular cortex, which processes emotion, was active when researchers brushed G.L.’s arm.” This tells us that in other subjects, two cortices are activated by the soft touch, while for G.L. only one cortex is activated, the insular cortex. This most closely matches Answer Choice A which says that G.L. differs from Olausson’s other test subjects in terms of the “number of cortices activated in the brain during

Page 196: SAT Full Reading Explanations (QAS 9-19) · SAT Full Reading Explanations (QAS 9-19) SAT #9 Reading 1 1.R.9 Answer Choice C is the correct answer because Lines 19-22 say, “Moths

gentle brushing.” Answer Choice B is incorrect because the Passage doesn’t make any remark about differences between the actual physical dimensions of the somatosensory cortex for G.L. and other of Olausson’s test subjects. Answer Choice C is incorrect because the Passage doesn’t mention a difference in the intensity of nerve signals required to first activate the insular cortex. Answer Choice D is incorrect because the Passage doesn’t talk about any effect that MRI scanning has on the function of the brain’s cortices.

How to solve this? The Question asks, “According to the passage, G.L. differed from Olausson’s other test subjects in terms of the [...]?” To solve this, we should go back to the Text to look for a specific sentence that compares G.L. to other of Olausson’s test subjects.

32.R.15

Answer Choice B is the correct answer because Lines 80-83 say, “This solidified the notion that CT fibers convey a more emotional quality of touch, rather than the conscious aspects that helps us describe what we are sensing,” which tell us that “humans experience an emotional aspect of touch when” these CT fibers are activated through soft touch. This most closely matches Answer Choice B which says that humans experience an emotional touch when “CT fibers are exposed to a stimulus.” Answer Choice A is incorrect because the Passage doesn’t make any connection between humans experiencing an emotional touch and brain cortices being shielded from nerve signals. Answer Choice C is incorrect because the Passage doesn’t talk about nerve fibers that sense pain being suppressed and their connection an emotional aspect of touch. Answer Choice D is incorrect because the Passage doesn’t make any connection between conscious aspects of sensation being ignored and humans experiencing an emotional aspect of touch.

How to solve this? The Question asks, “According to the passage, humans experience an emotional aspect of touch when [...]?” To solve this, we should go back to the Passage and look for a specific text that talks about humans experiencing an emotional aspect of touch and under which circumstances they do experience it.

Page 197: SAT Full Reading Explanations (QAS 9-19) · SAT Full Reading Explanations (QAS 9-19) SAT #9 Reading 1 1.R.9 Answer Choice C is the correct answer because Lines 19-22 say, “Moths

SAT #15, Reading 4

Passage Outline

Passage 1 — “March of the Flag”

● “it is a noble land that God has given us.” many wonderful things about this nation — beautiful land, strong people, god-sanctioned history, a history of expansion. (paragraph one)

● how positive it would be when Americans can expand into Hawaii, Puerto Rico and the Philippines, etc. (paragraph two)

Passage 2 — “The Paralyzing Influence of Imperialism”

● if the US thinks its right to take over the Philippines, they ought to say so. but should expect the people to protest. (paragraph one)

● Filipinos do not need encouragement form Americans. they will prefer liberty above everything. if they want to censure Filipino speech in opposition to the US, they should remember Patrick Henry. (paragraph two)

● they should also remember Jefferson, Washington and Lincoln. (paragraph three) ● the principles of the Declaration of Independence are timeless. God himself has made

men love liberty. (paragraph four) ● those who want to impose imperialism should consider its effect on Filipinos and also on

the US. (paragraph five)

33.R.15

Answer Choice C is the correct answer because Lines 1-5 say, “Fellow-Citizens: It is a noble land that God has given us; a land that can feed and clothe the world; a land whose coast lines would enclose half the countries of Europe; [...].” These Lines most clearly tell us what Beveridge thinks about ‘the resources and immensity of the United States,” and the phrase “It is a noble land that God has given us” most closely matches Answer Choice C which says that the United States is a “divine gift to the American people.” Answer Choice A is incorrect because neither the Passage nor our Text talk about how the resources and size of the US serve to safeguard it from foreign invasion. Answer Choice B is incorrect because, although the Passage does call the United States “a greater England with a nobler destiny,” it doesn’t quite say that is replicates the conditions of Europe. Answer Choice D is incorrect because, while plausible under Beveridge’s description of the United States, the Passage never directly states that the U.S. is a source of envy for other countries.

How to solve this? The Question asks, “In Passage 1, Beveridge asserts that the resources and immensity of the United States constitute a [...]?” To solve this, we should go back to the Passage to look for a claim or sentence that Beveridge makes about the resources and size of the U.S. Because this is the first question of the Passage, and because the next question references Lines 24-37, we should expect our Text to come from earlier in the Passage. Lines 1-5 say [...].

Page 198: SAT Full Reading Explanations (QAS 9-19) · SAT Full Reading Explanations (QAS 9-19) SAT #9 Reading 1 1.R.9 Answer Choice C is the correct answer because Lines 19-22 say, “Moths

34.R.15

Answer Choice B is the correct answer because the second paragraph entirely takes the form of a series of commands for the audience to “think” — think of Americans bringing safety justice to Hawaii and Puerto Rico, think of Americans establishing order and equity in the Philippines, etc. All of these scenarios are future-oriented and positive, so most closely match Answer Choice B which says that the commands serve to “anticipate the benefits of a proposed policy.” Answer Choice A is incorrect because the author isn’t directly emphasizing any specific civic responsibility, and isn’t reminding the audience of them. Answer Choice C is incorrect because the paragraph doesn’t give any indication of urgency. Answer Choice D is incorrect because the paragraph doesn’t look at any arguments from an opposition, or offer any refutations.

How to solve this? The Question asks, “In the second paragraph of Passage 1 (lines 24-37), the commands given by Beveridge mainly serve to [...]?” To solve this, we should go back to the second paragraph, identify the commands given, and understand what point they’re used to make. [...].

35.R.15

Answer Choice B is the correct answer because the word “recalled” in Line 65 is used to emphasize a “pulling back of a truth,” or “retraction.” Answer Choice A is incorrect because the Lines aren’t emphasizing that a truth can’t be repeated. Answer Choice C is incorrect because the Lines aren’t saying that you can’t reject a truth once it’s been spoken. Answer Choice D is incorrect because the Line wouldn’t be saying that a truth, once it’s been spoken, can’t be remembered.

● Some one has said that a truth once spoken can never be recalled. ● Some one has said that a truth once spoken can never be retracted.

How to solve this? The Question asks, “As used in Line 65, “recalled” most nearly means [...]?”

36.R.15 & 37.R.15

Answer Choices D and C are the correct answers because Lines 71-73 say, “He never made a race of people so low in the scale of civilization or intelligence that it would welcome a foreign master,” which most clearly tell us how Bryan would consider the preference for sovereignty, or self-government, over foreign rule. These Lines most clearly tell us that this desire for liberty and self-governance is God-given or innate, and most closely matches Answer Choice D which says that the preference is a “manifestation of an innate drive in humans toward self-rule.” For Question 37, Answer Choice A is incorrect because it speaks more particularly about the U.S. and Filipinos and the censure of speech than it does a general preference for sovereignty. Answer Choice B is incorrect because it talks generally about a truth that has been “once spoken” and not about a preference for self-government. Answer Choice D is incorrect because it asks Americans to consider the effect of their policy or argument on their own nation, not about a preference for sovereignty. For Question 36, Answer Choice A is incorrect because neither the Passage nor our Texts discuss how a preference for liberty comes as a reaction to

Page 199: SAT Full Reading Explanations (QAS 9-19) · SAT Full Reading Explanations (QAS 9-19) SAT #9 Reading 1 1.R.9 Answer Choice C is the correct answer because Lines 19-22 say, “Moths

imperialism, but rather presents it as universal and innate. Answer Choice B is incorrect because neither the Passage nor our Texts talk about this preference as a sign of a certain kind of belief. Answer Choice C is incorrect because the Passage, if anything, sets this innate preference at odds with United States foreign policy, and doesn’t claim that it derives from it.

How to solve this? Question 36 asks, “It can reasonably be inferred from Passage 2 that Bryan considers the preference for national sovereignty over foreign rule to be a [...]?” Because this is a Paired Question, to solve it we should begin by going through the answer choices for Question 37, looking for any texts that tell us about what Bryan thinks about a preference for national sovereignty, or self-government, over foreign rule. Answer Choice A says [...].

38.R.15

Answer Choice A is the correct answer because the word “calculate” in Line 76 is used to emphasize that people should consider or “evaluate” the effect of their decision on their own nation, emphasizing a kind of deliberation. Answer Choice B is incorrect because the Lines don’t mean that Americans should “design” the effects their policy would have on their own nation, but that they are natural consequences. Answer Choice C is incorrect because the Passage isn’t saying that Americans should “assume” or accept these effects, but understand or consider them. Answer Choice D is incorrect because the Lines don’t mean that Americans should multiply these effects, but simply keep them in mind as a kind of warning.

● Those who would have this Nation enter upon a career of empire must consider, not only the effect of imperialism on the Filipinos, but they must also calculate its effects upon our own nation.

● Those who would have this Nation enter upon a career of empire must consider, not only the effect of imperialism on the Filipinos, but they must also evaluate its effects upon our own nation.

How to solve this? The Question asks, “As used in line 76, “calculate” most nearly means [...]?”

39.R.15

Answer Choice A is the correct answer because both Passages express admiration for the “founding and history of the United States.” In Lines 12-14 from Passage 1, the author writes “it is a glorious history our God has bestowed upon His chosen people,” and 50-63, the author of Passage 2 references a number of historical figures from the U.S. including Patrick Henry, Jefferson, Washington and Lincoln. Answer Choice B is incorrect because neither Passage directly references or expresses admiration for the diversity of American culture. Answer Choice C is incorrect because both Passages focus more narrowly on independence relative to the United States, and don’t reference a global or worldwide history of struggles. Answer Choice D is incorrect because neither Passage references an idealism that runs throughout American society.

How to solve this? The Question asks, “In developing their respective arguments, Beveridge (Passage 1) and Bryan (Passage 2) both express admiration for the [...]?” To solve this, we should start with the answer choices given and use our memory of the Passage to help us

Page 200: SAT Full Reading Explanations (QAS 9-19) · SAT Full Reading Explanations (QAS 9-19) SAT #9 Reading 1 1.R.9 Answer Choice C is the correct answer because Lines 19-22 say, “Moths

predict which options are most likely. From there, we should try to find evidence for any of the plausible candidates and [negative evidence] for all of the other choices. Answer Choice A for example says [...].

40.R.15

Answer Choice B is the correct answer because Beveridge in Paragraph Two, after having outlined a host of positive qualities American liberty, makes the case that the conquest of other territories and nations will be positive as well, bringing “justice and safety,” “order and equity,” and “energy and industry” — claiming that becoming a part of the U.S. is the “highest honor liberty can bestow.” By contrast, Bryan in Lines 77-79 makes the claim that “we cannot repudiate the principle of self-government in the Philippines without weakening that principle here.” These two Lines present the respective views of both authors and most closely match Answer Choice B, which says that both authors differ in their views of American liberty because “Beveridge considers it so exemplary as to justify conquest of other regions, whereas Bryan warns that its exemplary quality would be undermined by imperial expansion.” Answer Choice A is incorrect because, while Beveridge does make a connection between the U.S. and Europe, he never quite makes the claim that American liberty is a direct inheritance of European colonization, and because Bryan doesn’t make any point about the relationship between American and European liberty. Answer Choice C is incorrect because Passage 1 doesn’t make the case that liberty in the United States has matured, and Passage 2 doesn’t emphasize that it has always existed from the beginning. Answer Choice D is incorrect because Bryan doesn’t believe that liberty is unique to the United States, but that it’s innate in all people.

How to solve this? The Question asks, “Which choice describes a central difference between how Beveridge (Passage 1) and Bryan (Passage 2) view the concept of liberty as it is realized in the United States?” To solve this, we should go through each of the answer choices [...].

41.R.15 & 42.R.15

Answer Choices D and A are the correct answers because Lines 40-43 say, “the Republican party ought to state its position and defend it, but it must expect the subject races to protest against such a policy and to resist to the extent of their ability.” These Lines most cleary match Answer Choice D which says that Bryan would criticize the vision of American governance of island territories by calling it “naive, since the islanders would object to being governed by Americans.” For Question 42, Answer Choice B is incorrect because it talks less directly than Choice A about American occupation of those islands and because no Choices from Question 41 match it. Answer Choice C is incorrect because it also less directly discusses American occupation of territories and doesn’t match any answer choices from Question 41. Answer Choice D is incorrect because it doesn’t directly tell us about American conquest of island territories and because it doesn’t match any Choices from Question 41. For Question 41, Answer Choice A is incorrect because Passage 2 never makes the case that most Americans wouldn’t be willing to relocate to distant islands. Answer Choice B is incorrect because Passage 2 never makes the case that economic domination would be the actual, true goal of American government. Answer Choice C is incorrect because, although it’s probably true that Bryan believes that islanders would insist upon an equal distribution of resources, he never makes that

Page 201: SAT Full Reading Explanations (QAS 9-19) · SAT Full Reading Explanations (QAS 9-19) SAT #9 Reading 1 1.R.9 Answer Choice C is the correct answer because Lines 19-22 say, “Moths

claim directly in the Passage or argues that this would make something Beveridge’s vision impractical.

How to solve this? Question 41 asks, “It can most reasonably be inferred from Passage 2 that Bryan would criticize the vision of American governance of island territories that Beveridge presents in Passage 1 for being [...]?” Because this is a Paired Question, to solve it we should begin by going through the answer choices for Question 42, looking for any texts that tell us about [...].

SAT #15, Reading 5

Passage Outline

“Control of Weeds by Plowing at Night”

● a long history of plowing to control weeds. it works, but often leads to new weed seedlings. (paragraph one).

● there are many weed seeds in soil, but seeds need light to germinate. does plowing allow light to reach these seeds? (paragraph two).

● two ecologists study this. their methodology. they conclude that soil disturbance does stimulate germination. (paragraph three).

● a scientist believes that plowing during the day exposes seeds. if farmers plowed at night, there would be less germination. (paragraph four).

● he was skeptical of this idea, because it was so simple he thought it would have been discovered. (paragraph five).

● he tested this theory on a field. the results were dramatic, and supported his hypothesis. (paragraph six).

● this method of weed control is being used in Germany, and studied in other countries. (paragraph seven).

43.R.15

Answer Choice A is the correct answer because Lines 26-28 say, “They concluded that soil disturbance gives weed seeds a ‘light break,’ and this stimulates their germination,” which most closely tells us that exposure to light allows these seeds to “begin to develop.” Answer Choice B is incorrect because the Passage doesn’t connect exposure to light to the seeds’ ability to absorb necessary nutrients. Answer Choice C is incorrect because the Passage doesn’t make the case that exposure to light allows seeds to withstand temperatures. And Answer Choice D is incorrect because the Passage doesn’t make the case that exposure to light allows seeds to achieve maximum growth.

How to solve this? The Question asks, “According to the passage, exposure to light allows seeds to [...]?” To solve this, because it is an [According to the passage] type question, we should begin by going through the Passage, looking for a Text that will tell us about how seeds react to exposure to light.

Page 202: SAT Full Reading Explanations (QAS 9-19) · SAT Full Reading Explanations (QAS 9-19) SAT #9 Reading 1 1.R.9 Answer Choice C is the correct answer because Lines 19-22 say, “Moths

44.R.15

Answer Choice B is the correct answer because the question in Lines 13-16 reads, “Do the blades of a plow, which can reach more than a foot beneath the soil surface, bring some of these buried seeds to the surface where their germination is induced by exposure to sunlight?” This question suggests a topic of research that ecologists don’t yet understand, and immediately leads into a study that address this question — the next paragraph begins with, “Two ecologists, Jonathan Sauer and Gwendolyn Struik, began to study this question in the 1960s.” This characterization of the question most closely matches Answer Choice B, which says that it serves to “introduce the specific research topic addressed in the passage.” Answer Choice A is incorrect because the question comes before the findings discussed in the passage, and serves to introduce them, not to emphasize their provisional, or temporary nature. Answer Choice C is incorrect because the question doesn’t suggest a possible impact that the studies discussed in the Passage might have. Answer Choice D is incorrect because the question doesn’t concern any research methods and doesn’t indicate any kind of disagreement about them.

How to solve this? The Question asks, “The question in the second paragraph (lines 13-16) primarily serves to [...]?” To solve this, we should go back to the question mentioned in the Passage and try to understand how it works in the Passage in terms of both content and structure.

45.R.15

Answer Choice D is the correct answer because the word “induced” means “started by” or “stimulated,” emphasizing that sunlight begins the process of germination. Answer Choice A is incorrect because it doesn’t make sense to say that exposure to sunlight “lured” a germination. Answer Choice B is incorrect because the Lines don’t mean that exposure to sunlight “established” or made germination, because it suggests that [germination as an establishment?] Answer Choice C is incorrect because it doesn’t make sense to say that the sunlight “convinced” germination to begin, as if germination could be convinced.

● Do the blades of a plow, which can reach more than a foot beneath the soil surface, bring some of these buried seeds to the surface where their germination is induced by exposure to sunlight?

● Do the blades of a plow, which can reach more than a foot beneath the soil surface, bring some of these buried seeds to the surface where their germination is stimulated by exposure to sunlight?

How to solve this? The Question asks, “As used in line 15, “induced” most nearly means [...]?”

Page 203: SAT Full Reading Explanations (QAS 9-19) · SAT Full Reading Explanations (QAS 9-19) SAT #9 Reading 1 1.R.9 Answer Choice C is the correct answer because Lines 19-22 say, “Moths

46.R.15

Answer Choice C is the correct answer because Lines 40-44 say, “Although even under these conditions hundreds of millions of photons strike each square millimeter of ground each second, this illumination is below the threshold needed to stimulate the germination of most seeds,” where “these conditions” refers to fields being plowed at night. These Lines most clearly tell us that “seeds present in the fields plowed at night are exposed to some amount of light.” Answer Choice A is incorrect because this refers to a claim the scientist believes about fields plowed during the day, and not at night. Answer Choice B is incorrect because it only talks about the light exposure that occurs during plowing at daytime. Answer Choice D is incorrect because it only concerns Hartmann’s skepticism of his own idea about nighttime plowing, given that he assumed it would have already been discovered, and not about exposure to light during the nighttime.

How to solve this? The Question asks, “Which choice best supports the idea that seeds present in the fields plowed at night are exposed to some amount of light?” To solve this, we should begin by going through each of the answer choices given, looking for any text that tell us about plowing at night and how weed seeds are still exposed to some light.

47.R.15 and 48.R.15

Answer Choices A and B are the correct answers because Lines 55-57 say, “No crops were planted in these pilot experiments, to avoid possible competition with the emerging weeds.” These Lines suggest that if crops, like wheat or corn, were planted on these strips that there would be competition between them and the emerging weeds, making it likely for there to be less of the strip covered by weeds. This most closely matches Answer Choice A, which says that if these crops were planted, the percentage of the surface of each strip would be “lower than the percentage that Hartmann found.”

For Question 47, Answer Choice B is incorrect because the Passage suggests that if crops were planted alongside the weeds, there would be competition between them, making it more likely that less weeds would grow, not that the strip would contain a higher percentage of them. answer Choice C is incorrect, because it makes a strong claim and because the Passage doesn’t suggest that it would be almost impossible to determine the percentage of weeds which would cover the strips. Answer Choice D is incorrect because the Passage suggests that competition would likely lower the amount of weeds present in the strips, not make them comparable to the amount found in the original projection.

For Question 48, Answer Choice A is incorrect because it only gives an element of the methodology of the experiment, and not something about the effect of growing other crops alongside the weeds. Answer Choice C is incorrect because it only describes the results as “dramatic,” without giving any clear evidence about the likely effect of growing crops on the strips. Answer Choice D is incorrect because it gives the final results of the experiment without crops, but doesn’t suggest how the presence of crops would likely affect the results.

How to solve this? The Question asks, “The passage suggests that if Seydel had planted wheat or corn on the two agricultural strips in Hartmann’s experiment, the percentage of the

Page 204: SAT Full Reading Explanations (QAS 9-19) · SAT Full Reading Explanations (QAS 9-19) SAT #9 Reading 1 1.R.9 Answer Choice C is the correct answer because Lines 19-22 say, “Moths

surface of each strip covered with weeds would likely have been [...]?” Because this is a Paired Question, to solve it we should begin by going through the answer choices for Question 48, looking for any texts that tell us about the likely effect of planting crops alongside the weeds.

49.R.15

Answer Choice C is the correct answer because the word “dramatic” in Line 58, most nearly means significant or “impressive,” emphasizing the strong difference between the 2% and 80% of weeds they found on the strips. Answer Choice A is incorrect because the Lines aren’t emphasizing the actual “theatrical” nature of the results. Answer Choice B is incorrect because the Lines don’t suggest that these results happened suddenly, and it doesn’t form a close synonym with dramatic. Answer Choice D is incorrect because the Lines don’t emphasize the emotional aspect of the findings.

● The results were dramatic. ● The results were impressive.

How to solve this? The Question asks, “As us in Line 58, ‘dramatic’ most nearly means [...]?” To solve this [...]?

50.R.15

Answer Choice A is the correct answer because the fourth column in the figure measures the number of seedlings that emerged in darkness, and the first row, for Sample A, shows 0 emerged seedlings.

51.R.15

Answer Choice C is the correct answer because [...].

52.R.15

Answer Choice D is the correct answer because the Figure measures the difference of emerged seedlings for those disturbed in the light and those disturbed in darkness. It most clearly shows that those disturbed in the night had far fewer emerged seedlings than those disturbed in light, matching Answer Choice D which says, “Thus the germination of weed seeds would be minimized if farmers simply plowed their fields during the night, when the photon fluence rate (the rate at which photons hit the surface) is below 1015 photons per square meter per second.” Answer Choice A is incorrect because it only makes a comment about farmers in the past plowing their fields to control weeds, not relating to the figure in any significant way. Answer Choice B is incorrect because it comments on the number of seeds that can be buried in a square meter of land, and doesn’t relate to the graph. Answer Choice C is incorrect because it talks about how seeds buried beneath an undisturbed surface don’t receive enough light to germinate, and doesn’t relate to the figure.

Page 205: SAT Full Reading Explanations (QAS 9-19) · SAT Full Reading Explanations (QAS 9-19) SAT #9 Reading 1 1.R.9 Answer Choice C is the correct answer because Lines 19-22 say, “Moths

How to solve this? The Question asks, “The data presented in the table most directly support which claim from the passage?” To solve this, we should first identify the main features of the figure, etc.

Page 206: SAT Full Reading Explanations (QAS 9-19) · SAT Full Reading Explanations (QAS 9-19) SAT #9 Reading 1 1.R.9 Answer Choice C is the correct answer because Lines 19-22 say, “Moths

SAT #16, Reading Full Passage

Reading 1

Passage Outline

“Through the Ivory Gate”

- “The novel’s main character, Virginia, has just found her old cello while unpacking after a move.”

● narrator hadn’t played since college, not real music. (paragraph one) ● she started playing at nine, on a tonette. (paragraph two) ● she had worked for months, waiting to choose a different instrument. (paragraph three) ● before she could, the family moved. and the selection of instruments became smaller

and lower quality. (paragraph four). ● she ends up choosing a violoncello, which she learned to appreciate. (paragraph five) ● a specific moment of insight, where she learned to play the violoncello differently, with

more intent. (paragraph six) ● a return to the present after the narrator’s memory. (paragraph seven)

1.R.16

Answer Choice A is the correct answer because Lines 4-6 say, “that wasn’t real music, music that made you forget where you were, made you forget where your arms and legs ended and luscious sound began,” which contrast popular music like the Beatles with more serious, “real music,” with the repetition of “made your forget” characterizing how the music makes you feel. This most closely matches Answer Choice A which says that the lines serve to, “emphasize the qualities Virginia associates with powerful music.” Answer Choice B is incorrect because these Lines refer to music that the narrator sets in contrast to Beatles songs. Answer Choice C is incorrect because the “made you forget” doesn’t refer to an actual loss of memory, especially not about the theater troupe, but refers generally to the way that music makes her feel. Answer Choice D is incorrect because the Lines don’t express any regret, nor regret specifically about Virginia ending her musical studies.

How to solve this? The Question reads, “The repetition of the phrase “made you forget” in Lines 5-6 primarily serves to [...]?” Because this is a [question], to solve it we should [...].

Page 207: SAT Full Reading Explanations (QAS 9-19) · SAT Full Reading Explanations (QAS 9-19) SAT #9 Reading 1 1.R.9 Answer Choice C is the correct answer because Lines 19-22 say, “Moths

2.R.16

Answer Choice B is the correct answer because Lines 19-21 say, “She had chafed through months of scales and simple songs, waiting for the moment when she would walk across the auditorium stage and choose [...].” These Lines most clearly tell us the narrator practiced her instrument diligently, and most closely match Answer Choice B which says that the narrator’s experience with the tonette illustrates “her early interest in and commitment to music.” Answer Choice is incorrect because, while the Passage does make the case that the tonette serves to determine the musical “aptitude” of certain children, it never quite makes the case that the narrator herself had an “extraordinary aptitude for music at a young age.” Answer Choice C is incorrect because the Passage doesn’t indicate that the narrator had any initial fear of failure when she began to learn to play. Answer Choice D is incorrect because the Passage doesn’t suggest that the narrator felt any sense of resentment at the time needed to practice music.

How to solve this? The Question asks, “In the passage, the description of Virginia’s experience with the tonette illustrates which aspect of her relationship with music?” To solve this, we should go back to the Passage and look for any Texts that [...].

3.R.16

Answer Choice C is the correct answer because Lines 35-38 say, “‘That leaves the strings,’ she sighed, leading the way back through the noonday blaze and into the main building, where the violins, violas, cellos and double basses were housed,” where “housed” most nearly means “kept” or “stored.” This most closely matches Answer Choice C which says that the word most clearly matches “stored” as used in Lines 27-28 wich say, “There, the music instruments were stored in a classroom trailer [...].” Answer Choice A is incorrect because the word “covered” in Line 15 is used to refer to how the narrator’s fingers closed off the holes of the tonette, and don’t mean “stored” or “kept.” Answer Choice B is incorrect because the word “moved” in Line 26 refers to the narrator’s family relocating, and doesn’t match the sense of “housed” in Line 38. Answer Choice D is incorrect because the word “opened” in Line 28 refers to the narrator opening a case, and doesn’t match the sense of “stored” or “kept” for the word “housed.”

How to solve this? The Question asks, “As used in Line 38, “housed” is most similar in meaning to which other word as used in the passage?”

Page 208: SAT Full Reading Explanations (QAS 9-19) · SAT Full Reading Explanations (QAS 9-19) SAT #9 Reading 1 1.R.9 Answer Choice C is the correct answer because Lines 19-22 say, “Moths

4.R.16

Answer Choice B is the correct answer because the Lines 27-33 describes Virginia’s negative reaction to the instruments in the classroom trailer, in particular the “glare bouncing off all the polished silver, those gloating caps and hinges” of the flute and the clarinet which looked “like an overdesigned walking stick,” and sounded “like a clown laughing.” This most closely matches Answer Choice B, which says that Virginia is “repelled by the appearance of both instruments and by the sound of the clarinet.” Answer Choice A is incorrect because, while Virginia certainly has a poor opinion of both instruments and does single out the clarinet, she less specifically expresses “skepticism” towards them and focuses on their quality. Answer Choice C is incorrect because Virginia doesn’t particularly focus on the poor condition of the classroom trailers, even though she may suggest so by contrasting them to the auditorium stage in Akron. Answer Choice D is incorrect because the scene regarding the clarinet and flute don’t illustrate Virginia’s frustration with playing the instruments. How to solve this? The Question asks, “Based on the passage, which choice best describes Virginia’s reaction to the flute and clarinet in the classroom trailer?” To solve this [...].

5.R.16

Answer Choice A is the correct answer because Lines 38-43 say, “There, by virtue of its sonorous name, Virginia asked for the violoncello — and was too intimidated by the teacher’s growing impatience to protests when what emerged from the back closets was something resembling not a guitar, but a childsized android.” This tells us that Virginia chose the violoncello because she felt pressured by the teacher and was afraid to ask to look at another instrument, and most clearly matches Answer Choice A which says that she accepted the instrument because “she is reluctant to request an alternative.” Answer Choice B is incorrect because the Passage doesn’t indicate the violoncello is the last remaining instrument in the trailer, and suggests that there may be more but that Virginia is reluctant to ask for them. Answer Choice C is incorrect because the Passage doesn’t describe the instrument as a tiger until later in the Passage, long after Virginia had already chosen the instrument. Answer Choice D is incorrect because Virginia doesn’t come to appreciate the instrument’s sound until Lines 46-48, after she had already chosen the instrument.

How to solve this? The Question asks, “According to the passage, Virginia allows herself to be assigned the violoncello because [...]?” To solve this [...].

Page 209: SAT Full Reading Explanations (QAS 9-19) · SAT Full Reading Explanations (QAS 9-19) SAT #9 Reading 1 1.R.9 Answer Choice C is the correct answer because Lines 19-22 say, “Moths

6.R.16 & 7.R.16

Answer Choices A and D are the correct answers because Lines 44-48 say, “It was not long, however, before she realized that she had made a good choice, for the sound of its name was synonymous with the throbbing complaint that poured out of its cumbersome body.” These Lines draw a connection between the sound of the name of the instrument and the sound that the instrument produces, and most clearly matches Answer Choice A which says that Virginia perceives a relationship in an instrument between “What it is called and how it sounds.” For Question 7, Answer Choice A is incorrect because Lines 19-25 discuss Virginia’s anticipation to choose an instrument, but don’t detail any relationship betweens aspects of an instrument. Answer Choice B is incorrect because it only describes Virginia’s reaction to the appearance of the clarinet in the classroom trailer, and doesn’t discuss any relationship between different aspects of an instrument. Answer Choice C is incorrect because it only describes a narrative scene where the music teacher begins to show Virginia the strings. For Question 6, Answer Choice B is incorrect because neither the Texts nor the Passage discuss a relationship between playing and maintaining an instrument. Answer Choice C is incorrect because the Passage doesn’t show Virginia making any connection between how an instrument looks and how popular it is. Answer Choice D is incorrect because the Passage never discusses any connection between the availability of an instrument and its ease of mastery.

How to solve this? Question 6 asks, “In the passage, the narrator suggests that Virginia perceives a relationship between which aspects of a musical instrument?” Because this is a Paired Question, to solve it we should begin by going through the answer choices for Question 7, looking for any texts that tell us about any relationship between aspects of an instrument.

8.R.16 & 9.R.16

Answer Choices C and D are the correct answers because Lines 66-68 say, “She had to love and not be scared, and show the cat that it did not need to growl to protect itself,” which comes in the context of the narrator seeing “for the first time” the shape of the cello like a tiger, and recognizing the need to draw it in close and tame it. These Lines indicate a scene where Virginia recognizes a need to change her attitude towards the cello, and shows her going from being “scared” to “love.” This most clearly matches Answer Choice C from Question 8, which describes her changing her attitude from “apprehension to calm affection.” For Question 9, Answer Choice A is incorrect because these Lines only indicate how long it took for Virginia to learn how to hold the cello, but don’t suggest any significant change in her attitude towards it. Answer Choice B is incorrect because, while it sets up the scene where Virginia changes her attitude, it only describes her setting the physical scene and doesn’t detail any significant change. Answer Choice C is incorrect because, while it does describe Virginia changing her physical posture while holding the instrument, it less clearly defines a change in her attitude. For Question 8, Answer Choice A is incorrect because the Text describes Virginia’s change in attitude from fear to love, which less clearly matches “uncertainty to firm commitment,” even if that transition captures the positive to negative change we want. Answer Choice B is incorrect because “dissatisfaction to reluctant acceptance” doesn’t capture the change of “fear to love” that we want, especially the latter “reluctant acceptance part.” Answer Choice D is incorrect because the Passage doesn’t describe Virginia as frustrated prior to her change in attitude.

Page 210: SAT Full Reading Explanations (QAS 9-19) · SAT Full Reading Explanations (QAS 9-19) SAT #9 Reading 1 1.R.9 Answer Choice C is the correct answer because Lines 19-22 say, “Moths

How to solve this? Question 8 asks, “In the sixth paragraph (lines 49-71), the narrator suggests that Virginia recognizes a need to change her attitude toward the cello from one of [...]?” Because this is a Paired Question, to solve it we should begin by going through the answer choices for Question 9, looking for any texts that tell us about Virginia changing her attitude towards the cello.

10.R.16

Answer Choice D is the correct answer because the italicized Lines 72-73 say, “Funny how fantasy works. And memory. I haven’t thought about that evening in years,” and mark the shift in the Passage from Virginia’s memory of the past to the present moment. This most clearly matches Answer Choice D which says that the lines “indicate a shift in time and perspective.” Answer Choice A is incorrect because the Lines don’t suggest that any of the memories are doubtful in accuracy. Answer Choice B is incorrect because the Passage doesn’t shift toward the point of view of a new character, but to Virginia herself, later in time. Answer Choice C is incorrect because the Passage doesn’t make a contrast between real or imagined events or indicate that any of the events presented in the Passage were imaginary.

How to solve this? The Question asks, “In the context of the passage as a whole, the italicized sentence in Lines 72-73 mainly serve to [...]?” To solve this [...].

Reading 2

Passage Outline

● many studies, including one involving multiple sclerosis patients, show that helping others increases well being. (paragraph one)

● an American and New Zealand survey both confirm this. (paragraph two) ● helping others may even prevent early death, as one study showed lower mortality rates

for people who volunteered. (paragraph three) ● a psychologist wanted to test these theories in a real world context, and designed an

experiment to do so. (paragraph four) ● she found that the amount of time between acts affected the happiness. altruistic acts

may need to be frequent to add benefit. (paragraph five) ● overall though, helping others and being generous can boost your mood and health.

(paragraph six)

Page 211: SAT Full Reading Explanations (QAS 9-19) · SAT Full Reading Explanations (QAS 9-19) SAT #9 Reading 1 1.R.9 Answer Choice C is the correct answer because Lines 19-22 say, “Moths

11.R.16

Answer Choice C is the correct answer because Lines 8-11 say, “After tracking the groups for three years, Schwartz found that the helpers — the people in the phone-call group — reported profound improvements in their self-worth and their moods.” These Lines most clearly tell us that those patients who offered emotional support reported improvements in their overall well-being, and most closely matches Answer Choice C which says that, “Givers of emotional support reported increased well-being.” Answer Choice A is incorrect because the Passage doesn’t indicate that both those who gave and received emotional support reported increased well-being, but only those who gave it. Answer Choice B is incorrect because the Passage doesn’t tell us either way how the recipients of emotional support responded, so we can’t assume that they showed no increased well-being. Answer Choice D is incorrect because the Passage doesn’t indicate that both givers and recipients reported initial well-being and because it doesn’t indicate that the well-being reported by givers was only temporary, and that they returned to their previous condition.

How to solve this? The Question asks, “Based on the passage, which choice best describes the relationship between emotional support and well-being as shown by Schwartz’s study?” Because this is a [based on the passage] type question, we can expect to find a specific sentence that will answer our question. Given the order of this question, and using the question order rule, we should expect our Text to come early in the Passage, and should look for a sentence that describes the relationship between emotional support and well-being as shown by Schwartz’s study. Lines [...].

12.R.16

Answer Choice D is the correct answer, because the word “positive” is used in Line 16 to emphasize the altruistic or “beneficial” nature of the support given. Answer Choice A is incorrect because the sentence isn’t highlighting the “confidence” of the impact on the world, but how it helps others. Answer Choice B is incorrect because the Lines aren’t emphasizing the practical as opposed to more abstract or theoretical nature of the impact. Answer Choice C is incorrect because the Lines aren’t emphasizing the “specificity” of the impact.

● They talked about how helping other people transformed their experience of multiple sclerosis from something that victimized them to something that enabled them to be a positive force in the world.

● They talked about how helping other people transformed their experience of multiple sclerosis from something that victimized them to something that enabled them to be a beneficial force in the world.

How to solve this? The Question asks, “As used in line 16, ‘positive’ most nearly means [...]?”

Page 212: SAT Full Reading Explanations (QAS 9-19) · SAT Full Reading Explanations (QAS 9-19) SAT #9 Reading 1 1.R.9 Answer Choice C is the correct answer because Lines 19-22 say, “Moths

13.R.16

Answer Choice A is the correct answer because the word “associations” in Line 42 most closely means correlations or “links,” emphasizing the connectedness between helping and happiness. Answer Choice A is incorrect because the Line isn’t emphasizing a degree of structure or organization that exists between helping and happiness. Answer Choice C is incorrect because the Lines aren’t emphasizing the “combinations” between helping and happiness, but the connectedness between them. Answer Choice D is incorrect because it ascribes too much [humanity] to the terms, as if they could partner together like people.

● While survey studies have found more or less strong associations between helping and happiness, the University of California, Riverside, psychologist Sonja Lyubomirsky wanted to test the connection in a real-world setting.

● While survey studies have found more or less strong links between helping and happiness, the University of California, Riverside, psychologist Sonja Lyubomirsky wanted to test the connection in a real-world setting.

How to solve this? The Question asks, “As used in line 42, ‘associations’ most nearly means [...]?”

14.R.16

Answer Choice D is the correct answer because Lines 57-59 say that “Lyubormirsky’s work suggests altruistic acts may need to be frequent in order to confer a lasting change in well-being.” This tells us that the author interprets her study as showing that only frequent good deeds cause long lasting happiness, and that infrequent altruistic acts are likely to bring any significant, lasting change, so any survey that showed otherwise would undermine this interpretation. This most closely matches Answer Choice D which says that the interpretation would be undermined by a study that showed that “occasional altruistic acts result in long-lasting increases in the personal happiness of those who perform them.” Answer Choice A is incorrect because this would confirm the author’s interpretation of the study, since she believes that frequent acts of kindness should result in long lasting happiness. Answer Choice B is incorrect because Lyubormirsky’s survey and the author’s interpretation of it don’t make any claims about the connection between happiness and the immediacy of benefits. Answer Choice C is incorrect because Lyubormirsky’s survey and the author’s interpretation of it don’t make any claims about the connection between happiness and the amount of efforts required by those acts.

How to solve this? The Question asks, “If true, which finding of a survey of the general population would most undermine the author’s interpretation of Lyubomirsky’s study?” To solve this, we should go back to the Passage to first determine what the author’s interpretation of the study is, and then determine which survey would contradict or undermine its central claim. Lines [...].

Page 213: SAT Full Reading Explanations (QAS 9-19) · SAT Full Reading Explanations (QAS 9-19) SAT #9 Reading 1 1.R.9 Answer Choice C is the correct answer because Lines 19-22 say, “Moths

15.R.16

Answer Choice B is the correct answer because the quote from the scientist in Lines 61-64 says, “it could be that there’s a very short — narrowly defined in time and space — bump in happiness that doesn’t shift your [overall] happiness in any meaningful way.” In context, this quote most clearly explains Lyubomirsky’s findings, offering a reason for them. Answer Choice A is incorrect because the Lines don’t present a hypothesis, and especially not one that Lyubomirsky’s study was meant to test. Answer Choice C is incorrect because the quote isn’t critical of Lyubomirsky, and doesn’t advocate for her to consider the benefits of short-term happiness. Answer Choice D is incorrect because the quotes don’t make any comparison between Lyubomirsky’s study and other studies about kind acts.

How to solve this? The Question asks, “In lines 61-64, the author includes the quotation from Borgonovi most likely to [...]?” To solve this [...].

16.R.16 & 17.R.16

Answer Choices A and C are the correct answers because Lines 65-67 say, “On balance, though, being generous boosts your mood and health because it strengthens your sense that you’re really doing something significant.” These Lines most clearly tell us a reason why people who perform altruistic acts benefit from them, and most closely matches Answer Choice A which says that people benefit because of the “beliefs that they hold about the effects of such acts,” namely that they find them to be beneficial. For Question 17, Answer Choice A is incorrect because it comes early in the Passage and because it only introduces a study, without giving any evidence for why people benefit from performing acts of altruism. Answer Choice B is incorrect because it only talks about what kinds of altruistic acts confer well-being, but doesn’t explain why they produce well-being in people. Answer Choice D is incorrect because it doesn’t match with any of the answer choices for Question 16. For Question 16, Answer Choice B is incorrect because neither the Passage nor our Texts indicate that people benefit from altruistic acts because of the feedback they receive from others. Answer Choice C is incorrect because, while one of the Texts does talk about brain chemistry, the benefits of altruism aren’t said to be afforded merely through thinking about such acts. Answer Choice D is incorrect because neither the Passage nor our Texts talk about how people benefit from altruistic acts because of social approval.

How to solve this? Question 16 asks, “The author most strongly suggests that people who perform altruistic acts benefit partly because of [...]?” Because this is a Paired Question, to solve it we should [...].

Page 214: SAT Full Reading Explanations (QAS 9-19) · SAT Full Reading Explanations (QAS 9-19) SAT #9 Reading 1 1.R.9 Answer Choice C is the correct answer because Lines 19-22 say, “Moths

18.R.16

Answer Choice C is the correct answer because Figure 1 shows the percent of respondents agreeing about certain survey statements, and the highest percentage of agreement was for the statement that “Volunteering enriches my sense of purpose in life.” This most closely matches Answer Choice C which says that the highest percentage of respondents agreed that volunteering has “made them feel a greater sense of purpose.” Answer Choice A is incorrect [...].

How to solve this? The Question asks, “According to figure 1, the highest percentage of respondents agreed that volunteering has [...]?”

19.R.16 & 20.R.16

Answer Choices B and D are the correct answers because Lines 71-74 say, “The boost we get from helping may also mute our stress response, causing us to release fewer jarring stress hormones such as cortisol and norepinephrine.” Because those surveyed in Figure 1 were those who had volunteered in the previous year, it makes sense that these findings about reductions in certain stress hormones would also apply to them, matching Answer Choice B which says that those surveyed “may have experienced decreases in the level of certain hormones after volunteering.” For Question 20, Answer Choice A is incorrect because, although it does reference the specific survey mentioned in Figure 1, it could only support the claim volunteering improved well-being, lowered stress levels and enhanced a sense of purpose, which doesn’t match any of the answer choices for Question 19. Answer Choice B is incorrect because it only discusses a second study of the Maori in New Zealand, which doesn’t relate to the participants in Figure 1. Answer Choice C is incorrect because it concerns a study about mortality rates and doesn’t offer any information about those presented in Figure 1. For Question 19, Answer Choice A is incorrect because, while plausible, neither the Passage nor the Texts make the claim that people value volunteering because they have a high degree of empathy. Answer Choice C is incorrect because the Passage indicates that mortality rates are likely to be lower for those who do volunteer, not for those who answer yes to the specific questions given in the survey. Answer Choice D is incorrect because it makes too strong a claim which, although perhaps true in general, isn’t true in every case.

How to solve this? Question 19 asks, “Based on information in the passage, it can reasonably be inferred that the majority of survey respondents represented in figure 1 [...]?” Because this is a Paired Question, to solve this we should begin by going through the answer choices for Question 20, looking for any texts that might [...].

Page 215: SAT Full Reading Explanations (QAS 9-19) · SAT Full Reading Explanations (QAS 9-19) SAT #9 Reading 1 1.R.9 Answer Choice C is the correct answer because Lines 19-22 say, “Moths

21.R.16

Answer Choice B is the correct answer because Figure 1 provides the specific percentages for given for the survey mentioned in Paragraph 2, while the second Figure gives details for the study given and discussed in Paragraphs 4 and 5, so that the figures support and offer specific details on two of the studies mentioned in the Passage. This most closely matches Answer Choice B which says that “both figures provide the specific results of studies discussed in the passage.” Answer Choice A is incorrect because the two figures describe the studies presented in the Passage and thus support its primary claim, not challenge it. Answer Choice C is incorrect because neither Figure 1 nor Figure 2 refer to the first study mentioned in the Passage in Paragraph 1. Answer Choice D is incorrect because the author doesn’t make the case that either study depicted by the Figures requires further study or evaluation.

How to solve this? The Question asks, “Which choice best states the relationship between the two figures and the passage?” To solve this [...].

Reading 3

Passage Outline

The Bionic Leaf

● two scientists have created a “bionic leaf,” which can convert solar energy into liquid fuel. they hope that it can be used in remote areas of the developing world. (paragraph one)

● the way that the artificial leaf works. it is continually being developed and improved. (paragraph two)

● the hydrogen it produces, however, is in the form of gas and not liquid, which are easier to use. this is why the bionic leaf is important. (paragraph three)

● in the bionic leaf, bacteria help convert the hydrogen gas to a liquid fuel. (paragraph four)

● combining an inorganic catalyst with biology, provides more new ways to join and build chemicals than either method alone. (paragraph five)

● scientists have continually worked on improving the bionic leaf. (paragraph six) ● the ultimate goal is not only to create fuel, but to create “high value commodities” like

drugs or vitamins. (paragraph seven)

Page 216: SAT Full Reading Explanations (QAS 9-19) · SAT Full Reading Explanations (QAS 9-19) SAT #9 Reading 1 1.R.9 Answer Choice C is the correct answer because Lines 19-22 say, “Moths

22.R.16

Answer Choice A is the correct answer because the Passage primarily concerns the history and development of the bionic leaf, as well as its overall importance. The Passage is titled, “The Bionic Leaf,” and Lines 1-2 begin the Passage by saying, “Harvard scientists have created a ‘bionic leaf’ that converts solar energy into a liquid fuel.” Taking all these together, we should choose Answer Choice A which says that the purpose of the passage is to “discuss the development and significance of the bionic leaf.” Answer Choice B is incorrect because the Passage doesn’t focus on or even discuss “current commercial uses of the bionic leaf,” but only suggests how it may be important in the future for serving rural, undeveloped areas. Answer Choice C is incorrect because the Passage doesn’t show any debate surrounding the bionic leaf and its effectiveness, but instead tracks its development and mechanics. Answer Choice D is incorrect because, while the Passage does discuss the differences between the artificial leaf and the bionic leaf, it doesn’t take this difference as a primary focus but instead discusses both to show the history and development of the bionic leaf.

How to solve this? The Question asks, “The primary purpose of the passage is to [...]?” Because this is a [big picture type question], asking about the primary purpose of the passage, we should begin by reading the subtitle for the Passage as well as the introductory paragraph, looking for any clear theses or topical sentences. The title [...].

23.R.16 & 24.R.16

Answer Choices D and C are the correct answers because Lines 7-11 say, “The pair, who began collaborating two years ago, share an interest in developing energy sources that might someday have practical application in remote locales in the developing word.” These Lines most clearly tell us what problem motivated the two scientists’ research and matches Answer Choice D, which says that they wanted to address the problem that “some communities lack adequate access to reliable energy sources.” For Question 24, Answer Choice A is incorrect because it only explains the fact that two scientists have created a bionic leaf, without explaining what motivated their research. Answer Choice B is incorrect because it only explains that the bionic leaf serves as a proof of concept for a new field and resulted from the collaboration of the two scientists from different backgrounds, without discussing a problem that initially motivated their research. Answer Choice D is incorrect because it explains the choice of the word “bionic,” but doesn’t suggest an underlying motivation for the scientists’ work. For Question 23, Answer Choice A is incorrect because, while the Passage does state the need for reliable fuel sources in developing parts of the world, it never explicitly makes the case that these countries lack the necessary natural resources, only that they lack access. Answer Choice B is incorrect because neither the Passage nor the Text make the case that a fact about liquid fuel motivated the scientists’ research, and because the Passage seems to suggest that creating liquid fuel was more difficult than creating gas fuels. Answer Choice C is incorrect because neither the Passage nor our Texts cite a difficulty of solar energy as the motivation for their research.

How to solve this? Question 23 asks, “The first paragraph implies that Silver and Noceara’s research was motivated in part by a desire to address which problem?” Because this is a Paired Question, to solve it we should begin by going through the answer choices for Question 24, looking for any texts that tell us about these two scientists’ initial motivation.

Page 217: SAT Full Reading Explanations (QAS 9-19) · SAT Full Reading Explanations (QAS 9-19) SAT #9 Reading 1 1.R.9 Answer Choice C is the correct answer because Lines 19-22 say, “Moths

25.R.16

Answer Choice B is the correct answer because the second paragraph mostly talks about how the bionic leaf works and some of its features. Lines 16-19, the opening sentence, says, “Nocera’s artificial leaf, which serves as the fuel source in the bionic leaf, works by sandwiching a photovoltaic cell between two thin metal oxide catalysts,” and Lines 19-25 continue to explain how the leaf works, with two sentences at the end explaining its efficiency. This most closely matches Answer Choice B, which says that the purpose of the second paragraph is to “explain the workings of a central component of the bionic leaf.” Answer Choice A is incorrect because the artificial leaf described in the second paragraph isn’t a competitor to the bionic leaf, but one of its parts. Answer Choice C is incorrect because the paragraph doesn’t take a primary focus on photosynthesis, because the artificial leaf only “mimics” photosynthesis and doesn’t actually perform it, and because it doesn’t mention how photosynthesis played a role specifically in the “development” of the bionic leaf. Answer Choice D is incorrect because, while the Passage does look at the efficiency of the artificial leaf, it only compares it to that of naturally occurring photosynthesis and not to that of the bionic leaf. How to solve this? The Question asks, “The main purpose of the second paragraph (lines 16-31) is to [...]?” To solve this, we should begin by going back to the second paragraph to look for [...].

26.R.16

Answer Choice A is the correct answer because Lines 21-25 say, “Current from the silicon solar wafer is fed to the catalysts, which split water molecules: oxygen bubbles off the catalyst on one side of the wafer, while hydrogen rises from the catalyst on the wafer’s other side.” This tells us that the artificial leaf splits water into two parts, and most closely matches Answer Choice A which says that the leaf “splits water into hydrogen and oxygen.” Answer Choices B, C and D are all incorrect because the only chemical process which the Passage describes is that of separating hydrogen and oxygen molecules.

How to solve this? The Question asks, “The passage indicates that the artificial leaf carries out which chemical process?” Because this is a [passage indicates] type question, we can expect that a single sentence will contain our answer to the question. Using the Question Order Rule [...].

Page 218: SAT Full Reading Explanations (QAS 9-19) · SAT Full Reading Explanations (QAS 9-19) SAT #9 Reading 1 1.R.9 Answer Choice C is the correct answer because Lines 19-22 say, “Moths

27.R.16

Answer Choice C is the correct answer because the word “captures” in Line 28 is used to indicate that plants can make “use” of a certain percentage of the sunlight’s energy. Answer Choice A is incorrect because it’s unclear what it would mean for the plants to “record” the sunlight’s energy. Answer Choice B is incorrect because it’s also unclear what it would mean for the plants to “describe” the sun’s energy, as if they could speak or write about it. Answer Choice D is incorrect because thej Lines don’t mean that the plants are conquering the sunlight’s energy, and is also unclear in meaning.

● Today, it is far more efficient than a field-grown plant, which captures only 1 percent of sunlight’s energy.

● Today, it is far more efficient than a field-grown plant, which uses only 1 percent of sunlight’s energy.

How to solve this? The Question asks, “As used in line 42, “trick” most nearly means [...]?” To solve this, we should begin by going back to Line 42 and look for how the word “trick” is used in context.

28.R.16

Answer Choice A is the correct answer because the word “trick” in Line 42 is used to describe the real process or “clever technique” that the scientists used to produce liquid fuel. Answer Choice B is incorrect because the Lines don’t mean that the scientists pulled a prank of bioengineering. Answer Choice C is incorrect because “trick” refers to a real process or technique, not an illusion. Answer Choice D is incorrect because there’s no deliberate deception mentioned in these Lines.

● The bacteria combine the hydrogen with carbon dioxide as they divide to make more cells, and then — through a trick of bioengineering pioneered by Anthony Sinskey [...] — produce isopropanol [...].

● The bacteria combine the hydrogen with carbon dioxide as they divide to make more cells, and then — through a clever technique of bioengineering pioneered by Anthony Sinskey [...] — produce isopropanol [...].

How to solve this? The Question asks, “As used in line 42, ‘trick’ most nearly means [...]?”

Page 219: SAT Full Reading Explanations (QAS 9-19) · SAT Full Reading Explanations (QAS 9-19) SAT #9 Reading 1 1.R.9 Answer Choice C is the correct answer because Lines 19-22 say, “Moths

29.R.16

Answer Choice D is the correct answer because the quotes by Colon in the fifth paragraph, make the case and support the claim that the bionic leaf is important because it combines inorganic catalysts with biology and produces an “unprecedented platform for chemical synthesis that you don’t have with inorganic catalysts alone.” He continues to remark on how effective biology can be at making chemical modifications and suggests future uses for this methodology of integration. This most closely matches Answer Choice D, which says that Colon’s remarks serve to “emphasize the innovative nature and great potential of the bionic leaf.” Answer Choice A is incorrect because Colon doesn’t focus specifically on the design or level of sophistication of the technology behind the bionic leaf, but instead looks at the innovation it employs and potential uses for it in other domains. Answer Choice B is incorrect because Colon doesn’t highlight any of the collaboration that led to the production and development of the leaf. Answer Choice C is incorrect because Colon also doesn’t suggest anything about the testing of the bionic leaf or its continuous improvements.

How to solve this? The Question asks, “Colon’s remarks in the fifth paragraph (lines 48-60) primarily serve to [...]?” Because this is a [serves to] type question, to solve it we should begin by [...].

30.R.16 & 31.R.16

Answer Choices B and D are the correct answers because Lines 56-60 say, “‘If you can use enzymes for building chemicals, you open the door to making many of the natural compounds we rely on every day,’ such as antibiotics, pesticides, herbicides, fertilizer, and pharmaceuticals.” The use of the qualifier “if,” tells us that the process of using enzymes to build other chemicals is still a hypothetical and not fully verified. This most closely matches Answer Choice B from Question 30, which says that the researchers make an assumption about the bionic leaf that it “can be used to produce chemical compounds other than isopropanol.” For Question 31, Answer Choice A is incorrect because it only explains the bacteria’s role in the bionic leaf, and doesn’t suggest an assumption that the researchers make. Answer Choice B is incorrect because it only explains the advantage of merging inorganic catalysts with biology, but doesn’t suggest any assumption that the researchers make. Answer Choice C is incorrect because it only explains why using biology in chemical synthesis is so important, but doesn’t offer an assumption that the researchers make. For Question 30, Answer Choice A is incorrect because the Passage already indicates that the bionic leaf has reached the efficiency of plant photosynthesis, and because none of the Texts reference the efficiency of the bionic leaf in comparison to photosynthesis. Answer Choice C is incorrect because neither the Texts nor the Passage suggest that the researchers will attempt to replace the artificial catalysts with natural ones, and actually highlight the importance of using both kinds of catalysts together. Answer Choice D is incorrect because the Passage already indicates that the liquid fuel produced by the bionic leaf can be used similarly to ethanol, and because none of the Texts make a point about the leaf’s generation of fuel.

How to solve this? Question 30 asks, “As presented in the passage, the researchers make which assumptions about the bionic leaf that has yet to be substantiated?” Because this is a

Page 220: SAT Full Reading Explanations (QAS 9-19) · SAT Full Reading Explanations (QAS 9-19) SAT #9 Reading 1 1.R.9 Answer Choice C is the correct answer because Lines 19-22 say, “Moths

Paired Question, to solve it we should begin by going through the answer choices for Question 31, looking for any texts that tell us about a possible assumption that the researcher makes.

Reading 4

Passage Outline

- “At the the time of these speeches, South Africa was in the process of transitioning from a British colony to an independent republic under a system of white-minority rule known as apartheid.

Passage 1

- “adapted from Albert Luthuli’s speech to the South African Congress of Democrats, delivered in 1958. [...] Luthuli was the president of the African National Congress, a group advocating equality for black South Africans [...].”

● those involved in freedom struggle have one “gospel” or message: democracy and freedom. (paragraph one)

● this must be the vision for South Africa. it is still under apartheid, which is the antithesis of democracy. (paragraph two)

● there are many justifications for this diversion from true freedom, including the claim that Africans should develop culture and society on their own. (paragraph three)

● to be able to develop, you need freedom and self determination. (paragraph four and five)

● the vision of democracy in South Africa is not just a dream, but realizable. because every person wants freedom, and not slavery. (paragraph six)

Passage 2

- “adapted from Harold Macmillan’s address to the South African Parliament, delivered in 1960. [...] Macmillan, the prime minister of Britain, was addressing the all-white South African Parliament.”

● change is happening in Africa, and politics must respond to it. (paragraph one) ● the audience should understand this, because they come from Europe, the home of

nationalism. a national consciousness is found in the achievements of Western civilization. (paragraph two)

● every person must do right in their area of responsibility. we British have learned what we think is right through experience, and have tried to raise the material quality of life and moral culture of every area they have taken responsibility for. (paragraph three)

● in countries with more than one race, the British have tried to find ways to bring the community together. (paragraph four)

Page 221: SAT Full Reading Explanations (QAS 9-19) · SAT Full Reading Explanations (QAS 9-19) SAT #9 Reading 1 1.R.9 Answer Choice C is the correct answer because Lines 19-22 say, “Moths

32.R.16

Answer Choice A is the correct answer because Lines 7-11 say, “It cannot in honesty be claimed that she is yet really democratic, when only about a third of her people enjoy democratic rights, and the rest — notwithstanding the fact that they constitute the majority — are still subjected to apartheid rule.” These Lines most clearly tell us that Luthuli believes that South Africa can’t be fully democratic until black South Aricans, the “majority,” receive the same rights as others. This most closely matches Answer Choice A, which says that South Africa will only become fully democratic when black South Africans “enjoy the same rights as white citizens.” Answer Choice B is incorrect because Luthuli doesn’t make the complaint that black South Africans enjoy political power, but lack economic power. Answer Choice C is incorrect because, while Luthuli does later talk about the necessity of self-determination, he more clearly connects the idea of the fullness of democracy to all citizens enjoying democratic rights. Answer Choice D is incorrect because black South Africans already do “constitute a majority of the government,” as Luthuli says in Lines 10-11.

How to solve this? The Question asks, “In Passage 1, Luthuli argues that South Africa will become a fully democratic country only when black South Africans [...]?” Because this is a [In passage, passage indicates, etc.] type Question, we should expect to find a single sentence to support our answer choice. Using the Question Order Rule, we should also expect to find this sentence towards the beginning of the Passage, probably before Line 17. Lines 7-11 say [...].

33.R.16

Answer Choice B is the correct answer because Lines 17-18 say that these “very nice and pretty phrases” are often used “to justify this diversion from the democratic road.” This suggests that the phrases hide or dismiss the fact that apartheid goes against true democracy, and most closely matches Answer Choice B, which says that the language is used in order to “obscure an indefensible growing system.” Answer Choice A is incorrect because the phrases aren’t trying to actually “rectify” or fix the situation, but rather to excuse it. Answer Choice C is incorrect because, while they may serve to ultimately undermine critics, the phrases are not addressed specifically to these critics but are used to justify and excuse a situation. Answer Choice D is incorrect because the Lines don’t “depict the daily experience of the majority of citizens.”

How to solve this? The Question asks, “Luthuli refers to ‘very nice and pretty phrases’ (line 17) primarily to show that language is being used in order to [...]?” To solve this, we should go back to the Line in question and determine its meaning in context. 34.R.16

Answer Choice C is the correct answer because Lines 35-37 say, “One might ask, ‘Is this vision of a democratic society in South Africa a realizable vision?. Or is it merely a mirage?’” Here we see Luthuli contrasting a “realizable vision” with something that is “merely a mirage,” emphasizing the reality of attaining or reaching that vision. This most closely matches Answer Choice C, which says that “realizable” means that the vision can be “achieved.” Answer Choice A is incorrect because “acquired” suggests that the vision would be taken from someplace else, or “acquired,” not develop within South Africa and achieved. Answer Choice B is incorrect because Luthuli doesn’t make the point that his vision of South Africa can be “comprehended,”

Page 222: SAT Full Reading Explanations (QAS 9-19) · SAT Full Reading Explanations (QAS 9-19) SAT #9 Reading 1 1.R.9 Answer Choice C is the correct answer because Lines 19-22 say, “Moths

but that we can actually reach it. Answer Choice D is incorrect because “realizable” is used to mean the actual accomplishment of creating this democratic vision, not merely its pursuit.

How to solve this? The Question asks, “When Luthuli describes the vision of a democratic society in South Africa as “realizable” (lines 36-37), he means that this vision can be [...]?”

35.R.16 & 36.R.16

Answer Choices C and A are the correct answers, because Lines 45-49 say, “The wind of change is blowing through this continent and whether we like it or not, this growth of national consciousness is a political fact. And we must all accept it as a fact, and our national policies must take account of it.” These Lines most clearly tell us about what Macmillan thinks about “the growth of national consciousness in Africa” and that he thinks it is an undeniable fact, and one which politicians have to recognize. This most closely matches Answer Choice C, which says that Macmillan thinks the growth of national consciousness is “inevitable, because nationalism in Africa is a force that cannot be stopped.” For Question 36, Answer Choice B is incorrect because it only makes the point that South Africans will be recognized as the first of the African nationalists, and doesn’t concern the new national consciousness. Answer Choice C is incorrect because, while it does mention national consciousness, this answer choice makes the case that the Western World is responsible for the African national consciousness, and doesn’t match an Answer Choice for Question 35. Answer Choice D is incorrect because it only concerns what the British feel is their responsibility towards areas they occupy, not a new national consciousness. For Question 35, Answer Choice A is incorrect because neither the Passage nor our Texts make the case that the growth of a national consciousness in South Africa is “baffling” and specifically because other African nations don’t need one. Answer Choice B is incorrect because neither the Passage nor our Texts make the case that this growth is invigorating or that the consciousness involves a willingness to embrace diversity. Answer Choice D is incorrect because neither the Passages nor the Text make the case that Europeans “doubted that nationalism would take hold in Africa.”

How to solve this? Question 35 asks, “In Passage 2, Macmillan implies that the growth of national consciousness in Africa is [...]?” Because this is a Paired Question, to solve it we should begin by going through the answer choices for Question 36, looking for any texts that tell us what Macmillan thinks about this growth. Lines 45-49 say [...].

37.R.16 & 38.R.16

Answer Choices A and D are the correct answers because Lines 65-68 say “Our judgment of right and wrong and of justice is rooted in the same soil as yours — in Christianity and in the rule of law as the basis of a free society.” These Lines most clearly match Answer Choice A, which says that Macmillan presents his argument by “asserting that Britain and South Africa share certain important values.” For Question 37, Answer Choice B is incorrect because neither the Passage nor the Texts make the case that Britain should take a role in leadership in the African nations, but seems to suggest that these countries will assume a new kind of national sovereignty. Answer Choice C is incorrect because neither the Passage nor our Texts make the case that “South Africa faces greater challenges than does Britain.” Answer Choice D is incorrect because neither the Passage nor our Texts make the point that Britain has had trouble

Page 223: SAT Full Reading Explanations (QAS 9-19) · SAT Full Reading Explanations (QAS 9-19) SAT #9 Reading 1 1.R.9 Answer Choice C is the correct answer because Lines 19-22 say, “Moths

sustaining free and just societies, but seems to actually suggest the opposite. For Question 38, Answer Choice A is incorrect because it talks less clearly about shared values between the South Africans and the British, but only makes the point that the British should understand the new African nationalism. Answer Choice B is incorrect because it only makes the case about responsibility and doing what one thinks is right, while not addressing the point of shared values. Answer Choice C is incorrect because it only explains how the British determine what is right, but doesn’t mention any shared values.

How to solve this? Question 37 asks, “In Passage 2, Macmillan presents his argument to the South African government by [...]?” Because this is a Paired Question [...].

39.R.16

Answer Choice B is the correct answer because in Lines 68-78, Macmillan makes the case that it is important to create in a society “the opportunity of an increasing share in political power and responsibility.” Based on Luthuli’s overall argument, he would seem to agree with this in sentiment, while also making the case that until apartheid is ended and more than “a third of her people enjoy democratic rights,” this vision will not be realized. This most closely matches Answer Choice B, which says that Luthuli would most likely respond to Macmillan’s demand by arguing that “such a society is impossible as long as apartheid exists in South Africa.” Answer Choice A is incorrect because Luthuli never makes a distinction between economic and political power in the Passage, and especially doesn’t make the case that economic power is more important to black South Africans. Answer Choice C is incorrect because Luthuli doesn’t make the case about black South African’s unwillingness to participate in the political system, and on the basis of corruption. Answer Choice D is incorrect because Luthuli wouldn’t make the argument that black South Africans have already achieved the vision expressed by Macmillan nor does he make the case that they already have significant political responsibilities.

How to solve this? The Question asks, “Luthuli would most likely respond to Macmillan’s demand for a society in which all individuals have a ‘share in political power and responsibility’ (lines 75-76) by arguing that [...]?” To solve this, we should go back to the Lines cited and [...].

40.R.16

Answer Choice A is the correct answer because Lines 29-31 say that, “the essence of development along your own lines is that you must have the right to develop, and the right to determine how to develop,” and Lines 68-72 say, “This experience of our own explains why it has been our aim in the countries for which we have borne responsibility, not only to raise the material standards of life, but to create a society that respects the rights of individuals, a society in which men are given the opportunity to grow to their full stature [...].” These Lines, and others, most clearly tell us that both authors believe that a just society must be one which allows individuals to be free and develop as they will, and most closely matches Answer Choice A which says that both authors would agree that “just societies give people the freedom to develop as individuals.” Answer Choice B is incorrect because, while the sentiment of both Passages roughly matches the sentiment of the answer choice, it makes too strong a claim that isn’t directly supported by both Passages. Answer Choice C is incorrect because neither

Page 224: SAT Full Reading Explanations (QAS 9-19) · SAT Full Reading Explanations (QAS 9-19) SAT #9 Reading 1 1.R.9 Answer Choice C is the correct answer because Lines 19-22 say, “Moths

Passage makes distinctions between political, economic and social freedom or talks about the proper order of their development. Answer Choice D is incorrect because only Passage 2 discusses nationalism.

How to solve this? The Question asks, “Luthuli and Macmillan would most likely agree on which statement about freedom [...]?” To solve this, we should [...].

41.R.16

Answer Choice B is the correct answer because Luthuli uses the collective pronouns “us” and “we” throughout the essay, referring to South Africans, and because he places the weight of change at the individual level, discussing “those of who are in the freedom struggle” and the fact that “the germ of freedom is in every individual.” Meanwhile, Macmillan addresses South African leaders and emphasizes Britain’s own leadership role in the region, while also making the case that “whether we like it or not, this growth of national consciousness is a political fact. And we must all accept it as a fact, and our national policies must take account of it.” These most clearly tell us that Luthuli focuses more at the level of the individual citizen, while Macmillan focuses more at the level of politicians, which most closely matches Answer Choice B which says that the two authors differ in their approach to social change in that “Luthuli implies that the people of South Africa themselves will initiate social change, while Macmillan emphasizes the role played by those in positions of power.” Answer Choice A is incorrect because, while Macmillian does make the case that “significant change is imminent,” Luthuli nowhere suggests that “major social change in South Africa is unlikely to happen soon,” and indeed implies that its a current issue. Answer Choice C is incorrect because only Passage 1 mentions apartheid, because Luthuli, although he does say it is necessary, doesn’t make the specific case that eliminating apartheid is only the first step, and because Macmillan doesn’t discusss apartheid as the ultimate goal. Answer Choice D is incorrect because Luthuli doesn’t make the case that change will occur through collective action, but instead references the individual, and because Macmillan doesn’t advocate for the necessity of change at the individual level, but more directly addresses leaders and politicians.

How to solve this? The Question asks, “The speeches of Luthuli (Passage 1) and Macmillan (Passage 2) differ in their approach to social change in that [...]?” To solve this [...].

Page 225: SAT Full Reading Explanations (QAS 9-19) · SAT Full Reading Explanations (QAS 9-19) SAT #9 Reading 1 1.R.9 Answer Choice C is the correct answer because Lines 19-22 say, “Moths

Reading 5

Passage Outline

The Story of Earth: The First 4.5 Billion Years, from Stardust to Living Planet

● according to conventional wisdom, the moon is very dry. and different evidence shows this. (paragraph one)

● however, different observations began to suggest that there is water on the moon. now it is an established fact. (paragraph two)

● scientist Erik Hauri uses an ion microprobe to look for water in the glass beads from the moon. scientists had looked before, but not with this ion microprobe. it showed evidence for a large amount of water.(paragraph three)

● it suggests that a lot of water may be locked inside the moon, and because the moon came from the Earth, it’s likely that our interior also contains water. (paragraph four)

42.R.16

Answer Choice D is the correct answer because Lines 13-15 say, “Eventually someone will challenge what everyone else knows to be true, and once in a while something really interesting will be found.” This claim most closely matches Answer Choice D which says that challenging the conventional wisdom “sometimes leads to significant new insights.” Answer Choice A is incorrect because [neither] the Passage [nor our Text] tells us that “unexpected outcomes” are the results of challenging conventional wisdom, but instead that it may lead to “something really interesting.” Answer Choice B is incorrect because the author doesn’t make this point and actually suggests that the challenge to conventional wisdom comes within the domain of the scientific circle. Answer Choice C is incorrect because the Passage doesn’t make any point about the likelihood of a challenge to conventional wisdom results in a technological innovation or not.

How to solve this? The Question asks, “According to the author, challenging the conventional wisdom [...]?” To solve this [...].

43.R.16

Answer Choice C is the correct answer because Lines 15-19 say, “In 1994 a single flyby of the Clementine spacecraft mission produced radar measurements that were consistent with water ice, though many planetary scientists were unconvinced.” These Lines most clearly tell us that this mission was the first to find suggestive evidence that the moon contained water, and most closely matches Answer Choice C, which says that the mission “offered preliminary indications of water on the moon.” Answer Choice A is incorrect because the Passage doesn’t make the point that evidence from this mission was included in the three Science magazine articles on the moon, and suggests that only later, more contemporaneous evidence was included in the 2009 issue. Answer Choice B is incorrect because the Passage doesn’t tell us anything about the original design of the 1994 study, and whether it was looking for water or not. Answer Choice D is incorrect because the Passage also doesn’t say anything about the radar technology used in that mission.

Page 226: SAT Full Reading Explanations (QAS 9-19) · SAT Full Reading Explanations (QAS 9-19) SAT #9 Reading 1 1.R.9 Answer Choice C is the correct answer because Lines 19-22 say, “Moths

How to solve this? The Question asks, “According to the passage, which choice is true about the 1994 Clementine spacecraft mission?” To solve this [...].

44.R.16 & 45.R.16

Answer Choices C and D are the correct answers because Lines 23-25 say, “Still, many experts pointed to implanted hydrogen ions from the Sun’s solar wind as a more likely source of the signal.” These Lines tell us a reason why scientists continued to believe in the aridity, or dryness, of the Moon, since they could rationalize the signal as caused by something other than water on the Moon. This most closely matches Answer Choice C, which says that the idea of the Moon’s aridity was reinforced because “evidence that might have contradicted this notion could be explained in another way.” For Question 45, Answer Choice A is incorrect because, while it does strongly suggest a reason why scientists believed in the Moon’s aridity, it doesn’t match any answer choice from Question 44. Answer Choice B is incorrect because it only makes general claims about conventional scientific wisdom, and doesn’t provide a reason why scientists continued to believe in the Moon’s aridity. Answer Choice C is incorrect because it concerns the first piece of evidence that suggested the existence of water on the Moon, not of its aridity. For Question 44, Answer Choice A is incorrect because, while this might be a true point made by the Passage overall, it doesn’t match any of the answer choices from Question 45. Answer Choice B is incorrect because it doesn’t match any of the answer choices from Question 45 and because scientists’ willingness to challenge conventional wisdom would point to disconfirming the aridity of the Moon, not reinforcing it. Answer Choice D is incorrect because none of the answer choices from Question 45 concern the Apollo Moon rocks, and because the first paragraph indicates that they were already available for scientists.

How to solve this? Question 44 asks, “It can reasonably be inferred from the passage that the idea that the Moon was completely arid was reinforced in part because [...]?” Because this is a Paired Question, to solve it we should begin by going through the answer choices for Question 45, looking for any texts that tell us why the idea of the Moon’s aridity was reinforced. Lines [...].

46.R.16 (correct this one)

Answer Choice A is the correct answer because the word “resolve” in Line 44 is used to describe a strong detection capacity, or ability to find water. Only Answer Choice A captures this sense of discernment or discrimination. Answer Choice B is incorrect because the Lines don’t suggest that the ion microphobe had the ability to “change into” or transform. Answer Choice c is incorrect because the Lines don’t mean “convert to.” Answer Choice D is incorrect because the word “resolve” isn’t set to mean “clear from,” etc.

● Other scientists had examined the glass beads for signs of water decades earlier, but their detection capacities were no match for the ion microphobe’s ability to resolve measurements [.///

● Other scientists had examined the glass beads for signs of water decades earlier, but their detection capacities were no match for the ion microphobe’s ability to distinguish between measurements [.///

Page 227: SAT Full Reading Explanations (QAS 9-19) · SAT Full Reading Explanations (QAS 9-19) SAT #9 Reading 1 1.R.9 Answer Choice C is the correct answer because Lines 19-22 say, “Moths

How to solve this? The Question asks, “As used in line 44, ‘resolve’ most nearly means [...]?”

47.R.16

Answer Choice B is the correct answer because the word “drive” in Line 59 is used to suggest that the water moved or “fueled” the volcanism from billions of years ago. Answer Choice A is incorrect because it uses a word too closely associated with human interaction. Answer Choice C is incorrect because the word “drive” doesn’t mean to “transport” or carry away. Answer Choice D is incorrect because the word “drive” isn’t used to mean “maneuver,” [as if the water were moving through “volcanism” etc.]

● [...] — a lot of water, comparable to many volcanic rocks on Earth, and more than enough to drive surface volcanism that would have dispersed magma in explosive eruptions billions of years ago.

● [...] — a lot of water, comparable to many volcanic rocks on Earth, and more than enough to drive surface volcanism that would have dispersed magma in explosive eruptions billions of years ago.

How to solve this? The Question asks, “As used in line 59, ‘drive’ most nearly means [...]?” To solve this [...].

48.R.16 & 49.R.16

Answer Choices A and D are the correct answers because Lines 65-69 say, “And since the Moon formed primarily by the wholesale excavation of Earth’s primordial mantle during a collision with another massive object, our planets deep interior likely holds prodigious amounts of unseen water as well.” These Lines most clearly tell us that the water that currently resides on the Moon originally came from Earth, and most closely matches Answer Choice A which says that water currently on the Moon “had its primary source on Earth.” For Question 49, Answer Choice A is incorrect because it only explains how Erik Hauri used an ion microphobe to look at the glass beads from the Moon, but doesn’t suggest anything about the water that currently resides there. Answer Choice B is incorrect because it talks primarily about the amount of water that the glass beads suggest used to reside on the Moon, not about the water that exists there currently. Answer Choice C is incorrect because it only makes the point that the Moon’s water is trapped in its frozen interior, but doesn’t match any of the answer choices for Question 48. For Question 49, Answer Choice B is incorrect because the Passage doesn’t suggest that the Moon’s water resides mainly in glass beads, but suggests that much of it lies inside its frozen interior. Answer Choice C is incorrect because neither the Passage nor our Texts make any point about the water increasing in volume. Answer Choice D is incorrect because the Passage suggests that the water on the Moon currently resides only in ice form, not also in liquid form.

How to solve this? Question 48 asks, “The author implies that any water currently present on the Moon [...]?” Because this is a Paired Question, to solve it we should [...].

Page 228: SAT Full Reading Explanations (QAS 9-19) · SAT Full Reading Explanations (QAS 9-19) SAT #9 Reading 1 1.R.9 Answer Choice C is the correct answer because Lines 19-22 say, “Moths

50.R.16

Answer Choice D is the correct answer because the Figure measures water concentration in parts per million on the y-axis, and distance from the core in micrometers on the x-axis. Following a straight line across from a water concentration of 15 parts per million, we should find that it intercepts the line at around 100-120 micrometers. This most closely matches Answer Choice D, which says 100 micrometers.

How to solve this? The Question asks, “According to the figure, at what distance from the core is the water concentration within lunar glass bead green #5 approximately 15 parts per million?”

51.R.16

Answer Choice C is the correct answer because at the core of the bead the water concentration is at about 30 parts per million, while at 120 micrometers from the core the water concentration is at about 15 parts per million. This means that the water concentration is about half that at the core at 120 micrometers, which most closely matches Answer Choice C which says of the glass bead that “its water concentration at 120 micrometers is approximately half that at its core.” Answer Choice A is incorrect because the figure does show water detectable beyond 100 micrometers from the core, at both 110 and 120. Answer Choice B is incorrect because it makes too strong a claim for the figure to measure, and because these values may have fluctuated over time. Answer choice D is incorrect because the figure doesn’t show us how the glass bead varied in its water concentration over time, so we can’t say how much it has declined.

How to solve this? The Question asks, “Based on data in the figure, which choice is a reasonable conclusion about lunar glass bead green #5?” To solve this [...].

52.R.16

Answer Choice D is the correct answer because the figure shows the relationship between the water concentration of the bead and the distance from the core, showing the overall trend that the core of the bead has a relatively high degree of water concentration while the outer rim of the bead has much less. This most closely matches Lines 48-51, which say “The beads’ outer rims proved to be very dry, with only a few parts per million water, but the cores of the largest beads have as much as [46 parts per million].” Answer Choice A is incorrect because it only cites the multiple sources of evidence for the claim about the Moon’s aridity, a fact disproven by the glass beads like that in the Figure. Answer Choice B is incorrect because it only concerns a specific piece of evidence again meant to confirm the belief that the Moon contains no water, a fact disproven by the Figure. Answer Choice C is incorrect because it concerns another mission that demonstrated the possible existence of water on the Moon, but doesn’t concern the glass beads in the Figure.

How to solve this? The Question asks, “The figure best supports which claim from the passage [...]?” To solve this [...].

Page 229: SAT Full Reading Explanations (QAS 9-19) · SAT Full Reading Explanations (QAS 9-19) SAT #9 Reading 1 1.R.9 Answer Choice C is the correct answer because Lines 19-22 say, “Moths

SAT #17, Reading Full

Reading 1

Passage Outline

● arrival of the narrator at Celia’s apartment. (Lines 1-8) ● description of Celia’s home. (Lines 9-22) ● description of Celia. (Lines 23-34) ● Celia’s discussion of things to wear, services provided, gossip about neighbors. (Lines

35-54) ● Celia’s discussion of her own two sons. (Lines 55-73) ● the narrator’s discussion of her daughter. (Lines 74-90) ● the feeling of connection the narrator has for Celia. (Lines 91-98)

1.R.17

Answer Choice A is the correct answer because Line 9 says of Celia’s apartment, “I loved how full her home felt.” The paragraph continues to describe many of the features and items in the apartment, all communicating a sense of warmth and hominess. This most closely matches Answer Choice A, which says the description serves to “evoke a sense of coziness and comfort.” Answer Choice B is incorrect because the description doesn’t concern Celia’s overall building and doesn’t communicate any sense of being hectic or unpredictable, a fairly strong claim. Answer Choice C is incorrect because the Passage doesn’t make the point that the items in Celia’s home suggest the availability of goods in the United States. Answer Choice D is incorrect because the Passage doesn’t make the case that the eclecticism and fullness of Celia’s apartment suggests that she is an avid collector, a point that would have to be made more explicitly.

How to solve this? The Question asks, “The description of Celia’s apartment in lines 9-20 is primarily intended to [...]?” To solve this [...].

2.R.17

Answer Choice D is the correct answer because Lines 46-48 say, “She said that Quisqueya dyed her hair, which was hardly news — I had assumed as much when I met her.” This most clearly tells us something the narrator knew about Celia’s neighbors before she visited the apartment, and most closely matches Answer Choice D, which says she knew that Quisqueya had dyed her hair. Answer Choices A, B and C are all incorrect because the Passage doesn’t suggest that the narrator knew anything about each of these facts prior to coming to Celia’s apartment.

How to solve this? The Question asks, “According to the passage, which fact about Celia’s neighbors does the narrator know before she visits Celia’s apartment [...]?” To solve this…

Page 230: SAT Full Reading Explanations (QAS 9-19) · SAT Full Reading Explanations (QAS 9-19) SAT #9 Reading 1 1.R.9 Answer Choice C is the correct answer because Lines 19-22 say, “Moths

3.R.17

Answer Choice A is incorrect because Lines 51-54 say, “Quisqueya is a busybody, but it’s only because she’s so insecure. She doesn’t know how to connect with people. Don’t let her put you off.” These Lines contain the phrase in question, “put you off,” and seems to suggest that Quisqueya may inappropriately try to get into the narrator’s business or interact oddly with her, and most closely matches Answer Choice A which describes the phrase as meaning that the narrator shouldn’t “be offended by Quisqueya.” Answer Choice B is incorrect because the description of Quisqueya doesn’t suggest her capacity for deception. Answer Choice C is incorrect because the phrase “don’t let her put you off” doesn’t mean that the narrator should let Quisqueya avoid her, which seems unlikely given that Quisqueya is a busybody. Answer Choice D might seem plausible, because a busybody is likely to “impose” on someone else, but it makes a stronger, more specific claim than Choice A without the requisite information to back it up.

How to solve this? The Question asks, “When Celia tells the narrator not to let Quisqueya ‘put you off’ (line 54), she most nearly means that the narrator should not [...]?”

4.R.17 & 5.R.17

Answer Choices C and B are the correct answers because Lines 59-62 say, “‘I have two boys,’ she said. ‘Both of them were born there. Enrique, my oldest, is away at college on a soccer scholarship. And there’s Mayor, who you met.’” This last point about Mayor most clearly tells us about the only relationship the narrator had with Celia’s sons, and most closely matches Answer Choice C which says that, “The narrator has seen Mayor in person, but she has seen Enrique only in Celia’s photo.” For Question 5, Answer Choice A is incorrect because it only shows the narrator asking a question about Celia’s son, and doesn’t give any evidence of their relationship. Answer Choice C is incorrect because it also doesn’t suggest anything about the narrator’s relationship to Celia’s son, but only describes the origin of the photo that Celia shows. Answer Choice D is incorrect because it only shows Celia asking a question of the narrator, and doesn’t depict the relationship the narrator has with the sons. For Question 4, Answer Choice A is incorrect because neither the Passage nor the Texts suggest that Celia’s sons are friends with the narrator’s daughter. Answer Choice B is incorrect because neither the Passage nor the Texts suggest that the narrator’s daughter goes to school with Mayor. Answer Choice D is incorrect because neither the Passage nor the Texts suggest that the narrator has seen either son play soccer.

How to solve this? Question 4 asks, “Which choice best describes the narrator’s relationship with Celia’s sons?” Because this is a Paired Question, to solve it we should [...].

Page 231: SAT Full Reading Explanations (QAS 9-19) · SAT Full Reading Explanations (QAS 9-19) SAT #9 Reading 1 1.R.9 Answer Choice C is the correct answer because Lines 19-22 say, “Moths

6.R.17

Answer Choice C is the correct answer because Lines 62-64 say, “‘He’s nothing at all like his brother. Rafa thinks we might have taken the wrong baby home from the hospital.’ She forced a smile. ‘Just a joke of course.’” Because Celia mentions this joke just after making the claim that Mayor is nothing like his brother, it seems most plausible to suggest that the joke serves to emphasize this difference, and most closely matches Answer Choice C which says that the joke serves to “stress how different her sons are.” Answer Choices A and B are incorrect because the description of Rafa’s joke seems more directly given to emphasize the difference between the sons. Answer Choice D is incorrect because there’s no tense situation which Celia could be trying to defuse.

How to solve this? The Question asks, “Celia recounts Rafa’s joke (lines 61-64) primarily in order to [...]?”

7.R.17 & 8.R.17

Answer Choices B and A are the correct answers because Lines 79-80 say, “‘And she’s going …’ Celia trailed off, as though she didn’t want to say it out loud.” Given the context of the exchange, this last part, “as though she didn’t want to say it ou loud” most clearly tells us that Celia already knows where the narrator’s daughter goes to school, but just doesn’t want to say. This, in turn, most clearly matches Answer Choice B from Question 7 which says that Celia already knows the answer to the question, “Where does the narrator’s daughter go to school?” For Question 8, Answer Choice B is incorrect because it only describes Celia’s and the narrator’s reaction to the mention of the school the narrator’s daughter attends, but less directly tells us that Celia knows in advance about the narrator’s family. Answer Choice C is incorrect because it shows Celia describing her own family’s experience, but doesn’t suggest anything that she knows about the narrator’s family. Answer Choice D is incorrect because it also doesn’t suggest anything that Celia knows about the narrator’s family, but only asks a general question about the relationship between parents and children. For Question 7, Answer Choice A is incorrect because no Text indicates that Celia knows about how many children the narrator has. Answer Choice C is incorrect because none of the Texts suggest that Celia know’s the narrator’s profession. Answer Choice D is incorrect because there’s also no evidence that Celia knows how long the narrator and her family have lived in the United States.

How to solve this? Question 7 asks, “Based on the passage, it is most reasonable to infer that Celia knows the answer to which question about the narrator’s family before the narrator visits her apartment?” Because this is a Paired Question, to solve it we should [...].

Page 232: SAT Full Reading Explanations (QAS 9-19) · SAT Full Reading Explanations (QAS 9-19) SAT #9 Reading 1 1.R.9 Answer Choice C is the correct answer because Lines 19-22 say, “Moths

9.R.17

Answer Choice D is the correct answer because Celia’s mention of her family’s arrival from Panama comes directly after the narrator says, “I hope so. It’s why we came.” After a brief moment of reflection, Celia responds with her own story, which seems intended to demonstrate empathy and understanding for the narrator. This most closely matches the more general Answer Choice D, which says that Celia tells the narrator about leaving Panama primarily to “assure the narrator that she understands and empathizes with her.” Answer Choice A is incorrect because there’s no suggestion or mention that Celia is trying to encourage the narrator to share her story, and more evidence is required to back up this more specific claim. Answer Choice B is incorrect because its more specific than Answer Choice D, and because Celia’s story, while somewhat similar to the narrator’s, only concerns her own reasons for immigration. Answer Choice C is incorrect because Celia’s story doesn’t mention how her family has prospered since immigrating to the United States.

How to solve this? The Question asks, “Based on the passage, it can reasonably be inferred that Celia tells the narrator about leaving Panama (lines 91-96) primarily to [...]?” To solve this [...].

10.R.17

Answer Choice C is the correct answer because Lines 97-98 say, “She put her hand on mine. A benediction. From then, we were friends.” These Lines suggest that the narrator finds Celia’s gesture to be a benediction, or blessing, in terms of its significance, and most clearly matches Answer Choice C, which says that the word serves to “emphasize how meaningful Celia’s gesture was for the narrator.” Answer Choice A is incorrect because, although the word “benediction” is used, nothing in the surrounding passage suggests its used here to emphasize the importance of religion. Answer Choice B is incorrect because the Lines emphasize the connection between the two women, not the establishment of Celia as an authority figure. Answer Choice D is incorrect because the word “benediction” doesn’t suggest any kind of eloquence that might be ascribed to Celia.

How to solve this? The Question asks, “In line 97, the word ‘benediction’ primarily serves to [...]?” To solve this [...]?

Page 233: SAT Full Reading Explanations (QAS 9-19) · SAT Full Reading Explanations (QAS 9-19) SAT #9 Reading 1 1.R.9 Answer Choice C is the correct answer because Lines 19-22 say, “Moths

Reading 2

Passage Outline

● most voters have strong positions on policy, but not clear understanding of them in action. (paragraph 1)

● this is called the “illusion of explanatory depth.” (paragraph 2) ● one scientist wanted to shake this confidence by making people explain their policies in

detail. (paragraph 3) ● the methodology of the experiment. (paragraph 4) ● the scientist thought that having to explain an idea in detail would shake that person’s

confidence, and that’s what happened. (paragraph 5) ● the scientists wanted to try this in another experiment, where people simply listed the

reasons that they believed something. (paragraph 6) ● this exercise had less of an effect on people’s beliefs. (paragraph 7) ● people who already believe something strongly, tend to reinforce those beliefs. but

having candidates give specific and concrete plans can help against this. (paragraph 8)

11.R.17

Answer Choice B is the correct answer because Lines 50-53 say, “Trying — and failing — to explain complex policies undermined the extremists’ illusions about being well-informed. They became more moderate in their views as a result.” These Lines most clearly explain the results of the experiment, and the central idea discussed in the passage, and most closely match Answer Choice B which says that “the process of describing an issue in detail can make people more moderate in their views about the issue.” Answer Choice A is incorrect because the Passage suggests that people only weaken theirs views of an issue, not that they come to decide that they are wrong. Answer Choice C is incorrect because the Passage doesn’t ever suggest that people aren’t interested in understanding complex ideas, only that they often fail to think through their beliefs completely. Answer Choice D is incorrect because the Passage doesn’t indicate that people understate their opinions to avoid offending others.

How to solve this? The Question asks, “A central idea discussed in the passage is that [...]?”

Page 234: SAT Full Reading Explanations (QAS 9-19) · SAT Full Reading Explanations (QAS 9-19) SAT #9 Reading 1 1.R.9 Answer Choice C is the correct answer because Lines 19-22 say, “Moths

12.R.17

Answer Choice B is the correct answer because the first two paragraphs (Lines 1-14) introduce the phenomenon of the “illusion of explanatory depth,” the next five paragraphs (Lines 15-62) discuss two experiments related to that phenomenon, and the last paragraph (Lines 69-78) suggests a possibility at mitigating the effects of that phenomenon. This breakdown reflects the best way at dividing the Passage into parts, and most clearly matches Answer Choice B which says that focus of the passage shifts from “an introduction of a phenomenon, to a description of experiments concerning that phenomenon, and then to a recommendation based on the results of the experiments.” Answer Choice A is incorrect because the Passage doesn’t specifically characterize the bias it explores as a “long-standing problem,” and doesn’t discuss previous attempts at addressing that bias. Answer Choice C is incorrect because the Passage doesn’t discuss competing theories for a behavior, doesn’t attempt to determine which of those two theories is correct and doesn’t explore the future of any academic field. Answer Choice D is incorrect because the Passage contains two experiments not mentioned in this description and because it doesn’t end with a proposal for further research.

How to solve this? The Question asks, “Over the course of the passage, the main focus shifts from [...]?”

13.R.17

Answer Choice A is the correct answer because the word “illusion” is used in Line 9 to represent the misunderstanding that people have about the depth of their knowledge, and can best be replaced by “misconception.” Answer Choice B is incorrect because the word “dream” fails to capture the right sense of misunderstanding or confusion. Answer Choice C is incorrect because it implies an intentional misrepresentation which doesn’t capture the correct sense of misunderstanding. Answer Choice D is incorrect because “phantom” doesn’t clearly replace the word “illusion” in the sentence.

● Psychological scientists have a name for this easy, automatic, simplistic thinking: the illusion of explanatory depth.

● Psychological scientists have a name for this easy, automatic, simplistic thinking: the misconception of explanatory depth.

How to solve this? The Question asks, “As used in line 9, ‘illusion’ most nearly means [...]?”

Page 235: SAT Full Reading Explanations (QAS 9-19) · SAT Full Reading Explanations (QAS 9-19) SAT #9 Reading 1 1.R.9 Answer Choice C is the correct answer because Lines 19-22 say, “Moths

14.R.17

Answer Choice C is the correct answer because the word “shape” used in Line 13 is used to emphasize the idea that beliefs can mold or impact us, and can most nearly be replaced by “influence.” Answer Choice A is incorrect because the Lines don’t mean that our beliefs can “plan” our actions, but simply influence them. Answer Choice B is incorrect because the Lines emphasize the cause and effect relationship between our beliefs and actions, not represented by the word “model.” Answer Choice D is incorrect because it less clearly captures the sense of an overall influence, and suggests more particular, modular adjustments.

● We strongly believe that we understand complex matter, when in fact we are clueless, and these false and extreme beliefs shape our preferences, judgments, and actions — including our votes.

● We strongly believe that we understand complex matter, when in fact we are clueless, and these false and extreme beliefs influence our preferences, judgments, and actions — including our votes.

15.R.17

Answer Choice C is the correct answer because Lines 26-42 primarily detail the methodology of the experiment in question, introducing the group of volunteers, the steps they were asked to follow and more. This most clearly matches Answer Choice C, which says that the main purpose of the fourth paragraph is to “outline the specifics of an experiment.” Answer Choice A is incorrect because the fourth paragraph isn’t used to support any theory, but simply to outline the methodology of an experiment. Answer Choice B is incorrect because the Passage doesn’t discuss the need for a research study or attempt to defend that need. Answer Choice D is incorrect because the Passage only explores the methodology behind an experiment, and not the complexity of that issue.

How to solve this? The Question asks, “The main purpose of the fourth paragraph (lines 26-42) is to [...]?”

Page 236: SAT Full Reading Explanations (QAS 9-19) · SAT Full Reading Explanations (QAS 9-19) SAT #9 Reading 1 1.R.9 Answer Choice C is the correct answer because Lines 19-22 say, “Moths

16.R.17 & 17.R.17

Answer Choices D and D are the correct answers because Lines 44-49 say, “Fernbach’s idea was that such an exercise would force many to realize just how little they really know about cap and trade, and confronted with their own ignorance, they would dampen their own enthusiasm.” These Lines tell us clearly that Fernbach expected individuals to have a poor understanding of the complex ideas which they were enthusiastic about, so we would expect him to be surprised at the opposite finding. This most closely matches Answer Choice D which says that Fernbach would have been surprised if, “When volunteers were asked questions about complex issues, those with the most extreme views were found to have the best overall understanding of them.” For Question 17, Answer Choice A is incorrect because it only explains the features of the group used in the experiment and doesn’t suggest something which Fernbach might have been surprised by. Answer Choice B is incorrect because it also only explains the methodology of the experiment, and not something that Fernbach might be surprised by. Answer Choice C is incorrect because it also only discusses the overall methodology of the experiment, and not something that would have surprised Fernbach. For Question 16, Answer Choice A is incorrect because neither the Passage nor the Texts suggest that Fernbach believed that there was a link between the complexity of an issue and the strength of volunteers’ position on it, so it’s doubtful that he would be surprised by not finding that link. Answer Choice B is incorrect because Fernbach didn’t expect volunteers’ understanding of an issue to increase after analyzing an issue, so he’d be unlikely to be surprised if it didn’t increase. Answer Choice C is incorrect because neither the Passage nor our Texts say that Fernbach expected that listing reasons for a policy would affect volunteers’ views in any significant way.

How to solve this? Question 16 asks, “The passage implies that when conducting his laboratory work, Fernbach would have been most surprised by which finding [...]?” To solve this [...].

18.R.17 & 19.R.17

Answer Choices B and D are the correct answers because Lines 76-78 say, “But forcing the candidates to provide concrete and elaborate plans might be a start; it gives citizens a starting place,” referencing the possibility of limiting political extremism. These Lines tell us that, based on the passage, making political candidates discuss their plans in specific detail may help curb extremism, and most closely matches Answer Choice B which says that the action most likely to reduce political extremism is “requiring that politicians explain their proposed policy in detail before an election is held.” For Question 19, Answer Choice A is incorrect because it only explains how individuals who gave reasons for their beliefs only strengthened their views, but doesn’t suggest a way to curb political extremism. Answer Choice B is incorrect because it only explains how polarization tends to increase people’s conviction in their beliefs, but doesn’t suggest a possible way to reduce extremism. Answer Choice C is incorrect because it continues to explain how polarization reinforces itself, without offering a solution. For Question 18, Answer Choice A is incorrect because neither the Passage nor the Texts suggest that forming groups who share strong opinions will limit political extremism, and in fact make the opposite claim. Answer Choice C is incorrect because neither the Passage nor the Texts suggest that the promotion of worthwhile charities will lead to the reduction of political extremism. Anwer Choice

Page 237: SAT Full Reading Explanations (QAS 9-19) · SAT Full Reading Explanations (QAS 9-19) SAT #9 Reading 1 1.R.9 Answer Choice C is the correct answer because Lines 19-22 say, “Moths

D is incorrect because, while it may be true, neither the Passage nor the Texts suggest that events that require people with opposing viewpoints to interact will curb political extremism.

How to solve this? Question 18 asks, “Based on the passage, which action would most likely reduce political extremism among the citizenry?” Because this is a Paired Question, to solve it we should [...].

20.R.17

Answer Choice C is the correct answer because Figure 1 measures the effect of analyzing and explaining a policy and listing reasons for or against a policy on the individual’s assessment of how well they understand that policy. The first two columns show that after being asked to analyze and explain a policy, individual’s assessment of their understanding dropped. This most closely matches Answer Choice C which says that after being asked to analyze and explain a policy, the volunteers “became less confident in their understanding of the policy.” Answer Choice A is incorrect because the Figure doesn’t measure extremism. Answer Choice B is incorrect because the Figure doesn’t measure an individual’s reluctance to justify their own viewpoint. Answer Choice D is incorrect because the Figure doesn’t measure interest.

How to solve this? The Question asks, “According to figure 1, which statement best describes the change in the mean self-ratings of the volunteers after they were asked to analyze and explain a policy?

21.R.17

Answer Choice A is the correct answer because Figure 2 measures the volunteers’ self-rating of their position on policy, and shows that listing reasons for or against a policy had almost no effect on that position. Lines 67-68 also say that those individuals “remained extreme in their passion for their position.” These most closely match Answer Choice A which says that both the Figure and the Passage suggest that listing reasons for or against a policy has little impact on an individual’s “position on an issue.” Answer Choice B is incorrect because Figure 2 doesn’t measure interest in an issue, and because the Passage doesn’t mention any effect of the exercise on that interest. Answer Choice C is incorrect because neither the Figure nor the Passage look at the opinions people have on those holding different views. Answer Choice D is incorrect because neither the Figure nor the Passage discuss the effect of this task on an individual’s likelihood to vote in an election.

How to solve this? The Question asks, “According to both the passage and figure 2, making a list of reasons for or against a policy has little impact on an individual’s [...]?”

Page 238: SAT Full Reading Explanations (QAS 9-19) · SAT Full Reading Explanations (QAS 9-19) SAT #9 Reading 1 1.R.9 Answer Choice C is the correct answer because Lines 19-22 say, “Moths

Reading 3

Passage Outline

Rare Woodland Plant Uses ‘Cryptic Coloration’ to Hide from Predators

● everyone knows animals use camouflage, but what about plants? (paragraph 1) ● plants using coloration to attract food or pollinators is well known, but not using it to

avoid predators. one scientists explored this. (paragraph 2) ● quote from the scientist about a specific plant, that has a broader range of possible

colors. (paragraph 3) ● methodology of the scientists. cut away the dried bracts that cover the plant, to see if

they function like camouflage. they found that they do and help the plants avoid predators. (paragraph 4)

● quote from the scientist that they have demonstrated that plants use camouflage to avoid predators. (paragraph 5)

● however this plant also needs animals to pollinate. how does it do this while still remaining camouflaged? they emit a strong odor to attract pollinators. (paragraph 6)

22.R.17

Answer Choice C is the correct answer because the Passage primarily focuses on a single plant, Monotropsis odorata, and explores whether its dried outside “might serve a similar purpose as the stripes on a tiger or the grey coloration of the wings of the peppered moth: namely, to hide.” This tells us most clearly that the Passage focuses on the camouflage on an individual plant, and most closely matches Answer Choice C which says that the purpose of the passage is to “describe a study illuminating a defensive strategy of a particular species of a plant.” Answer Choice A is incorrect because the Passage doesn’t focus especially on photosynthesis as a defining feature of Monotropsis odorata, nor does it compare the plant’s activities with those of other plants. Answer Choice B is incorrect because the Passage primarily concerns a plant’s ability to camouflage, not how it attracts pollinators. Answer Choice D is incorrect because the Passage’s experiment doesn’t compare plants and animals.

How to solve this? The Question asks, “The main purpose of the passage is to [...]?”

Page 239: SAT Full Reading Explanations (QAS 9-19) · SAT Full Reading Explanations (QAS 9-19) SAT #9 Reading 1 1.R.9 Answer Choice C is the correct answer because Lines 19-22 say, “Moths

23.R.17

Answer Choice B is the correct answer because Lines 2-7 say, “Individuals that are able to blend in to their surroundings and avoid being eaten are able to survive longer, reproduce, and thus increase their fitness (pass along their genes to the next generation) compared to those who stand out more.” These Lines most clearly tells us that “camouflage can provide an evolutionary advantage” to a species. Answer Choice A is incorrect because it only makes the case that many animals use camouflage to avoid predators, without elaborating on the evolutionary advantage this affords them. Answer Choice C is incorrect because it only asks whether plants use camouflage in the same way that animals do, without elaborating on the general advantages afforded by camouflage. Answer Choice D is incorrect because it only establishes that its well known that plants use coloration to attract pollinators, but doesn’t discuss the evolutionary advantage of using camouflage.

How to solve this? The Question asks, “Which choice best supports the idea that the ability of a species to use camouflage effectively can provide an evolutionary advantage?” To solve this [...].

24.R.17

Answer Choice D is the correct answer because Lines 14-15 say, “However, variation in pigmentation as a means of escaping predation has received little attention.” These Lines contrast with the fact that “the use of coloration or pigmentation as a vital component of acquiring food [...] has been well studied,” and most clearly match Answer Choice D, which says that in comparison to other functions of coloration in plants, camouflage has “been the subject of a smaller number of scientific investigations. Answer Choice A is incorrect because the Passage doesn’t make the case that understanding camouflage has allowed scientists to have a deeper understanding of potential food sources. Answer Choice B is incorrect because the Passage doesn’t make the case that plants use more distinctive shades of color in camouflage than in other functions. Answer Choice C is incorrect because the Passage contrasts camouflage functions of coloration as a means of avoiding predators with other color functions as a means of attracting pollinators, and makes the case that these camouflage colorations are effective.

How to solve this? The Question asks, “The passage indicates that compared with other functions of coloration in plants, camouflage in plants has [...]?” To solve this [...].

Page 240: SAT Full Reading Explanations (QAS 9-19) · SAT Full Reading Explanations (QAS 9-19) SAT #9 Reading 1 1.R.9 Answer Choice C is the correct answer because Lines 19-22 say, “Moths

25.R.17

Answer Choice A is the correct answer because Lines 19-21 say that scientist Matthew Klooster wanted to know if some of the features of Monotropsis odorata “might serve a similar purpose as the stripes on a tiger or the grey coloration of the wings of the peppered moth: namely, to hide.” Here, the comparison to the tiger and moth are used as examples species other than Monotropsis odorata that use camouflage to hide, and most closely matches Answer Choice A which says that the reference serves to “provide examples of animal species with characteristics analogous to those of the plant investigated in the passage.” Answer Choice B is incorrect because the Passage doesn’t indicate a distinction between the camouflage of animals and that of plants in the Lines given, but uses the two animals as a reference point for the function of camouflage. Answer Choice C is incorrect because the Passage isn’t making the case that the research done on plants can be applied to the animal world or these two species. Answer Choice D is incorrect because the reference doesn’t serve as a defense against criticism, and because the tiger and the moth weren’t included in the interpretation of the findings of the study.

How to solve this? The Question asks, “In Lines 20-21, the references to the tiger and the moth serve mainly to [...]?”

26.R.17 & 27.R.17

Answer Choices B and C are the correct answers because Lines 25-30 say, “‘Because this plant no longer requires photosynthetic pigmentation (i.e., green coloration) to produce its own energy, it is free to adopt a broader range of possibilities in coloration, much like fungi or animals.’” These Lines set other plants in contrast to Monotropsis odorata which receives all its nutrients from a specific fungus, and tell us that because these other plants rely on photosynthesis, they have a smaller range of coloration possibilities. This most closely matches Answer Choice B for Question 26, which says that the nutrient requirements of many plants have the consequence of “limiting the plants’ defensive options.” For Question 27, Answer Choice A is incorrect because it only introduces scientist Matthew Klooster and explains his research project, without explaining anything about the nutrient requirements of other plants. Answer Choice B is incorrect because it more directly explains how Monotropsis odorata receives its nutrients, without making a claim about how other plants do so and the related consequences. Answer Choice D is incorrect because it only explains the methodology of the study conducted, and not about the nutrient requirements of other plants. For Question 26, Answer Choice A is incorrect because the Text says that plants that rely on photosynthesis have “a smaller range of coloration possibilities,” but not that their coloration patterns are therefore exaggerated. Answer Choice C is incorrect because neither the Passage nor the Texts make the case that plants other than Monotropsis odorata have increased energy consumption. Answer Choice D is incorrect because neither the Passage nor our Texts make a connection between a plant’s nutrient requirements and that plant’s potential habitats.

How to solve this? Question 26 asks, “It can most reasonably be inferred from the passage that the nutrient requirements of many plants have the consequence of [...]?” Because this is a Paired Question, to solve it [...].

Page 241: SAT Full Reading Explanations (QAS 9-19) · SAT Full Reading Explanations (QAS 9-19) SAT #9 Reading 1 1.R.9 Answer Choice C is the correct answer because Lines 19-22 say, “Moths

28.R.17

Answer Choice A is the correct answer because the word “broader” as used in Line 28, serves to emphasize the more inclusive, expansive or “extensive” set of possibilities for coloration for certain plants. Answer Choice B is incorrect because the Lines don’t mean that the range of possibilities is more obvious. Answer Choice C is incorrect because the word “tolerant” is best used to describe human behavior. Answer Choice D is incorrect because it focuses too literally on physical space and not abstract set size.

● “Because this plant no longer requires photosynthetic pigmentation (i.e., green coloration) to produce its own energy, it is free to adopt a broader range of possibilities in coloration, much like fungi or animals.”

● “Because this plant no longer requires photosynthetic pigmentation (i.e., green coloration) to produce its own energy, it is free to adopt a more extensive range of possibilities in coloration, much like fungi or animals.”

How to solve this? The Question asks, “As used in line 28, “broader” most nearly means more [...]?” To solve this [...].

29.R.17

Answer Choice C is the correct answer because the word “worked” in Line 48 most nearly means served or “functioned,” emphasizing the use of the camouflage to the plant. Answer Choice A is incorrect because it implies that the camouflage “changed” from some original purpose to the purpose of hiding the plant, which doesn’t make sense. Answer Choice B is incorrect because the Line doesn’t make sense if you plug in the word, and because the camouflage is not exerting some influence that hides the plant, but hides the plant directly. Answer Choice D is incorrect because the word “manipulated” has too many human connotations and because it doesn’t make sense to say that the plant’s camouflage “manipulated” to hide the plant.

● Furthermore, they experimentally demonstrated that this camouflage actually worked to hide the plant from its predators and increased its fitness.

● Furthermore, they experimentally demonstrated that this camouflage actually functioned to hide the plant from its predators and increased its fitness.

How to solve this? The Question asks, “As used in line 48, ‘worked’ most nearly means [...]?” To solve this [...].

Page 242: SAT Full Reading Explanations (QAS 9-19) · SAT Full Reading Explanations (QAS 9-19) SAT #9 Reading 1 1.R.9 Answer Choice C is the correct answer because Lines 19-22 say, “Moths

30.R.17

Answer Choice A is the correct answer because figure 1 measures the reflectance and wavelength for four features of Monotropsis odorata. The dark dotted Line indicates these measurements for the flower petal of Monotropsis odorata, and at 600 nanometers corresponds to 5 percent on the y-axis. Answer Choice B is incorrect because [...].

How to solve this? The Question asks, “According to figure 1, the percent reflectance of the flower petal of the Monotropsis odorata plants for light at a wavelength of 600 nanometers was approximately? To solve this [...].

31.R.17

Answer Choice D is the correct answer because figure 1 measures the reflectance and wavelength for four features of Monotropsis odorata, and the line measuring “bract” and that which measures the “reproductive stem” diverge significantly, most closely matching Answer Choice D which says that “the bract reflectance pattern differs substantially from the reproductive stem reflectance pattern.” Answer Choice A is incorrect because the flower petal and bract reflectance patterns do not follow one another. Answer Choice B is incorrect because the trend where the leaf litter line decreases while the flower petal line increases isn’t represented by the graph, and in fact all four lines increase together. Answer Choice C is incorrect because the line measuring the stem reflectance pattern differs significantly from that measuring the leaf litter reflectance pattern.

How to solve this? The Question asks, “Which statement about reflectance patterns in Monotropsis odorata is best supported by the data presented in figure 1?” To solve this [...].

32.R.17

Answer Choice B is the correct answer because Figure 2 measures the proportion of floral stems suffering from herbivore damage for two years, 2006 and 2007. Each year also has two columns, where the light gray columns show “plants with bracts intact” and the dark gray columns show “plants with bracts removed.” For the plants with intact bracts in 2007 (the third, light gray column), the mean proportion of damage rises to about .17. Answer Choice A is incorrect because [...].

How to solve this? The Question asks, “According to figure 2, the mean proportion of floral stems with herbivore damage per plant for Monotropsis odorata plants with intact bracts in 2007 was about [...]?” To solve this [...].

Page 243: SAT Full Reading Explanations (QAS 9-19) · SAT Full Reading Explanations (QAS 9-19) SAT #9 Reading 1 1.R.9 Answer Choice C is the correct answer because Lines 19-22 say, “Moths

Reading 4

Passage Outline

“Reflections on the Revolution in France” — In 1789, the French formed a new governmental body known as the National Assembly, ushering the tumultuous period of political and social change known as the French Revolution.

● it is easy to make a government with one person in control, or one where there is no leadership and perfect freedom. but it’s very hard to make a government with leadership and freedom for individuals. to do so requires a lot of talent and character, something which the National Assembly lacks. it is nothing but a popularity contest, where they follow the people and don’t lead them, have become extreme and can’t make any proper laws. (paragraph 1)

● isn’t there something good that the National Assembly has done? in a way, because if everything is destroyed some bad things will be destroyed as well and you allow the possibility for good things to grow in their place. but you have to assume there was no other way to get rid of the bad things. the “improvements” the National Assembly has made are only surface level, but their errors are deep. (paragraph 2)

● britain should avoid following France in their revolution, but instead recommend their constitution to them. britain owes its happiness to the entire constitution, and not to one part, and they should protect it from too much change. the constitution was built with an eye towards the ignorance and fallibility of people, and that kind of humility should be respected. britain should value that foundation and not follow France.

33.R.17 & 34.R.17

Answer Choices C and C are the correct answers because Lines 13-18 say, “But when the leaders choose to make themselves bidders at an auction of popularity, their talents, in the construction of the state, will be of no service. They will become flatterers instead of legislators, the instruments, not the guides, of the people.” These Lines tell us clearly that Burke disapproves of the National Assembly’s interest in popularity over leadership, which most closely matches Answer Choice which says that Burke is upset by the Assembly’s decision to “seek the approval of the public.” For Question 34, Answer Choice A is incorrect because these Lines only explain the necessary qualities that leaders need to manage a free government. Answer Choice B is incorrect because these Lines only show Burke making the claim that if the National Assembly were actually as bad as they seem, it would put them below the “common level of human understanding.” Answer Choice D is incorrect because it only explains that the good that the National Assembly has done is merely surface level, while their errors are fundamental. For Question 33, only Answer Choice C matches Lines 13-18. Answer Choice A is incorrect because Burke would actually support to limiting of the king’s power in the service of creating a free government. Answer Choice B is incorrect because Burke doesn’t mention the National Assembly expanding the size of the government. Answer Choice is incorrect because, although this might seem a plausible point for Burke to meet, he doesn’t make the point in either the Texts or the Passage.

Page 244: SAT Full Reading Explanations (QAS 9-19) · SAT Full Reading Explanations (QAS 9-19) SAT #9 Reading 1 1.R.9 Answer Choice C is the correct answer because Lines 19-22 say, “Moths

How to solve this? Question 33 asks, “It can reasonably be inferred from the passage that Burke is particularly upset with the National Assembly’s decision to [...]?” Because this is a Paired Question, to solve it we should [...].

35.R.17

Answer Choice B is the correct answer because the word “instruments” in Line 17 is used to emphasize how members of the National Assembly don’t lead the people but are used by them, most closely matching ‘tools.” Answer Choice A is incorrect because the word “representatives” implies a more active leadership role and loses the negative and passive connotation of “instruments.” Answer Choice C is incorrect because it too specifically casts the Assembly as counselors or guides, and loses the passive sense of “instruments.” Answer Choice D is incorrect because it doesn’t make sense to call the Assembly the “navigators” of the people.

● They will become flatterers instead of legislators, the instruments, not the guides, of the people.

● They will become flatterers instead of legislators, the tools, not the guides, of the people.

How to solve this? The Question asks, “As used in line 17, ‘instruments’ most nearly means [...]?” To solve this [...].

36.R.17

Answer Choice D is the correct answer because Lines 18-22 say, “If any of them should happen to propose a scheme of liberty, soberly limited and defined with proper qualifications, he will be immediately outbid by his competitors who will produce something more splendidly popular.” These Lines tell us that Burke believes that French leaders who advocate more moderate positions are sensible and qualified but will ultimately be outbid by those who are more popular, and most closely match Answer Choice D which says that Burke finds these leaders are “sensible, but are likely to be undermined.” Answer Choice A is incorrect because, although Burke does suggest that these leaders will raise suspicions or be distrusted, he never clearly calls them “brave.” Answer Choice B is incorrect because Burke doesn’t ever call these leaders “cowardly” and suggests that they’ll receive a more negative reaction, not that they’ll be praised. Answer Choice C is incorrect because Burke moreso calls these leaders “sensible” than the more specific “virtuous,” and suggests that they’ll be outbid, not necessarily “ignored.”

How to solve this? The Question asks, “Based on the passage, Burke believes that French leaders who would advocate moderate positions are [...]?” To solve this [...].

Page 245: SAT Full Reading Explanations (QAS 9-19) · SAT Full Reading Explanations (QAS 9-19) SAT #9 Reading 1 1.R.9 Answer Choice C is the correct answer because Lines 19-22 say, “Moths

37.R.17

Answer Choice D is the correct answer because the word “sober” in Line 30 is used to emphasize how sensible or “reasonable” the purposes are, in contrast to more sensational and popular ones. Answer Choice A is incorrect because the lines aren’t emphasizing the seriousness or solemness of the purposes. Answer Choice B is incorrect because the Lines aren’t emphasizing the improvement or repurposing of the purposes. Answer Choice C is incorrect because the Lines aren’t emphasizing the earnestness or lack of pretense of the purposes.

● [...] the popular leader is obliged to become active in propagating doctrines and establishing powers that will afterwards defeat any sober purpose at which he ultimately might have aimed.

● [...] the popular leader is obliged to become active in propagating doctrines and establishing powers that will afterwards defeat any reasonable purpose at which he ultimately might have aimed.

How to solve this? The Question asks, “As used in line 30, ‘sober’ most nearly means [...]?”

38.R.17

Answer Choice B is the correct answer because Lines 59-64 say, “I think our happy situation owing to our constitution, but owing to the whole of it, and not to any part singly, owing in great measure to what we have left standing in our several reviews and reformations as well as to what we have altered or superadded,” and Lines 69-72 say, “In what I did, I should follow the example of our ancestors. I would make the reparation as nearly as possible in the style of the building.” These Lines tell us that Burke believes the Britsh owe their political situation to the fact that they have left their constitution mostly unchanged with only a few modifications and revisions. This most closely matches Answer Choice D which says that Burke’s central claim in the last paragraph is that the British have “left their government essentially unchanged for hundreds of years.” Answer Choice A is incorrect because Burke doesn’t mention how the British have regarded their rights. Answer Choice C is incorrect because, although Burke does suggest that the British encourage France to adopt a position similar to their own rather than be influenced by them, he never makes the case that they have already actively tried to do so. Answer Choice D is incorrect because Burke does make the case that the British have made additions and amendments to their government over the years, just that they’ve been moderate in those changes that they’ve made.

How to solve this? The Question asks, “Burke’s central claim in the last paragraph is that the British have [...]?”

Page 246: SAT Full Reading Explanations (QAS 9-19) · SAT Full Reading Explanations (QAS 9-19) SAT #9 Reading 1 1.R.9 Answer Choice C is the correct answer because Lines 19-22 say, “Moths

39.R.17

Answer Choice D is the correct answer because when Burke, in Lines 70-72, says, “I would make the reparation as nearly as possible in the style of the building,” he means to emphasize that changes to a government should be made cautiously and still preserve the underlying structure. This most closely matches Answer Choice D, which says that building is mentioned to make the point that “changes to a government should maintain that government’s essential properties.” Answer Choice A is incorrect because Burke cautions against making too many changes to the government and doesn’t make the case that they must be updated every now and then to stay relevant. Answer Choice B is incorrect because in these Lines Burke is making the opposite case, that governments should be repaired little by little and not completely replaced. Answer Choice C is incorrect because, although Burke may believe this point, the specific image of the repair of the building is used primarily to emphasize his belief that governments should be changed in part, not that all of their flaws can be fixed.

How to solve this? The Question asks, “Burke refers to the repair of a building (lines 70-72) to make the point that [...]?” To solve this [...].

40.R.17 & 41.R.17

Answer Choices B and D are the correct answers because Lines 81-84 say, “Let us imitate their caution if we wish to deserve their fortune or to retain their bequests. Let us add, if we please, but let us preserve what they have left; [...],” where “they” refers to “our forefathers.” These Lines tell us that Burke has immense respect for former British leaders. For Question 41, Answer Choice A is incorrect because these Lines show Burke suggesting to the British that they encourage France to follow their own example and not follow France’s. Answer Choice B is incorrect because it only shows Burke admitting that the British may have some reason to complain, but argues that this is because of their own behavior and not their constitution. Answer Choice C is incorrect because these Lines show Burke making the case that the British will find their own expression of freedom and patriotism in how they protect their own constitution, and not anything about something that he respects. For Question 40, Answer Choice A is incorrect because the “they” from Lines 81-84 refers to Burke’s “forefathers” or former British leaders, and not the general voting public. Answer Choice C is incorrect because Burke advises the British against taking inspiration from the French, and doesn’t suggest any respect for them. Answer Choice D is incorrect because Burke doesn’t suggest any respect for the leaders of France’s former government in either the Passage or Texts.

How to solve this? The Question asks, “In the passage, Burke displays the greatest respect for which of the following?” Because this is a Paired Question, to solve it [...].

Page 247: SAT Full Reading Explanations (QAS 9-19) · SAT Full Reading Explanations (QAS 9-19) SAT #9 Reading 1 1.R.9 Answer Choice C is the correct answer because Lines 19-22 say, “Moths

42.R.17

Answer Choice A is the correct answer because in Lines 75-77, which say, “Not being illuminated with the light of which the gentlemen of France tell us they have got so abundant a share, they have acted under a strong impression of the ignorance and fallibility of mankind,” Burke contrasts the “gentlemen of France” and their belief in an internal guiding light and insight to Britain’s forefather’s humility. This is clearly a negative opinion of these “gentlemen of France,” and most closely matches the description of “scornful.” Answer Choice B is incorrect because burke doesn’t take an understanding or “sympathetic” stance towards the “gentlemen of France,” but a clearly negative one. Answer Choice C is incorrect because Burke doesn’t express envy towards these “gentlemen of France,” but a disrespect or disdain. Answer Choice D is incorrect because Burke doesn’t express a timidity or “apprehension” towards these “gentlemen of France.”

How to solve this? The Question asks, “Burke’s attitude toward the ‘gentlemen of France’ (lines 76-77) would best be described as [...]?”

Reading 5

Passage Outline

Passage 1 — “Blast in the Past?”

● a group of scientists gathered to argue that a comet or asteroid exploded above earth and caused temperatures to drop. (paragraph 1)

● this group argues that this hypothesis makes sense because it explains a lot of things. (paragraph 2)

● however, there are several other hypotheses that could explain these events. (paragraph 3)

● but new evidence from geochemical analysis makes this hypothesis about an asteroid or comet more plausible. (paragraph 4)

Passage 2 — “What Caused a 1300-Year Deep Freeze?”

● proponents of the Younger Dryas impact have a lot of evidence, but not everyone finds this convincing. (paragraph 1)

● now new evidence about the dating of different sites attacks this hypothesis even more. (paragraph 2)

● quote from scientist who says that the hypothesis fails for a number of reasons, although people want to believe it because it’s so simple and dramatic. (paragraph 3)

● but believers in the hypothesis still remain convinced.

Page 248: SAT Full Reading Explanations (QAS 9-19) · SAT Full Reading Explanations (QAS 9-19) SAT #9 Reading 1 1.R.9 Answer Choice C is the correct answer because Lines 19-22 say, “Moths

43.R.17

Answer Choice B is the correct answer because Lines 15-17 say, “A change in ocean circulation is generally thought to have brought about the onset of the millenium-long cooling, which is known as the Younger Dryas.” These Lines tell us about one of the “leading hypotheses [that] explain each of these three events” mentioned in Lines 13-14, and most clearly tell us that “the impact hypothesis is not the only possible explanation for the sudden change in Earth’s climate.” Answer Choice A is incorrect because it only explains the logic behind those who believe in the impact hypothesis. Answer Choice C is incorrect because it only explains how new evidence supports the impact hypothesis, but doesn’t suggest the existence of other possible explanatory theories. Answer Choice D is incorrect because it only continues the explanation of how new evidence supports the impact hypothesis, and doesn’t suggest the existence of any other possible explanatory hypotheses.

How to solve this? The Question asks, “Which choice provides the best support for the claim that the impact hypothesis is not the only possible explanation for the sudden change in Earth’s climate?” To solve this [...].

44.R.17

Answer Choice D is the correct answer because if an element like osmium, which is rare on earth but common in asteroids, similar to the iridium mentioned in Lines 30, were found in the geologic record from 13,000 years ago, when the scientists believed an asteroid hit, it would suggest an exterrestrial impact like a comet or asteroid and support the impact hypothesis. Answer Choice A is incorrect because, while an impact crater that contains high levels of iridium would support the occurrence of some kind of impact like a comet or asteroid, the impact hypothesis argues that the comet or asteroid caused the Younger Dryas (Lines 5-7 & 15-17), not that it hit well afters its start. Answer Choice B is incorrect because, while glass and carbon spherules were found at several archaeological sites the impact hypothesis team used as evidence, it was the presence of iridium that primarily suggested an extraterrestrial impact, so its absence wouldn’t support the hypothesis. Answer Choice C is incorrect because if temperatures were shown to begin declining well before the Younger Dryas, this wouldn’t support the impact hypothesis which suggests that an asteroid or comet caused falling temperatures right at the start of the Younger Dryas.

How to solve this? The Question asks, “High levels of osmium, which is rare on Earth but relatively common in asteroids, are observed in the geologic record from approximately 13,000 years ago.

Page 249: SAT Full Reading Explanations (QAS 9-19) · SAT Full Reading Explanations (QAS 9-19) SAT #9 Reading 1 1.R.9 Answer Choice C is the correct answer because Lines 19-22 say, “Moths

45.R.17

Answer Choice C is the correct answer because Lines 31-34 say, “In addition, the rocks contain black layers of carbonized material, which the team says are the remains of wildfires that swept across the continent after the impact.” These Lines most clearly tell us that the black carbonized material found in some sedimentary layers were caused by “rapidly spreading fires,” matching Answer Choice C. Answer Choices A, B and D are incorrect because the Passage only cites wildfires as the cause of the black carbonized material.

How to solve this? The Question asks, “According to Passage 1, the team of scientists believes that the black carbonized material found in certain sedimentary layers were caused by which phenomenon following a cosmic collision?” To solve this [...].

46.R.17

Answer Choice C is the correct answer because the word “sharply” in Line 42 is used to emphasize emphasize the conviction of the other specialists or how “strongly” they made their argument. Answer Choice A is incorrect because the word “sharply” isn’t used to emphasize the immediate timing of their argument. Answer Choice B is incorrect for the same reason, because the word “sharply” doesn’t emphasize the timing of their response. Answer Choice D is incorrect because the word “sharply” doesn’t emphasize the intention or deliberation of their argument.

● These claims were sharply contested by some specialists in the relevant fields, however, who either did not detect such evidence or argued that the deposits had other causes than a cosmic impact.

● These claims were strongly contested by some specialists in the relevant fields, however, who either did not detect such evidence or argued that the deposits had other causes than a cosmic impact.

How to solve this? The Question asks, “As used in line 42, ‘sharply’ most nearly means [...]?” To solve this, [...].

47.R.17 & 48.R.17

Answer Choices A and D are the correct answers because Lines 62-69 say, “The team argues that when the quality and accuracy of the dating — which was based on radiocarbon and other techniques — is examined closely, only three of the 29 sites actually fall within the time frame of the Younger Dryas onset [...].” These Lines most clearly tell us about evidence that Meltzer and his team used to challenge the impact hypothesis and tell us that some of the sites used to support the hypothesis actually have a later date than previously realized. This most closely matches Answer Choice A which says that Meltzer and his team found evidence to challenge the Younger Dryas impact hypothesis after “a reevaluation of the dates assigned to sites thought to display signs of the proposed impact.” For Question 48, Answer Choice A is incorrect because these Lines only make the claim that supporters of the impact hypothesis have various evidence to support their theory. Answer Choice B is incorrect because these Lines give general evidence for why other scientists don’t believe the impact hypothesis, but don’t specifically focus

Page 250: SAT Full Reading Explanations (QAS 9-19) · SAT Full Reading Explanations (QAS 9-19) SAT #9 Reading 1 1.R.9 Answer Choice C is the correct answer because Lines 19-22 say, “Moths

on the reasons used by Meltzer and his team. Answer Choice C is incorrect because these Lines also give evidence from other scientists and not specifically from Meltzer and his team.

How to solve this? The Question asks, “Based on Passage 2, Meltzer and his team relied on what evidence to challenge the Younger Dryas impact hypothesis?” Because this is a Paired Question, to solve this [...].

49.R.17

Answer Choice B is the correct answer because the phrase “more nuanced” in Lines 78 means more complex, sophisticated or subtle, which most directly contrasts with the word “simple.” Answer Choice A is incorrect because “theoretical” emphasizes the abstract, purely idea-based nature of something and doesn’t contrast directly with “more nuanced.” Answer Choice C is incorrect because “complex” is almost a synonym for “more nuanced” and doesn’t contrast directly with it. Answer Choice D is incorrect because “broad” emphasizes the wide scope of something and contrasts with “narrow,” not with “more nuanced.”

How to solve this? The Question asks, “The phrase ‘more nuanced’ (Line 78) contrasts most directly with which word in Meltzer’s quoted remarks in Lines 70-77?”

50.R.17

Answer Choice A is the correct answer because Passage 1 primarily explores the details of the Younger Dryas impact hypothesis while Passage 2 discusses specific criticisms of that hypothesis. This most closely matches Answer Choice A, which describes the relationship between the two passages as one where “Passage 2 presents a critique of the central hypothesis described in Passage 1.” Answer Choice B is incorrect because Passage 2 primarily provides a critique of the hypothesis in Passage 1, and doesn’t look more generally at the underlying scientific question of what caused the Younger Dryas cooling. Answer Choice C is incorrect because Passage 2 doesn’t look at the implications of the impact hypothesis, but instead provides a critique of it. Answer Choice D is incorrect because Passage 2 doesn’t look at evidence in favor of the impact hypothesis but provides reasons to criticize it.

How to solve this? The Question asks, “Which statement best describes the relationship between the two passages?”

Page 251: SAT Full Reading Explanations (QAS 9-19) · SAT Full Reading Explanations (QAS 9-19) SAT #9 Reading 1 1.R.9 Answer Choice C is the correct answer because Lines 19-22 say, “Moths

51.R.17

Answer Choice D is the correct answer because both Passages make the point that the Younger Dryas impact hypothesis is “controversial in the scientific community.” In Passage 1, for example, Lines 13-14 say, “Not all will be convinced. Several leading hypothesis already explain each of these three events,” while Passage 2 says in Lines 42-43 that the claims of the Younger Dryas impact hypothesis “were sharply contested by some specialists in the relevant fields [...].” Answer Choice A is incorrect because only Passage 2 makes the case that the hypothesis is founded on reliable evidence, while Passage 1 explores new evidence it presents as convincing (lines 23-34). Answer Choice B is incorrect because Passage 1 presents the hypothesis as convincing and Passage 2 doesn’t make the sympathetic case that the hypothesis is interesting but just difficult to conclusively evaluate. Answer Choice C is incorrect because only Passage 2 makes the case that the impact hypothesis is more appealing to the public than to specialists, a point that Passage 1 doesn’t address. How to solve this? The Question asks, “The authors of both passages characterize the impact hypothesis as [...]?”

52.R.17

Answer Choice A is the correct answer because Meltzer’s primary finding is that the sites which the impact hypothesis uses as evidence were dated incorrectly, suggesting that spherules of glass and carbon found in these sites may be from an earlier or later date, most closely matching Answer Choice A which says that these spherules “could have been formed at a time other than the beginning of the Younger Dryas.” Answer Choice B is incorrect because Meltzer in Passage 2 doesn’t make any point about what was likely to form as a result of the global cooling during the Younger Dryas period. Answer Choice C is incorrect because Meltzer makes the case that several sites date to different periods than earlier suggested, but doesn’t suggest that glass and carbon spherules would be found in higher concentrations at Clovis archaeological sites. Answer Choice D is incorrect because Passage 2 doesn’t make any case about the tool technology of the Clovis people nor does Passage 1 suggest that the glass and carbon spherules were tools used by them.

How to solve this? The Question asks, “If Meltzer’s findings (Passage 2) are accurate, what can most reasonably be inferred about the glass and carbon spherules mentioned in the last paragraph of Passage 1?”

Page 252: SAT Full Reading Explanations (QAS 9-19) · SAT Full Reading Explanations (QAS 9-19) SAT #9 Reading 1 1.R.9 Answer Choice C is the correct answer because Lines 19-22 say, “Moths

SAT #18, Full Test

Reading 1

Passage Outline

“The Master” — The novel is based on the life of writer Henry James (1843-1916)

● Henry and Mr. Milson developed a relationship over their shared interest in homes. Henry wants to live in a home in Point Hill, but no houses are for sale. (paragraph 1)

● Henry’s view of his conversation with Mr. Milson as a kind of play. He receives a letter from Milson about a house that has opened up. Henry fears he’ll lose it because this is the house he wanted so badly. (paragraph 2)

● Henry calls for help from a friend and wonders if anyone knew how important this moment was for him. He had wanted a house for so long, as a place to shield him from the outer world. (paragraph 3)

1.R.18

Answer Choice D is the correct answer because the first Paragraph primarily concerns Mr. Milson and Henry discussing homes, and Milson looking for a suitable home for Henry (Lines 15-22). The second Paragraph, however, moves to Milson’s announcing that he has found a home for Henry, and Henry’s reflection on what that means (Lines 70-ish). This most closely matches Answer Choice D, which says that the main focus of the narrative shifts from “an account of Mr. Milson’s search for a suitable property for Henry to a portrait of Henry’s musings on the meaning of having a home.” Answer Choice A is incorrect because the first part of the Passage doesn’t describe Henry’s unhappiness in Rye, and the later part of the Passage isn’t accurately described as Henry trying to find happiness there. Answer Choice B is incorrect because the first part of the Passage focuses primarily on Henry and Milson’s discussion of buying a home in Rye, not a depiction of life there. Answer Choice C is incorrect because the Passage doesn’t directly contrast the two men’s personality, nor does it then emphasize a similarity between them.

How to solve this? The Question asks, “Over the course of the passage, the main focus of the narrative shifts from [...]?”

Page 253: SAT Full Reading Explanations (QAS 9-19) · SAT Full Reading Explanations (QAS 9-19) SAT #9 Reading 1 1.R.9 Answer Choice C is the correct answer because Lines 19-22 say, “Moths

2.R.18

Answer Choice B is the correct answer because Lines 8-11 say, “Since Mr. Milson enjoyed talking, and since he was not interested in literary matters, and since he had not been to America and knew no other Amercians, and since Henry’s knowledge or ironmongery was rudimentary, the two men discussed houses [...].” These Lines most clearly match Answer Choice B, which says of the two men that “they have little in common; therefore, they talk about houses in the area that may be of interest to Henry.” Answer Choice A is incorrect because it’s too strong to describe Henry as desperate in the Passage, and Mr. Milson, in the first paragraph, only tells him about homes that “remained firmly in [their] owner’s hands.” Answer Choice C is incorrect because the Passage doesn’t say anything about Mr. Milson not liking new arrivals or of being insincere with Henry. Answer Choice D is incorrect because the Passage doesn’t indicate that Mr. Milson is new to Rye or that he also needs a place to live.

How to solve this? The Question asks, “Which choice best summarizes what is learned about Henry and Mr. Milson in the first paragraph of the passage?” To solve this [...].

3.R.18

Answer Choice C is the correct answer because Lines 25-27 show Henry describing his conversation with Mr. Milson as “a way of drinking in England, allowing its flavors to come to him in phrases, turns of speech and local references,” and use “flavors” as a way of describing all the particular features of England. This most closely matches Answer Choice C which says that the literal meaning of the figurative “flavors” is, “The aspects of a place that give it its particular character.” Answer Choice A is incorrect because “flavor” is used to describe a place and not an “object,” and because these “flavors” aren’t characterized as the thing that makes the place memorable. Answer Choice B is incorrect because while “flavors” is used to describe the features of an environment, it doesn’t emphasize those features that appeal most to the senses. Answer Choice D is incorrect because, while the word does refer to the “characteristics of a locale,” it doesn’t connect these to the reason for the places popularity.

How to solve this? The Question asks, “Which choice most closely captures the literal meaning of the figurative ‘flavors’ referred to in Line 26?” To solve this [...].

Page 254: SAT Full Reading Explanations (QAS 9-19) · SAT Full Reading Explanations (QAS 9-19) SAT #9 Reading 1 1.R.9 Answer Choice C is the correct answer because Lines 19-22 say, “Moths

4.R.18

Answer Choice B is the correct answer because Lines 27-34 say, “Thus even when he opened the letter which arrived at his London address, having noticed that the handwriting on the envelope was not that of someone accustomed to writing letters, and even when he saw the name Milson as the sender, he was still puzzled by its provenance. Only when he read it a second time did he realize who it was from and then, as though he had received a blow in the stomach, he understood what the letter said.” These Lines tell us most clearly that Henry didn’t recognize or understand the importance of the letter when he first received it, which most clearly matches Answer Choice B, which says that “he initially fails to appreciate the letter’s significance.” Answer Choice A is incorrect because the Passage doesn’t suggest that Henry hopes the letter came from someone else. Answer Choice C is incorrect because the Passage doesn’t tell us that Henry thinks the letter will contain bad news. Answer Choice D is incorrect because the Passage doesn’t tell us that Henry thinks the letter will contain false information.

How to solve this? The Question asks, “The passage indicates that Henry has which reaction when he receives Mr. Milson’s letter?” To solve this [...].

5.R.18 & 6.R.18

Answer Choices C and C are the correct answers because Lines 33-34 say, “He wondered if his ironmonger was freely broadcasting the news of this vacancy to all comers,” where “his ironmonger” refers to Mr. Milson. These Lines most clearly tell us that Henry “suspects that Mr. Milson may not be looking out exclusively for his best interests.” For Question 6, Answer Choice A is incorrect because it refers to the moment when Henry has read but not fully understood Mr. Milson’s letter. Answer Choice B is incorrect because, while these Lines tell us about Henry after reading and understanding the letter, they don’t match any answer choice from Question 5. Answer Choice D is incorrect because these Lines only tell us that Mr. Henry wanted to move into this particular house badly, a point which less directly comes after understanding Milson’s letter and that doesn’t match any of the answer choices from Question 5. For Question 5, Answer Choice A is incorrect because neither the Passage nor the Texts suggest that Henry thinks Mr. Milson may not be knowledgeable about the property in question. Answer Choice B is incorrect because neither the Passage nor the Texts make the point that Henry regrets how little he paid attention to the house in question. Answer Choice D is incorrect because neither the Passage nor the Texts show Henry questioning Mr. Milson’s motivation.

How to solve this? Question 5 asks, “The passage suggests that after reading and understanding Mr. Milson’s letter, Henry [...]?” To solve this [...].

Page 255: SAT Full Reading Explanations (QAS 9-19) · SAT Full Reading Explanations (QAS 9-19) SAT #9 Reading 1 1.R.9 Answer Choice C is the correct answer because Lines 19-22 say, “Moths

7.R.18

Answer Choice B is the correct answer because Lines 45-48 say, “Nothing had ever come easily, magically like this. He could do what he liked, he could send a cable, he could take the next train, but he remained sure that he would lose it.” These Lines tell us clearly that Henry thinks he will lose the possibility to live in the Lamb house because things don’t come this easily, which most closely matches Answer Choice B which says that Henry “cannot believe that a wish he feels so intensely could possibly be fulfilled.” Answer Choice A is incorrect because the Passage doesn't suggest that commitments in London will delay his travels, but says that he would take the next train. Answer Choice C is incorrect because the Passage doesn’t indicate that Henry treated Mr. Milson badly or that he worries this treatment will hurt his chances of getting the house. Answer Choice D is incorrect because the Passage shows Henry being resolute in his desire to purchase the house and simply worrying it will be taken before he gets there, and not that he fears his own indecisiveness will affect him.

How to solve this? The Question asks, “It can be inferred that Henry fears that he will lose the possibility of being able to live in Lamb House because he [...]?”

8.R.18

Answer Choice A is the correct answer because Lines 35-36 say, “Nothing had ever come easily, magically like this.” These Lines tell us most clearly that Henry is accustomed to struggling for things that he wants and not receiving them easily. Answer Choice B is incorrect because these Lines only show Henry realizing that the only thing he can do is try to make it to Lamb House and hope for the best, but don’t indicate that he’s used to struggling to attain things. Answer Choice C is incorrect because these Lines only show Henry wondering if anyone else knows how important this moment was for him, not that he’s accustomed to struggling for what he wants. Answer Choice D is incorrect because these Lines only show Henry expressing that he knows this wouldn’t be as big a deal to others.

How to solve this? The Question asks, “Which choice best supports the claim that Henry feels that his life has been characterized by a struggle to attain things that he desired?” To solve this [...].

Page 256: SAT Full Reading Explanations (QAS 9-19) · SAT Full Reading Explanations (QAS 9-19) SAT #9 Reading 1 1.R.9 Answer Choice C is the correct answer because Lines 19-22 say, “Moths

9.R.18

Answer Choice B is the correct answer because the last paragraph primarily focuses on Henry’s thoughts about why getting a house is so important to him, how long he’s been looking for one and what having one would mean to him. This most clearly matches Answer Choice B, which says that the last paragraph serves to “provide context that explains Henry’s particular aspiration,” or gives background for why he wants to buy the house so badly. Answer Choice A is incorrect because the last paragraph doesn’t primarily sketch a series of events that led Henry to this moment. Answer Choice C is incorrect because the last paragraph doesn’t create a mood of anticipation by making the moment more suspenseful or uncertain. Answer Choice D is incorrect because the Passage doesn’t primarily describe the Lamb’s House and it’s not yet certain that Henry will get to live there and that it will be the setting for his new experiences.

How to solve this? The Question asks, “The last paragraph mainly serves to [...]?” To solve this [...].

10.R.18

Answer Choice A is the correct answers because the words “shell,” “exposure,” “facade,” and “frontage” all serve to explain the features of the house that will protect Henry’s psychology and which he feels he lacks. In this case, he makes a clear connection between these actual features of the house and features of his own personality or lack of those features. This most clearly matches Answer Choice A which says that the words serve to “establish a parallel between aspects of Henry’s personality and certain features of houses.” Answer Choice B is incorrect because these words don’t refer to specific features of Lamb House, and Henry never makes the point that it is these features in particular that attract him. Answer Choice C is incorrect because these words aren’t used to single out Lamb House from other houses in Rye. Answer Choice D is incorrect because these words are metaphors for psychological features, but not pessimism, and it’s unclear to describe Henry as being pessimistic at this point in the Passage.

How to solve this? The Question asks, “The words ‘shell,’ ‘exposure,’ ‘facade,’ and ‘frontage’ in Lines 50-52 primarily serve to [...]?”

Page 257: SAT Full Reading Explanations (QAS 9-19) · SAT Full Reading Explanations (QAS 9-19) SAT #9 Reading 1 1.R.9 Answer Choice C is the correct answer because Lines 19-22 say, “Moths

Reading 2

Passage Outline

World Development Report 2015: Mind, Society, and Behavior

● a water tunnel collapsed in Bogota, causing a public emergency of a shortage of water. the city responded by warning people of the threat. (paragraph 1)

● however, this intervention didn’t work. the city’s strategy actually increased water consumption. (paragraph 2)

● so, the government changed its strategy. they placed stickers around faucets to remind people of the crisis, and daily reports were issued about the situation. (paragraph 3)

● the city launched entertaining campaigns to teach people more about water conservation. (paragraph 4)

● the city also publicly rewarded houses that conserved the most water and punished households or businesses that used an excessive amount. (paragraph 5)

● the assumption behind this new campaign was that people would conserve water if there were greater social rewards and punishments involved. this strategy worked and water consumption remained low. (paragraph 6)

11.R.18

Answer Choice D is the correct answer because the Passage primarily concerns the successful response of the city of Bogota to a critical water shortage, which most closely matches Answer Choice D, which says that the purpose is to “discuss an effective city initiative to address a critical situation.” Answer Choice A is incorrect because the Passage doesn’t look at the reasons behind why Bogota experienced such an environmental challenge. Answer Choice B is incorrect because the Passage looks at Bogota’s response to one emergency and not to two. Answer Choice C is incorrect because the Passage doesn’t talk about technology that was used to help the city cope with its difficulty, but instead looks at the systems of rewards and punishments it uses.

How to solve this? The Question asks, “The main purpose of the passage is to [...]?”

Page 258: SAT Full Reading Explanations (QAS 9-19) · SAT Full Reading Explanations (QAS 9-19) SAT #9 Reading 1 1.R.9 Answer Choice C is the correct answer because Lines 19-22 say, “Moths

12.R.18

Answer Choice A is the correct answer because Paragraph 1 primarily focuses on the problem of the collapse of a water tunnel and the problem that ensues, Paragraph 2 focuses on a failed response to the crisis, and Paragraphs 3-6 discuss the successful response to the crisis. This pattern most closely matches Answer Choice A, which says the structure of the passage is that, “a complex problem is described, a failed attempt to resolve that problem is summarized, and then the details of a successful resolution are presented.” Answer Choice B is incorrect because the water shortage is not presented as a long-standing dilemma, two proposals for a solution are not presented at the same time but one after the other and the Passage doesn’t look at a study that compares the costs of the two options. Answer Choice C is incorrect because the water shortage isn’t presented as a recurrent conflict, and the Passage doesn’t look at two separate solutions to the problem where one is popular and the other seemingly outdated. Answer Choice D is incorrect because the Passage doesn’t analyze the obstacles to solving the problem of the crisis and doesn’t present an argument for why the program can be articulated.

How to solve this? The Question asks, “Which choice best describes the overall structure of the passage?”

13.R.18

Answer Choice D is the correct answer because Lines 27-29 say, “The mayor himself appeared in a TV ad taking a shower, explaining how the tap could be turned off while soaping.” This example is the only one to come in the paragraph that describes the city launching “engaging and entertaining campaigns to teach individuals the most effective techniques for household water conservation.” So, unfortunately, while not the most creative act in the world, it’s really our best bet. Answer Choice A is incorrect because these Lines only describe the initial collapse of the water tunnel and the city’s first, incorrect strategy. Answer Choice B is incorrect because these Lines only explain the assumption that undergird the city’s unsuccessful first method of intervention. Answer Choice C is incorrect because, while this example is part of the successful campaign its obviously way less creative than watching the mayor shower.

How to solve this? The Question asks, “Which choice best supports the claim that creative tactics were employed to encourage people to conserve water?” To solve this [...].

Page 259: SAT Full Reading Explanations (QAS 9-19) · SAT Full Reading Explanations (QAS 9-19) SAT #9 Reading 1 1.R.9 Answer Choice C is the correct answer because Lines 19-22 say, “Moths

14.R.18

Answer Choice B is the correct answer because Lines 12-15 say, “Many people did not change their behavior because they did not think they could make a difference and did not know which steps were most important.” These Lines tell us most clearly that the first intervention failed because citizens didn’t understand how to respond to the crisis, which most closely matches Answer Choice B which says that the first intervention failed to “explain to residents how to make the best use of the new information.” Answer Choice A is incorrect because the Passage doesn’t indicate that the conservation measures weren’t applied consistently across all neighborhoods. Answer Choice C is incorrect because [it less clearly matches the appropriate text, even though it's kind of true that the lack of a mechanism for demonstrating caused the first one to fail]. Answer Choice D is incorrect because the Passage doesn’t indicate that the city warned residents about the long-term dangers of the water crisis, but says that the residents were unsure of what to do with this knowledge.

How to solve this? The Question asks, “The passage implies that the city’s first attempt to address the water shortage was inadequate in that it failed to [...]?”

15.R.18

Answer Choice A is the correct answer because the word “steps” in Line 12 is used to refer to actions or behaviors, which most closely matches “measures.” Answer Choice B is incorrect because “strides” could either mean actual long steps or major advances in progress, neither of which closely matches steps. Answer Choice C is incorrect because “steps” doesn’t mean literal “stair.” Answer Choice D is incorrect because the Lines don’t mean that people didn’t understand what “phases” or windows or progress were most important.

● Many people did not change their behavior because they did not think they could make a difference and did not know which steps were most important.

● Many people did not change their behavior because they did not think they could make a difference and did not know which measures were most important.

How to solve this? The Question asks, “As used in line 12, ‘steps’ most nearly means [...]?”

Page 260: SAT Full Reading Explanations (QAS 9-19) · SAT Full Reading Explanations (QAS 9-19) SAT #9 Reading 1 1.R.9 Answer Choice C is the correct answer because Lines 19-22 say, “Moths

16.R.18

Answer Choice A is the correct answer because Lines 24-25 say, “The stickers made the need to conserve water at all times salient,” which most closely matches Answer Choice A which says that the purpose of the stickers was to “remind people of the ongoing need to avoid wasting water.” Answer Choice B is incorrect because the Passage doesn’t suggest that the stickers were used to warn people about another possible crisis. Answer Choice C is incorrect because the Passage doesn’t indicate that the stickers were used as a means to inspire pride. Answer Choice D is incorrect because the Passage doesn’t suggest that the stickers rewarded people for water conservation, which is something the posters mentioned later did.

How to solve this? The Question asks, “According to the passage, the purpose of the stickers distributed by the Bogota city government was to [...]?”

17.R.18

Answer Choice B is the correct answer because the word “cuts” in Line 38 is used to mean that squanderers had to go through more days with less water, most closely matching “reductions.” Answer Choice A is incorrect it doesn’t make sense to say that squanderers were subject to an extra day of water channels. Answer Choice C is incorrect because [...].

● While the sanctions were minor — squanderers had to participate in a water-saving workshop and were subject to an extra day of water cuts — they were nevertheless effective because they targeted highly visible actors.

● While the sanctions were minor — squanderers had to participate in a water-saving workshop and were subject to an extra day of water reductions — they were nevertheless effective because they targeted highly visible actors.

How to solve this? The Question asks, “As used line 38, ‘cuts’ most nearly means [...]?”

Page 261: SAT Full Reading Explanations (QAS 9-19) · SAT Full Reading Explanations (QAS 9-19) SAT #9 Reading 1 1.R.9 Answer Choice C is the correct answer because Lines 19-22 say, “Moths

18.R.18 & 19.R.18

Answer Choices D and C are the correct answers because Lines 41-44 say, “The assumption underlying the new strategy was that conservation would improve if the city created a greater scope for social rewards and punishments that helped to reassure people that achieving the public good — continued access to water — was likely.” These Lines most clearly tell us a conclusion that can be drawn from Bogota’s campaign, and make the point that people made changes when they that an underlying systems of rewards and punishments would help regulate them. This most closely matches Answer Choice D, which says that the conclusion best drawn from the campaign is that people are “willing to adopt new behaviors if adequate incentives are provided.” For Question 19, Answer Choice A is incorrect because these Lines only tell us a feature of the city’s overall campaign and not a more general conclusion that can be drawn from it. Answer Choice B is incorrect because these Lines also only tell us about a more specific feature of the city’s campaign, without providing a conclusion about humans in general. Answer Choice D is incorrect because these Lines tell us one of the consequences of the effective campaign, but not a general conclusion about human nature. For Question 18, Answer Choice A is incorrect because neither the Passage nor our Texts make the specific case that praise is a more powerful incentive than punishment and highlight both equally in the Passage. Answer Choice B is incorrect because neither the Passage nor the Texts talk about people’s interest in learning new subjects if the information is available, but focus more specifically on how people change their actions. Answer Choice C is incorrect because the Passage suggests that people are not good on their own at educating each other about emergencies, but require an underlying system like the City of Bogota’s to help motivate them to action.

How to solve this? Question 18 asks, “The conclusion best drawn from the results of the Bogota city government’s campaign is that people are [...]?” Because this is a Paired Question, to solve it [...].

20.R.18

Answer Choice B is the correct answer because in the figure the X-axis measures the year and the y-axis measures the Water Demand in cubic meters. For the year 2005, the corresponding point on the graph rises to about halfway between 14 and 14.5, making 14.25 the best answer. Answer Choices A, C and D are incorrect because [...].

How to solve this? The Question asks, “According to the figure, water demand in Bogota in 2005 was approximately [...]?”

Page 262: SAT Full Reading Explanations (QAS 9-19) · SAT Full Reading Explanations (QAS 9-19) SAT #9 Reading 1 1.R.9 Answer Choice C is the correct answer because Lines 19-22 say, “Moths

21.R.18

Answer Choice C is the correct answer because from 2002-2004 the graph shows a steady decline from just under 15 to just over 14, making it correct to say it “declined steadily from 2002-2004.” Answer Choice A is incorrect because from 1999 to 2002 the graph goes from just under 15, makes a small dip and then comes back to the same level, and doesn’t “drop considerably.” Answer Choice B is incorrect because from 2000 to 2001 the graph only barely moves up, making it incorrect to say that it rose dramatically. Answer Choice D is incorrect because the graph rises from about 14.75 to over 15 between 2007 and 2009, making it incorrect to say that it “remained stable.”

How to solve this? The Question asks, “According to the data presented in the figure, water demand in Bogota is best described as having [...]?” To solve this [...].

Reading 3

Passage Outline

“Razzle Dazzle ‘Em”

● it used to be thought that camouflage worked by matching to surroundings. then Thayer argued that animals with obvious markings like a zebra used camouflage by disrupting their outlines and edges, a feature predators look for. (paragraph 1)

● this hypothesis, however, was untested until 2005, when two scientists used paper moths to test the claim and found it to be true. (paragraph 2)

● another scientist asked volunteers to track moths on a computer and also showed that Thayer was right and people use outlines to track moths. (paragraph 3)

● however, because humans don’t track moths, experiments in the wild are being conducted. (paragraph 4)

● people can get better at detecting animals with camouflage if they focus less on outline and more on markings. (paragraph 5)

● its unclear whether non-human predators can do this, but if so, it would show that this kind of camouflage only works in certain conditions.

22.R.18

Answer Choice C is the correct answer because the Passage primarily looks at several studies that test and elaborate on the disruptive camouflage hypothesis, matching the general Answer Choice C which says that the purpose of the passage is to “discuss the implications of several scientific studies concerning disruptive camouflage.” Answer Choice A is incorrect because the Passage doesn’t focus on how predators still hunt animals using disruptive camouflage and only hints at a possibility in the last paragraph. Answer Choice B is incorrect because the Passage doesn’t say that disruptive camouflage helps scientists to track the wing coloration patterns in moths. Answer Choice D is incorrect because the Passage doesn’t attempt to defend any theory, like that of Thayer’s, from critics.

How to solve this? The Question asks, “The main purpose of the passage is to […]?”

Page 263: SAT Full Reading Explanations (QAS 9-19) · SAT Full Reading Explanations (QAS 9-19) SAT #9 Reading 1 1.R.9 Answer Choice C is the correct answer because Lines 19-22 say, “Moths

23.R.18

Answer Choice C is the correct answer because Lines 45-48 say, “But if predators can learn to see through disruptive camouflage, it would suggest that this concealment strategy is more likely to evolve in prey that face short-lived or generalist predators than long-lived or specialist ones.” These Lines, along with others that the author endorses, tell us most clearly that the author believes that disruptive camouflage is effective against some predators, matching the more general Answer Choice C which says that this camouflage is “a viable defense against particular predators.” Answer Choice A is incorrect because disruptive camouflage works most effectively when a predator scans for outlines, and isn’t neutralized by it. Answer Choice B is incorrect because the Passage doesn’t indicate that this form of camouflage has become obsolete but only suggests that it may be effective only in particular cases. Answer Choice D is incorrect because, although the Passage does indicate that disruptive camouflage is effective when it uses high contrast markings, it also makes the case that it is less effective when that contrast exceeds the surrounding environment.

How to solve this? The Question asks, “The author’s central claim about disruptive camouflage is that it is [...]?” To solve this [...].

24.R.18

Answer Choice A is the correct answer because the sequence of scientists and studies mentioned in the passage begins with (1) Thayer making the hypothesis about disruptive camouflage, to (2) Cuthill and Stevens testing this with paper moths, to (3) Webster showing more specifically that its effectiveness does come from breaking up outlines, to (4) a final in-field study. Each one of these studies progressively builds on the other, most closely matching Answer Choice A which says that the passage shows how scientists “increased their understanding of a particular camouflage strategy.” Answer Choice B is incorrect because the Passage doesn’t show the pattern of scientists accepting and then rejecting different possible camouflage mechanisms. Answer Choice C is incorrect because the Passage doesn’t show scientists eventually losing their interest in camouflage research. Answer Choice D is incorrect because, although humans were brought into the series of experiments midway through, the Passage doesn’t emphasize their increasing inclusion and the final study doesn’t contain humans at all.

How to solve this? The Question asks, “The passage provides a chronological account of how scientists [...]?” To solve this [...].

Page 264: SAT Full Reading Explanations (QAS 9-19) · SAT Full Reading Explanations (QAS 9-19) SAT #9 Reading 1 1.R.9 Answer Choice C is the correct answer because Lines 19-22 say, “Moths

25.R.18 & 26.R.18

Answer Choices A and B are the correct answers because Lines 9-12 say, “But although disruptive camouflage was cited in countless textbooks, it remained largely untested until 2005, when Innes Cuthill, Martin Stevens and their colleagues at the University of Bristol, United Kingdom, devised an experiment using fake moths made from paper triangles.” These Lines most clearly tell us about a relationship between Thayer’s work and Cuthill and Steven’s work, suggesting that Cuthill and Stevens were the first to provide experimental data about Thayer’s hypothesis. This most closely matches Answer Choice A which says that “Cuthill and Stevens provided empirical support for Thayer’s theory.” For Question 26, Answer Choice A is incorrect because these Lines only concern Thayer, and don’t tell us anything about the relationship between him and Cuthill and Stevens. Answer Choice C is incorrect because these Lines only expand on the findings of Cuthill and Stevens but don’t specifically make a point about their relationship to Thayer’s work. Answer Choice D is incorrect because it also continues to elaborate on Cuthill and Steven’s findings without referencing Thayer. For Question 25, Answer Choice B is incorrect because neither the Passage nor the Texts tell us that Cuthill and Stevens worked on the logic of Thayer’s assumptions, but only that they were the first to test his initial hypothesis. Answer Choice C is incorrect because the Passage tells us that Cuthill and Stevens were the first to look at Thayer’s hypothesis experimentally, so it wouldn’t make sense to say that they continued his experimental procedure. Answer Choice D is incorrect because neither the Passage nor the Texts say that Cuthill and Stevens looked at an idea of Thayer’s that was largely overlooked.

How to solve this? Question 25 asks, “Which choice best states the relationship between Cuthill and Stevens’s work and Thayer’s work [...]?”

27.R.18

Answer Choice B is the correct answer because Lines 9-10 say, “But although disruptive camouflage was cited in countless textbooks, it remained largely untested until 2005 [...].” In these Lines, “countless” is used to emphasize the fact Thayer’s theory was generally accepted as true despite not being empirically tested, which most closely matches Answer Choice B which says that the author uses the word to “emphasize the widespread and long-standing acceptance of an untested theory of camouflage.” Answer Choice A is incorrect because the Lines aren’t emphasizing the different kinds of textbooks or variety of textbooks that cited Thayer’s theory. Answer Choice C is incorrect because the word “countless” refers to textbooks and not to camouflage patterns. Answer Choice D is incorrect because the Lines aren’t emphasizing the impracticality of studying camouflage, but the fact that this widely accepted theory was yet to be verified.

How to solve this? The Question asks, “The author most likely uses the word ‘countless’ to [...]?” To solve this [...].

Page 265: SAT Full Reading Explanations (QAS 9-19) · SAT Full Reading Explanations (QAS 9-19) SAT #9 Reading 1 1.R.9 Answer Choice C is the correct answer because Lines 19-22 say, “Moths

28.R.18

Answer Choice C is the correct answer because the word “uniform” in Line 14 is used to depict birds who have the same coloration across their entire body, which most closely matches “unvarying.” Answer Choice A is incorrect because the Lines aren’t emphasizing that the colors can’t be distinguished from one another. Answer Choice B is incorrect because the Lines aren’t emphasizing the simplicity of the colors. And Answer Choice D is incorrect because it’s unclear what it would mean for the Lines to be emphasizing the symmetry of the colors.

● By pinning them to oak trees, the researchers found that “moths” with black markings on their edges were less likely to be attacked by birds than those with central markings or uniform colors.

● By pinning them to oak trees, the researchers found that “moths” with black markings on their edges were less likely to be attacked by birds than those with central markings or unvarying colors.

How to solve this? The Question asks, “As used in line 14, ‘uniform’ most nearly means [...]?”

29.R.18 & 30.R.18

Answer Choices A and D are the correct answers because Lines 44-45 say, “Whether non-human predators adopt the same tactic is hard to say. They may not even see camouflage markings in the same way that we do.” These Lines tell us most clearly that humans and nonhumans probably differ in ability to “see camouflage markings in the same way.” This most closely matches Answer Choice A, which says that they differ in their ability to “perceive visual patterns.” For Question 30, Answer Choice A is incorrect because these Lines only show that volunteers had trouble detecting moths with disrupted edges, and don’t show any difference between how them and non-human predators. Answer Choice B is incorrect because these Lines only say that the question about whether non-human predators react to disruptive coloring in the same way as humans or not has yet to be answered, and fails to show a definitive difference. Answer Choice C is incorrect because it only shows how human volunteers can gradually get better at detecting moths with disruptive camouflage, but doesn’t make any comparison between them and non-human predators. For Question 29, only Answer Choice A matches our Text. Answer Choices B, C and D are incorrect because neither the Passage nor the Texts talk about a difference in the perception of small motions, ability to locate ground-nesting birds or hear high-pitched sounds.

How to solve this? Question 29 asks, “In the passage, the author suggests that humans and nonhuman predators may differ in their ability to [...]?”

31.R.18

NA

Page 266: SAT Full Reading Explanations (QAS 9-19) · SAT Full Reading Explanations (QAS 9-19) SAT #9 Reading 1 1.R.9 Answer Choice C is the correct answer because Lines 19-22 say, “Moths

32.R.18

Answer Choice A is the correct answer because Figure 2 measures number of edge patches on the X-axis and mean search time in seconds on the Y-axis. The point that corresponds to five edge patches on the x-axis rises up to two seconds on the Y-axis, matching Answer Choice A. Answer Choices B, C and D are incorrect because [...].

Reading 4

Passage Outline

On the Adoption of the Universal Declaration of Human Rights

● Important to remember what the Declaration of Human Rights is. It is not a treaty, statement of law, or legal obligation. (paragraph 1)

● It is an important event. (paragraph 2) ● It is significant that so many countries have signed and agreed to peace and common

human rights. (paragraph 3) ● A quote. Humans are moral creatures with a desire for freedom. Having freedom is the

mark of a civilization. (paragraph 4) ● There is more work to be done, but this a great step forward. (paragraph 5)

The Case against Human Rights

● Many people think that international human rights’ laws, like the Declaration, were great achievements. It’s said that they provide people with protection from the state. (paragraph 1)

● Yet, there is a lot of evidence that countries continue to violate these rights. (paragraph 2)

● Human rights law has failed because of assumptions like that human rights were universal and could be forced on countries. (paragraph 3)

● The Declaration of Human Rights was the first major milestone. (paragraph 4) ● The weaknesses of human rights law in general were in the Declaration too. It was not

binding, it wasn’t ratified or made official and it was vague. (paragraph 5)

Page 267: SAT Full Reading Explanations (QAS 9-19) · SAT Full Reading Explanations (QAS 9-19) SAT #9 Reading 1 1.R.9 Answer Choice C is the correct answer because Lines 19-22 say, “Moths

33.R.18

Answer Choice B is the correct answer because Lines 11-13 say, “At a time when there are so many issues on which we find it difficult to reach a common basis of agreement, it is a significant fact that 58 states have found such a large measure of agreement in the complex field of human rights.” These Lines tell us most clearly that the development of the Declaration is significant and “marks a major accomplishment.” Answer Choice A is incorrect because these Lines only explain what the Declaration is not, and don’t make any comment on its significance. Answer Choice C is incorrect because they explain what the foundation for the Declaration is but don’t tell us that it marks a significant moment. Answer Choice D is incorrect because these Lines only explain that more work remains to be done.

How to solve this? The Question asks, “Which choice from Passage 1 best supports the idea that in Roosevelt’s view, the cooperation of various nations in the development of the Universal Declaration of Human Rights marks a major accomplishment?” To solve this [...]. 34.R.18

Answer Choice A is the correct answer because the word “testimony” in Line 14 emphasizes the fact that the signing of the Declaration should be taken as evidence for, or “public affirmation” of the common aspiration of the United Nations. Answer Choice B is incorrect because the word “testimony” isn’t referring to something like an official legal record. Answer Choice C is incorrect because the word “testimony” refers to evidence in general about a common aspiration and not a personal account. Answer Choice D is incorrect because “testimony” isn’t used to refer to any kind of divine rule.

● This must be taken as testimony of our common aspiration first voiced in the Charter of the United Nations to lift men everywhere to a higher standard of life and to a greater enjoyment of freedom.

● This must be taken as public affirmation of our common aspiration first voiced in the Charter of the United Nations to lift men everywhere to a higher standard of life and to a greater enjoyment of freedom.

How to solve this? The Question asks, “As used in line 14, “testimony” most nearly means [...]?” To solve this [...].

35.R.18

Answer Choice D is the correct answer because in Paragraph 4, the author of Passage 1 introduces a quote from Murray that makes the case that humans are moral creatures and that freedom is the defining feature of civilization. Immediately after this quote, in Lines 27-29, Roosevelt says, “This Declaration is based upon the spiritual fact that man must have freedom in which to develop his full stature and through common effort to raise the level of human dignity.” This tells us that Roosevelt mentions Murray to support her case that freedom is essential for civilization and human dignity, and most closely matches Answer Choice D, which says that she brings up the quote to “introduce her claim that certain freedoms are required for fulfilling humanity’s full potential.” Answer Choice A is incorrect because the Murray quote doesn’t focus on different rights for men and women. Answer Choice B is incorrect because the

Page 268: SAT Full Reading Explanations (QAS 9-19) · SAT Full Reading Explanations (QAS 9-19) SAT #9 Reading 1 1.R.9 Answer Choice C is the correct answer because Lines 19-22 say, “Moths

Murray quote and Passage 1 don’t focus on the impact of technology on freedom. Answer Choice C is incorrect because Roosevelt doesn’t make the case that individuals should use their freedom to serve the state, but merely that freedom is necessary for a fully developed nation.

How to solve this? The Question asks, “In Passage 1, Roosevelt uses the quotation from Gladstone Murray primarily to [...]?” To solve this [...].

36.R.18

Answer Choice D is the correct answer because Lines [...] say, “Because human rights law gives rights to all people regardless of nationality, it deprives governments of their traditional riposte when foreigners criticise them for abusing their citizens — namely “sovereignty” (which is law-speak for “none of your business”).” Here, the author takes both a negative and [slightly humorous / casual] stance towards the term, turning an imposing, legal sounding word like “sovereignty” into how its essentially used, the casual “none of your business.” This most closely matches Answer Choice D, which says that the use of the phrase creates a “wry tone that conveys a disapproving view of how a term has been used.” Answer Choice A is incorrect because the phrase doesn’t create a thoughtful tone and isn’t used to highlight how the term has changed over time. Answer Choice B is incorrect because there’s nothing in the Lines to suggest a “suspicious tone” or to convey doubt about how the term has changed over time. Answer Choice C is incorrect because it’s too strong of claim to say that the term is “comical” compared to the more general “wry.”

How to solve this? The Question asks, “In Passage 2, Posner’s use of the phrase “none of your business” (line 47) primarily serves to create a [...]?” To solve this [...].

37.R.18

Answer Choice B is the correct answer because Lines 52-54 say, “Although the modern notion of human rights emerged during the 18th century, it was on December 10, 1948, that the story began in earnest, with the adoption of the Universal Declaration of Human Rights by the UN general assembly.” These Lines tell us most clearly that the way we understand human rights today, emerged in the 18th century, which most closely matches Answer Choice B. Answer Choices A, C and D are incorrect because none of them [...].

How to solve this? The Question asks, “According to Passage 2, when did the idea of human rights as we now understand them first begin to develop [...]?”

Page 269: SAT Full Reading Explanations (QAS 9-19) · SAT Full Reading Explanations (QAS 9-19) SAT #9 Reading 1 1.R.9 Answer Choice C is the correct answer because Lines 19-22 say, “Moths

38.R.18 & 39.R.18

Answer Choices C and D are the correct answers because Lines 61-62 say, “Moreover, the rights were described in vague, aspirational terms, which could be interpreted in multiple ways.” These Lines tell us that one flaw of the Declaration was that its term weren’t described clearly or precisely, and most closely matches Answer Choice C which says that the one flaw of the Declaration is that it “presents the rights such that they lack clear and precise applications.” For Question 39, Answer Choice A is incorrect because these Lines only tell us that there were weaknesses without describing what those specific weaknesses were. Answer Choice B is incorrect because these Lines tell us that the treaty failed because it wasn’t legally binding, which doesn’t match any of the Answer Choices from Question 38. Answer Choice C is incorrect because these Lines tell us that the Declaration was never ratified, which also doesn’t match any answer choices from the previous question. For Question 38, Answer Choice A is incorrect because neither the Texts nor Passage 2 talk about how the Declaration exploited humanity’s fear of war. Answer Choice B is incorrect because neither the Texts nor the Passage talk about how the Declaration focused too narrowly on specific violations. Answer Choice D is incorrect because neither the Passage nor the Texts talk about how the Declaration excluded certain other values from the United Nations Charter.

How to solve this? The Question asks, “Passage 2 most strongly suggests that a significant flaw of the Universal Declaration of Human Rights is that it [...]?”

40.R.18

Answer Choice B is the correct answer because in Lines 3-5, Roosevelt says of the Declaration that, “It is not a treaty; it is not an international agreement. It is not and does not purport to be a statement of law or of legal obligation.” In Lines 58-59, Posner says “The universal declaration was not a treaty in the formal sense; no one at the time believed that it created legally binding obligations.” Both these Lines tell us that the Passages clarify the nature of the Universal Declaration of Human Rights by stating that it was not a legal document, matching Answer Choice B which says that they distinguished it “from legally binding documents.” Answer Choice A is incorrect because neither Passage compares the Declaration to notable agreements from the past. Answer Choice C is incorrect because only Passage 1 clearly makes the case that the Declaration has its basis in key democratic principles. Answer Choice D is incorrect because only Passage 1 really emphasizes its wide adoption by diverse governments, while Passage 2 focuses more on the few countries that haven’t adopted it.

How to solve this? The Question asks, “Both passages clarify the nature of the Universal Declaration of Human Rights by [...]?” To solve this [...].

Page 270: SAT Full Reading Explanations (QAS 9-19) · SAT Full Reading Explanations (QAS 9-19) SAT #9 Reading 1 1.R.9 Answer Choice C is the correct answer because Lines 19-22 say, “Moths

41.R.18

Answer Choice C is the correct answer because Roosevelt in Passage 1 expresses concern for those living in oppressed conditions, mentioning the “flagrant violations of human rights by Nazi and Fascist countries” and arguing that “man must have freedom in which to develop his full stature and through common effort to raise the level of human dignity.” In Passage 2, Posner expresses concern for a number of issues in Lines 40-45, like torture, authoritarianism, subjection of women and child labor. This tells us most clearly that both passages reflect a concern for the “welfare of people living under oppressive social and political circumstances,” matching Answer Choice C. Answer Choice A is incorrect because neither Passage talks about how United Nations officials will react to new human rights initiatives. Answer Choice B is incorrect because only Passage 2 talks about how some of the underlying assumptions for the idea of universal human rights were incorrect or unfounded. Answer Choice D is incorrect because neither Passage talks about the dangers of having to step in for citizens living under authoritarian rule. How to solve this? The Question asks, “In their discussions of human rights, both passages reflect an underlying concern with the [...]?”

42.R.18

Answer Choice A is the correct answer because Passage 1 speaks in primarily positive terms about human rights law and only the first paragraph from Passage 2 does the same, moreso highlighting the arguments in favor of human rights than giving the author’s own opinion. Answer Choice B is incorrect because the second paragraph of Passage 2 primarily gives examples of ways in which universal human rights has failed, which wouldn’t match the view from Passage 1. Answer Choice C is incorrect because the third paragraph continues to make the case that universal human rights hasn’t helped people or improved their well being. Answer Choice D is incorrect because this paragraph explains how the underlying assumptions of universal human rights were incorrect or unfounded, a belief that Roosevelt wouldn’t hold.

How to solve this? The Question asks, “Which paragraph of Passage 2 presents a view of human rights law that is most consistent with Roosevelt’s view of human rights in Passage 1?” To solve this [...].

Page 271: SAT Full Reading Explanations (QAS 9-19) · SAT Full Reading Explanations (QAS 9-19) SAT #9 Reading 1 1.R.9 Answer Choice C is the correct answer because Lines 19-22 say, “Moths

Reading 5

Passage Outline

Can Sea Monkeys Stir the Sea?

● many natural forces move water in the ocean, pushing heat down and bringing up nutrients. but some sea creatures, like sea monkeys, may also help move the water. two scientists decided to test this. (paragraph 1)

● the methodology of the experiment. the researchers used lasers to attract and move the sea monkeys, taking care not to allow for interference and tracking them. (paragraph 2)

● the videos showed that the creatures caused big swirls, suggesting that they may be effective at mixing. (paragraph 3)

● if`organisms convert only part of the energy taken from the sun to mixing, the effect could be large. (paragraph 4)

● quote from a scientist who is positive about the study and methodology. (paragraph 5) ● some scientists disagree with the findings. quote from another scientist who has a

disagreeable take. (paragraph 6) ● the fact that there are different layers to the sea may affect the mixing. another study

measured water fleas in layered water and showed that this cancelled the mixing effect. (paragraph 7)

● the original scientists plan on testing the sea monkeys in layered water to see if the effect still holds. (paragraph 8)

43.R.18

Answer Choice C is the correct answer because the Passage primarily looks at a single experiment that looks at the mixing effect of sea monkeys and includes commentary on the findings from other scientists. The Passage is called “Can Sea Monkeys Stir the Sea?” and the first paragraph begins by talking about the mixing effect of some sea-creatures and then leads into the specific experiment with sea monkeys. This most closely matches Answer Choice C which is the most general answer choice and says that the purpose of the passage is to “discuss experimental research on a possible example of biomixing.” Answer Choice A is incorrect because “field observations” refers to studies made directly in the ocean, while the experiment in the Passage takes place in a lab. Answer Choice B is incorrect because scientists don’t come to any consensus or agreement in the Passage, but continue to disagree about the results. Answer Choice D is incorrect because the Passage doesn’t try to bring together two contradictory findings.

How to solve this? The Question asks, “The main purpose of the passage is to [...]?”

Page 272: SAT Full Reading Explanations (QAS 9-19) · SAT Full Reading Explanations (QAS 9-19) SAT #9 Reading 1 1.R.9 Answer Choice C is the correct answer because Lines 19-22 say, “Moths

44.R.18

Answer Choice B is the correct answer because Lines 18-21 say, “Dabiri and Wilhelmus used blue and green lasers to induce thousands of 5-millimeter-long brine shrimp to “migrate” to and from the bottom of a 1.2-meter-deep tank,” and Lines 21-23 say, “The researchers shone the blue laser into the tank and moved it slowly up and down to control the crustaceans’ vertical movements.” Both of these Lines tell us that the green and blue lasers were used to guide and move the brine shrimp around the tank, which most closely matches Answer Choice B which tells us that the purpose of the discussion is to “describe how the researchers attempted to guide the motions of the brine shrimp. Answer Choice A is incorrect because the Passage tells us that it was the red lasers that allowed the scientists to see the swirls made by the brine shrimp (Liness 27-31). Answer Choice C is incorrect because the Passage doesn’t talk about a difference between the brine shrimp used in the study and those from the ocean. Answer Choice D is incorrect because the Passage doesn’t look at the overall migration pattern of brine shrimp when it talks about the blue and green lasers.

How to solve this? The Question asks, “The main purpose of the discussion of blue and green lasers is to [...]?”

45.R.18 & 46.R.18

Answer Choices C and B are the correct answers because Lines 18-21 say, “The tank’s solid walls could strongly affect the flow patterns generated by the shrimp as they swam, so the researchers kept the shrimp away from the edges of the tank by shining the green laser beam directly down into the center.” These Lines tell us most clearly something that the researchers tried to avoid that could disrupt their experiment, and focus on the impact that the tank’s walls could have on the movement of the brine shrimp. This most closely matches Answer Choice C which says that the researchers tried to exclude the possibility that, “the apparatus in which the brine shrimp were held influenced the results of the experiment.” For Question 46, Answer Choice A is incorrect because these Lines only explain how the researchers used the blue light to move the brine shrimp, but doesn’t talk about measures they took to avoid interference. Answer Choice C is incorrect because these Lines only tell us what the researchers did to help them visualize the brine shrimp, not anything about what they did to try to preserve the experiment. Answer Choice D is incorrect because these Lines tell us about the results of the experiment and not about something the researchers did in the set up. For Question 45, Answer Choice A is incorrect because neither the Passage nor the Texts talk about the difference between the duration of the experiment versus the duration of the migration. Answer Choice B is incorrect because neither the Passage nor the Texts talk about the brine shrimps’ adaptation to their experimental conditions and how this might affect the data. Answer Choice D is incorrect because the Passage tells us that the red light “doesn’t seem to affect the shrimps’ behavior” (Lines 30-31).

How to solve this? Question 45 asks, “The passage most strongly suggests that in designing their experiment, Dabiri and Wilhelmus tried to exclude the possibility that [...]?”

Page 273: SAT Full Reading Explanations (QAS 9-19) · SAT Full Reading Explanations (QAS 9-19) SAT #9 Reading 1 1.R.9 Answer Choice C is the correct answer because Lines 19-22 say, “Moths

47.R.18

Answer Choice C is the correct answer because the word “captured” in Line 25 is used to refer to how the team used video to record the motion of the sea monkeys. Answer Choices A and B are incorrect because the word “captured” isn’t used in the literal sense of taken and kept, like either “seized” or “imprisoned.” Answer Choice D is incorrect because the word “captured” in the Lines doesn’t mean attracted.

● The team’s high-speed videos of the teeming, laser-lit migrations captured images of swirls much larger than that creatures themselves, which resulted from the interactions of smaller flows created by individuals.

● The team’s high-speed videos of the teeming, laser-lit migrations recorded images of swirls much larger than that creatures themselves, which resulted from the interactions of smaller flows created by individuals.

How to solve this? The Question asks, “As used in Line 25, ‘captured’ most nearly means [...]?” To solve this [...].

48.R.18

Answer Choice A is the correct answer because Noss, in Lines [...], says, “This is a really innovative experimental setup that provides a nice illustration of flow velocities,” while Yen, in Lines [...], says “I like the idea of using [the shrimps’] behavior to lure them to the camera.” Both of these statements express a positive opinion about the study expressed, especially the methodology it uses, which most closely matches Answer Choice A which says that the Lines serve to, “provide expert evaluations of the experimental methods used by Dabiri and Wilhelmus.” Answer Choice B is incorrect because neither of the scientists quoted here mention reservations or doubts that they have about the experiment. Answer Choice C is incorrect these two quotes don’t illustrate opposite sides of a debate, but both speak positively about the methodology of the experiment discussed. Answer Choice D is incorrect because these views don’t present a scholarly consensus and aren’t challenged by Dabiri and Wilhelmus.

How to solve this? The Question asks, “The quotations from Noss and Yen primarily serve to [...]?”

Page 274: SAT Full Reading Explanations (QAS 9-19) · SAT Full Reading Explanations (QAS 9-19) SAT #9 Reading 1 1.R.9 Answer Choice C is the correct answer because Lines 19-22 say, “Moths

49.R.18

Answer Choice A is the correct answer because Lines 52-55 say, “‘I wouldn’t want to say just yet that [biomixing] is important at a global scale’ solely based on a lab experiment, says Stephen Monismith, a fluid mechanicist at Stanford University.” These Lines tell us most clearly that Monosmith doesn’t think that the scientists’ lab results can be replicated in the greater environment. This most closely matches Answer Choice A, which says that Monismith is not convinced that Dabiri and Wilhelmus’s results can be “generalized to the natural environment.” Answer Choice B is incorrect because Monismith doesn’t make any claim about replication, or the ability for the same experiment to be repeated with the same results. Answer Choice C is incorrect because Monismith doesn’t make the claim that he thinks the researchers’ results won’t be verified or obtained by other researchers. Answer Choice D is incorrect because Monismith doesn’t make any claim that he doesn’t think that their findings won’t be reconciled or in sync with other findings.

How to solve this? The Question asks, “According to the passage, Monismith is not convinced that Dabiri and Wilhelmus’s results can be [...]?”

50.R.18

Answer Choice B is the correct answer because Lines 56-58 say, “‘Most of the energy [from the shrimp] probably goes into heating the water’ rather than mixing it, he says.” These Lines tell us that its likely that the sea monkeys movement contributes less to mixing than it does to simply increasing heat, an effect not suggested by Dabiri and Wilhelmus. Answer Choice A is incorrect because [its a text that was used previously in the question before it] and because it only tells us that it may be possible that [...]. Answer Choice C is incorrect because these Lines look at other researchers than Dabiri and Wilhelmus and an experiment involving water fleas, not sea monkeys. Answer Choice D is incorrect because these Lines tell us about a future study Dabiri and his colleagues will perform, and not about the fact that their previous research may have a different effect than expected.

How to solve this? The Question asks, “Which choice best supports the idea that the migration of brine shrimp may not have the effect suggested by Dabiri and Wilhelmus?” To solve this [...].

Page 275: SAT Full Reading Explanations (QAS 9-19) · SAT Full Reading Explanations (QAS 9-19) SAT #9 Reading 1 1.R.9 Answer Choice C is the correct answer because Lines 19-22 say, “Moths

51.R.18

Answer Choice D is the correct answer because the word “mildly” in the Lines mentioned is used to emphasize that the water is only somewhat stratified, which most closely matches “moderately.” Answer Choices A and B are incorrect because these are adjectives moreso used to describe humans, and its unclear how the water could be either “pleasantly” or “humbly” stratified. Answer Choice C is incorrect because its unclear what it would mean for the water to be just vaguely stratified, and the Lines are emphasizing the mildness of the stratification not that its difficult to confirm or identify.

● In that research, Noss and colleague Andreas Lorke, also of Koblenz-Landau, studied the effects of large crowds of aquatic creatures called Daphnia (commonly known as water fleas) as they migrated up and down in a tank of mildly stratified water.

● In that research, Noss and colleague Andreas Lorke, also of Koblenz-Landau, studied the effects of large crowds of aquatic creatures called Daphnia (commonly known as water fleas) as they migrated up and down in a tank of moderately stratified water.

How to solve this? The Question asks, “As used in line 53, ‘mildly’ most nearly means [...]?” To solve this [...].

52.R.18

Answer Choice D is the correct answer because the research about the water fleas is mentioned in Paragraph 6 where the guiding question is whether water stratification would “reduce the efficiency of any biomixing.” The paragraph concludes by saying in Lines [...], “As expected, the stratification squelched the biomixing generated by the swimming Daphnia, Noss says.” These Lines tell us most clearly that the study of water fleas is used as evidence for the idea water stratification could negatively affect the ability of biomixing. This most closely matches Answer Choice D, which says that the study of water fleas is used to support the idea that “water stratification reduces the likelihood of successful biomixing.” Answer Choice A is incorrect because, while this is a true claim of the Passage, this is not what the study of water fleas is intended to show. Answer Choice B is incorrect because the study of water fleas isn’t meant to show anything about the circulation of nutrients. Answer Choice C is incorrect because the discussion of the water fleas isn’t used to discuss the relative likelihood of biomixing in light versus heavy water.

How to solve this? The Question asks, “The information about the study of water fleas is provided primarily as evidence in support of the idea that [...]?”

Page 276: SAT Full Reading Explanations (QAS 9-19) · SAT Full Reading Explanations (QAS 9-19) SAT #9 Reading 1 1.R.9 Answer Choice C is the correct answer because Lines 19-22 say, “Moths

SAT #19, Full Test

Reading 1

Passage Outline

Mrs. Manstey’s View

● Mrs. Manstey spent many hours in front of her window, looking at her view. She loved the garden and yard she saw, even more than her few visitors. (paragraph 1)

● One day, her landlady came to visit her. (paragraph 2) ● Mrs. Manstey comments on the beauty of the magnolia in the next yard, something she

normally doesn’t do. Her landlady doesn’t appreciate it in the same way. (paragraphs 3-6)

● The landlady explains that Mrs. Manstey’s neighbor will build an extension on her home, right through the yard. (paragraphs 7-9)

● The extension is to be built as high as the roof, blocking Mrs. Manstey’s view. (paragraphs 10-13)

● Mrs. Manstey admires her view, knowing it will soon be blocked from her. (paragraph 14)

1.R.19

Answer Choice B is the correct answer because the word “idle” in Line 2 is used to express that Mrs. Manstey was still active while sitting in front of her window, and not “unoccupied.” Answer Choice A is incorrect because the Lines aren’t making the claim that she was engaged rather than indifferent. Answer Choice C is incorrect because the Lines aren’t making the point that Mrs. Manstey is more deep than superficial. Answer Choice D is incorrect because the Lines aren’t making the point that Mrs. Manstey has high energy, as opposed to being lethargic, but more specifically talks about her activity while sitting in front of the window.

● Mrs. Manstey, in the long hours which she spent at her window, was not idle. ● Mrs. Manstey, in the long hours which she spent at her window, was not unoccupied.

How to solve this? The Question asks, “As used in line 2, ‘idle’ most nearly means [...]?”

Page 277: SAT Full Reading Explanations (QAS 9-19) · SAT Full Reading Explanations (QAS 9-19) SAT #9 Reading 1 1.R.9 Answer Choice C is the correct answer because Lines 19-22 say, “Moths

2.R.19

Answer Choice B is the correct answer because the first paragraph primarily discusses the main character Mrs. Manstey and some of her daily habits, while the second paragraph, following Line 18, transitions into the scene where her landlady comes to the house. So, the focus of the Passage shifts from a general description of Mrs. Manstey to a specific moment in her life, and most closely matches Answer Choice B which says that the focus shifts from “a general description of the main character to a sustained narration of an episode in her life.” Answer Choice A is incorrect because, while the first paragraph could be understood as a description of Mrs. Manstey’s daily routine, the Passage doesn’t transition to a discussion of the importance of that routine. Answer Choice C is incorrect because the first paragraph doesn’t look at a dilemma which Mrs. Manstey faces, something which comes after Line 18. Answer Choice D is incorrect because the Passage after Line 18 doesn’t transition to a discussion of Mrs. Manstey attempting to develop new pastimes.

How to solve this? The Question asks, “At Line 18, the focus of the passage shifts from [...]?” To solve this [...].

3.R.19

Answer Choice C is the correct answer because Lines 28-44 show an exchange between Mrs. Manstey and her landlady where Mrs. Manstey expresses her enthusiasm and appreciation for the magnolia, and her landlady responds with relative indifference, not even remembering that there was magnolia. These Lines most clearly show the difference between how Mrs. Manstey and her landlady experience and pay attention to the yard and garden next door, and most closely match Answer Choice C which says that the lines serve to “illustrate the marked divergence in the attitudes of Mrs. Manstey and Mrs. Sampson toward their immediate surroundings.” Answer Choice A is incorrect because the Lines don’t mark any difference between how important an event is with how Mrs. Manstey looks at it. Answer Choice B is incorrect because the difference that these Lines highlight between the two women is in their appreciation of the garden, not in how they address a sensitive issue. Answer Choice D is incorrect because the Lines don’t illustrate any kind of realization that Mrs. Manstey has.

How to solve this? The Question asks, “The exchange between Mrs. Manstey and Mrs. Sampson regarding the magnolia (lines 28-44) serves primarily to [...]?” To solve this [...].

Page 278: SAT Full Reading Explanations (QAS 9-19) · SAT Full Reading Explanations (QAS 9-19) SAT #9 Reading 1 1.R.9 Answer Choice C is the correct answer because Lines 19-22 say, “Moths

4.R.19

Answer Choice C is the correct answer because the word “absorbing” in Line 31 is used to describe the absolute importance that Mrs. Manstey places on the natural life that surrounds her, which is best captured by the word “engaging.” Answer Choice A is incorrect because the Lines aren’t describing Mrs. Manstey’s interest as being “relaxing.” Answer Choice B is incorrect because the Lines are describing her interests as being “amusing.” And Answer Choice D is incorrect because the Lines don’t suggest that Mrs. Manstey’s interest is “transforming.”

● “The magnolia is out earlier than usual this year, Mrs. Sampson,” she remarked, yielding to a rare impulse, for she seldom alluded to the absorbing interest of her life.

● “The magnolia is out earlier than usual this year, Mrs. Sampson,” she remarked, yielding to a rare impulse, for she seldom alluded to the engaging interest of her life.

How to solve this? The Question asks, “As used in Line 31, “absorbing” most nearly means [...]?

5.R.19

Answer Choice A is the correct answer because Lines 44-59 describe how the neighbor, Mrs. Black, will begin construction on an extension to her house, most closely matching Answer Choice A which says that Mrs. Sampson announces that “a neighbor will expand her house.” Answer Choices B, C and D are all incorrect because the Passage only suggests that [...].

How to solve this? The Question asks, “In the passage, Mrs. Sampson announces that which change will occur next door?” To solve this [...].

6.R.19

Answer Choice B is the correct answer because Lines 60-62 say, “Mrs. Mastey had grown pale. She always spoke slowly, so the landlady did not heed the long pause which followed.” These Lines come immediately after Mrs. Sampson tells her the news about the neighbor and most clearly show us her response. This matches Answer Choice B, which says that the Lines serve to “indicate the strength of the impression that a piece of news makes on Mrs. Manstey.” Answer Choice A is incorrect because these Lines don’t particularly express Mrs. Sampson’s receptiveness to the news, even if they do illustrate Mrs. Manstey’s aversion to it. Answer Choice C is incorrect because the Passage doesn’t indicate that Mrs. Manstey misinterpreted the news given to her. And Answer Choice D is incorrect because these Lines, while they do show Mrs. Manstey being reluctant to talk at first, don’t concern a personal matter and don’t necessarily show her unwillingness to talk.

How to solve this? In the context of the passage, Lines 60-62 (“Mrs. Manstey … followed”) serve mainly to [...]?” To solve this [...].

Page 279: SAT Full Reading Explanations (QAS 9-19) · SAT Full Reading Explanations (QAS 9-19) SAT #9 Reading 1 1.R.9 Answer Choice C is the correct answer because Lines 19-22 say, “Moths

7.R.19 & 8.R.19

Answer Choices C and D because Lines 69-71 say, “‘I should say it would. But there’s no help for it; if people have got a mind to build extensions there’s no law to prevent ‘em, that I’m aware of.” These Lines most clearly define an attitude of Mrs. Sampson to Mrs. Black’s extension, and tell us that she believes there’s nothing she can do, most closely matching Answer Choice C which says that her attitude is one of “stoical acceptance.” For Question 8, Answer Choice A is incorrect because these Lines only show Mrs. Sampson telling Mrs. Manstey about the extension, but don’t give any sign of how she feels about it. Answer Choice B is incorrect because these Lines continue to show Mrs. Sampson elaborating on the fact of the extension and how far it will go to, but don’t clearly tell us about her attitude towards it. Answer Choice C is incorrect because these Lines only show Mrs. Sampson questioning how Mrs. Black can afford the expansion, but still don’t clearly show her attitude towards it. For Question 7, Answer Choice A is incorrect because neither the Passage nor the Texts suggest that Mrs. Sampson is angry about the extension, but only that she’s resigned to accept it. Answer Choice B is incorrect because neither the Passages nor the Texts show Mrs. Sampson trying to deny the fact of the extension or pretend that it won’t happen. Answer Choice D is incorrect because the Passage tells us that Mrs. Sampson finds the extension to be a great burden but feels like there’s nothing she can do about it.

How to solve this? Question 7 asks, “Based on the passage, the attitude of Mrs. Sampson towards Mrs. Black’s extension is best described as one of [...]?” To solve this [...].

9.R.19 & 10.R.19

Answer Choices A and D are the correct answers because Lines 83-86 say, “Between her eyes and them a barrier of brick and mortar would swiftly rise; presently even the spire would disappear, and all her radiant world would be blotted out.” These Lines tell us most clearly the reaction of Mrs. Manstey to the news, and show that she finds that her “radiant world would be blotted out.” These Lines most clearly match Answer Choice A which says that Mrs. Manstey “feels that the life she has constructed for herself is about to come to an end.” For Question 10, Answer Choice A is incorrect because these Lines only show Mrs. Manstey asking Mrs. Sampson how she feels about the extension, but don’t tell us how Mrs. Manstey herself reacted. Answer Choice B is incorrect because these Lines only describe Mrs. Manstey turning back to the window, but don’t tell us about her reaction to the news. Answer Choice C only describes Mrs. Manstey’s appreciation of the day, but doesn’t tell us about her overall reaction to the news. For Question 9, Answer Choice B is incorrect because although the Passage does describe Mrs. Manstey admiring the nature, it doesn’t tell us that she can still find solace in it and in fact we know that soon she won’t be able to. Answer Choice C is incorrect because neither the Passage nor the Texts tell us that Mrs. Manstey wishes she had responded more forcefully. Answer Choice D is incorrect because neither the Text nor the Passage tell us that Mrs. Manstey plans to address the difficulties imposed on her.

How to solve this? Question 9 asks, “Based on the passage, which choice best describes Mrs. Manstey’s reaction to Mrs. Black’s plans?” To solve this [...].

Page 280: SAT Full Reading Explanations (QAS 9-19) · SAT Full Reading Explanations (QAS 9-19) SAT #9 Reading 1 1.R.9 Answer Choice C is the correct answer because Lines 19-22 say, “Moths

Reading 2

Passage Outline

The Man with the Muck Rake

● The man with the muck rake symbolizes the person who only focuses on the negative things. It's important to pay attention to these things, but if you focus too much on it, it becomes a force for evil. (Paragraph 1)

● There should be exposure and attack on evil people, but only if it is true. It's almost a worse evil to attack a person’s character dishonestly than stealing, and maybe worse. Attacking people’s character without reason doesn’t do good. (Paragraph 2)

● It’s easy to misinterpret this idea as defending evil people. Attacking people without discrimination is better for the actually evil, criminal person. (Paragraph 3)

● Any excess causes a reaction. Because people can make money through attacking character, it makes things worse. It also prevents good men from entering public service. (Paragraph 4)

● Any person who takes on trying to build the Panama Canal will probably have his character assaulted. (Paragraph 5)

● Again, clarify the idea. Not to excuse people, but to make the case that exposure of character should always be truthful or it can cause more harm than good. (Paragraph 6)

11.R.19

Answer Choice B is the correct answer because Lines 10-14 say, “But the man who never does anything else, who never thinks or speaks or writes, save of his feats with the muck rake, speedily becomes, not a help, … but one of the most potent forces for evil.” These Lines most clearly tell us that the purpose of the Passage is to argue against the practice of “mudslinging” or untruthful assaults on people’s character, especially those in public service. These Lines also match Answer Choice B which says that the purpose of the passage is to “identify and denounce a harmful practice.” Answer Choice A is incorrect because the Passage primarily looks at the practice of “mudslinging,” which isn’t best characterized as a discovery. Answer Choice C is incorrect because Roosevelt isn’t trying to recruit anybody for a project or initiative. Answer Choice D is incorrect because Roosevelt is mostly criticizing people who expose others untruthfully, not praising those who try to reveal social problems.

How to solve this? The Question asks, “The primary purpose of the passage is to [...]?” To solve this [...].

Page 281: SAT Full Reading Explanations (QAS 9-19) · SAT Full Reading Explanations (QAS 9-19) SAT #9 Reading 1 1.R.9 Answer Choice C is the correct answer because Lines 19-22 say, “Moths

12.R.19

Answer Choice A is the correct answer because Lines 20-24 say, “I hail as a benefactor every writer or speaker, every man who, on the platform, or in a book, magazine, or newspaper, with merciless severity makes such attack, provided always that he in his turn remembers that the attack is of use only if it is absolutely truthful.” These Lines tell us that Roosevelt, in the Passage, makes the case that evil and corruption should be exposed, but only if it’s truthful. This most closely matches Answer Choice A, which says that the central claim of the passage is that “efforts to expose corruption are valuable only when conducted truthfully and responsibly.” Answer Choice B is incorrect because, while Roosevelt does argue against those who make wrongful accusations, he doesn’t ever make the case that there need to be measures to protect against them. Answer Choice C is incorrect because Roosevelt doesn’t talk about addressing complex social problems and how they relate to the presidency. Answer Choice D is incorrect because Roosevelt argues against the idea that criticizing “mudslinging” protects the guilty.

How to solve this? The Question asks, “Which choice best states the central claim of the passage?” To solve this [...].

13.R.19

Answer Choice C is the correct answer because Roosevelt talks about the Man with the Muck Rake in the first paragraph, saying that he “typifies the man who in this life consistently refuses to see aught that is lofty, and fixes his eyes with solemn intentness only on that which is vile and debasing.” These Lines tell us that the figure is used to represent the person who only and unjustly focuses on the negative, something Roosevfelt disapproves of. Answer Choice A is incorrect because Roosevelt doesn’t make the point that social problems can be difficult to identify, or use the figure of the Man with the Muck Rake to illustrate this. Answer Choice B is incorrect because, although Roosevelt does mention the contrast between spiritual and carnal things, his main point isn’t that spiritual matters are neglected but that focusing on negative and unjust claims is a social harm. Answer Choice D is incorrect because Roosevelt doesn’t particularly criticize a pitfall of public service and doesn’t make the case that what he criticizes is inevitable.

How to solve this? The Question asks, “Roosevelt mentions the figure of the Man with the Muck Rake primarily in order to [...]?” To solve this [...].

Page 282: SAT Full Reading Explanations (QAS 9-19) · SAT Full Reading Explanations (QAS 9-19) SAT #9 Reading 1 1.R.9 Answer Choice C is the correct answer because Lines 19-22 say, “Moths

14.R.19

Answer Choice D is the correct answer because the word “service” in Line 9 is used to refer to the task job or “work” of the man with the muck rake. Answer Choice A is incorrect because the word doesn’t mean ability or “facility.” Answer Choice B is incorrect because the Lines aren’t referring to the “combat” of the Man with the Muck Rake. Answer Choice C is incorrect because the Lines aren’t referencing any ceremony.

● There is filth on the floor, and it must be scraped up with the muck rake; and there are times and places where this service is the most needed of all the services that can be performed.

● There is filth on the floor, and it must be scraped up with the muck rake; and there are times and places where this work is the most needed of all the services that can be performed.

How to solve this? The Question asks, “As used in Line 9, ‘service’ most nearly means [...]?” To solve this [...].

15.R.19

Answer Choice A is the correct answer because Lines 17-24 sy, “There should be relentless exposure of and attack upon every evil man, whether politician or businessman, every evil practice, whether in politics, in business, or in social life. I hail as a benefactor every writer or speaker, every man who, on the platform, or in a book, magazine, or newspaper, with merciless severity makes such attack, provided always that he in his turn remembers that the attack is of use only if it is absolutely truthful.” These Lines tell us that the word “every” is used to further the claim that every evil or wrongdoing should be attacked. This most closely matches Answer Choice A, which says that the word “emphasizes the point that all wrongdoing should be condemned.” Answer Choice B is incorrect because the Lines mentioned don’t talk about or suggest that Roosevelt wants to appeal to a wide audience. Answer Choice C is incorrect because the Lines don’t single out any particular kind of person for being more likely to be involved in scandals. Answer Choice D is incorrect because the Lines don’t make the point that evil deeds have become have become more prevalent or widespread in the modern world.

How to solve this? The Question asks, “What main effect does the repetition of the word ‘every’ in line 18 and line 19; have on the presentation of Roosevelt’s argument?” To solve this [...].

Page 283: SAT Full Reading Explanations (QAS 9-19) · SAT Full Reading Explanations (QAS 9-19) SAT #9 Reading 1 1.R.9 Answer Choice C is the correct answer because Lines 19-22 say, “Moths

16.R.19 & 17.R.19

Answer Choices B and D are the correct answers because Lines 20-24 say, “I hail as a benefactor every writer and or speaker, every man who, on the platform, or in a book, magazine, or newspaper, with merciless severity makes such attack, provided always that he in his turn remembers that the attack is of use only if it is absolutely truthful.” These Lines tell us that Roosevelt supports any criticism, so long as it’s accurate, but is likely to object to journalism that covers a scandal inaccurately. This most closely matches Answer Choice B, which says that Roosevelt would object to the approach of “distorting certain details to heighten the effect of the story.” For Question 17, Answer Choice A is incorrect because it tells us that Roosevelt thinks that the person who only focuses on the negative can become a force for evil, which less clearly relates to journalistic coverage and doesn’t match any of the answer choices for Question 16. Answer Choice B is incorrect because these Lines only make the assertion that there are many political and social evils that are worth addressing, but doesn’t offer any indication of what kind of coverage Roosevelt would object to. Answer Choice C is incorrect because these Lines tell us that Roosevelt thinks there should be relentless attack on true evil practice, without offering any indication of what kind of coverage he would object to. For Question 16, Answer Choice A is incorrect because neither the Passage nor the Texts talk about journalists taking incriminating photographs. Answer Choice C is incorrect because Roosevelt doesn’t criticize journalists for their complex presentation of technical information in either the Passage or the Text. Answer Choice D is incorrect because Roosevelt doesn’t talk about criminal acts committed with good intentions.

How to solve this? Question 16 asks, “Based on the passage, Roosevelt would be most likely to object to which of the following approaches to journalistic coverage of a scandal?” To solve this [...].

Page 284: SAT Full Reading Explanations (QAS 9-19) · SAT Full Reading Explanations (QAS 9-19) SAT #9 Reading 1 1.R.9 Answer Choice C is the correct answer because Lines 19-22 say, “Moths

18.R.19 & 19.R.19

Answer Choices A and C are the correct answers because Lines 38-40 say, “and both the interested individuals who need whitewashing, and those others who practice mud-slinging, like to encourage such confidence of ideas,” where “such ideas” refers to misinterpreting Roosevelt’s point as trying to protect those who do wrong (Lines 33-37). These Lines tell us that Roosevelt feels like his argument could be at risk of being misunderstood by journalist who “mud-sling” and those who dislike being exposed, matching Answer Choice A which says that Roosevelt feels his argument to be at risk of “deliberate misrepresentation by corrupt public figures and journalists.” For Question 19, Answer Choice A is incorrect because these Lines only talk about how bad it is to incorrectly attack and accuse honest people, without offering any potential risk to Roosevelt’s argument. Answer Choice B is incorrect because the Lines continue to talk about how indiscriminate or unfocused attacks on people’s character does harm and not good, without offering a potential risk to the overall argument. Answer Choice D is incorrect because these Lines talk about the reaction to assault upon the character of good people, and don’t clearly mention any potential risk to Roosevet’s argument. For Question 18, Answer Choice B is incorrect because neither the Passage nor the Texts talk about possible unintentional misinterpretation. Answer Choice C is incorrect because neither the Passage nor the Texts talk mention Roosevelt's argument being published for financial profit. Answer Choice D is incorrect because Roosevelt doesn’t ever make the specific case that he thinks that convicted criminals will endorse his argument.

How to solve this? Question 18 asks, “Based on the passage, Roosevelt believes that his argument about efforts to expose wrongdoing is subject to which risk?” To solve this [...].

20.R.19

Answer Choice B is the correct answer because Lines 70-73 say, “remember that even in the case of crime, if it attacked in sensational, lurid and untruthful fashion, the attack may do more damage to the public mind than the crime itself.” These Lines tell us that Roosevelt assumes that social critics can influence public sentiment, matching Answer Choice B which says that social critics are “capable of using their influence to shape public opinion.” Answer Choice A is incorrect because the Passage doesn’t make the case that social critics are immune to political pressure. Answer Choice C is incorrect because the Passage doesn’t say that social critics are responsible for the prosecution of guilty officials, and moreso makes the case that they’re responsible for the unjust criticism of certain officials. Answer Choice D is incorrect because Roosevelt doesn’t make the case that social critics are unable to investigate rumors properly, instead making the case that they deliberately choose to not investigate rumors properly.

How to solve this? The Question asks, “In the passage, Roosevelt makes which assumption about social critics’ relationship to society?” To solve this [...].

Page 285: SAT Full Reading Explanations (QAS 9-19) · SAT Full Reading Explanations (QAS 9-19) SAT #9 Reading 1 1.R.9 Answer Choice C is the correct answer because Lines 19-22 say, “Moths

21.R.19

Answer Choice C is the correct answer because the last paragraph primarily repeats Roosevelt’s argument and attempts to clarify his point to avoid confusion. This most closely matches Answer Choice C, which says that the last paragraph serves to “restate an important distinction in anticipation of a potential point of confusion.” Answer Choice A is incorrect because the last paragraph doesn’t offer any alternative solution to the problem of unjust social criticism. Answer Choice B is incorrect because the last paragraph doesn’t introduce anything personal about Roosevelt. Answer Choice D is incorrect because Roosevelt doesn’t suggest any possible shortcomings or flaws in his analysis.

How to solve this? The Question asks, “In the context of the passage as a whole, the last paragraph serves mainly to [...]?” To solve this [...].

Reading 3

Passage Outline

Pigment or Bacteria? Researchers Re-examine the Idea of ‘Color’ in Fossil Feathers

Passage 1

● paleontologists think that certain microscopic structures tell us about the color of ancient birds, but new research shows it might be impossible to predict if those are from bacteria or not. (paragraph 1)

● melanosomes, which are round and oblong, are the structures that scientists think tell us about coloration. but these could also be microbes. (paragraphs 2 and 3)

● a scientist wanted to find out if we could know which these were definitively. she designed an experiment using chickens to answer this. (paragraph 4)

● she came to the conclusion that it was impossible to tell which structure was what, and thinks that to answer this question more data, like fine scale chemical data, is needed. (paragraph 5)

Passage 2

● many fossils have microscopic structures on them, which could either be melanosomes or bacteria. (paragraph 1)

● a scientist and his team concluded that the structures are melanosome containing pigments. (paragraph 2)

● they accomplished this by analyzing the chemistry of the structures. theirs was the first study to conclusively show that it was pigmentation in the skin of fossilized animals. (paragraph 3)

● to accomplish this, they fired ions at the samples and measured the chemical composition of fragments. (paragraph 4)

● this research opens up the possibility of studying the coloration of animals without feathers, something that was previously difficult to do. (paragraph 5)

Page 286: SAT Full Reading Explanations (QAS 9-19) · SAT Full Reading Explanations (QAS 9-19) SAT #9 Reading 1 1.R.9 Answer Choice C is the correct answer because Lines 19-22 say, “Moths

22.R.19

Answer Choice C is the correct answer because the word “drove” in Line 19 is used to emphasize how microbes were the direct “cause” of decomposition. Answer Choice A is incorrect because it doesn’t make sense to say that bacteria would “chase” composition. Answer Choice B is incorrect because it also doesn’t make sense to say that the bacteria “prodded” decomposition. Answer Choice D is incorrect because it also doesn’t make sense to say that the bacteria “transported” decomposition.

● In fact, the microbes that drove the decomposition of the animal prior to fossilization share the same size and shape as melanosomes, and they would also be present in feathers during decay.

● In fact, the microbes that caused the decomposition of the animal prior to fossilization share the same size and shape as melanosomes, and they would also be present in feathers during decay.

How to solve this? The Question asks, “As used in line 19, ‘drove’ most nearly means [...]?”

23.R.19 & 24.R.19

Answer Choices A and C are the correct answers because Lines 25-29 say, “Using black and brown chicken feathers — chickens are one of the closest living relatives to both dinosaurs and ancient birds — Moyer grew bacteria over them to replicate what we see in the fossil record.” These Lines tell us most clearly that Moyer used chicken feathers because they’re the most closely related to dinosaurs and ancient birds and can provide the best insight into those species. This most closely matches Answer Choice for Question 23, which says that “conclusions drawn from research on chicken feather pigmentation might inform understanding of the pigmentation of avian dinosaurs.” For Question 24, Answer Choice A is incorrect because these Lines only introduce the scientific problem of the Passage, and don’t tell us anything about why Moyer chose to use chicken feathers for her study. Answer Choice B is incorrect because these Lines only explain what melanosomes are, without telling us anything about why the Moyer used chicken feathers. Answer Choice D is incorrect because these Lines only talk about the final step in the methodology of Moyer’s experiment, without giving any insight into her choice of using chicken feathers. For Question 23, Answer Choice B is incorrect because neither the Texts nor the Passage say anything about bacteria interfering with research on melanosomes or how chicken feathers are resistant to that bacteria. Answer Choice C is incorrect because neither the Passage nor the Texts talk about the stability of chicken feathers, and why they’re conducive to being studied. Answer Choice D is incorrect because neither the Passage nor the Texts talk about how chicken feathers are widely studied and how those are important to the research at hand.

How to solve this? The Question asks, “It can most reasonably be inferred from Passage 1 that Moyer’s study involved chicken feathers in part because [...]?” To solve this [...].

Page 287: SAT Full Reading Explanations (QAS 9-19) · SAT Full Reading Explanations (QAS 9-19) SAT #9 Reading 1 1.R.9 Answer Choice C is the correct answer because Lines 19-22 say, “Moths

25.R.19

Answer Choice C is the correct answer because the word “determined” in Lines 53-54 most nearly means identified or “established,” emphasizing how the team found out what those pigments were. Answer Choice A is incorrect because the Lines aren’t saying that the team “restricted” the pigments, and it’s unclear what that would mean. Answer Choice B is incorrect because the Lines don’t mean that the researchers “regulated” the pigments, and it’s also unclear what that would mean. Answer Choice D is incorrect because “arbitrate” has connotations of authority and in deciding between different views on an issue, which doesn’t quite match the researchers finding out what the pigments were.

● By studying the molecular composition of the pigments, Johan Lindgren of Lund University and his team not only concluded that the deposits are pigment remains, but also determined what those pigments were.

● By studying the molecular composition of the pigments, Johan Lindgren of Lund University and his team not only concluded that the deposits are pigment remains, but also determined what those pigments were.

How to solve this? The Question asks, “ As used in Lines 53-54, ‘determined’ most nearly means [...]?” To solve this [...].

26.R.19

Answer Choice B is the correct answer because Lines 66-76 say, “It was enough to provide the first unequivocal evidence of pigmentation in the skin of a fossilized animal, says Maria McNamara of the University of Bristol, who was not involved in the study.” These Lines tell us that McNamara finds Lindgren’s evidence “unequivocal,” or beyond doubt. This most closely matches Answer Choice B, which says that McNamara finds the evidence “clear and persuasive.” Answer Choice A is incorrect because McNamara thinks the results are convincing, not doubtful. Answer Choice C is incorrect because McNamara is positive about the results, and doesn’t think they’re unreliable. Answer Choice D is incorrect because McNamara doesn’t characterizie the results as unexpected.

How to solve this? The Question asks, “Based on Passage 2, McNamara most likely considers the results of Lindgren’s team’s study, described in the third paragraph (lines 66-76) to be [...]?

Page 288: SAT Full Reading Explanations (QAS 9-19) · SAT Full Reading Explanations (QAS 9-19) SAT #9 Reading 1 1.R.9 Answer Choice C is the correct answer because Lines 19-22 say, “Moths

27.R.19

Answer Choice D is the correct answer because the Passage tells us that, instead of relying on a merely visual approach, Lindgren’s team analyzed the chemistry of the samples. Lines 69-76 say, “The ions broke up the material and sent fragments flying into a detector, which analyzed their chemical composition and confirmed the dark deposits were eumelanin. Under the microscope, Lindgren’s team showed that concentrations of eumelanin peaked in areas with the highest density of the tiny egg-shaped structures — suggesting the structures were indeed melanosomes, not bacterial cells.” These Lines tell us most clearly that the strongest evidence for the identification of the structures as melanosomes was the detection of eumelanin. This most closely matches Answer Choice D, which says that the most convincing evidence was the “high concentrations of eumelanin near the egg-shaped structures.” Answer Choice A is incorrect because the Passage doesn’t mention that the lack of damage to the egg-shaped structures suggested they were melanosomes. Answer Choice B is incorrect because the Passage doesn’t talk about how the presence of egg-shaped structures in reptile fossils confirmed that they were melanosomes. Answer Choice C is incorrect because the Passage makes the case that the dark appearance could suggest either melanosomes or bacteria.

How to solve this? The Question asks, “As presented in Passage 2, which finding provided the most convincing evidence that the egg-shaped structures found in the fossils of the marine reptiles are probably the remains of melanosomes?” To solve this [...].

28.R.19

Answer Choice A is the correct answer because both Passage focus on specific research projects that seek to identify whether certain structures in fossils are melanosomes or bacteria, and describe the process behind each project. This most closely matches Answer Choice A which says that the main purpose of both Passages is to “report on the methods and findings of specific laboratory observations.” Answer Choice B is incorrect because the Passages only focus on the development of a single topic in micropaleontology and don’t look at the field broadly. Answer Choice C is incorrect because there’s no specific, prevailing assumption about what these structures are, but instead an open question which both Passages address. It’s also unclear if the issue can be characterized as one specifically about “dinosaur anatomy.” Answer Choice D is incorrect because neither Passage makes the direct case the science benefits from the adoption of new technologies.

How to solve this? The Question asks, “One of the main purposes of both Passage 1 and Passage 2 is to [...]?” To solve this [...].

Page 289: SAT Full Reading Explanations (QAS 9-19) · SAT Full Reading Explanations (QAS 9-19) SAT #9 Reading 1 1.R.9 Answer Choice C is the correct answer because Lines 19-22 say, “Moths

29.R.19

Answer Choice B is the correct answer because both authors believe that the coloration of dinosaurs can be determined from looking at the melanosomes they contain. In Passage 1, the first and second paragraphs show the author clearly stating that these microscopic melanosomes can tell us about the coloration of dinosaurs, like, for example, in Lines 10-12 which say of melanosomes that “they contain melanin, which can give feathers colors ranging from brownish-red to gray to solid black.” In Passage 2, the second paragraph clearly states that analysis of the fossils can tell us about the coloration of dinosaurs, like, for instance, in Lines 52-53 which say that Lindgren and his team “not only concluded that the deposits are pigment remains, but also determined what those pigments were.” This most closely matches Answer Choice B, which says that both authors agree that dinosaur coloration “could plausibly be deduced from the remains of certain microscopic structures.” Answer Choice A is incorrect because, while both authors might agree with this, they never directly make the claim in either Passage. Answer Choice C is incorrect because neither Passage makes the case that dinosaur coloration was limited to certain shades, and Passage 1 says that the pigments in melanosomes could range “from brownish-red to gray to solid black.” Answer Choice D is incorrect because neither Passage directly makes the case that dinosaur coloration is unlikely to have left a trace in fossilized skin and because Passage 2 makes the point that “Lindgren’s study opens the door to reconstruct coloration in a wider range of species, including non feathered dinosaurs.”

How to solve this? The Question asks, “Moyer (Passage 1) and Lindgren’s team (Passage 2) would most likely agree that dinosaur coloration [...]?”

30.R.19

Answer Choice A is the correct answer because Moyer concludes that that it’s “impossible to say with certainty what these structures are without more data, including fine scale chemical data,” and Lindgren’s team analyzed “the chemistry of the structures and pigments in the samples.” This most clearly matches Answer Choice A, which says that “Moyer identifies a need for chemical analysis, which Lindgren’s team supplies.” Answer Choice B is incorrect because the Passage doesn’t show that Lindgren’s team used Moyer’s research as a basis for their own, and instead take an entirely different route in their analysis. Answer Choice C is incorrect because Moyer doesn’t reach any conclusion, and because Lindgren’s team doesn’t challenge her. Answer Choice D is incorrect because Passage 2 doesn’t show Lindgren’s team addressing Moyer’s research at all or questioning its appropriateness.

How to solve this? The Question asks, “Which choice best describes a particular relationship between Moyer’s study (Passage 1) and the study by Lindgren’s team (Passage 2)? To solve this [...].

Page 290: SAT Full Reading Explanations (QAS 9-19) · SAT Full Reading Explanations (QAS 9-19) SAT #9 Reading 1 1.R.9 Answer Choice C is the correct answer because Lines 19-22 say, “Moths

31.R.19

Answer Choice C is the correct answer because Lines 59-60 say, “Previous studies relied on a visual identification of those egg-shaped melanosomes,” referencing the kind of study that Moyer conducted, when she “used three different types of microscopy to examine the patterns of biofilm growth, and then compared those structures to melanosomes inside of chicken feathers that she had sliced open.” Answer Choice A is incorrect because these Lines only indicate that many fossils have dark deposits on them, but don’t tell us anything about Moyer’s research methodology. Answer Choice B is incorrect because these Lines only tell us that some scientists think that these dark deposits are due to bacteria and not melanosomes, without telling us anything about Moyer’s methodology. Answer Choice D is incorrect because these Lines only tell us about Lindgren’s team’s methodology, and don’t tell us anything about Moyer.

How to solve this? The Question asks, “Which statement from Passage 2 best describes an important feature of the research methodology that Moyer (Passage 1) uses?” To solve this [...].

Reading 4

Passage Outline

The Surprising Benefits of Sarcasm

● Defining sarcasm, as a way to humorously convey disapproval or scorn. Two scientists have found that using sarcasm could contribute to greater creativity. (Paragraphs 1 & 2)

● Most people interpret sarcasm as more critical than literal comments. People can also often misinterpret sarcasm, and overestimate how much people will understand them when they use it. (Paragraphs 3-5)

● However, one study shows a positive for sarcasm. In a study where participants used sarcasm, they performed better on a questionnaire that tests creativity. (Paragraphs 6-8)

● Some questions about why irony or sarcasm could enhance creativity. Perhaps because it requires using tone to sound serious but show that you’re joking, which stimulates creativity. (Paragraph 9)

32.R.19

Answer Choice D is the correct answer because Lines 4-6 say, “Often we use it to humorously convey disapproval or scorn. ‘Pat, don’t work so hard!’ a boss might say, for example, on catching his assistant surfing the Web.” These Lines tell us that a common use of sarcasm is to show disapproval or negative opinion, and most closely match Answer Choice D which says that sarcasm is commonly used to “communicate negative feedback.” Answer Choices A, B and C are incorrect because the Passage never directly marks these instances as common uses of sarcasm.

How to solve this? The Question asks, “According to the passage, a common use of sarcasm is to [...]?”

Page 291: SAT Full Reading Explanations (QAS 9-19) · SAT Full Reading Explanations (QAS 9-19) SAT #9 Reading 1 1.R.9 Answer Choice C is the correct answer because Lines 19-22 say, “Moths

33.R.19

Answer Choice C is the correct answer because Lines 28-30 say, “They gave 30 pairs of university students a list of statements, half of which were sarcastic and half serious,” where “serious” is used to describe statements that were straightforward in contrast to “sarcastic” statements with disguised meanings. This most closely matches the word “direct” in Lines 18-19, which say, “[...] most perceive such comments as critical compared with more direct utterances.” Answer Choice A is incorrect because the word “unexpected” in Line 9 is used to describe something surprising, which doesn’t match the meaning of the word “serious.” Answer Choice B is incorrect because the word “creative” in Line 14 is used to describe the ability to generate new ideas, which doesn’t match “serious.” Answer Choice D is incorrect because “overconfident” is used to describe people’s strong sense of how they’ll be interpreted, which doesn’t match the word “serious.” How to solve this? The Question asks, “As used in the passage, which word is most similar in meaning to “serious” (line 30)?” To solve this [...].

34.R.19

Answer Choice A is the correct answer because the word “badly” in Lines 38-39 is used to describe the strength or “severity” of the participant’s overestimation. Answer Choice B is incorrect because the Lines don’t mean that they inadequately overestimated their ability, as if there were an adequate amount they should have overestimated. Answer Choice C is incorrect because the Lines don’t mean that their overestimation indicated something unfavorable about their attitude towards their ability. Answer Choice D is incorrect because the Passage doesn’t give us any reason to think that the participants had regret about their overestimation.

● That is, they badly overestimated their ability to communicate the tenor of these statements via email.

● That is, they severely overestimated their ability to communicate the tenor of these statements via email.

How to solve this? The Question asks, “As used in Line 38, ‘badly’ most nearly means [...]?” To solve this [...].

Page 292: SAT Full Reading Explanations (QAS 9-19) · SAT Full Reading Explanations (QAS 9-19) SAT #9 Reading 1 1.R.9 Answer Choice C is the correct answer because Lines 19-22 say, “Moths

35.R.19

Answer Choice B is the correct answer because the phrase “negative picture of sarcasm” comes immediately after the discussion of several issues with sarcasm, like people perceiving it as more critical than direct statements or strongly misinterpreting it. This tells us that this “negative picture” emerges because people are liable to miscommunicate while using it, closely matching Answer Choice B which says that the “negative picture” emerges because sarcasm can “obscure the actual intention of a communication.” Answer Choice A is incorrect because the Passage doesn’t directly make the point that sarcasm specifically emphasizes differences over similarities. Answer Choice C is incorrect because the Passage doesn’t mention any cross-cultural comparisons. Answer Choice D is incorrect because, while perhaps true, the Passage doesn’t directly make a point about the use of sarcasm during solemn occasions.

How to solve this? The Question asks, “The passage suggests that before the author’s research, the ‘negative picture of sarcasm’ (line 44) emerged because sarcasm can [...]?” To solve this [...]?

36.R.19

Answer Choice D is the correct answer because Lines 55-57 say, “Not surprisingly, the participants exposed to sarcasm reported more interpersonal conflict than those in other groups.” To support the fact that this claim is not surprising, we should find a text that tells us clearly that sarcasm is associated with feelings of interpersonal conflict. Answer Choice D most clearly fulfills this, and says, “Consistently, participants rated sarcasm to be more condemning than literal statements.” Answer Choice A is incorrect because Lines 1-2 only introduce the concept of sarcasm, without saying anything about how it increases feelings of conflict. Answer Choice B is incorrect because these Lines talk about a positive aspect of sarcasm and how it promotes creativity, without telling us anything about how it increases feelings of conflict. Answer Choice C is incorrect because these Lines only describe the methodology of an experiment, without giving us any finding that tells about the relationship between sarcasm and feelings of conflict.

How to solve this? The Question asks, “Which choice best supports the claim made by the author in Lines 55-57 that a particular result of her research was not surprising?” To solve this [...].

Page 293: SAT Full Reading Explanations (QAS 9-19) · SAT Full Reading Explanations (QAS 9-19) SAT #9 Reading 1 1.R.9 Answer Choice C is the correct answer because Lines 19-22 say, “Moths

37.R.19 & 38.R.19

Answers Choices A and D are the correct answers because the author’s 2015 study showed an association between the use of sarcasm and creativity. However, Lines 58-61 say, “This effect emerged for both the deliverer and recipient in the simulated conversation but only when the recipient had picked up on the sarcasm in the script,” making it plausible that those who can pick up on sarcasm tend to be more creative. This most closely matches Answer Choice A for Question 37, which says that one possible interpretation is that “people who perceive sarcasm well tend to be creative.” For Question 38, Answer Choice A is incorrect because it provides little justification for a possible interpretation and merely describes the methodology of the experiment. Answer Choice B does the same thing. Answer Choice C tells us directly that those who engaged in sarcastic conversation did better on creativity tasks, but doesn’t offer much that would provide an interpretation of the matter. For Question 37, Answer Choice B is incorrect because neither the Passage nor the Texts draw any connection between sarcasm and being logical. Answer Choice C is incorrect because neither the Passage nor the Texts draw an association between creativity and combativeness. Answer Choice D is incorrect because the Passage also doesn’t draw any connection between people seeking logical connections and begin conversational.

How to solve this? Question 37 asks, “Based on the passage, which choice presents a possible interpretation arising from the results of the 2015 study conducted by the author and her colleagues?” To solve this [...].

39.R.19

Answer Choice C is the correct answer because the last paragraph primarily explores the question of why sarcasm or verbal irony enhances creativity, and offers one possible explanation. This most closely matches Answer Choice C, which says that the main purpose of the final paragraph is to “put forth a potential explanation for results discussed in the passage.” Answer Choice A is incorrect because the last paragraph doesn’t offer any possible practical applications. Answer Choice B is incorrect because the last paragraph doesn’t even mention any possible challenges explored in the passage. Answer Choice D is incorrect because the Passage doesn’t mention two interpretations or advocate for any kind of compromise between them.

How to solve this? The Question asks, “The main purpose of the last paragraph is to [...]?” To solve this [...].

40.R.19

Answer Choice B is the correct answer because [...].

41.R.19

Answer Choice A is the correct answer because [...].

Page 294: SAT Full Reading Explanations (QAS 9-19) · SAT Full Reading Explanations (QAS 9-19) SAT #9 Reading 1 1.R.9 Answer Choice C is the correct answer because Lines 19-22 say, “Moths

Reading 5

Passage Outline

City Rabbits, Like Humans, Live in Smaller Homes

● Just like humans, who live in smaller homes in the city, rabbits build smaller homes in urban environments. (Paragraph 1)

● The rabbit population in the countryside has shrunk, but has grown in cities. A graduate student studied these rabbits in Frankfurt. (Paragraph 2)

● The team found 191 rabbit burrows, and ranked them in terms of how urban they are. They found these homes using two different methods. (Paragraphs 3 & 4)

● As these burrows became more “urban,” they became smaller, and more clumped together. The researcher said that the results could have been the opposite. (Paragraph 5)

● Rabbit homes may be smaller because heat is less of an issue since the city is already hot. There are also less predators so there can be less escape routes. Rabbits also live in large groups when there are few resources, so the city may provide more resources than the countryside. (Paragraph 6)

43.R.19

Answer Choice D is the correct answer because the word “regular” in Line 6 is used to emphasize the fact that the houses appear in a more predictable or “consistent” manner. Answer Choice A is incorrect because the word “usual” suggests that the houses appear in an order that is typical or expected. answer Choice B is incorrect because it doesn’t make sense to say that the intervals in which the houses occur is “traditional.” Answer Choice C is incorrect because the Lines don’t mean that the houses appear in an order that is a habit or “habitual.”

● As you approach a city (finally!), rows of houses appear at regular intervals instead of clumping.

● As you approach a city (finally!), rows of houses appear at consistent intervals instead of clumping.

44.R.19

Answer Choice A is the correct answer because Lines 8-11 say, “European rabbits, it turns out, build their homes in a similar way — and since these animals are disappearing in the countryside, understanding their urban planning strategy matters to humans trying to conserve them.” These Lines tell us that understanding rabbits burrowing patterns can help in understanding how to conserve them, or preserve their population, matching Answer Choice A. Answer Choice B is incorrect because the Passage doesn’t mention constructing burrows for rabbits anywhere. Answer Choice C is incorrect because the Passage also doesn’t talk about transferring rabbit populations. Answer Choice D is incorrect because the Passage doesn’t talk about the goal of trying to urbanize the rabbits’ habitats.

Page 295: SAT Full Reading Explanations (QAS 9-19) · SAT Full Reading Explanations (QAS 9-19) SAT #9 Reading 1 1.R.9 Answer Choice C is the correct answer because Lines 19-22 say, “Moths

How to solve this? The Question asks, “According to the passage, information about rabbits’ burrowing patterns could aid in [...]?” To solve this [...].

45.R.19

Answer Choice D is the correct answer because Lines 41-43 say, “And while rural burrows were spread out and clumped, like the rural houses on our imaginary road trip, urban burrows were spaced out more evenly.” These Lines tell us most clearly that, compared to rural burrows, urban burrows were more evenly spaced out and therefore easier for scientists to predict their relative positions. Answer Choice A is incorrect because these Lines only tell us how the =rural rabbit populations has been driven down, without telling us anything about the relative positions of urban rabbit burrows. Answer Choice B is incorrect because these Lines only tell us about these scientists rated the burrows for “urbanity,” without telling us how it’s easier to identify the relative positions of urban burrows compared to rural ones. Answer Choice C is incorrect because these Lines only tell us that these urban burrows became more frequent or “common,” and tell us less clearly than Answer Choice D that its easier to identify their relative positions.

How to solve this? The Question asks, “Which choice best supports the idea that it is easier to predict the relative positions of burrows in areas with a high urbanity rating than it is in areas with a low urbanity rating?” To solve this [...].

46.R.19

Answer Choice D is the correct answer because the word “artificial” in Line 28 is used to emphasize the the man-made or non-natural quality of objects like pavement or playground turf, which most closely matches “synthetic.” Answer Choice A is incorrect because “alternate” would require [...]. Answer Choice B is incorrect because the word “counterfeit” implies that something is pretending to be something else, which doesn’t make sense in the Lines. Answer Choice C is incorrect because the word “affected” is used to either to express that something is influenced by something or of someone who’s trying to impress someone, neither of which makes sense in the Lines.

● And how much of the ground is covered by something artificial, such as pavement or playground turf?

● And how much of the ground is covered by something synthetic, such as pavement or playground turf?

How to solve this? The Question asks, “As used in Line 28, ‘artificial’ most nearly means [...]?” To solve this [...].

Page 296: SAT Full Reading Explanations (QAS 9-19) · SAT Full Reading Explanations (QAS 9-19) SAT #9 Reading 1 1.R.9 Answer Choice C is the correct answer because Lines 19-22 say, “Moths

47.R.19 & 48.R.19

Answers Choices A and D are the correct answers because Lines 36-37 say, “They also counted burrow entrances to estimate how big each home was.” These Lines tell us that the researchers assumed that by looking at burrow entrances, they could determine roughly how big the burrow was. This most closely matches Answer Choice A for Question 47, which says that the researchers assumed that “the size of a burrow can be estimated by certain aspects of the burrow’s outward appearance.” For Question 48, Answer Choice A is incorrect because these Lines only tell us where the researchers looked for rabbit burrows, don’t offer a clear indication of any assumption the researchers made and don’t match any of the answer choices for Question 47. Answer Choice B is incorrect because these Lines only tell us about one of the methods the researchers used to identify rabbit burrows without offering any clear indication of an assumption. Answer Choice C is incorrect because these Lines only tell us that researchers counted how many rabbits came and went from their burrows, which doesn’t offer any clear assumption or match any of our answer choices. For Question 47, Answer Choice B is incorrect because the Passage doesn’t compare experimental settings with natural settings, or offer any judgment on how they affect behavior. Answer Choice C is incorrect because neither the Passage nor the Texts talk about the different activities rabbits engage in during dawn and dusk. Answer Choice D is incorrect because neither the Passage nor the Texts make a comparison between the populations of rabbits in park spaces and the countryside.

How to solve this? Question 47 asks, “It can reasonably be inferred from the Passage that the researchers’ data-collection methods depended on an assumption that [...]?” To solve this [...].

49.R.19

Answer Choice A is the correct answer because Lines 49-52 say, “Big groups of rabbits keep their burrows toastier in the winter — but cities are a little warmer to begin with, so living with a lot of warm bodies might not be as important.” These Lines tell us that in the countryside, which tends to be colder than the city, rabbits are more likely to live together in big burrows to keep warm. This most closely matches Answer Choice A which says that in rural areas, European rabbits “cluster in densely populated burrows for warmth.” Answer Choice B is incorrect because the Passage doesn’t tell us that rural rabbits avoid going aboveground when its cold. Answer Choice C is incorrect because, while the Passage does talk about how rabbits in rural areas avoid predation, it doesn’t mention specific strategies used for the winter. Answer Choice D is incorrect because the Passage doesn’t talk about how rabbits delay their breeding for the warmer spring months.

How to solve this? The Question asks, “The passage suggests that the comparatively lower temperatures in rural areas influence European rabbits to [...]?” To solve this [...].

Page 297: SAT Full Reading Explanations (QAS 9-19) · SAT Full Reading Explanations (QAS 9-19) SAT #9 Reading 1 1.R.9 Answer Choice C is the correct answer because Lines 19-22 say, “Moths

50.R.19

Answer Choice C is the correct answer because Lines 55-56 say, “Finally, rabbits tend to live in large groups resources are limited.” This tells us that the opposite is also true, that, when resources are abundant, rabbits tend to live in smaller groups. This most closely matches Answer Choice C, which says that “an abundance of food in a given area correlates with a low average population per burrow.” Answer Choice A is incorrect because the Passage doesn’t talk about the genetic diversity of rabbits or relate it to the size of their burrows. Answer Choice B is incorrect because the Passage tells us the opposite, that a burrow tends toward greater complexity when predation threat increases. Answer Choice D is incorrect because the Passage doesn’t relate the size of burrows to the availability of land.

How to solve this? The Question asks, “Based on the passage, which inverse relationship is demonstrated by the burrowing patterns of European rabbits?” To solve this [...].

51.R.19

Answer Choice C is the correct answer because the point closest to an urbanity rating of 1, as measured by the x-axis, rises to a level of about 3.2, which falls “between 3 and 4.” The other Answer Choices are wrong because [...].

How to solve this? The Question asks, “According to figure 1, the site with an urbanity rating closest to 1 was found to have how many burrows per hectare?” To solve this [...].

52.R.19

Answer Choice C is the correct answer because [...].